Download tests on dermatology with 1 correct answer 1. What medications

Survey
yes no Was this document useful for you?
   Thank you for your participation!

* Your assessment is very important for improving the workof artificial intelligence, which forms the content of this project

Document related concepts
Transcript
tests on dermatology with 1 correct answer
1. What medications often cause erythema fixed
A. Sulfadimetoksin*
B. Analginum
C. aspirin
D. Ampicillin
E. No correct answer
2. What should be differentiated disease spread
toxicoderma
A. herpes
B. Pyoderma
C. Pemphigus vulgaris*
D. Disease Borowski
E. No correct answer
3. Enter the typical localization at multiforme exudative
Erythema
A. Scalp
B. Rear feet*
C. The neck
D. Interdigital spaces of hands
E. No correct answer
4. What factors are important in the development multiforme
exudative erythema
A. Hereditary
B.Photosensitivity
C.Foci of chronic infection*
D. Hyperactivity of the sebaceous glands
E. No correct answer
5. What dermatoses differentiate erythema multiforme
A. Lyell's syndrome*
B. Strofulyus
C. Scrapie
D.psoriasis
E. No correct answer
6. Specify the pathogenetic factors of pink lichen Gibert
A.The introduction into the skin mites
B.Yeasts
C. Idiosyncrasy to iodine
D.Viral infection*
E. No correct answer
7. What kinds of microbial eczema
A. Adult
B.Paratravmaticheskaya*
C.Idiopathic
D. Horn
E. No correct answer
8. How is a clinical sign is characterized by a rash in true
A.eczema
B. Symptom <serous wells>*
C. Tubercles asymmetry
D.Induratum edema
E. No correct answer
9. What are the symptoms seen with discoid lupus
A. Net Wickham
B. Collar Biett
C. Sign "ladies' heel"*
D. Paired elements
E. No correct answer
10. Specify the variety of scleroderma
A. Vulgar
B. White spot disease*
C. Infiltrative
D.Intertriginoznoy
E. No correct answer
11. Lichen planus is characterized by
A. Absence of pruritus
B. Tubercles
C. Mesh Wickham*
D. Urticaria rashes
E. No correct answer
12. planus characterized by
A. Swelling of the spinous layer of intercellular
B. Akantolizis
C. Vacuolar degeneration
D. Irregular thickening of the granular layer of the epidermis*
E. No correct answer
13. What are the clinical symptom pathognomonic for red
A.lichen planus "honeycomb Celsus*
B.Net Wickham
C.Cells Ttsanka
D. A symptom auspices
E. No correct answer
14. What diseases do not belong to a group of viral dermatoses
A. Warts
B. Genital warts
C. Psoriasis*
D. shingles
E. No correct answer
15. The vascular patch is
A.Erythema*
B. Chloasma
C. Nevus
D. Leucoderma
E. No correct answer
16. Primary morphological elements
A. scar
B. Bubble*
C. erosion
D. ulcer
E. No correct answer
17. Secondary morphological elements
A. Roseola
B. Lace
C. Node
D.Ulcer *
E. No correct answer
18. The bubble is formed at
A.Urticaria
B. CPL
C. Herpes simplex*
D. Psoriasis
E. No correct answer
19. The outcome of the node is
A. Spot*
B. Atrophy
C. Ulcer
D. Scar
E. No correct answer
20. exudative morphological element is
A. Node
B. Tubercle
C. Vesicle*
D. Papule
E. No correct answer
21. Specify dosage forms for topical treatment in accordance with step
Eczema:
A. Weeping – gadgets*
B. The stage of secondary pyoderma gadgets
C. Bubble stage ointments
D. Exudation powders
E. No correct answer
22. What diseases are at neyrodermatozov:
A. Pink zoster Gibert
B. Dermatitis Duhring
C. Lichen planus
D. Scrapie*
E. No correct answer
23.Specify the reasons that may be the cause of localized
itch:
A. Itch mites
B. Violation of the endocrine glands
C. Hemorrhoids*
D. Ketoacidosis
E. No correct answer
24. What preparations have antipruritic effect:
A. Anesthetics*
B.Sulfonamides
C. Antibiotics
D. Cytostatics
E. No correct answer
25. What comorbidities are common in
atopic dermatitis:
A. hepatitis
B. Toxoplasmosis
C. tuberculosis
D. Vasomotor rhinitis*
E. No correct answer
26. In what diseases there is white dermographism:
A. scabies
B. Allergic dermatitis
C. Scrapie*
D. psoriasis
E. No correct answer
27. Describe the main pathogenetic mechanisms of development of children
scrapie:
A. Hereditary fermentopathy*
B. Center focal infection
C. Endocrine disorders
D.Immune deficiency
E. No correct answer
28. What are the rash characteristic of psoriasis:
A. Vegetating
B. Erosive and ulcerative
C. Monomorphic pustular
D. Papular
E. No correct answer
29. What areas are distinguished in focal neurodermatitis:
A. Peripheral zone pustular rash
B. Center Zone lihenifikatsii*
C. Central erythematoussquamous
D. Average a zone of depigmentation
E. No correct answer
30. What are the signs characteristic of urticaria:
A. Secondary atrophic changes in the skin
B. Monomorphic papules
C. Monomorphic blisters*
D. White dermographism
E. No correct answer
31 patients with acute angioedema starting asphyxia. What preparations
Rata be appointed to help:
A. Korglikon 0.5 / m
B. Gemodez 200.0 drip into a vein
C. Bronholitin
D. Prednisone 6080 mg into a vein*
E. No correct answer
32. What diseases are at bullous dermatosis:
A. Stafilodermiya
B. Pemphigus vulgaris*
C. shingles
D. chickenpox
E. No correct answer
33. Under what bullous dermatosis observed eruptions:
A. herpes
B. Stafilodermiya
C. Lyell's syndrome*
D. Rosacea
E. No correct answer
34. The classification of true pemphigus include:
A. Pemphigus vulgaris*
B. Exudative
C. Handfoot
D. Disgidroticheskaya
E. No correct answer
35. What are the symptoms characteristic of true pemphigus:
A. Symptom GorchakovHardy
B. A symptom of "apple jelly"
C. Symptom Nikolsky*
D. Symptom Pospelov
E. No correct answer
36. In what diseases there is a positive sign
Nikolsky:
A. Contact dermatitis bullous stage
B. Pemphigus vulgarisB*
C. Dermatitis Duhring
D. Bullous streptoderma
E. No correct answer
37. What are the basic rules of corticosteroids in the treatment of hormone
pemphigus:
A. Hormones are prescribed only for external treatment
B. Treatment is initiated with a bolus
C. Treatment start with a gradual increase in the dose until the shock
D. Taking hormones administered in the evening
E. No correct answer
38. What are the complications observed in the treatment of pemphigus:
A. The development of hypersensitivity
B. Oppression of the hematopoietic system
C. cirrhosis of the liver
D. Cushing's syndrome*
E. No correct answer
39. What are the typical clinical signs of dermatitis Dühring:
A. The defeat of the oral mucosa
B. A positive sign of Nikolsky
C. Bunching rash*
D. Infiltrative lesions are composed of elements
E. No correct answer
40. What are the morphological features are characteristic rash
dermatitis Dühring:
A.Vegetation
B. Units
C. Bumps
D. Papular*
E. No correct answer
41. What are the laboratory findings are characteristic of dermatitis Dühring:
A. Eosinophils in the cystic fluid*
B. Lymphocytosis
C. Leykotsitozv cystic fluid
D. Histomorphological acantholysis
E. No correct answer
42. Which dermatosis characterized by the appearance of rashes on
oral cavity:
A. psoriasis
B. Candidiasis*
C. Pink zoster Gibert
D. Hydradenitis
E. No correct answer
43. Name the rash with a clear viral etiology:
A. Dermatitis Duhring
B. Lichen planus
C. Herpes*
D. Pemphigoid
E. No correct answer
44. What are the characteristics of lesions in the herpes simplex:
A. Subjectively itching prevails over the soreness and burning
B. Polymorphism rash
C. Bubble elements of monomorphic*
D. Grouped cystic lesions
E. No correct answer
45. What are the features of genital herpes from a solid
chancre:
A. No solid infiltrate the base
B. Positive PB
C. Painless erosions
D. Regional bubo
E. No correct answer
46. What are the clinical forms of herpes zoster:
A. Disgidroticheskaya
B. Generalized*
C. Vegetating
D. Neuralgia
E. No correct answer
47. Specify the drug has an antiviral
action:
A. Metatreksat
B. Sulphur cleared
C. FIBS
D. Tsiklovir*
E. No correct answer
48. Assign the drug to patients with a diagnosis of herpes zoster:
A. Sodium bromo drops
B. Prednisolone ointment
C. Analgesics*
D. Ampicillin
E. No correct answer
49. Specify the characteristics of molluscum contagiosum:
A. Transmission transmissive
B. Nodules contained within the white curd*
C. Subjectively itching sensation
D. A symptom of "net Wickham" positive
E. No correct answer
50. What forms of lupus are distinguished according to the classification:
A. Discoid*
B. Vegetating
C. Squamous
D. Pustular
E. No correct answer
51. What are the cardinal symptoms of the rash of lupus erythematosus:
A. Lihenifikatsiya
B. Keloid scars
C. Pustular infiltration
D. Infiltrative erythema*
E. No correct answer
52. What are the symptoms observed in discoid lupus erythematosus:
A. "Grid" Wickham
B. "Butterfly"*
C. "Honeycomb" Celsus
D. A terminal film
E. No correct answer
53. In what diseases it is necessary to differentiate the red
lupus:
A. Scleroderma
B.Dermatitis Duhring
C. Photodermatosis*
D. eczema
E. No correct answer
54. What are the stages are distinguished in the clinical course of scleroderma:
A. Hyperkeratosis
B.exudation
C. Ulceration
D. Pack*
E. No correct answer
55. What dermatoses included in the group of connective tissue:
A. Discoid lupus erythematosus*
B. Dermatitis Duhring
C. Genital warts
D. Leyshmanmoz
E. No correct answer
56. What are considered benign neoplasms of the skin:
A. carcinoma
B. Lipoma*
C. melanoma
D. sarcoma
E. No correct answer
57. Favourite localization of molluscum contagiosum:
A. Underarm area
B. Hands and feet
C. Person
D. Mucosas
E. No correct answer
58. What are the clinical varieties of acne vulgaris:
A. Spherical
B. Erythematoussquamous
C. Erosive and ulcerative
D. Pustular*
E. No correct answer
59. Enter the current stage of rosacea:
A. Papulespustular*
B.Follicular hyperkeratosis
C. The stage of atrophy
D. Stage seals
E. No correct answer
60. What are the exogenous factors that may cause abnormal
condition in the skin:
A. Disorders of the nervous system
B. Hypersensitization
C. Pathogenic fungi*
D.Autoimmune
E. No correct answer
61. endogenous physiological factors that may cause
inflammatory changes in the skin include:
A. Viruses
B. Chemical
C. Physical
D. Metabolic disorders*
E. No correct answer
62. The primary morphological elements can be:
A. Proliferative*
B. Induratum
C. Serous
D. Follicular
E. No correct answer
63. Which of the following applies to exudative lesions:
A. roseola
B. papule
C.Tubercle
D. Blister*
E. No correct answer
64. What are the primary proliferative elements:
A. Knot*
B. blister
C. vesicle
D. Bull
E. No correct answer
65. Stain a primary element, resulting from:
A. As a result of hyperkeratosis
B. As a result of accumulation of melanocytes at a certain area of the skin*
C. Changes in skin color due to akantolizisa
D. It changes color as a result of hyperfunction of the sebaceous glands
E. No correct answer
66. What types of spots are distinguished:
A. Papillomatoznye
B. Iron deficiency
C. Melanocytic
D. Pigment*
E. No correct answer
67. What kinds of vascular spots:
A. Hyperpigmentation
B.Leucoderma
C. Telangiectasia*
D. Tattoo
E. No correct answer
68. What kinds of hemorrhagic spots:
A. nevus
B. Petechiae*
C. lentigo
D. Chloasma
E. No correct answer
69. What are the characteristics of a blister:
A. Bespolostnoy element resulting from edema of the papillary layer
Dermis*
B. Is the result of akantolizisa
C. Characteristic of autoimmune diseases
D. Is not accompanied by subjective sensations
E. No correct answer
70. What are the characteristics of a bubble:
A. Formed as a result of edema papillary dermis
B. Is a proliferative element
C. Exudative primary element comprises a liquid*
D. Pitting with ulceration and atrophy
E. No correct answer
71. What is different from the bubble of the bubble:
A. Localization of elements
B. The contents of exudate
C. Subjective sensations
D. Size less than 0.5 cm bubble, the bubble more*
E. No correct answer
72. Name the layer of the epidermis:
A. Horn*
B. Papillary
C. reticulate
D. fatty
E. No correct answer
73. refers to the skin appendages:
A. Dermal papilla
B. epidermis
C. Hair*
D. Subcutaneous fat
E. No correct answer
74. What are the primary morphological elements:
A. atrophy
B. Papule*
C. Follicle
D. ulcer
E. No correct answer
75. What kinds of pustules, you know:
A. Ecchymosis
B. Petechiae
C. Impetigo*
D. Skrofuloderma
E. No correct answer
76. What types of papules vary in size:
A. Lichenoid
B. Dwarf
C. Tuberculoid
D. Miliary*
E. No correct answer
77. What are the characteristics of the nodule:
A. Bespolostnoy primary element*
B. It is ephemeral
C. It contains serous ekksudat
D. The involution leaves atrophic scar
E. No correct answer
78. What are the characteristic features of tubercle and assembly:
A. It refers to the elements of the exudative
B. Infiltrate located in the epidermis
C. Is a members bespolostnoy*
D.Are ephemeral formations
E. No correct answer
79. Write the primary element and the corresponding pathological process:
A. Vial atrophy
B. Tuberkulum secondary depigmentation
C.Node Erosion
D. Bubble – acantholysis*
E. No correct answer
80. Locate the secondary morphological elements:
A. papule
B. Erosion*
C. Conflict
D.Tubercle
E. No correct answer
81. What are the characteristic features of lihenifikatsii:
A. Atrophic changes
B. Rash on the hearth
C.Anesthesia on the hearth
D. Enhanced skin pattern*
E. No correct answer
82. What are the primary elements and their corresponding inverse
the development of secondary elements:
A. Node secondary depigmentation
B. Bubble – Erosion*
C. Papule scar atrophy
D.Spot flaking and crust
E. No correct answer
83. What pyoderma characteristic of childhood:
A. Hydradenitis
B.Basal cell carcinoma
C. Epidemic pemphigus newborns*
D. Genital herpes
E. No correct answer
84. What factors contribute to the development of pyoderma:
A. Diabetes*
B. Violation of the mechanism of division epidermotsitov
C. Akantolizis
D. Hyperactivity of the sweat glands
E. No correct answer
85. What are the main clinical signs of scabies:
A. A symptom of Pospelov
B. Sgruppirovannnye bubbles
C. Paired papular or vesicular papules elements*
D. Bites itch mites
E. No correct answer
86. What tools are used to treat scabies:
A.Solution Dimexidum
B. Method Dem'yanovich*
C. Gray mercurial ointment
D. Kerosene mixed with vegetable oil
E. No correct answer
87. What are the main clinical signs of lice:
A. Finding ticks
B. Paired papular elements
C.A symptom GorchakovHardy
D. Finding lice*
E. No correct answer
88. With what diseases differentiate scabies:
A. Pruritus*
B. shingles
C.Leishmaniasis
D. Exudative erythema
E. No correct answer
89. What are the rash characteristic of the lepromatous type of leprosy:
A. Vesicular elements
B. Leproma*
C. Gummy
D. Vegetating papules
E. No correct answer
90. What are the symptoms are caused by a variety of motor
disorders in leprosy:
A. A symptom of "fish eggs"
B.A symptom of "butterfly"
C. "Mask of St. Anthony"*
D.shatayuschayasya gait
E. No correct answer
91. What laboratory and instrumental methods used to
diagnosis of leprosy:
A. Leprominovaya trial*
B.Mantoux test
C. Reaction Cana
D. Trial Jadassohn
E. No correct answer
92. Which of the provisions are considered to be correct when setting the sample:
A. Type of tuberculosis a negative test
B. Lepromatous positive test
C. Lepromatous type – negative*
D. Undifferentiated type resistant sample
E. No correct answer
93. Enter any skin diseases considered contagious:
A. psoriasis
B. Pink zoster Gibert
C. Dermatitis Duhring
D. Pedikulё*
E. No correct answer
94. Specify links in the epidemiological chain of the disease Borowski:
A. Wand Hansen
B. Ground Squirrel*
C. cattle
D. Ticks
E. No correct answer
95. Specify the reservoir of infection in leishmaniasis:
A. Poultry
B.Mosquitoes and fleas
C. Ground Squirrels*
D. cattle
E. No correct answer
96. List the basic preventive measures when
leishmaniasis:
A. Deratization work in the centers*
B. Disinfection of the premises and linen
C. The destruction of stray dogs
D. Vaccination of cattle
E. No correct answer
97. What laboratory and instrumental methods of diagnosis
are used for the diagnosis of tuberculosis of the skin:
A. Trial Minor
B. Trial Jadassohn
C. Mantoux*
D. Research on the LEcells
E. No correct answer
98. Specify the localized forms of lupus:
A. Papulonecrotic
B. Ulcerative tuberculosis*
C. Indurative
D. Miliary
E. No correct answer
99. What dermatosis known as psoriasis:
A. Allergic dermatitis
B. Toxicoderma
C. Psoriasis*
D.Trihofitii
E. No correct answer
100. Specify where indicated correctly matching the pathogen diagnosis:
A. Leprosy Hansen bacillus*
B. Nevi papillomatoznye viruses
C. Microsporia fungi genus Candida
D. Leishmaniasis leptospira
E. No correct answer
101. Indicate where indicated correctly matching the pathogen diagnosis:
A. Microsporia mycoplasma
B. Pink eels genus Demodex mites*
C.Trihofitii Trichomonas
D. Leprosy Leishmania
E. No correct answer
102. Identifying the factors causing exogenous
histopathological changes in the skin:
A.Immune deficiency
B. Autoimmunnization
C.Status of hypersensitivity
D. Acid*
E. No correct answer
103. Identifying the factors causing endogenous nature
histopathological changes in the skin:
A.Parazitar Mites
B. Dysbacteriosis*
C.Substance Oncogenic action
D.Patologic Mushrooms
E. No correct answer
104. What segments are distinguished in the epidermis:
A. Reticulate
B. Papillary
C. Grainy*
D.Lipid
E. No correct answer
105. As relates to the skin appendages:
A. Melanocytes
B. Keratinocytes
C. The connective tissue of the dermis
D. Nails*
E. No correct answer
106. Specify where the skin of the human body is most located
apocrine sweat glands:
A. The skin of the palms and soles of the skin
B. axilla*
C. On the face
D. The extensor surfaces of the forearms
E. No correct answer
107. What are the physiological functions of the skin are performed:
A. Participation in the synthesis of sex hormones
B.The function of the depot steroids
C. Protective*
D. Enzyme
E. No correct answer
108. Specify morphologic primary elements:
A. Vesicle*
B. Erosion
C.Atrophy
D. Crust
E. No correct answer
109. Specify the morphological elements which do not belong to the primary:
A. Abscess
B. Hemorrhagic spot
C. Tubercle
D. Excoriation*
E. No correct answer
110. Specify the pericardial primary element:
A.Papule
B.Tubercle
C. Vesicle*
D. Lace
E. No correct answer
111. Specify the primary proliferative element:
A.Vesicle
B. Lace*
C. Excoriation
D. Atrophic scar
E. No correct answer
112. What kind of spots are distinguished:
A. PU*
B. Exudative
C. Hypertrophic
D. Dishydrotic
E. No correct answer
113. Which of the following refers to vascular spots:
A. Secondary pigmentation
B. Roseola*
C. Vibitsess
B. Leucoderma
E. No correct answer
114. With what diseases must be differentiated leishmaniasis:
A. Disease Borowski
B. Duhring Dermatitis
C. Warts simple
D. Leprosy*
E. No correct answer
115. What tests should be appointed to confirm the diagnosis
skrofuloderma:
A. Biopsy
B. Lepra trial
C. Mantoux*
D. Sample Minor
E. No correct answer
116. What types of fungal infections are classified according to the classification Arievicha
Sheklakova:
A. Warts
B. Sporamikosys
C. Dermatomycoses*
D. Blastomycosis
E. No correct answer
117. What diseases are at dermatomycoses:
A. Inguinal athlete*
B. psoriasis
C. Lichen planus
D. Pink zoster
E. No correct answer
118. What are the clinical forms rubromikoza distinguished:
A. Rubromicosys oral mucosa
B. Rubromicosys scalp
C. Rubromicosys palms and soles*
D. Zoonotic rubromicosys
E. No correct answer
119. At what diseases may occur as a symptom of alopecia:
A. Rubromikoz
B. Leishmaniasis
C. Psoriasis
D. Microsporia*
E. No correct answer
120. What are the factors leading to activation of Candida infections:
A. Longterm antibiotic therapy*
B. Status of hypersensitivity
C. Autonomic dysfunction
D. Cholesterolemia
E. No correct answer
121. Find what the disease does not correspond to the designated
preparation:
A. Trihofitii griseofulvin
B. Candidiasis
C. Metronizadol*
D. Pemphigus – polkortolon
E. Pediculosis benzylon
122. What are hemorrhagic spots:
A. Leucoderma
B. Erythema
C. Petechiae*
D.Telangiectasia
E. No correct answer
123. What symptoms characteristic of a blister:
A. Ephemeral rashes*
B. Deep pleural primary element
C. Accompanied by a feeling of pain
D. Characteristic of viral diseases
E. E. No correct answer
124. Specify pustules varieties:
A. Folliculitis*
B. Granuloma
C. Comedones
D. Gumma
E. E. No correct answer
125. Specify piznaki characteristic papules:
A. Ephemeral element fugitive
B. Leaving behind scar atrophy
C. Protruding above the skin*
D. Contains purulent serous exudate
E. E. No correct answer
126. Which histopathological changes in the skin may lead to
papule formation:
A. Akantoliz
B. Acanthosis *
C. Disgidroz
D. Anaplaziya
E. E. No correct answer
127. What papules vary in size:
A. Skutulyarnye
B. Miliary *
C. Hypertrophic
D. Dwarf
E. E. No correct answer
128. Which histopathological changes in the skin underlying
A. bubble formation and bubble:
B. Anaplasia
C. Gidrotsitoz
D. Acantholysis*
E. Profileratsiya
129. Choose the signs typical for the site and hillocks:
A. There are secondary syphilis
B. Is characterized by ephemerality
C. Has a soft consistency
D. Bespolostnoy, proliferative elements*
E. E. No correct answer
130. At what diseases occur roseola:
A. Secondary syphilis *
B. Erysipelas
C. Hemorrhagic vasculitis
D. The progressive stage of psoriasis
E. E. No correct answer
131. At what diseases papule occurs:
A. Erythrasma
B. Leishmaniasis
C. Red lichen planus*
D. Hydradenitis
E. E. No correct answer
132. At what diseases common node and bump:
A. Leishmaniasis *
B. Lichen planus
C. Neurodermatitis
D. Dermatitis Duhring
E. E. No correct answer
133. For what diseases clinic characteristic bubble and bubbles:
A. Leishmaniasis
B. Genital herpes*
C. Exudative psoriasis
D. Lichen planus
E. E. No correct answer
134. Specify the correct instructions, where histopathological changes
correspond to the element:
A. Bubble papillomatosis
B. Abscess specific granulomatous inflammation
C. Bump spongiosa
D. Blister intracellular edema of the papillary dermis*
E. E. No correct answer
135. Find morfologicheskie secondary elements:
A. Abscess
B. A bottle
C. Erosion *
D. Node
E. E. No correct answer
136. For any clinic diseases characterized by excoriations:
A. Pink zoster Gibert
B. Scabies*
C. Colorful lichen
D. Leishmaniasis
E. E. No correct answer
137. Specify the secondary morphological elements:
A. Bubble
B. Abscess
C. Node
D. Ulcer *
E. E. No correct answer
138. What types of crusts are distinguished:
A. Purulent *
B. Atrophic
C. Scar
D. Scale
E. No correct answer
139. Specify the characteristic signs of the hearth lihenifikatsii:
A. Atrophic scars
B. Infiltration and roughness of the skin on the hearth*
C. Easy to scrape off the silverwhite scales
D. Secondary vegetation on the hearth
E. No correct answer
140. Specify signs of vegetation:
A. Localized in the extensor surface of the forearm
B. Solid of consistency of The Center has umbilicated
C. Soft texture, reminiscent of cauliflower*
D. surface and hypo giperpigmentnyh spots
E. No correct answer
141. Find the right indicate where the primary element matches
Secondary:
A. Bubble scar atrophy
B. Hemorrhagic spot hemorrhagic crusts
C. Nodule ulceration with the outcome of atrophy
D. A bottle – Erosion*
E. No correct answer
142. Specify the primary and corresponding secondary element:
A. Blister vegetation
B. Pustules erosion and ulcerative defect*
C. A bottle atrophy
D. Lace scar atrophy
E. No correct answer
143. pathogens which cause purulent inflammation of the skin (piodermity):
A. Stafilllococci*
B. Spirochetes
C. Herpes viruses
D. Sticks Hansen
E. No correct answer
144. Specify pustules varieties:
A. Comedones
B. Leproma
C. Ecchymosis
D. Impetigo*
E. No correct answer
145. Find the incorrect information:
A. Acne purulent inflammation of the sebaceous glands
B. Ecthyma superficial pustule*
C. Furunkul Inflammation of the hair follicle and surrounding tissue
D. Rupee deep pustule
E. No correct answer
146. Choose the signs stafillokokkovogo abscess:
A. Abscess with sluggish tire
B. The fluid abscess serous
C. Relationships with hair follicles*
D. It occurs mostly in women and children
E. No correct answer
147. Specify which piodermity characteristic of childhood:
A. Sycosis
B. Hydradenitis
C. Acne vulgaris
D. Pseudofuruncullosys Finger*
E. No correct answer
148. Specify the types of deep abscesses:
A. Furuncle*
B. Ostiofollicullit
C. Conflict
D. Impetigo
E. No correct answer
149. Specify superficial abscesses:
A. Furuncle
B. Ecthyma
C. Conflict*
D. Hydradenitis
E. No correct answer
150. Specify streptococcal species:
A. Dishydrotic form
B. Zayed*
C. Discoid Form
D. hyperceratotic form
E. No correct answer
151. Specify the characteristic signs of sycosis:
A. The causative agents are Streptococcus
B. In general ill children
C. Sprinkler elements are phlyctenas
D. It occurs mainly in men*
E. No correct answer
152. With what diseases it is necessary to differentiate the boil:
A. Pemphigus
B. Hives
C. Carbuncle*
D. Syphilitic papules
E. No correct answer
153. What are the signs correspond to the clinic pseudofuruculosys:
A. The causative agents are staphylococci
B. The causative agents are Streptococcus
C. It occurs mostly in women
D. There is a purulent inflammation of the sweat glands eksocrine
E. No correct answer
154. What is the characteristic signs of scabies mites for biology:
A. In an environment viable 30 days
B. Propagate by division and sporulation
C. Parasites in the dermis and hypodermis
D. Multiply by laying eggs*
E. No correct answer
155. In what ways can be infected with scabies:
A. Transmissible by
B. Direct contact*
C. Alimentar
D. Airborne
E. No correct answer
156. Where lesions are localized mainly in scabies:
A. The skin of the scalp
B. On the skin of the soles
C. On the face
D. On the skin interdigital folds*
E. No correct answer
157. Specify the characteristic clinical signs of scabies:
A. Itching amplified at night*
B. The rash consisting of small blisters pair
C. Paroxysmal itching associated with nervous stress
D. Finding nits and lice
E. No correct answer
158. Specify the medicines used to treat
Scabies:
A. Mixture solution of kerosene or vegetable oil
B. 33% sulfuric ointment
C. Liquid Castellani*
D. Net sulfuric ointment
E. No correct answer
159. What distinguishes types of lice agents pediculosis
A. Intertriginosys
B. Hair
C. Head*
D. Axillary
E. No correct answer
160. Specify the clinical signs of pediculosis
A. Finding nits*
B. Finding ticks
C. Itching worsens at night
D. Symptom Gorchakov Hardy
E. E. No correct answer
161. What purpose are recommended for the treatment of lice:
A. A solution of chlorhexidine
B. A solution of formaldehyde
C. Liquid Castellani
D. vinegar hair combing and fine scallops*
E. No correct answer
162. Specify the complication of scabies:
A. State toxicoderma
B. Pyoderma*
C. Skin atrophy
D. Keloids
E. No correct answer
163. What medications are used to treat scabies in children:
A. 20% sulfuric ointment
B. Simple sulfuric ointment
C. Zinc paste
D. White mercury ointment*
E. No correct answer
164. Specify the current stage of psoriasis:
A. Recurrent
B. Latent
C. Progressive*
D. Infiltrative
E. No correct answer
165. Which of the following are contagious dermatosis:
A. Psoriasis
B. Microbial eczema
C. Lupus erythematosus
D. Streptoderma*
E. No correct answer
166. Specify the characteristic localization of lesions in scabies:
A. The skin of the scalp
B. Extensor surfaces of the forearms*
C. On the skin surface of the forearm bending
D. At the oral mucosa
E. No correct answer
167. What are the clinical forms of psoriasis are distinguished:
A. Remittent
B. Infiltrativesuppurative
C. Papillomatosys
D. Arthropathic*
E. No correct answer
168. Specify the characteristic features of psoriatic plaques:
A. Silver white scales on the surface of papules*
B. If there is soreness papules
C. umblicuous impression in the center of papules
D. Papules conical shape without the tendency to peripheral growth
E. No correct answer
169. What are the symptoms of the triad is Auspittsa:
A. "Hidden peeling"
B. "Grid Wickham"
C. "Stearin spot"*
D. Symptom "thimble"
E. No correct answer
170. Specify the pathognomonic symptoms of psoriasis:
A. Grid Wickham
B. Triad Auspittsa*
C. Symptom Jadassohn
D. Symptom "falling through the probe"
E. No correct answer
171. What symptoms characteristic of the progressive stage of psoriasis:
A. Secondary hyper and hypopigment spots
B. Positive symptom GorchakovHardy
C. Pseudoatrophic rim around papules
D. Red halo around papules without peeling *
E. E. No correct answer
172. By the simple dermatitis do not lead:
A. Oral medication*
B. Friction pressure
C. Action rays
D. High and low temperature
E. No correct answer
173. Genital warts are transmitted:
A. Blood transfusion
B. After kissing
C. Sexual transmission*
D. Airborne droplets
E. No correct answer
174. Symptom negative Nikolsky
A. Vegetating pemphigus
B. Limited neurodermatitis*
C. True pemphigus
D. Pemphigus foliaceous
E. No correct answer
175. What drugs can cause skin reaction fototoksicheskuyu:
A. Nizoral
B. Delagil
C. Tetracycline
D. Puvalen*
E. No correct answer
176. Which drugs contribute to the emergence of fixed erythema
A. Sulfadimetoksin*
B. Kanamycin
C. Penicillin
D. Tetracycline
E. No correct answer
177. Local treatment of a simple contact dermatitis
A. Cryosurgery
B. PUVA therapy
C. Ointments*
D. Laser
E. No correct answer
178. What is included in the professional dermatosis of chemical factors
A. Lichen planus
B. Contact dermatitis
C. Psoriasis
D. Neurodermatitis*
E. No correct answer
179. Specify the clinical varieties of psoriasis
A. Tuberculoid
B. Glandular
C. Arthropathic*
D. System
E. No correct answer
180. The diagnosis of leprosy helps
A. RIT, RIF
B. Research on the tubercle bacillus
C. IFA
D. Scraping of leprosy *
E. No correct answer
181. specific morphological element in leprosy is
A. Bumps*
B. Polygonal papules
C. Grouped vesicles
D. Urticaria
E. No correct answer
182. In some material from the patient often obaruzhivayutsya mycobacteria
A. Leprosy
B. Secret sweat glands
C. B scrapings from leprosy*
D. In urine
E. Semen
183. What are the rash characteristic of the artificial dermatitis
A. Bugorkova
B. Vesicular
C. Erythema with sharp edges
D. Urticaria*
E. No correct answer
184. With what diseases should be differentiated leishmaniasis skin
A. Colorful lichen
B. Fixed erythema
C. Hives
D. Leprosy*
E. No correct answer
185. In the case of evidence of the patient the nature of trichomonas urethritis
A. Treatment sldeuet conduct the following medications
B. Tetracycline
C. Nystatin
D. Plus metronidazole*
E. Acyclovir
186. When gonorrheal urethritis effective drugs listed groups
A. Besides
B. Desensitizing*
C. Tetracycline
D. Macrolides
E. Cephalosporins
187. Indicate preparations which can be used in the treatment of
A. a patient suffering from Trichomonas urethritis
B. Trichopolum*
C. Doxycycline
D. Nystatin
E. Suprastin
188. pathogens infections, sexually transmitted diseases are
A. Wand Hansen
B. Treponema pallidum *
C. Bacillus
D. Rickettsia
E. E. No correct answer
189. Drugs used for individual STI
A. Potassium Premanganat
B. Potassium Premanganat
C. Miramistin*
D. Potassium Premanganat
E. E. No correct answer
190. What antibiotics to applicable treponema
Diflucan
A. Penicillin *
B. Orungal
C. Nystatin
D. No correct answer
191. Absolute symptoms of late congenital syphilis
A. Symptom fish eggs
B. Symptom rosary
C. Getchin teeth*
D. Symptom apple jelly
E. No correct answer
192. What rash characteristic of atopic dermatitis:
A. Erythema *
B. Ulcerative elements
C. Vegetation
D. Bumps
E. No correct answer
193. Lyell's syndrome is characterized by:
A. Hereditary diseases
B. Positive iodine sample Balzer
C. Only the defeat of the mucous membranes
D. Positive sign of Nikolsky*
E. No correct answer
194. What forms of eczema are distinguished according to the classification:
A. Viral
B. Microbial*
C. Arthropathic
D. Intertriginosys
E. No correct answer
195. What kinds of eczema are distinguished according to the classification:
A. Urticaria
B. Infiltrative
C. True*
D. Squamous
E. No correct answer
196. What distinguishes the stage for eczema:
A. weeping*
B. Bullous
C. Infiltrative
D. Allergic
E. No correct answer
197. What are the characteristics of microbial eczema clinic:
A. Centers of the fuzzy border
B. Soak is not observed
C. Monomorphic rash consisting of pustules
D. Developed on the site of the centers of pyoderma or venous ulcers*
E. No correct answer
198. Which solutions can be used for the topical treatment lotions
Eczema:
A. salicylic alcohol
B. weak solution of potassium permanganate*
C. Hypertonic solution
D. 1% iodine solution
E. No correct answer
199. Pri late congenital syphilis may have the following
A. dystrophic changes
B. Wide warts
C. simptom ladies heel
D. sign Avsitidi*
E. triada psoriatic
200. Signs of late congenital syphilis
A. Absolute*
B. Stigmy
C. Wide Warts
D. Vegetatsii
E. No correct answer
201. for syphiloma characteristic
A. test consistency
B. stellate scar*
C. kaseosys necrosis
D. stamped scars
E. No correct answer
202. sing of early congenital syphilis in infants are
A. Pemphigus *
B. Chancre
C. tuberculi
D. Gummy
E. No correct answer
203. for the manifestation of tertiary syphilis is characterized by:
A. The tendency to decay symptoms with extensive destruction*
B. Limitation Defeats
C. Only at persons of the same sex
D. Only children
E. No correct answer
204. The special features of the manifestations of tertiary syphilis include
A. From certain professions
B. An insignificant infectiousness*
C. Only in men
D. The defining large number treponem
E. E. No correct answer
205. Evaluation of patients with tertiary syphilis must include
A. sureon
B. ophthalmolog
C. Pediatrician
D. Therapist*
E. No correct answer
206. The factors preventing syphilis include:
A. Overalls
B. Use of condoms*
C. Application Antihistamines
D. injection
E. No correct answer
207. syphilitic tubercles characteristic:
A. Pink Yellow
B. Bluishred color*
C. eliminated Completely
D. Test Consistency
E. No correct answer
208. The clinical picture of secondary syphilis may occur
A. Papuly
B. Leykoderma
C. Bumps*
D. Rozeoly
E. No correct answer
209. The special features of tertiary syphilis should include
A. prone To fix lesions at the site of injury
B. Low tension immunity*
C. In Men only
D. Hurt Only a certain age
E. No correct answer
210. The late forms of syphilis include
A.Seropozitivny Latent syphilis
B.Vtorichny Recurrent sifilis
C.Vtorichny Fresh syphilis
D.Neurosyphilis *
E. No correct answer
211. Tertiary siiflis may appear:
A. Gum*
B. Stamped Scar
C. Leykodermo
D. All Of the above is true
E. E. No correct answer
212. syphilitic alopecia secondary syphilis has the following
varieties:
A. Seborea
B. Pseudopelada Brock
C. As stumps
D. Mixed*
E. E. No correct answer
213. pustular eruptions with secondary syphilis have the following
varieties:
A. Sifilitic Gunma
B. Sifilitic Bumps
C. Eel*
D. Furuncle
E. No correct answer
214. papular eruption in secondary syphilis are as follows
varieties
A. Nummular*
B. Wide Warts
C. Acute end Warts
D. In the form of vegetations
E. No correct answer
215. syphilides secondary syphilis have the following characteristics
A. Syphilides quickly dissolve under the influence of treatment antisyphyllitic*
B. Feature of Burning sensation
C. Only Monomorphic
D. Having Duty plaques
E. No correct answer
216. secondary fresh syphilis is characterized by:
A. Eritematosys Angina, extensive warts
B. Leykoderma, Extensive warts folds
C. Erythematous angina, remnants of the chancre*
D. groupede Papules
E. No correct answer
217. secondary recurrent syphilis is characterized by:
A. Eritematosys Angina, remnants of the chancre
B. Leucoderma, extensive wartsfolds*
C. Eritematosys Angina, roseola
D. complicated Located, multiple roseola
E. No correct answer
218. Specify signs of secondary syphilis
A. Furuncle
B. Leucoderma*
C. Itch moves
D. Tuberculi
E. E. No correct answer
219. The differential diagnosis is carried out with a syphilitic roseola
A. Seboreey
B. Piedra
C. Toxicoderma*
D. Leprosy
E. No correct answer
220. Infectious lesions in secondary syphilis
A. Leykoderma
B. Rozeola Trunk
C. Baldness
D. Extensive warts around the anus*
E. no correct answer
221.condary recurrent syphilis is characterized by:
A. Piedra
B. Leucoderma*
C. Pink Zoster
D. Herpes
E. No correct answer
222. Signs indicating the possible presence of secondary
recurrent syphilis
A. Hair Break off in the form of stumps
B. Psoriasis
C. Red Planus
D. Extensive warts*
E. No correct answer
223. For erythematous syphilitic angina is characterized by:
A. Availability Purulent plugs
B. Harsh border*
C. Nonaccurate Border
D. Acute illness
E. E. No correct answer
224. What is characteristic for secondary syphilides
A. illness
B. Assimetric Rash
C. Considerable acuteinflammatory reaction
D. No acute inflammatory elements*
E. No correct answer
225. The differential diagnosis is carried pustular syphilis
A. Vulgar impetigo*
B. Vulgarny Psoriasis
C. Furuncle
D. Carbuncle
E. No correct answer
226. Diseases, which is not carried out differential diagnosis
A. pusutulosus syphilis
B. Vulgar Impetigo
C. Vulgar Ecthyma
D. Psoriasis vulgaris*
E. Vulgar Acne
227.uptions on the skin in patients with secondary syphilis:
A. Pigmentous
B. Nodulus
C. Vesiculous
D. Roseolous*
E. No correct answer
228. Specify the disease, which differentiate papular
syphilides
A. Psoriasis*
B. Pink Zoster
C. Piedra Zoster
D. Acute end Warts
E. No correct answer
229. In the primary period of syphilis patient may occur
A. following signs
B. Wide Warts
C. Solid chancre*
D. Papuly
E. Pustuly
230. What are the clinical symptoms in patients with primary
syphilis
A. Shankr Felon
B. Roseolous rash*
C. Shankr Amygdalite
D. regionar adenitis
E. No correct answer
231. For a typical characteristic chancre
A. Brilliant lacquered bottom*
B. Polycyclic Outlines
C. An ample purulent discharge
D. An ample foamy discharge
E. No correct answer
232. erosive chancre should first
to differentiate
A. vesiculous
B. Squamous cell carcinoma*
C. Nifiltrative Suppurative trichophytosis
D. Dermatozom Dühring
E. No correct answer
233. For noncoplication chancre is typical:
A. Purulent discharge
B. infiltration at the base*
C. bubble Discharge
D. bubbleduor
E. E. No correct answer
234. What are the clinical symptoms in patients with primary
syphilis
A. Plotnoelastichesky Infiltration at the base
B. Papulae palms and soles*
C. Podrytye Edge
D. Blestyaschee Lacquered bottom
E. No correct answer
235. In primary syphilis is characterized by regional lymphadenitis
clinical signs
A. Limfatic Components soldered
B. Dermahemia
C. Suppuration of lymph nodes
D. The skin over the lymph nodes is not changed*
E. No correct answer
236. For a typical chancre is not typical
A. Is a regular round or oval outline
B. Smooth brilliant bottom
C. Morbidity *
D. Edge
E. No correct answer
237. Specify the typical localization of lesions in the pink lichen
A. Trunk*
B. Serous Shell
C. Palm And soles
D. Genitals
E. No correct answer
238. In the treatment of 240 total alopecia use
A. corticosteroid
B. cytostatici
C. Vitamins*
D. Antihistamin
E. No correct answer
239. What local resources are used in the treatment of acne
A. Kuriozin*
B. iodine
C. brillianto greens
D. ointment
E. E. No correct answer
240. Specify the severe forms of acne vulgaris
A. Pink Acne
B. Papulosys Acne
C. Abscessed acne*
D. Bromid Acne
E. No correct answer
241. Specify the complications of acne vulgaris
A. Toxicodermia
B. Persistent Pigmentation*
C. Mikrobus Eczema
D. Gipertrihoz
E. No correct answer
242. Select the morphological features characteristic of the vulgar
A. acne Teleangiektazii
B. Diskoidnye Spots
C. Erythema
D. Follicular pustules*
E. No correct answer
243. Which of the following drugs can cause
acne medication
A. Preparaty Imidazole group
B. Iodine*
C. Sulfanilamidy
D. Antibiotiki
E. No correct answer
244. Specify the typical localization of seborrhea
A. Buttocks
B. Genitals
C. The upper third of the back and chest*
D. Slizistaya Mouth
E. No correct answer
245. Describe the main pathogenetic links of acne vulgaris
A. Pischevaya Allergies
B. Oily seborrhea*
C. Glistnaya Invasion of exposure to high and low temperatures
D. Mikrobnaya Eczema
E. No correct answer
246. in the treatment of acne vulgaris using all physiotherapy
procedure, except
A. Mechanical cleaning*
B. PUVA Therapy
C. UF Exposure
D. Kriomassazh Snow Coal kislorty
E. No correct answer
247. in the pathogenesis of acne vulgaris are essential all
A. factors other than contamination Mikrobnoy
B. Changes The amount of sebum
C. Changes The quality of sebum
D. Gain keratinization of the follicular channel*
E. No correct answer
248. Specify the clinical signs of acne vulgaris
A. Evolyutsionny Polymorphism rash
B. Flikteny
C. The presence of comedones*
D. Teleangiektazii
E. No correct answer
249. Specify the clinical signs of acne vulgaris
A. Evolyutsionny Polymorphism rash
B. Flikteny
C. The presence of comedones*
D. Teleangiektazii
E. No correct answer
250. Kakie drugs often cause erythema fixed
A. sulfadimetoksin *
B. analginum
C. aspirin
D. tavegil
E. No correct answer
251. S how widespread the disease should be differentiated
A. toxicoderma *
B. herpes
C. pyoderma
D. pemphigus vulgaris a
E. disease Borowski
252. Choose the typical localization at multiforme exudative
A. erythema scalp
B. Rear feet*
C. The neck interdigital spaces of hands
D. No correct answer
E. pemphigus vulgaris a
253. What factors are important in the development multiforme
A. exudative erythema
B. Hereditary
C. Photosensitivity
D. Foci of chronic infection*
E. No correct answer
254. What dermatoses differentiate erythema
A. multiforme *
B. Lyell's syndrome
C. strofulyus
D. Scrapie Psoriasis
E. No correct answer
255. Specify the pathogenetic factors of pink lichen Gibert
A.The introduction of ticks into
B.the skin yeasts
C.idiosyncrasy to iodine
D.viral infection
E. no correct answer
256. Specify the species of microbial eczema
A.Adults*
B. Paratravmatic
C. idiopathic horn
D.Scrapie Psoriasis
E. No correct answer
257. What clinical sign is characterized by a rash in true
A. eczema
B. Symptom <serous wells*
C. Tubercles asymmetry
D. Induratum edema
E. No correct answer
258. What are the symptoms seen with discoid lupus
A. Net Wickham
B. Collar Biett
C. Sign "ladies' heel"*
D. Paired elements
E. No correct answer
259. Choose varieties of scleroderma
A. Vulgar
B. White spot disease*
C. Infiltrative
D. Intertriginosous
E. No correct answer
260. Lichen planus is characterized by
A. Absence of pruritus
B. Tubercles
C. Mesh Wickham*
D. Urticaria rashes
E. No correct answer
261. For planus characterized by
A. Swelling of the spinous layer of intercellular
B. Akantolizis
C. Vacuolar degeneration
D. Irregular thickening of the granular layer of the epidermis*
E. No correct answer
262. What clinical symptom pathognomonic for red
A. lichen planus "honeycomb Celsus*
B. Net Wickham
C. Cells Ttsanka
D. A symptom auspices
E. No correct answer
263. What diseases do not belong to a group of viral dermatoses
A. Warts
B. Genital warts
C. Psoriasis*
D. Shingles
E. No correct answer
264. The vascular patch is
A. Erythema *
B. Chloasma
C. Nevus
D. Leucoderma
E. No correct answer
265. Primary morphological elements
A. scar
B. Bubble*
C. erosion
D. ulcer
E. No correct answer
266. Secondary morphological elements
A. Roseola
B. Lace
C. Node
D. Ulcer *
E. No correct answer
267. The bubble is formed at
A. Urticaria
B. CPL
C. Herpes simplex*
D. Psoriasis
E. No correct answer
268. The outcome of the node is
A. Spot*
B. Atrophy
C. Ulcer
D. Scar
E. No correct answer
269. exudative morphological element is
A. Node
B. Tubercle
C. Vesicle*
D. Papule
E. No correct answer
270. Indicate dosage forms for topical treatment in accordance with step
Eczema:
A. Weeping – gadgets*
B. The stage of secondary pyoderma gadgets
C. Bubble stage ointments
D. Exudation powders
E. No correct answer
271. What diseases are at neyrodermatozov:
A. Pink zoster Gibert
B. Dermatitis Duhring
C. Lichen planus
D. Scrapie*
E. No correct answer
272. Specify the reasons that may be the cause of localized
itch:
A. Itch mites
B. Violation of the endocrine glands
C. Hemorrhoids*
D. Ketoacidosis
E. No correct answer
273. What preparations have antipruritic effect:
A. Anesthetics*
B. Sulfonamides
C. Antibiotics
D. Cytostatics
E. No correct answer
274. What comorbidities are common in
atopic dermatitis:
A. hepatitis
B. Toxoplasmosis
C. tuberculosis
D. Vasomotor rhinitis*
E. No correct answer
275. At what diseases there is white dermographism:
A. scabies
B. Allergic dermatitis
C. Scrapie*
D. Psoriasis
E. No correct answer
276. Describe the main pathogenetic mechanisms of development of children
scrapie:
A. Hereditary fermentopathy*
B. Center focal infection
C. Endocrine disorders
D. Immune deficiency
E. No correct answer
277. What are the rash characteristic of psoriasis:
A. Vegetating
B. Erosive and ulcerative
C. Monomorphic pustular
D. Papular*
E. No correct answer
278. What areas are distinguished in focal neurodermatitis:
A. Peripheral zone pustular rash
B. Center Zone lichenification*
C. Central erythematoussquamous
D. Average a zone of depigmentation
E. No correct answer
279. What are the signs characteristic of urticaria:
A. Secondary atrophic changes in the skin
B. Monomorphic papules
C. Monomorphic blisters*
D. White dermographism
E. No correct answer
280. patients with acute angioedema starting asphyxia. What preparations
Rata be appointed to help:
A. Korglikon 0.5 / m
B. Gemodez 200.0 drip into a vein
C. Bronholitin
D. Prednisone 6080 mg into a vein*
E. No correct answer
281. What diseases are at bullous dermatosis:
A. Stafilodermiya
B. Pemphigus vulgaris*
C. shingles
D. chickenpox
E. No correct answer
282. Under what bullous dermatosis observed eruptions:
A. herpes
B. Stafilodermiya
C. Lyell's syndrome*
D. Rosacea
E. No correct answer
283. The classification of true pemphigus include:
A. Pemphigus vulgaris*
B. Exudative
C. Handfoot
D. No correct answer
E. Dyhydrotic
284. What are the symptoms characteristic of true pemphigus:
A. Symptom GorchakovHardy
B. A symptom of "apple jelly"
C. Symptom Nikolsky*
D. Symptom Pospelov
E. No correct answer
285. At what diseases there is a positive sign
Nikolsky:
A. Contact dermatitis bullous stage
B. Pemphigus vulgaris*
C. Dermatitis Duhring
D. Bullous streptoderma
E. No correct answer
286. What are the main rules of corticosteroids in the treatment of hormone
pemphigus:
A. Hormones are prescribed only for external treatment
B. Treatment is initiated with a bolus*
C. Treatment start with a gradual increase in the dose until the shock
D. Taking hormones administered in the evening
E. No correct answer
287. What are the complications observed in the treatment of pemphigus:
A. The development of hypersensitivity
B. Oppression of the hematopoietic system
C. cirrhosis of the liver
D. Cushing's syndrome*
E. No correct answer
288. What are the typical clinical signs of dermatitis Dühring:
A. The defeat of the oral mucosa
B. A positive sign of Nikolsky
C. Bunching rash*
D. Infiltrative lesions are composed of elements
E. No correct answer
289. What are the morphological features are characteristic rash
dermatitis Dühring:
A. Vegetation
B. Units
C. Bumps
D. Papular*
E. No correct answer
290. What are the laboratory findings are characteristic of dermatitis Dühring:
A. Eosinophils in the cystic fluid*
B. Lymphocytosis
C. Leukotcytosys cystic fluid
D. Histomorphological acantholysis
E. No correct answer
291. Which dermatosis characterized by the appearance of rashes on
oral cavity:
A. psoriasis
B. Candidiasis*
C. Pink zoster Gibert
D. Hydradenitis
E. No correct answer
292. For what dermatoses characterized by defeat and nail polish
rollers
A. Mycosis
B. Skrofuloderma
C. Pink zoster Gibert
D. Rubromikosys*
E. No correct answer
293. A sign of some dermatoses may be desquamation, hyperkeratosis,
A. cracks in the skin of the palms and fingertips
B. Tuberkuloid Form of leprosy
C. Rubromikoz*
D. Sindrom Lyell
E. StevensZhonsona
294. What stage are distinguished within rosacea (rosacea)
A. Infiltrativesuppurative
B. Eritematosquamous
C. Atrophic
D. Erythematous*
E. No correct answer
295. What are the clinical signs characteristic of rosacea at
an objective examination
A. ochagi lichenification
B. Telangiectasia*
C. Phlyctenas
D. .gipo and giperpigment spots
E. No correct answer
296. Specify the causal factor in the pathogenesis of rosacea
A. Fungal infection
B. Demodex mite parasitism*
C. Renal failure
D. defitsit iodine
E. No correct answer
297. What diseases are characterized by localized skin rash
A. person
B. Rosacea *
C. Lichen planus
D. Multicolored shingles
E. Hydradenitis
298. Specify the clinical signs of transformation in the pigment nevus
melanoma
A. appearance roseolous common rash
B. Accelerating exophytic growth chamber after mechanical trauma insolation*
C. gipo and anesthesia on the hearth
D. keloidnoe scarring on the hearth
E. No correct answer
299. Specify the correct answers, which corresponds to a survey diagnosis
A. blood eosinophilia
B. Eczema seeding tank with focus
C. Scabies a survey on ticks Demodiks
D. Multicolored lichen Trial Balzer*
E. No correct answer
300. Specify the correct answer, which corresponds to the designated drug
A. diagnosis
B. ekzema sulfuric ointment
C. Photodermatitis photosensitizing drugs
D. Leprosy sulfone drugs*
F. No correct answer
301. If any of the disease in the skin can be observed atrophic
changes and scarring
A. vitiligo
B. lupus erythematosus*
C. dermatit Dühring
D. athlete
E. No correct answer
302. for the diagnosis of disease which you
requires biopsy method
A. eczema
B. shingles
C. Scleroderma
D. Infiltrativesuppurative trichophytosis*
E. No correct answer
303. Specify the diseases where tests are assigned to fungi
A. Microsporia*
B. Mycosis fungoides
C. shingles
D. Psoriasis psoriasis
E. No correct answer
304. Specify the correct answers which correspond to the route of infection
diagnosis
A. Transfusion shingles
B. Transmissible – leishmaniasis*
C. Airborne warty tuberculosis of the skin
D. Transmissible trichophytosis
E. No correct answer
305. Specify the focal forms of skin tuberkuleza
A. Lichenoid tuberculosis skin
B. Induratum erythema
C. Miliary tuberculosis
D. Lupus*
E. No correct answer
306. Specify disseminirovannnye TB skin
A. Vulgar lupus
B. Miliary tuberculosis*
C. Infiltrative tuberculosis
D. Skrofuloderma
E. No correct answer
307. What are the signs characteristic of cutaneous leishmaniasis
A. Finding the virus cells to the lesion
B. A symptom of "apple jelly"
C. Transmissible transmission*
D. Reservoir of infection is cattle
E. No correct answer
308. In what dermatoses appointed by the local corticosteroid ointment
A. Atopic dermatitis*
B. Leishmaniasis
C. Colorful lichen
D. Shingles
E. No correct answer
309. For what dermatoses uncharacteristically strong subjective feeling
itching
A. Duhring Dermatitis
B. Microbial eczema
C. Hives
D. Shingles*
E. No correct answer
310. Find the mismatch harkter rash diagnosis
A. Typhoid fever roseolous
B. Colorful lichen roseolouspapular*
C. Strofulyus papules, urticaria, vesicular
D. Duhring Dermatitis polymorphic herpeticum
E. No correct answer
311. Find the appropriate diagnosis the nature of the rash
A. Microbial eczema monomorphic pustular
B. Systemic lupus erythematosus roseolous squamous skin atrophy
C. Skrofuloderma – Bugorkova*
D. Leishmaniasis skin papular patchy
E. No correct answer
312. For what dermatoses characterized by defeat and nail polish
A. rollers
B. Skrofuloderma
C. Athlete *
D. Pink zoster Gibert
E. Duhring Dermatitis
313. A sign of some dermatoses may be desquamation, hyperkeratosis,
cracks in the skin of the palms and fingertips
A. Lyell Syndrome
B. StevensZhonsona
C. Disseminated form of lupus erythematosus
D. Ichthyosis *
E. No correct answer
314. What are the most common opportunistic infections in AIDS
A. Leishmania
B. Korinobakterii
C. Pneumocystis infection*
D. Trichomonas
E. No correct answer
315.at forms of Kaposi's sarcoma you know
A. latespinning
B. AIDSrelated*
C. Undifferentiated
D. Paratravmatic
E. No correct answer
316. form on the prevalence of AIDS symptoms in the clinical
A. picture (according to WHO)
B. Pulmonary *
C. Gepatolienal
D. Gangrenous
E. Lymphoproliferative
317. Specify the ratio of Thelper / Tsuppressor in carriers of HIV
A. Within 10*
B. Within the 15
C. <1
D. More than 10
E. No correct answer
318. What laboratory methods allow to confirm the diagnosis
A. Vulgar lupus
B. Smears
C. Mantoux test*
D. Sample Jadassohn
E. Reaction of Wassermann
319. What diseases are at bullous dermatosis
A. Pemphigus *
B. chickenpox
C. shingles
D. Streptoderma
E. No correct answer
320. In what bullous dermatosis observed eruptions
A. eczema
B. Lichen planus
C. Streptoderma
D. Lyell's syndrome*
E. No correct answer
321. The classification of true pemphigus included
A. Handfoot
B. Pemphigus vulgaris*
C. Dyshydrotic
D. Pustular
E. No correct answer
322. What are the symptoms characteristic of true pemphigus
A. Nikolsky *
B. Pospelov
C. "Apple jelly"
D. GorchakovHardy
E. No correct answer
323. In what diseases there is a positive sign
A. Nikolsky
B. Pemphigus *
C. Dermatitis Duhring
D. Bullous streptoderma
E. Nodules doilschits (false cow ospa)
324. What are the main rules of the COP in the treatment of hormone
pemphigus
A. Use pulse therapy
B. Taking hormones administered on the evening
C. Treatment is initiated with a bolus*
D. Hormones are prescribed only for external treatment
E. No correct answer
325. What are the complications observed in the treatment of pemphigus
A. Oppression of the hematopoietic system
B. Cushingoid syndrome*
C. amyloidosis
D. Paralysis
E. No correct answer
326. What are the characteristic clinical signs of dermatitis Duhring
A. Infiltrative lesions are composed of elements
B. A positive sign of Nikolsky
C. Hypersensitivity to cereals*
D. The defeat of the oral mucosa
E. No correct answer
327. What are the morphological features are characteristic rash
A. dermatitis Duhring
B. Vesicular*
C. Bumps
D. Units
E. No correct answer
328. What are the laboratory findings are characteristic of dermatitis Duhring
A. Lymphocytosis
B. Leykotsitozv cystic fluid
C. Eosinophils in the blood*
D. Histomorphological acantholysis
E. No correct answer
329. Which dermatosis characterized by the appearance of rashes on
A. oral mucosa
B. psoriasis
C. shingles
D. Pemphigus *
E. Hydradenitis
330. Call rash with a clear viral etiology
A. Dermatitis Duhring
B. Lichen planus
C. Pemphigoid
D. Herpes*
E. No correct answer
331. What are the characteristics of lesions in the herpes simplex
A. Bubble elements of monomorphic*
B. Subjectively itching prevails over the soreness and burning
C. Grouped cystic lesions
D. Polymorphism rash
E. No correct answer
332. Call features of genital herpes from a solid
chancre
A. Painless erosions
B. Soreness*
C. Regional bubo
D. The presence of a solid infiltrate the base erosion
E. No correct answer
333. What are the clinical forms of herpes zoster
A. Vegetating
B. Abortive*
C. Intercostal
D. Infiltrativesuppurative
E. No correct answer
334. Specify the drug has prtivovirusnym
A. action
B. Sulphur cleared
C. FIBS
D. Rifampicin
E. Acyclovir *
335. Assign the drug to patients with a diagnosis of herpes zoster
A. Viroleks pills*
B. Ampicillin
C. Aevitum
D. Sodium bromo drops
E. No correct answer
336. Specify the characteristics of molluscum contagiosum
A. The scales on the surface of papules
B. A symptom of "net Wickham" positive
C. Meets especially in children*
D. Transmissiontransmissive
E. No correct answer
337. What forms of lupus are distinguished according to the classification
A. Pustular
B. Discoid*
C. Annular
D. Atrophic
E. No correct answer
338. cardinal symptoms of lupus rash
A. Pustular infiltration
B. Follicular hyperkeratosis*
C. Keloid scars
D. Lichenification
E. No correct answer
339. What are the symptoms observed in discoid lupus erythematosus
A. "Grid" Wickham
B. "Honeycomb" Celsus
C. A terminal film
D. "Ladies' heel"*
E. No correct answer
340. What are the laboratory findings are characteristic of SLE (acute form)
A. "LE" – cells*
B. Positive PB
C. Bilirubinemia
D. Ketoacidosis
E. No correct answer
341. With what diseases should be differentiate red
A. erythematosus
B. eczema
C. Tuberculous lupus*
D. pemphigus
E. Molluscum Contagiosum
342. What are the stages are distinguished in the clinical course of scleroderma
A. exudation
B. Hyperkeratosis
C. Spinning
D. Atrophy*
E. No correct answer
343. What dermatitis are at kollagenozov
A. Dermatitis Duhring
B. Genital warts
C. Sklerdermiya*
D. Leishmaniasis
E. No correct answer
344. What are considered benign neoplasms of the skin
A. melanoma
B. Basal cell carcinoma
C. Paget's disease
D. Lipoma*
E. No correct answer
345. Specify malignant skin lesions
A. papilloma
B. Lipoma
C. Melanoma*
D. Lymphangioma
E. No correct answer
346. What are the clinical varieties of acne vulgaris
A. Abscessed*
B. Conical
C. Eritematosquamous
D. Erosive and ulcerative
E. No correct answer
347. Specify the current stage of rosacea
A. Follicular hyperkeratosis
B. The stage of atrophy
C. Erythematous*
D. Stage seals
E. No correct answer
348. Call exogenous factors which may cause pathological
A. condition in the skin
B. immunodeficiency
C. Pathogenic fungi*
D. Hypersensitization
E. No correct answer
349. Endogenous physiological factors that may cause
inflammatory include changes in the skin
A. Hyperthermia*
B. Chemical
C. Physical
D. Bacteria
E. No correct answer
350. Primary morphological elements can be
A. Induratum
B. Exudative*
C. Follicular
D. Nonfollikular
E. No correct answer
351. Which of the following applies to exudative lesions
A. Node
B. Bubble*
C. Tuberkulum
D. papule
E. No correct answer
352. What are the primary elements of infiltrative
A. blister
B. vesicle
C. Bull
D. Knot*
E. No correct answer
353. is the primary element, resulting from
A. Hyperkeratinization
B. Hemorrhage *
C. Changes in skin color due to acantholysis
D. Changes in the color due to melanocytes hyperfunction
E. No correct answer
354. What kind of spots are distinguished
A. Dishydrotic
B. Serous
C. Melanocytic
D. Pigment*
E. No correct answer
355. species of vascular patches
A. Petechiae
B. lentigo
C. Roseola*
D. Leucoderma
E. No correct answer
356. varieties of hemorrhagic spots
A. lentigo
B. Chloasma
C. Petechiae*
D. Telangiectasia
E. No correct answer
357. What are the characteristics of voldyrya
A. Is the result of acantholysis
B. Is an ephemeral (quickly appears and disappears without a trace)*
C. In the process of return to secondary depigmentation
D. Typical for autoimm disease
E. No correct answer
358. What are the characteristics of a bubble
A. Pitting with ulceration and atrophy
B. Is a proliferative element
C. Formed as a result of edema papillary dermis
D. Exudative is a primary element and contains a liquid*
E. No correct answer
359. How different from the bubble bubble
A. Subjective sensations
B. Size less than 05 cm bubble, the bubble more*
C. The contents of exudate
D. Localization of elements
E. No correct answer
360. layers of the epidermis
A. Horn*
B. Papillary
C. Reticular
D. fatty
E. No correct answer
361. that belongs to the skin appendages
A. Subcutaneous fat
B. epidermis
C. Nails*
D. Dermal papilla
E. No correct answer
362. What are the primary elements of morphologic
A. Papule*
B. atrophy
C. Vegetation
D. excoriation
E. No correct answer
363. What kinds of pustules you know
A. Skrofuloderma
B. excoriation
C. Petechiae
D. Folliculitis*
E. No correct answer
364. As a result, any process can pathohistomorphologic
A. form a bundle
B. Anaplasia
C. Akantosys*
D. Vacuolar degeneration
E. Acantholysis
365. What types of papules vary in size
A. Lichenoid
B. Miliary*
C. Dwarf
D. Tuberculoid
E. No correct answer
366. What are the characteristics of the nodule
A. It is ephemeral
B. It contains serous eccsudat
C. noncavernous primary element*
D. Infiltrate located in the dermis and hypodermis
E. No correct answer
367. What are the characteristics of the node, and for tubercle
A. It refers to the elements of the exudative
B. Infiltrate located in the epidermis
C. Is a members *
D. Are ephemeral formations
E. No correct answer
368. Write by appropriate primary cells and their secondary
A. elements
B. Vial atrophy
C. Bubble – acantholysis*
D. Tuberkulum secondary depigmentation
E. Node Erosion
369. Find a secondary morphological elements
A. papule
B. Vegetation*
C. Conflict
D. Tuberkulum
E. No correct answer
370. What are the characteristics of lihenifikatsii
A. Pronounced skin pattern*
B. Rash on the hearth
C. Anesthesia on the hearth
D. Warty growths on the hearth
E. No correct answer
371. What are the primary elements and their corresponding inverse
the development of secondary cells
A. Bubble – Erosion*
B. Vial atrophic changes
C. Papule scar atrophy
D. Spot flaking and crust
E. No correct answer
372. Call pyoderma specific to childhood
A. Hydradenitis
B. Basal cell carcinoma
C. Epidemic pemphigus*
D. Discoid lupus erythematosus
E. No correct answer
373. clinical variants of streptococcal impetigo
A. Surface felon*
B. Centrifugal
C. Papules, necrotic
D. Seborrheic
E. No correct answer
374. What are the main clinical signs of scabies
A. A symptom of Pospelov
B. Sgruppirovannnye bubbles
C. Paired papular or vesicular papules elements*
D. Bites itch mites
E. No correct answer
375. What tools are used to treat scabies
A. Chlorhexidine
B. Gray mercurial ointment
C. 33 Rate of sulfuric ointment*
D. Kerosene mixed with vegetable oil
E. No correct answer
376. What are the main clinical signs of lice
A. Finding nits*
B. Finding ticks
C. Paired papular elements
D. A symptom GorchakovHardy
E. No correct answer
377. With what diseases differentiate scabies
A. shingles
B. Pruritus*
C. Exudative erythema
D. psoriasis
E. No correct answer
378. What rash characteristic of the lepromatous type of leprosy
A. Gummy
B. Units*
C. Vegetating papules
D. Pockets lihenifikatsii
E. No correct answer
379. What are the symptoms occur due to various motor
disorders in leprosy
A. "Paw" monkey*
B. A symptom of "butterfly"
C. Honeycomb Celsus
D. Staggering gait
E. No correct answer
380. What laboratory and instrumental methods used to
diagnosis of leprosy
A. Mantoux test
B. Reaction Cana
C. Lepro trial*
D. Trial Jadassohn
E. No correct answer
381. Which of the provisions are considered correct
A. Type of tuberculosis a negative test
B. Lepromatous negative test
C. Undifferentiated type trial
D. Lepromatous type – negative*
E. No correct answer
382. Specify which considered contagious skin disease
A. Scabies*
B. psoriasis
C. Pink zoster Gibert
D. Dermatitis Duhring
E. No correct answer
383. Specify zveno in the epidemiological chain of the disease Borowski
A. Wand Hansen
B. Leishmania *
C. cattle
D. Fleas
E. No correct answer
384. Specify the reservoir of infection in the leishmaniasis
A. Ground Squirrels*
B. Mosquitoes and fleas
C. Leishmania
D. Cockroaches
E. No correct answer
385. List the basic preventive measures when
leishmaniasis
A. Disinfection of the premises and linen
B. Timely detection and treatment of patients*
C. The destruction of stray dogs
D. Immunizations
E. No correct answer
386. What laboratory and instrumental methods of diagnosis
are used for the diagnosis of tuberculosis of the skin
A. Trial Jadassohn
B. Smearimprint
C. Research on the LEcells
D. Sample Pirke*
E. No correct answer
387. Specify the localized forms of tuberculosis of the skin
A. Papulonecrotic
B. Vulgar lupus*
C. Indurative
D. Miliary
E. No correct answer
388. What are the current stage of psoriasis:
A. Latent
B. Incubation
C. Progressive*
D. A progressive
389. Specify the most frequent localization of sites on the skin
psoriatic lesions:
A. skin on the face
B. Extensor surfaces of the lower extremities*
C. Flexor surfaces of the upper extremities
D. Leather back
E. No correct answer
390. clinical variants of psoriasis:
A. Papular psoriasis
B. Exfoliative psoriasis (leafshaped)
C. Common psoriasis*
D. Infiltrativesuppurative form
E. No correct answer
391. clinical variants of psoriasis:
A. Disgidroticheskaya form
B. Papules necrotic form
C. Pustular psoriasis*
D. Psoriatic lymphadenopathy
E. No correct answer
392. Call osobennno psoriatic plaques:
A. In the central parts have umbilicated
B. Papules covered with silverywhite scales*
C. Covered with serous crusts
D. Are placed are grouped (herpetiformis location)
E. No correct answer
393. What is included in the triad Auspittsa:
A. A symptom of a thimble
B. A symptom of apple jelly
C. A symptom of the oil spot
D. Symptom of terminal films*
E. No correct answer
394. Specify the pathognomonic symptoms of psoriasis:
A. A symptom of Jadassohn
B. A symptom of "butterfly"
C. Triad Auspittsa*
D. Grid Wickham
E. No correct answer
395. Which nail changes observed in the defeat of their
psoriatic process:
A. Sign "thimble"*
B. Purulent inflammation of the nail ridges
C. Cyanotic nail plate
D. Affects only the nail plate brushes
E. No correct answer
396. List the clinical manifestations common to psoriasis and lichen planus:
A. Acantholysis
B. Papules*
C. The formation of infectious granuloma Langhans cells
D. Spongiosa
E. No correct answer
397. With what diseases it is necessary to differentiate psoriasis:
A. Papular syphilis*
B. Syphilitic roseola
C. Herpes simplex
D. Dermatitis Duhring
E. No correct answer
398. What diseases are characterized by monomorphic eruptions:
A. Psoriasis*
B. eczema
C. Dermatitis Duhring
D. Erythema multiforme ekssudativeya
E. No correct answer
399. What drugs are used to treat psoriasis:
A. Sulfuric simple ointment
B. Griseofulvin
C. Hormonal ointments*
D. Anticoagulants
E. No correct answer
400. What rash characteristic of a simple contact dermatitis:
A. tuberculous
B. Vesicular
C. Urticaria
D. Erythema with sharp edges*
E. No correct answer
401. By the proliferative histomorphological changes include:
A. Karyorrhexis
B. Spongiosa
C. Akantolizis
D. Hyperkeratosis*
E. No correct answer
402. There are no sweat glands in the skin;
A. Red border*
B. Armpits
C. Palms
D. Soles
E. No correct answer
403. There are no sebaceous glands in the skin;
A. Nasolabial triangle
B. Palms, soles*
C. Scalp
D. Interscapulum
E. No correct answer
404. Which element is a kind of blisters:
A. Conflict*
B. Telangiectasia
C. excoriation
D. Bull
E. No correct answer
405. What kind of spots:
A. Acne
B. rupee
C. Roseola*
D. Conflict
E. No correct answer
406. What morphological element is characterized by the ephemeral:
A. pustule
B. papule
C. Tuberkulum
D. Blister*
E. No correct answer
407. Where are the "sebaceous glands":
A. Internal piece of the foreskin*
B. Red border
C. The skin of breast
D. Interscapulum
E. No correct answer
408. "Meibomian" sebaceous glands are located:
A. On derma of thorax
B. On the edge of the eyelids*
C. On the skin of the ears
D. On the red border of lips
E. No correct answer
409. Which areas of the skin sebaceous glands open directly into the epidermis:
A. Glans penis*
B. Scalp
C. The skin of the nose
D. Breast skin
E. No correct answer
410. Akantolizis (acantholysis) is:
A. Increased cell proliferation thorny layer
B. Dysfunction of keratinization
C. Loss of communication between the cells of the thorny layer*
D. Atypical nuclei of epithelial cells
E. No correct answer
411. parakeratosis is incomplete keratinization epidermotsitov accompanied by dystrophy:
A. Granular layer*
B. The basal layer
C. The papillary layer
D. Mesh layer
E. No correct answer
412. granulosa is:
A. Thickening of the granular layer*
B. Thinning of the granular layer
C. Structural changes in squamous cell carcinomas
D. Intracellular edema
E. No correct answer
413. anaplasia is:
A. Epithelial nuclei atypia in tumors*
B. Graincytoplasmatic cells
C. Loss of communication between the cells of the thorny layer
D. Vacuolization of the cytoplasm
E. No correct answer
414. Spongiosis is:
A. Intercellular edema vMalpigievom layer*
B. Cheesy disintegration of tissues
C. Vacuolar degeneration in the Malpighian layer
D. Degenerative changes in the dermis
E. No correct answer
415. Acanthosis not properties:
A. Scleroderma*
B. Lichen planus
C. Neurodermatitis
D. Psoriasis
E. No correct answer
416. Under what dermatoses marked acantholysis:
A. eczema
B. Contact dermatitis
C. Ecthyma vulgar
D. Pemphigus vulgaris*
E. No correct answer
417. Under what dermatoses granulosa notes:
A. Planus*
B. Turniol
C. Lichen simplex
D. Genital warts
E. No correct answer
418. Which function is not peculiar to the skin:
A. suction
B. Education toxins*
C. Thermoregulation
D. sense organ
E. No correct answer
419. Angular stomatitis is a variation:
A. Streptoderma*
B. Stafilodermii
C. Candidiasis
D. Pemphigus
E. No correct answer
420. Chankriform pyoderma most often begins:
A. C erosion*
B. With spots
C. With bubbles
D. With a bundle
E. No correct answer
421. Which group of dermatoses is shankriform pyoderma:
A. Diseases transmitted sexually
B. Pyoderma*
C. Cystic dermatosis
D. Dermatitis
E. No correct answer
422. Ritter exfoliative dermatitis is a type of:
A. Streptoderma
B. Contact dermatitis
C. Toxicoderma
D. Stafilodermii*
E. No correct answer
423. When exfoliative dermatitis Ritter can be detected positive symptom:
A. Nikolsky *
B. GorchakovHardy
C. Sisto
D. Kebnera
E. No correct answer
424. Who is more common psevdofurunkulosys Finger:
A. Y newborns and infants*
B. Old people
C. Teenagers
D. among women
E. No correct answer
425. What factors contribute to the development of acne:
A. Hyperactivity of the sebaceous glands*
B. Frequent colds
C. Breach of thermoregulation
D. physical exercise
E. No correct answer
426. With a variety of acne supporting factor are mites Zheleznitsa:
A. Acne vulgaris
B. Rosacea*
C. Spherical acne
D. Necrotic acne
E. No correct answer
427. What kind of spots characteristic of the erythematous stage demodekoza:
A. Telangiectasia*
B. Petechiae
C. Ecchymosis
D. Vibitsess
E. No correct answer
428. For Rosacea is characterized by the absence of:
A. Komedonov*
B. Follicular papules
C. Pustule
D. Rhinophyma
E. No correct answer
429. What are the morphological features characteristic of hidradenitis sup:
A. Thick, welded to the skin painful nodes*
B. Compact, mobile, painless nodes
C. Blisters
D. Lenticular papules
E. No correct answer
430. The autopsy units hidradenitis stands at:
A. Gummy exudate opalescent
B. Creamy pus mixed with blood*
C. Serous fluid
D. Purulent necrotic core
E. No correct answer
431. favus called:
A. Virus Filter
B. Corynebacteria
C. Chlamydia
D. Antropofilnymi fungus*
E. No correct answer
432. The typical localization at erythrasma:
A.Inguinal folds*
B.Scalp
C.nearnail roller
D.Interdigital gaps feet
E.No correct answer
433. Which symptom characteristic of infiltrativesuppurative trihofitii:
A. Glow in the lamplight Wood
B. Bee combs*
C. A symptom Pincus
D. A symptom Nikolsky
E. No correct answer
434. In the laboratory diagnosis of fungal infections is used:
A. Microsporia damaged hair and flakes*
B. Histological examination
C. Fluorescent Diagnostics
D. Smears from lesions
E. No correct answer
435. What is characteristic of psoriasis:
A. Net Wickham
B. Symptom falling through probe
C. Triad Auspittsa*
D. Trial Balzer
E. No correct answer
436. The source of infection with the disease Borowski is urban:
A. Rodents
B. Hymenoptera
C. People*
D. Artiodactyls
E. No correct answer
437. The source of infection with the disease Borowski rural type is:
A. human
B. Rodents*
C. wild animals
D. cattle
E. No correct answer
438. Indicate the average length of the current urban cutaneous leishmaniasis:
A. 2 weeks
B. 34 months
C. Plus about a year
D. a few days
E. No correct answer
439. Indicate the average length of the current cutaneous leishmaniasis rural type:
A. 26 months*
B. About a year
C. some years
D. a few days
E. No correct answer
440. The source of infection of leprosy is:
A. wild animals
B. People*
C. Rodents
D. Insects
E. No correct answer
441. In some material from the patient is most often found Mycobacterium leprae:
A. Secret sweat glands
B. In urine
C. In sebum
D. B scrapings from leprosy and nasal mucosa*
E. No correct answer
442. Which viruses cause genital warts:
A. Filter viruses*
B. Herpesviruses
C. Herpes simplex virus
D. CMV
E. No correct answer
443. Molluscum contagiosum is ...
A. erythema
B. pustule
C. vesicle
D. Papule*
E. No correct answer
444. Indicate the true variety of eczema:
A. Pruriginous*
B. Nummular
C. Seborrheic
D. Varicose
E. No correct answer
445. Which symptom seen with pink zoster Gibert:
A. A symptom of chips
B. Sign "parent" plaque*
C. A symptom of ladies' Heel
D. Koebner phenomenon
E. No correct answer
446. Monomorphic rash may be at:
A. Erythema multiforme exudative
B. Microbial eczema
C. Dermatitis Duhring
D. Psoriasis*
E. No correct answer
447. Akantolysis observed at:
A. Herpes simplex
B. True pemphigus*
C. Shingles
D. Atopic dermatitis
E. No correct answer
448. Monomorphic papular rash occurs when:
A. Planus*
B. hives
C. eczema
D. lupus erythematosus
E. No correct answer
449. Acanthosis typical for:
A. Planus*
B. Turniol
C. Lichen simplex
D. Simple herpes
E. No correct answer
450. Hyperkeratosis is typical for:
A. Bullous impetigo
B. Warts *
C. Folliculitis
D. Demodekosys
E. No correct answer
451. The most superficial skin are:
A. ointment
B. paste
C. patch
D. Gadgets*
E. No correct answer
452. What type of treatment should be external
choose when moist skin inflammation:
A. ointment
B. Chatterbox
C. Gadgets*
D. Varnishes
E. No correct answer
453. What type of lotions should be selected for the weeping of the infected skin surface:
A. Zinc
B. Rivanolous*
C. Tannic
D. Boric acid
E. No correct answer
454. What type of external therapy to choose in chronic infiltrative inflammation of the skin:
A. Zinc paste
B. The oil mash
C. 5% Naftalan ointment
D. Drying up wet bandag*
E. No correct answer
455. What kind of external agents have antipruritic property:
A. tar
B. Menthol *
C. salicylic acid
D. zinc
E. No correct answer
456. Which means external treatment has cauterizing property:
A. A solution of sodium giposulfata
B. A solution of podofillina
C. Solution furatsillina
D. Methylene blue solution*
E. No correct answer
457. For the destruction of the skin are used:
A. Liquid nitrogen*
B. Zinc oxide
C. Potassium permanganate
D. Paint Castellane
E. No correct answer
458. The method is suitable for cryoablation treatment:
A. Extensive warts
B. Plaque psoriasis
C. Limited neurodermatitis
D. For warts*
E. No correct answer
459. In the study of skin biopsy revealed fusion of epithelial intercellular bridges in the spinous
layer of cells detected epidermis Ttsanka. Specify the nature of the disease process:
A. Granulosa
B. Acantholysis*
C. Parakeratosis
D. Papillomatosis
E. No correct answer
460. What factors contribute to the development of pyoderma:
A. Gene mutations
B. Irradiation of ultraviolet irradiation
C. Microfracture*
D. Worm infestation
E. No correct answer
461. What factors contribute to the development of pyoderma:
A. Diabetes*
B. hemophilia
C. Violation of the mechanism of division epidermocyt
D. Violation of keratinization
E. No correct answer
462. To stafilodermiyam include:
A. Hydradenitis*
B. Lichen simplex
C. Slit impetigo
D. Ecthyma vulgar
E. No correct answer
463. What pyoderma found only in newborns and infants:
A. Hydradenitis
B. Pemphigus vulgaris
C. Chronic ulcerative pyoderma
D. Veziculopustulosys*
E. No correct answer
464. What pustular skin diseases do not occur in the newborn:
A. Epidemic pemphigus
B. Hydradenitis*
C. Finger psevdofurunculosys
D. Dermatitis Ritter
E. No correct answer
465. What pustular disease does not occur in adults:
A. Boils
B. Turniol
C. Angular stomatitis
D. Exfoliative dermatitis Ritter*
E. No correct answer
466. What are the morphological features characteristic of dermatitis Ritter:
A. Papules, blisters
B. Bumps, crust
C. Crust bubbles*
D. Vesicles, papules
E. No correct answer
467. What dermatoses in the first place should be differentiated eksfolliativny dermatitis Ritter:
A. Dermatitis herpetiformis Duhring
B. Bullous form strofulyusa
C. Epidermolysis bullosa*
D. Herpes simplex
E. No correct answer
468. Multiple abscesses in children causes:
A. Streptobatsilla DyukreyaPetersen
B. Staphylococcus aureus*
C. Calf Donovan
D. Streptococcus
E. No correct answer
469. Psevdofurunculosys Finger refers to the group:
A. Stafilodermy*
B. Parasitic skin diseases
C. Viral diseases
D. Complications of scabies
E. No correct answer
470. What factors contribute to the development psevdofurunkuleza Finger:
A. Overheating newborn*
B. Disruption of the sebaceous glands
C. Endocrinopathies
D. Pregnancy
E. No correct answer
471. What factors contribute to the development of acne:
A. Frequent colds
B. Breach of thermoregulation
C. Dysfunction of the sebaceous glands*
D. Dysfunction of the sweat glands
E. No correct answer
472. What kind of spots characteristic of the erythematous stage demodekoza:
A. Telangiectasia, erythema*
B. Ecchymosis, spots
C. erythema
D. Roseola, pustules
E. No correct answer
473. pineal "the nose is typical for:
A. Acne vulgaris
B. Rosacea*
C. Discoid lupus
D. Leprosy
E. No correct answer
474. What factors are important in the development of seborrhea:
A. Sweat gland dysfunction ekkrinnyh
B. Dysfunction of the apocrine sweat glands
C. Thyroid dysfunction
D. Dysfunction of the sebaceous glands*
E. No correct answer
475. Choose varieties of oily seborrhea:
A. Dry
B. Mukovidous
C. Liquid*
D. The leaf
E. No correct answer
476. The development of dermatosis facilitated by the presence of seborrhea:
A. Trihofitii
B. Baldnes*
C. impetigo
D. Pink lichen Gibert
E. No correct answer
477. For a dermatosis characterized by the presence of comedones:
A. Rosacea
B. Limited neurodermatitis
C. Psoriasis
D. Acne vulgaris*
E. No correct answer
478. At what diseases of the skin may experience alopecia:
A. psoriasis
B. Hydradenitis
C. Planus*
D. Erythema nodosum
E. No correct answer
479. What factors contribute most to the development of alopecia areata:
A. Neuroendocrine disorders*
B. In beriberi
C. With beriberi
D. Increased insolation
E. No correct answer
480. What factors contribute to the development of hidradenitis sup:
A. Sweating*
B. Professional skin contamination
C. Hypersensitivity to iodine
D. Micronutrient deficiencies
E. No correct answer
481. Specify the location of nodes in the hydradenitis:
A. Nasal bridge, sky
B. Shin
C. Armpits*
D. Interscapulum
E. No correct answer
482. Favourite localization rash scabies:
A. Interdigital gaps feet
B. Interdigital folds of brushes*
C. Facial skin, neck
D. Leather torso
E. No correct answer
483. For scabies is characterized by:
A. Paired papules vesicular elements*
B. Polygonal papules
C. grouped small bubbles
D. Peeling
E. No correct answer
484. Specify a rare localization of scabies rash in children:
A. Scalp*
B. The skin of the genitals
C. The skin of breast
D. The skin of the abdomen
E. No correct answer
485. Specify the drug used in the pemphigus vulgaris:
A. Prednisone*
B. Dapsone
C. Monomitsin
D. Nystatin
E. No correct answer
486. What is the primary morphological element is preceded by erosion:
A. papule
B. Vesicle*
C. Urtica
D. Macula
E. No correct answer
487. What disease is not included in the group of dermatophytosis:
A. Favus
B. Rubrofitii
C. Trichophytosis
D. Chromomycosis*
E. No correct answer
488. Specify the drug used in leprosy:
A. Norsulfazol
B. Monomitsinа
C. Delagil
D. Dapsone*
E. No correct answer
489. In the treatment of candidiasis is not used:
A. Acyclovir*
B. Nizoral
C. Nystatin
D. Diflucan
E. No correct answer
490. In the treatment of candidiasis is not used:
A. Acyclovir*
B. Nizoral
C. Nystatin
D. Diflucan
E. No correct answer
491. Symptom Nikolsky positive at:
A. Trepemphigus*
B. Skrofuloderma
C. Dermatitis herpetiformis Duhring
D. Focal neurodermatitis
E. No correct answer
492. pathognomonic for eczema sprinkler elements are:
A. erythema
B. Peel
C. Oozing erosion*
D. Papules
E. No correct answer
493. What are the main clinical signs are not specific for chronic eczema:
A. infiltration
B. Lichenification
C. Bright redness, swelling*
D. Venous congestion
E. No correct answer
494. If any of these viral diseases oral mucosa are not affected:
A. Molluscum contagiosum*
B. chicken pox
C. Simple herpes
D. Shingles
E. No correct answer
495. What is not affected in rubrofitii:
A. Soles
B. Hair*
C. Nail brush
D. Nails feet
E. No correct answer
496. The method of staining for ZiehlNielsen which pathogens are defined by:
A. Chlamydia
B. Pale treponema
C. Koch's bacillus*
D. Ureaplasma
E. No correct answer
497. Specify the characteristic appearance and location of the rash of pink lichen:
A. Congested spot on the trunk*
B. blister
C. vesicle
D. on the face
E. no correct answer
498. Specify the clinic at toxicoderma:
A. Syndrome asthenoneurotic
B. Sezary syndrome
C. Cushing's syndromeItsengo
D. Lyell's syndrome*
E. no correct answer
499. Specify the drug used in psoriasis:
A. Blemaren
B. Ethambutol
C. Plus dexamethasone*
D. Dithranol
E. no correct answer
500. Specify the symptoms of lupus erythematosus
A. A symptom Gorchakov Ardi
B. A symptom of asbestosHansen
C. A symptom Nikolsky
F. Sym no correct answer
D. ptom Besnier Mescherskogo
501. After some initial elements formed by erosion:
A. Abscess*
B. spot
C. Tubercle
D. Node
E. no correct answer
502. What are the phenomena included in psoriatic triad:
A. Ladies' heel
B. The phenomenon of apple jelly
C. Phenomenon stearin spots*
D. Mask of St. Anthony
E. no correct answer
503. Streptococcal pyoderma differentiated with what disease:
A. Simple herpes*
B. Erythrasma
C. CPL
D. Psoriasis
E. no correct answer
504. Deep trichophytosis differentiated c what diseases:
A. Erythrosis
B. eczema
C. Microsporia
D. Psoriasis
E. no correct answer
505. At what diseases are not observed site:
A. Leprosy
B. Herpes*
C. Cutaneous tuberculosis
D. Tertiary syphilis
E. no correct answer
506. In the dermis layers are distinguished:
A. grainy
B. horny
C. Papillary*
D. Basal
E. no correct answer
507. By the atypical chancre include:
A. Extensive warts
B. Balanoposthitis
C. Chancreamygdala*
D. Balanit
E. no correct answer
508. Choose the wrong statement: primary syphiloma can make complication
A. Orchitis
B. Paraphimosis
C. Gangrenizatsiya
D. Fagedinizmom
E. no correct answer
509. Choose the wrong statement, the possible reason is simple dermatitis
A. Low temperature
B. high temperatures
C. Ingestion of medicines*
D. Friction pressure
E. no correct answer
510. For localized scleroderma is uncommon stage:
A. Induration
B. Nonfollicular hyperkeratosis*
C. Atrophy
D. Edema
E. no correct answer
511. In the treatment of candidiasis is not used:
A. Nystatin
B. Polkortolon*
C. Intrakenazol
D. no correct answer
E. Levorin
512. In the treatment of head lice is not used:
A. Mikozolon*
B. 1020% emulsion benzyl
C. Spregal
D. Nittifor
E. no correct answer
513. erythrodermic psoriasis can develop after:
A. 5% topical ointment salicylic acid
B. The use of ultraviolet irradiation in patients with psoriasis type summer
C. Emotional experiences
D. Designation ACTH*
E. no correct answer
514. Specify the secondary elements that are the result of transformation of the node:
A. Scar atrophy
B. A secondary pigmentation*
C. ulcer
D. scar
E. no correct answer
515. Specify the clinical signs are not specific to tertiary syphilis:
A. Gumma palate
B. Mezaortit
C. Units
D. Extensive warts*
E. no correct answer
516. What are the main clinical signs are not specific for chronic eczema:
A. Bright hyperemia*
B. infiltration
C. Lichenification
D. Cracks
E. no correct answer
517. Under soak for topical treatment of eczema only apply:
A. Creams
B. Gadgets*
C. Powders
D. Powder
E. no correct answer
518. Specify the feature is not characteristic for discoid lupus erythematosus form:
A. Symptom "butterfly".
B. Symptom BesnierMeshchersky.
C. The symptom of "apple jelly"*
D. Follicular hyperkeratosis.
E. no correct answer
519. What disease is not included in the group neyrodermatozov:
A. Scrapie
B. hives
C. Itching
D. Allergiichesky dermatitis*
E. no correct answer
520. What is not typical in a clinical picture of rosacea:
A. Follicular hyperkeratosis*
B. erythema
C. Infiltration of the skin of the nose
D. An increase in the size of the nose
E. no correct answer
521. Specify the most characteristic feature of scabies:
A. Serous well
B. Evening and night itching*
C. Infiltration lesions
D. Polyadenylation
E. no correct answer
522. Choose among the following surface shape stafilodermii:
A. furuncle
B. Osteofolliculit*
C. carbuncle
D. Hydradenitis
E. no correct answer
523. In therapy planus can be used:
A. Novopassit*
B. Prednisolone
C. Androgens
D. All of the above
E. no correct answer
524. In the primary period of syphilis patients can meet all of the features krome
A. Negative Wasserman
B. Lymphangitis
C. Chancre
D. Nonerosive papules*
E. no correct answer
525. Choose the most effective means for the treatment of chlamydial urethritis:
A. penicillin
B. Tetracycline*
C. Metronidazole
D. Retarpen
E. no correct answer
526. Because of the clinical signs are not typical of AIDS patients:
A. Lymphoma
B. Recurrent herpes
C. diarrhea
D. Vitiligo*
E. no correct answer
527. Specify the drug used in the pemphigus vulgaris:
A. Monomitsin
B. Orungal
C. Prednisone*
D. Dapsone
E. no correct answer
528. Specify a drug used to treat scabies:
A. Vishnevsky ointment
B. 10% ointment sintomitsinovaja
C. Aerosol Spregal*
D. 2% salicylic ointment
E. no correct answer
529. Specify the drug used in skin candidiasis:
A. Orungal*
B. Furazolidon
C. Bactrim
D. Biseptol
E. no correct answer
530. Specify the drugs used in the progressive stage of psoriasis:
A. 2% ointment salicylat
B. 10% ointment salicylat*
C. Ointment dermatol
D. 10% ointment ichtiol
E. no correct answer
531. Specify the phenomena characteristic of lichen planus:
A. Fish roe
B. cigarette paper
C. Mesh Wickham*
D. "Ladies' heel"
E. no correct answer
532. Specify the symptoms characteristic of the multicolored lichen:
A. Sample Wickham
B. Sample Balzer*
C. cigarette paper
D. Triad auspices
E. no correct answer
533. Specify the drug that you can not appoint resistant white dermographism:
A. Sodium thiosulfate
B. Calcium gluconate*
C. Medicine Pavlova
D. Potassium iodide
E. no correct answer
534. Specify the drug used with shingles:
A. Delagil
B. Dapsone
C. Oxoline*
D. Prednisolone
E. no correct answer
535. Select the clinical signs characteristic of sycosis:
A. Chronic relapsing course*
B. Rheumatic pain
C. Nodules
D. Excessive sweating
E. no correct answer
536. Clinical signs of herpes simplex virus does not apply:
A. Keratoconjunctivitis
B. Lichen sclerosus*
C. Eczema herpeticum
D. Meningoencephalitis
E. no correct answer
537. Which of the diseases are not a manifestation of HIV infection:
A. Kaposi's sarcoma
B. diarrhea
C. Herpes simplex
D. Vitiligo*
E. no correct answer
538. What is not typical for psoriasis:
A. May first appear after birth
B. Can be cause by laboratory animals*
C. Stressrelated
D. In the event of the disease plays a role heredity
E. no correct answer
539. Specify the characteristic sign of a blister:
A. Is due to intracellular edema*
B. Leaves behind erosion
C. Peeling on the surface of the blister
D. No subjective sensations
E. no correct answer
540. What are the clinical manifestations characteristic of secondary recurrent syphilis:
A. Parenchymal keratitis
B. Swelling indurative
C. Fagedenesation
D. Leucoderma*
E. no correct answer
541. What are the clinical symptoms in patients with primary syphilis:
A. Swelling indurative
B. Extensive warts*
C. Regional lymphangitis
D. Ulcerative chancre
E. no correct answer
542. Specify the characteristic signs of uncomplicated chancre:
A. Saped edge
B. Dense infiltrate the base of the ulcer*
C. Lack of treponemes in the discharge sores
D. Purulent discharge
E. no correct answer
543. For patients with chancroid is unusual:
A. Signs of inflammation
B. Noted a tendency to autoinnokulyation
C. Solid infiltrate*
D. Lesions are formed after 23 days onsite implementation streptobatsill
E. no correct answer
544. In the treatment of bacterial vaginosis is used:
A. penicillin
B. Erythromycin
C. Plus metronidazole
D. Ampicillin
E. no correct answer
545. For the diagnosis of the disease applied research using Wood's lamp:
A. Microsporia*
B. Pink zoster.
C. Vitiligo.
D. Psoriasis.
E. no correct answer
546. What is the primary morphological element is preceded by erosion:
A. Abscess*
B. None of the above
C. Tubercle
D. Node
E. no correct answer
547. What is the primary morphological element is preceded by an ulcer:
A. bubble
B. blister
C. Hump*
D. None of the above
E. no correct answer
548. Specify the characteristic symptoms of uncomplicated chancre:
A. Saped edge
B. Purulent discharge
C. Dense infiltrate the base of the ulcer*
D. Signs of inflammation
E. no correct answer
549. What pustular skin diseases do not occur in the newborn:
A. Sycosis*
B. Vezikulopustulosis
C. Finger psevdofurunkulosis
D. Exfoliative dermatitis Ritter
E. no correct answer
550. Choose varieties of seborrhea:
A. The leaf
B. Dry*
C. Papulonecrotic
D. Scaly
E. no correct answer
551. What clinical symptom seen with lupus erythematosus:
A. "Ladies' heel"*
B. "Blood dew"
C. A symptom Pincus
D. Grain Trela
E. no correct answer
552. Name the atypical form of psoriasis:
A. Scalp
B. Intertriginosis*
C. Coin
D. Numular
E. no correct answer
553. Specify the most severe form of psoriasis:
A. geographical
B. Erythrodermic*
C. Numualr
D. Handfoot
E. no correct answer
554. Name the localization elements in pustular psoriasis Barbera:
A. Red border
B. Soles*
C. Leather body
D. Scalp
E. no correct answer
555. What kind of pustular psoriasis:
A. Lichenoid
B. Psoriasis Tsumbusha*
C. figured
D. Erythrosis
E. no correct answer
556. What clinical symptom characteristic of dermatitis Dühring:
A. Small focal baldness
B. Sclerodactyly
C. Esophagitis
D. Eosinophilia content bladder*
E. no correct answer
557. What clinical varieties of cutaneous leishmaniasis you know:
A. Undifferentiated type
B. Erythematoussquamous type
C. Lepromatous type
D. Lateulcerated type*
E. no correct answer
558. What kinds of clinical disease Borowski:
A. Urban style*
B. Undifferentiated
C. Infiltrative suppurative
D. Papulosys
E. no correct answer
559. Which layer is not part of the epidermis:
A. grainy
B. Papillary*
C. basal
D. horny
E. no correct answer
560. Choose varieties of seborrhea:
A. The leaf
B. Fat*
C. Vulgar
D. Dyshtdrotic
E. no correct answer
561. What diseases are a group of stafilodermy:
A. Epidemic pemphigus*
B. Impetigo nail ridges
C. Slit ecthyma vulgar impetigo
D. Botryo mycoma
E. no correct answer
562. What means have antipruritic property:
A. tar
B. Menthol *
C. sulfur
D. Ihtiola
E. no correct answer
563. What pyoderma found only in newborns and infants:
A. Hydradenitis
B. furuncle
C. Chronic ulcerative pyoderma
D. Vezikulopustulosis*
E. no correct answer
564. What are the morphological features characteristic of dermatitis Ritter:
A. Papules
B. Bubbles*
C. Small bubbles
D. Blisters
E. no correct answer
565. Localization rash scabies:
A. The skin of the neck
B. Leather around the navel, inner thighs*
C. Interdigital gaps feet
D. Places where sebaceous glands
E. no correct answer
566. Name the secondary morphological element:
A. Vegetation*
B. Bundle
C. Abscess
D. Tubercle
E. no correct answer
567. What proliferative morphological element:
A. Abscess
B. Vial
C. Knot*
D. Bubble
E. no correct answer
568. acantholysis is morphological
A. based on symptoms:
B. Of asbestos Hansen*
C. Pospelov
D. Wickham
E. Andogsky
569. What is the clinical evidence harakterendlya dermatitis Dühring:
A. Cryoglobulinemia
B. Leukopenia
C. Eosinophils in the blood*
D. Leukocytosis
E. no correct answer
570. What tests are used for the diagnosis of dermatitis herpetiformis Duhring:
A. 3 cups of the sample Thompson*
B. Iodine sample Balzer
C. Trial Minor
D. 50% potassium iodide ointment
E. no correct answer
571. Which drugs are most effective in dermatitis Dühring:
A. Biseptol
B. VAT*
C. Cytostatic
D. Antibiotics
E. no correct answer
572. Specify papules especially when red zoster:
A. Excessive flaking
B. Peeling edge
C. Umbilicated*
D. Peripheral roll
E. no correct answer
573. What kind of pustular psoriasis:
A. Psoriasis sarcoma
B. Lichenoid
C. Ringed
D. Psoriasis Barbera*
E. no correct answer
574. What are the areas of skin are usually free from lesions in the CPL:
A. Palms, soles*
B. Flexor surface of the limbs
C. Wrist joint
D. Flexor surface of the forearm
E. no correct answer
575. What should be differentiated dermatoses lichen planus:
A. The artificial dermatitis
B. Chromophytosis
C. Psoriasis*
D. Dermatitis Duhring
E. no correct answer
576. Name the localization elements in pustular psoriasis Barbera:
A. Palms, soles*
B. Scalp
C. Red border
D. Exposed areas of the body
E. no correct answer
577. Indicate the true variety of eczema:
A. Pruriginous, disgidroticheskaya*
B. Nummular
C. Seborrheic, varicose
D. Mycotic
E. no correct answer
578. rash of erythema multiforme exudative located:
A. Grouped along the nerves
B. Pairs
C. In the form of rings*
D. Asymmetrically platform
E. no correct answer
579. Specify the characteristic signs of scabies:
A. Confluent papules
B. Symptom Gorchakov Hardy*
C. Grouped vesicles plaque
D. Net Wickham
E. no correct answer
580. What drugs are used to lecheniyachesotki:
A. 20% benzyl benzoate ointment*
B. 5% of Naftalan ointment
C. 5% sulfuric ointment
D. Hydrofoil
E. no correct answer
581. With what diseases should be differentiated scabies:
A. Atopic dermatitis*
B. Pink zoster
C. shingles
D. Multicolored shingles
E. no correct answer
582. What obligate factors for dermatitis:
A. Gold jewelery
B. Detergents
C. Concentrated alkali*
D. Jewelry made of platinum
E. no correct answer
583. Allergic reactions to the medication may occur as:
A. A symptom Pilnova
B. A symptom of Jadassohn
C. Erythema *
D. A symptom Balzer
E. no correct answer
584. Choose varieties toxicoderma:
A. Fixed erythema*
B. Psoriasis
C. Perfrigeration
D. Pellagrosis dermatitis
E. no correct answer
585. What medications often cause erythema fixed:
A. Tavegil
B. Tetracycline
C. Analginum
D. Sulfadimetoksin*
E. no correct answer
586. What disease should be differentiated common toxicoderma:
A. Exfoliative dermatitis*
B. Pyoderma
C. Rosacea
D. Disease Borowski
E. no correct answer
587. Choose the typical localization exudative erythema multiforme at:
A. Scalp
B. Interdigital spaces of hands
C. Mucosa*
D. The neck
E. no correct answer
588. What factors are important in the development of exudative erythema multiforme:
A. Hypersensitivity to fluoride
B. Foci of chronic infection*
C. Hypersensitivity to iodine
D. Photosensitivity
E. no correct answer
589. What dermatoses differentiate erythema multiforme:
A. Pemphigus vulgaris*
B. Strofulyus
C. Skrofuloderma
D. Induratum erythema Bazin
E. no correct answer
590. Specify the pathogenetic factors of pink lichen Gibert:
A. Yeasts
B. Hyperactivity of the sebaceous glands
C. Viral infection*
D. Idiosyncrasy to iodine
E. no correct answer
591. Specify the species of microbial eczema:
A. Varicose*
B. Adult
C. Idiopathic
D. Horn
E. no correct answer
592. What clinical sign is characterized by a rash in true eczema:
A. Sign "serous wells"*
B. Tubercles
C. Monomorphic papules
D. Ephemeral blisters
E. no correct answer
593. What are the symptoms seen with discoid lupus:
A. Collar Biett
B. Sign "ladies' heel"*
C. Net Uithema
D. Silvery white, easily removable scales
E. no correct answer
594. Choose varieties of scleroderma:
A. Vulgar
B. Erythematous
C. Infiltrative
D. White spot disease*
E. no correct answer
595. Lichen planus is characterized by:
A. Polymorphism
B. Absence of pruritus
C. Mesh Wickham*
D. Urticaria rashes
E. no correct answer
596. For planus is characterized by:
A. Parakeratosis
B. interdermal mikroabcsess
C. Irregular thickening of the granular layer of the epidermis*
D. Akantolizis
E. no correct answer
597. What are the clinical symptoms pathognomonic planus:
A. Grain Trela
B. A symptom auspices
C. Umbilicated*
D. "Honeycomb Celsus"
E. no correct answer
598. What diseases do not belong to a group of viral dermatoses:
A. Molluscum Contagiosum
B. shingles
C. Simple herpes
D. Psoriasis*
E. no correct answer
599. The vascular patch is:
A. Erythema *
B. Lentigo
C. Leucoderma
D. Nevus
E. no correct answer
600. Primary morphological elements:
A. Lichenification
B. atrophy
C. ulcer
D. Bubble*
E. no correct answer
601. Secondary morphological features:
A. bubble
B. Crust*
C. blister
D. Bundle
E. no correct answer
602. The bubble is formed by:
A. Psoriasis
B. CPL
C. Multicolored shingles
D. Simple herpes*
E. no correct answer
603. The outcome of the node is:
A. scar
B. Scale*
C. atrophy
D. crust
E. no correct answer
604. exudative morphological elements are:
A. Abscess*
B. Node
C. papule
D. Tubercle
E. no correct answer
605. The layers of the epidermis:
A. reticulate
B. Papillary
C. fatty
D. Granular*
E. no correct answer
606. Have cavity morphological elements:
A. Bubble*
B. papule
C. Node
D. Blister
E. no correct answer
607. The rash polymorphic at:
A. Warts
B. Eczema*
C. Lichen planus
D. Molluscum Contagiosum
E. no correct answer
608. monomorphic rash when dermatoses:
A. Hives*
B. Dermatitis Duhring
C. Tertiary syphilis
D. Eczema
E. no correct answer
609. Clinical features of psoriasis:
A. Koebner phenomenon*
B. Bubbles
C. Haemorrhagic spots
D. Blisters
E. no correct answer
610. Clinical signs of lichen planus:
A. Vorotnichek Biett
B. Blisters
C. Umblicuous depressions in the center*
D. Bumps
E. no correct answer
611. Means, used locally for the treatment of psoriasis:
A. Paint Castellani
B. Naftalan ointment
C. Benzyl benzoate ointment
D. Salicylic ointment*
E. no correct answer
612. The phenomena characteristic of psoriasis:
A. Stearin spots*
B. Paired elements
C. Wickham
D. Ladies' Heel
E. no correct answer
613. Signs of true polymorphism rash when dermatitis Dühring:
A. Bubbles*
B. blister
C. Tubercle
D. Node
E. no correct answer
614. Clinical varieties of pemphigus:
A. Pustular
B. Vulgar*
C. Exudative
D. Papular
E. no correct answer
615. For the characteristic symptoms of lupus:
A. Erythema +
B. Net Wickham
C. Peeling mukovinos
D. A symptom Kebnera
E. no correct answer
616. For the characteristic symptoms of lupus:
A. Net Wickham
B. Follicular hyperkeratosis*
C. A symptom Kebnera
D. Triad auspices
E. no correct answer
617. Clinical forms of athlete's foot:
A. Psoriasifor
B. Erased
C. Dyshydrotic*
D. Common
E. no correct answer
618. Diseases related to dermatophytes:
A. Trichophytosis*
B. Erythrasma
C. Chromomycosis
D. Impetigo
E. no correct answer
619. Antibiotics are used to treat candidiasis:
A. Macrolides
B. Flunol*
C. Doxycycline
D. Kanamycin
E. no correct answer
620. When artropaticheskom advisable to appoint all of the above, krome
A. Essentiale
B. Antioxidants
C. Mineralocorticoid drugs
D. Nizoral *
E. no correct answer
621. Typical sprinkler elements planus have the following features, besides
A. Preferential localization in the flexor surface of the lower extremities
B. Symptom Pilnova*
C. Waxy luster
D. Wickham grid on the surface of papules
E. no correct answer
622. Monomorphic rash may be at:
A. Psoriasis*
B. Erythema multiforme exudative
C. Microbial eczema
D. Dermatitis Duhring
E. no correct answer
623. Hyperkeratosis is typical for:
A. Ichthyosis*
B. Bullous impetigo
C. sycosis
D. Folliculitis
E. no correct answer
624. A woman 25 years old, married 3 year The examination revealed chronic endocervicitis,
adnexitis. In smears leukocytosis. Gonorrhea and trichomonas are not found. The daughter of 1.5
years of acute vulvovaginal gonorrheal etiology. The correct tactics of the doctor:
A. Designation combined provocations followed by bacteriological and bacterioscopic*
B. research
C. The appointment of antihistamines
D. Antifungals
E. vaccination
625. What type of treatment should be external
choose when moist skin inflammation:
A. ointment
B. Aniline dyes*
C. cream
D. Varnishes
E. no correct answer
626. The patient was diagnosed with anterior urethritis chlamydial infection. Antibiotic a group
assign the patient:
A. Aminoglycosides*
B. Malaria
C. Fluorinated quinolones
D. Ingeneration cephalosporins
E. no correct answer
627. Girls vulvovaginitises trichomonas etiology characterized by all of the above
symptoms besides
A. Cheesy discharge from the genital slit*
B. Infiltrative foci
C. Frequent and painful urination
D. Itching in the vulva
E. no correct answer
628. What type of lotions should choose
for weeping infected surfaces
minute skin:
A. C potassium permanganate*
B. Zinc
C. Lead
D. Tannic
E. no correct answer
629. Diagnostic tests are tests of allergic dermatitis.
A. Balzer
B. Jadassohn
C. Scarification*
D. Minor
E. no correct answer
630. By the frequent complications of atopic dermatitis are all listed, except
A. Hyperthyroidism +
B. A secondary infection in the outbreak of atopic dermatitis
C. Skin atrophy during prolonged treatment
D. Poor sleep, irritability
E. no correct answer
631. Diagnosis of acute prostatitis is based on the results of these studies krome
A. Palpation of the prostate
B. Ultrasound of the prostate
C. Blood chemistry*
D. Analysis of urine 3 servings
E. no correct answer
632. Examination of children with atopic dermatitis is to find out all of the above, except
A. Identify the state immunodeficiency
B. Identify the source of microbial sensitization*
C. Identify other allergenic factors
D. Assessment of endogenous intoxication
E. no correct answer
633. What are the exogenous factors contribute to the development of pyoderma:
A. Gene mutations
B. Hypervitaminosis
C. Hypothermia and hyperthermia*
D. Worm infestation
E. no correct answer
634. What are the endogenous factors contribute to the development of pyoderma:
A. Gipovitaminoz*
B. Eosinophilia
C. hemophilia
D. Violation of the mechanism of division epidermotitis
E. no correct answer
635. In acute and subacute uncomplicated gonorrhea in men and women in the complex
Treatment includes all of the above, krome
A. Antispasmodic
B. Gonovaccine*
C. Sulfonamides
D. Antigoncoccus antibiotics
E. no correct answer
636. Choose the correct tactics of treatment of chronic and torpid gonorrhea infektsii
A. Antihistamines
B. Calcium supplements
C. Detoxification drugs
D. Antibacterial*
E. no correct answer
637. Choose the correct tactics of treatment of chronic and torpid gonorrhea infektsii
A. Antihistamines
B. Calcium supplements
C. Local treatment*
D. Antifungal
E. no correct answer
638. To stafilodermiyam include:
A. Carbuncle*
B. Bullous impetigo
C. Impetigo nail ridges
D. Lichen simplex
E. no correct answer
639. Clinical symptoms of discoid lupus eritomatosis
A. Bubbles
B. Bubbles
C. Atrophy*
D. Pustules
E. no correct answer
640. Koebner phenomenon is observed in the following diseases:
A. leprosy
B. Folliculitis
C. Planus*
D. Allergic dermatitis
E. no correct answer
641. Under what dermatoses rash element is the bubble:
A. psoriasis
B. Pink zoster Gibert
C. pemphigus
D. Herpes zoster*
E. no correct answer
642. The true characteristic of polymorphism:
A. Eczema*
B. Psoriasis
C. Planus
D. Tinea versicolor
E. no correct answer
643. Find the appropriate answers: proliferative morphological Changes
A. Spongiosa,
B. Hyperkeratosis*
C. Vacuolar degeneration
D. Balloon degeneration
E. no correct answer
644. Diffuse hyperkeratosis characteristic of:
A. Pemphigus
B. Ichthyosis*
C. Atopic dermatitis
D. Vitiligo
E. no correct answer
645. What factors contribute to the development of pyoderma:
A. Carbohydrate metabolism*
B. Increased blood clotting
C. Hypervitaminosis
D. exercise stress
E. no correct answer
646. For scabies is characterized by:
A. Polygonal papules with central retraction
B. Follicular hyperkeratosis
C. Paired papules vesicular elements*
D. A symptom of "apple jelly"
E. no correct answer
647. From the ectoderm develop:
A. Hair muscles
B. Vessels of the skin
C. Cover the epidermis*
D. Dermis
E. no correct answer
648. For hidradenitis sup characterized by:
A. Rupioidnye pustules
B. Favourite localization in the armpits*
C. Papules
D. Paired elements
E. no correct answer
649. Specify the kind of clinical mange (form):
A. Erythematous
B. Squamous
C. Chronic
D. Norwegian *
E. no correct answer
650. For mixed pyoderma include:
A. Vulgar impetigo*
B. Ostiofollikulit
C. carbuncle
D. furuncle
E. no correct answer
651. Specify the clinical varieties akantolicheskoy pemphigus:
A. The leaf *
B. Annular
C. Stevens Johnson
D. Epidemic pemphigus newborns
E. no correct answer
652. At what dermatosis morphological element is the primary node:
A. psoriasis
B. Leprosy*
C. Angioedema
D. Hives
E. no correct answer
653. Under what dermatoses main element of the rash is pimple:
A. pemphigus
B. Planus*
C. Pink zoster Gibert
D. Shingles
E. no correct answer
654. Clinical symptoms of discoid lupus erythematosus:
A. Atrophy*
B. Bubbles
C. Bubbles
D. Blister
E. no correct answer
655. Koebner phenomenon is observed in the following diseases:
A. leprosy
B. Folliculitis
C. Psoriasis*
D. Allergic dermatitis
E. no correct answer
656. Under what dermatoses main element of the rash is pimple:
A. Psoriasis*
B. pemphigus
C. shingles
D. Erythrasma
E. no correct answer
657. Under what dermatoses rash element is the bubble:
A. psoriasis
B. Eczema*
C. pemphigus
D. Erythematosus
E. no correct answer
658. At what dermatosis morphological element is the primary node:
A. psoriasis
B. Leishmaniasis skin*
C. Angioedema
D. Hives
E. no correct answer
659. True polymorphism characteristic for
A. Dermatitis Duhring*
B. Psoriasis
C. Planus
D. Tinea versicolor
E. no correct answer
660. Name the atypical form of primary syphiloma:
A. Chancreamygdala*
B. Diffuse pigmentation
C. Fagedenizm
D. Phimosis
E. no correct answer
661. Specify the secondary elements that are the result of transformation of the node:
A. Scar atrophy
B. Complete disappearance*
C. ulcer
D. scar
E. no correct answer
662. Complication chancre:
A. Balanopostitis*
B. impetigo
C. Stafilodermia Bockhart
D. Osteofolliculit
E. no correct answer
663. What drugs are used for photochemotherapy in psoriasis:
A. Acyclovir
B. Puvalen*
C. Nystatin
D. Lamisil
E. no correct answer
664. What type of external therapy is prescribed for the progressive stage of psoriasis
A. corticosteroid ointment*
B. 20% benzyl benzoate
C. Ointment "psoriasin"
D. Method Dem'yanovich
E. no correct answer
665. For secondary syphilis is characterized by:
A. Bumps
B. Roseolous rash*
C. Chancre and polyadenylation
D. Infiltration Gohzingera
E. no correct answer
666. What preparation is effective for genital herpes:
A. Lamisil
B.Trichopolum *
C.Ulkaril
D.Diprospan
E.no correct answer
667. The clinical picture of the acute form of athlete's foot:
A. Redness*
B. Bumps
C. Peel
D. Lichenification
E. no correct answer
668. The clinical picture of infiltrativesuppurative trihofitii:
A. Lichenification
B. Blisters
C. Inflammatory infiltrate*
D. Units
E. no correct answer
669. Clinical symptoms of jock itch:
A. Blisters
B. Units
C. Bumps
D. Red spots*
E. no correct answer
670. The most frequent localization of the surface of the skin candidiasis:
A. Corners of the mouth*
B. Flexor surfaces of the wrists
C. vulva
D. Cuticles
E. no correct answer
671. Fungal diseases of the skin causing:
A. Trihofiton*
B. Corynebacterium
C. Mycobacterium Hansen
D. Mycobacterium Koch
E. no correct answer
672. The main clinical signs of mucosal candidiasis:
A. Mucous papules
B. Bumps
C. White films*
D. Units
E. no correct answer
673. The antibiotics used in the treatment of fungal infections:
A. Linkomitsin
B. penicillin
C. Nizoral *
D. Lindamitsin
E. no correct answer
674. The localization of lesions in scabies:
A. Scalp
B. Nails
C. Palm
D. Interdigital folds of brushes*
E. no correct answer
675. The main clinical signs of scabies:
A. Itching in the evening and at night*
B. Units
C. Acantholysis
D. Blisters
E. no correct answer
676. Drugs used to treat scabies:
A. Iodine tincture
B. Prednisolone ointment
C. Nistatyn ointment
D. 60% sodium thiosulfate and hydrochloric acid 6%*
E. no correct answer
677. morphological elements specific to the clinic uncomplicated scabies:
A. spot
B. Tubercle
C. Abscess
D. Knot*
E. no correct answer
678. In the typical symptom of scabies:
A. Gorchakov Hardy*
B. Besnier Meshcherskiy
C. Nikolsky
D. Of asbestos Hensen
E. no correct answer
679. Pathogens pustular skin diseases:
A. Proteus
B. Tubercle bacillus
C. Aureus*
D. Corynebacterium
E. no correct answer
680. Streptococcus causes of the disease:
A. Red zoster
B. Simple herpes
C. sycosis
D. Impetigo*
E. no correct answer
681. Variety of strep impetigo:
A. Zayed*
B. furuncle
C. Simple herpes
D. Epidemic pemphigus
E. no correct answer
682. The ability to localize boil:
A. Neck*
B. Oral mucosa
C. Lips
D. Soles
E. no correct answer
683. Clinical signs typical of staphylococcal deep pustules:
A. Liquid pus
B. Located on the smooth skin
C. Surrounded by a rim of hyperemia
D. Riddled hair*
E. no correct answer
684. hydradenitis localized:
A. Palm
B. Shin
C. Soles
D. Armpits*
E. no correct answer
685. The main clinical forms of staphylococcal skin lesions:
A. Chromophytosis
B. shingles
C. Ecthyma vulgar
D. Folliculitis*
E. no correct answer
686. Clinical varieties of strep impetigo:
A. Bullous impetigo*
B. sycosis
C. Deep folliculitis
D. Simple herpes
E. no correct answer
687. Complications occurring in patients with the localization of boils on his face:
A. Phlebitis cerebral vessels*
B. Neuritis of the facial nerve
C. endocarditis
D. neuralgia
E. no correct answer
688. Deep streptococcal form:
A. Strep impetigo
B. Vulgar ecthyma*
C. Bullous impetigo
D. Lichen simplex
E. no correct answer
689. The factors that cause toxicoderma:
A. Sulfa drugs*
B. Acid
C. Alkalis
D. Metals (cobalt, nickel, chromium)
E. no correct answer
700. Clinical symptoms of hives:
A. Blister*
B. Node
C. pain
D. Pustule
E. no correct answer
701. stimuli, causing a simple contact dermatitis:
A. Drugs per os
B. Nutritional
C. Physical (high and low temperature)*
D. Psychoemotional
E. no correct answer
702. For atopic dermatitis is characterized by:
A. Bumps
B. Units
C. blister
D. Erythema *
E. no correct answer
703. Symptoms characteristic of true eczema:
A. Blisters
B. Scar atrophy
C. True polymorphism*
D. Bumps
E. no correct answer
704.Dlya eczema is characterized by:
A. Moisture "serous wells"*
B. Bumps
C. Units
D. Gumma
E. no correct answer
705. Clinical manifestations of fresh secondary period of syphilis:
A. Roseola*
B. Bubbles
C. Blisters
D. Extensive warts
E. no correct answer
706. Clinical signs of syphilitic roseola:
A. Itchy
B. Painful
C. Does not merge*
D. Purple
E. no correct answer
707. Clinical signs of tinea versicolor:
A. Defurfuration*
B. Papules
C. Bumps
D. Units
E. no correct answer
708. The source of infection at mikrosporii:
A. Cats, dogs*
B. cattle
C. Rodents
D. Water
E. no correct answer
709. candidiasis affects:
A. The muscular system
B. Follicular unit
C. Skeletal system
D. Mucous*
E. no correct answer
710. The primary elements in the herpes simplex:
A. blister
B. Abscess
C. Bubble*
D. Bundle
E. no correct answer
711. For viral diseases include:
A. Herpes zoster*
B. eczema
C. lupus erythematosus
D. Vulgar sycosis
E. no correct answer
712. For zoster is characterized by:
A. Units
B. Damage along the nerve endings*
C. Blisters
D. Nodules
E. no correct answer
713. Clinical manifestations characteristic of occupational eczema:
A. Bumps
B. Units
C. Moisture *
D. Vegetation
E. no correct answer
714. The bubble is allowed to form:
A. Lihenifikatsiya
B. Vibitsess
C. Erosion*
D. Ulcer
E. no correct answer
715. Have cavity morphological elements:
A. papule
B. blister
C. spot
D. Bubble*
E. no correct answer
716. The rash polymorphic at:
A. Psoriasis
B. Eczema
C. Lichen planus
D. Molluscum Contagiosum*
E. no correct answer
717. Monomorphic rash when dermatoses:
A. Erythema multiforme exudative
B. eczema
C. Dermatitis Duhring
D. Psoriasis*
E. no correct answer
718. Clinical features of psoriasis:
A. The presence of papules*
B. Haemorrhagic spots
C. Bubbles
D. Bumps
E. no correct answer
719. Clinical signs of lichen planus:
A. Biett collar
B. umblicuous depressions in the center*
C. Blisters
D. Node
E. no correct answer
720. The main clinical forms of psoriasis:
A. Subacute
B. Atrophic
C. hemorrhagic
D. Vulgar*
E. no correct answer
721. The histological signs characteristic of psoriasis:
A. Spongiosa
B. Acantholysis*
C. Parakeratosis
D. Ballooning degeneration
E. no correct answer
722. Clinical stage course of psoriasis:
A. Subacute
B. Progressive*
C. Acute
D. Chronic
E. no correct answer
723. Clinical varieties of lichen planus:
A. Arthropathic
B. Atrophic*
C. Pemfigoidnaya
D. Papulose
E. no correct answer
724. The most common form of psoriasis with the flow:
A. Autumn
B. Offseason
C. Winter*
D. General
E. no correct answer
725. Psoriasis must be differentiated from:
A. Pemphigus
B. Eczema
C. Warts
D. Lichen planus*
E. no correct answer
726. The phenomena characteristic of psoriasis:
A. Blood dew
B.Besnier Meshcherskiy
C.Nikolsky
D.Of asbestos Hansen
727. For the characteristic symptoms of lupus:
A.Net Wickham
B.Follicular hyperkeratosis
C.Depigmentation
D.A symptom Kebnera
E. No correct answer
728. Stages of scleroderma:
A. peeling
B.Pack*
C.Hyperkeratosis
D.Parakeratosis
E.No correct answer
729. Drugs used to treat scleroderma:
A.a nicotinic acid
B.+ Lidasa
C.Erythromycin
D.Biyohinol
E.No correct answer
730 Connective tissue diseases:
A.+ Lupus erythematosus
B.psoriasis
C.Allergic vasculitis
D.Lichen planus
E.No correct answer
731. Clinical varieties of lupus:
A.Seborrheic
B.Discoid*
C.Osteoarticular
D.Muscle
E.No correct answer
732. The period of syphilis:
A.acute*
B.Subacute
C.Primary
D.All right
E.No correct answer
733. The main clinical forms of leprosy:
A.Lepromatous
B.Kollikvation*
C.Acute
D.Chronic
E. No correct answer
734. For tinea versicolor is characterized by:
A.Sample Balzer*
B.A symptom Besnier Meshcherskiy
C.Green glow of the luminescence
D.roseola
E No correct answer
735. Clinical forms of athlete's foot:
A.Erased*
B.Intertriginosis
C.Chronic
D.Subacute
E. no correct answer
736. Diseases related to dermatophytes:
A.Athlete's groin*
B.Psoriasis
C.Lichen simplex
D.Erythrasma
E. No correct answer
737. What kind of pustules:
A.Folliculitis
B.Telangiectasia*
C.excoriation
D.vesicle
E.No correct answer
738. What kind of spots:
A.Roseola*
B.Acne
C.papule
D.Conflict
E.No correct answer
739. The lesion with indistinct borders, the size of 45 cm, pink, not protruding above the skin.
When the pressuresensitive element disappears.
A.Specify the morphological element:
B.Spot*
C.Bundle
D.blister
E.Tubercle
740. The spot size of 45 cm in diameter. Specify the kind of morphological elements:
A.roseola
B.Erythema*
C.purpura
D.Vibitsecc
E.No correct answer
741. What dermatological manifestations may be markers of AIDS:
A.Dermatitis herpetiformis Duhring*
B.Neurofibromatosis
C.Lichen planus
D. KS
E.No correct answer
742. At the stage of established AIDS observed:
A.SarkomaKaposhi*
B.gonorrhea
C.Dermatitis Duhring
D.Triad Hutchinson
E.No correct answer
743. The patient on the skin surface of the forearm extensor bespolostnoe proliferative
precipitation measuring 0.5 0,7sm diameter, thick consistency, raised above the skin. The
elements are clear lines, flat, dull shape.
Specify the morphological element:
A.Knot*
B.Vial
C.blister
D.bubble
E.No correct answer
744. What morphological element has the ephemeral:
A.spot
B.Node
C.Blister
D.Tubercle
E.No correct answer
745. In place of the node is allowed:
A.ulcer
B.Pigmentation
C.crack
D.scar
E.No correct answer
746. Which skin diseases characterized by monomorphic papuleznayasyp:
A.Psoriasis*
B.pemphigus
C.Vulgar sycosis
D.herpes
E.No correct answer
747. papulosus proliferative cavity element towering above the skin, often ulcerate and end up
with scar or scar atrophy
Find erroneous statements in this definition:
A.Ulcerate
B.Proliferative*
C.Cavity
D.Ends with scarring
E.No correct answer
748. Find the right signs of tubercle:
A.Proliferative
B.nonelevated
C.nonfuyt
D.Ends without scar pigmentation
E.No correct answer
749. The tubercle is:
A.Infectious granuloma occurring in the reticular dermis*
B.Cavity element occurs in the epidermis
C.acute inflammatory edema of the papillary dermis
D.Acute inflammatory cavity element of the papillary dermis
E.No correct answer
750. When any skin diseases occur tubercles:
A.psoriasis*
B.Secondary syphilis
C.Dermatitis Duhring
D.Lupus
E. No correct answer
751. The patient in the face and chest are abdominal elements protruding above the skin filled
with serous fluid, the size of 0.30.5 cm in diameter.
Identify morphological element:
A. abscess
B. vial *
C. papule
D. bubble
E.No correct answer
752. In the formation of the bubble histologically observed:
A. vakuolnaya degeneration *
B. acanthosis
C. granulosa
D. akantolizis
E. No correct answer
753. The patient in the forearm skin lesion size of 5x6 cm, bright red, do not rise above the level
of the skin. Specify the nature of the morphological elements:
A. bump
B. knot
C. Spot *
D. Lichenification
E. No correct answer
754. The size of 5x6 cm, towering above the skin, bright red color:
A. roseola
B. erythema*
C. purpura
D. Lentigo
E. No correct answer
755. Specify the characteristic morphological element sulfa erythema:
A. spot *
B. knot
C. Node
D. Bump
E. No correct answer
756. The patient in the back of the neck lesion size 5x6 cm. The neck skin is dry, rough, thick.
Skin pattern in the outbreak is significantly strengthene
Specify the nature of the morphological elements:
A. ecscoriacia
B. vegetation
C. lichenification*
D. scar atrophy
E. No correct answer
757. Lichenification character for:
A. shingles
B. simple bubble stripping
C. pruriginous eczema
D. neurodermatitis*
E. No correct answer
758. Lichenification appears:
primary
A. a second time at the confluence papular elements*
B. primarily due to prolonged irritation of the skin when scratching
C. primarily as a result of burns
D. guticar
E. No correct answer
759. The patient in the tibia lesion, accompanied by a deep defect in the epidermis and dermis, the
size of
23 cm oval, smooth edges, with seropurulent discharg
Specify the nature of the morphological elements:
A. Erosion
B. crack
C. ulcer *
D. excoriation
E. No correct answer
760. In place of the ulcer will been resolved:
A. scar *
B. pigmentation
C. excoriation
D. lichenification
E. No correct answer
761. In what appears ulcer skin diseases:
A. tertiary syphilis*
B. lichen planus
C. scabies
D. psoriasis
E. No correct answer
762. The patient skin rashes torso multiple pale pink in color, the size of 0.30.5 cm, towering
above the skin, disappearing when presse
Specify the kind of morphological elements:
A. roseola*
B. erythema
C. petechiae
D. ecchymosis
E. No correct answer
763. Specify the nature of roseola:
A. vascular hemorrhagic spot
B. primary lentigo
C. inflammatory vascular spot*
D. lenticular papule
E. No correct answer
764. What are the diseases of the skin accompanied by roseola:
A. erythema multiforme exudative
B. secondary syphilis *
C. psoriasis
D. planus
E. No correct answer
765. The patient in the skin of the trunk multiple lesions rounded shapes, the size of a penny coin
2, proliferative, protruding above the skin surface, dens
Specify the nature of the morphological elements:
A. nummular papules*
B. lenticular papules
C. Spot
D. Blisters
E. No correct answer
766. papules located:
A. AV epidermis with the capture of the papillary dermis
B. the reticular dermis
C. Surface only within the epidermis*
D. in the subcutaneous fat
E. No correct answer
767. The patient in the red border of lips abdominal accumulation of small items that contain
serous exudat
Specify the nature of the morphological elements:
A. bull
B. vesicles *
C. urticaria
D. pustules
E. No correct answer
768. To what morphological elements include vesicle:
A. primary effusion*
B. primary proliferative
C. secondary exudative
D. specific granuloma
E. No correct answer
769. At what diseases of the skin may appear vesicle:
A. lichen planus
B. microsporia
C. leprosy
D. simple herpes *
E. No correct answer
770. At the site revealed the remains of vesicles:
A. erosion *
B. excoriation
C. scar
D. ulcer
E. No correct answer
771. Histological examination of the preparation of the skin revealed a significant thickening of
the granular layer of the epidermis.
Specify the nature of the disease process:
A vacuolar degeneration
A. granulёz *
B. acantholysis
C. hyperkeratosis
D. ekcimatosys
E. No correct answer
772. The granular layer comprises:
A. 24 series *
B. 510ryadov
C. 56 rows of cells
D. 1012 rows of cells
E. No correct answer
773. The cytoplasm of the cells of the granular layer comprises:
A. eleidin
B. keratin
C. keratohyalin *
D. net correct answer
E. No correct answer
774. In the study of skin biopsy revealed fusion of epithelial intercellular bridges in the spinous
layer of cells found epidermisa Ttsank
Specify the nature of the disease process:
A. parakeratosis
B. papillomatosis
C. acanthosis
D. acantholysis *
E. No correct answer
775. The spinous layer composed of:
A. 36 rows of cells*
B. 1018 series
C. 1215 series
D. 2025 series
E. No correct answer
776. The skin biopsy revealed a violation of keratinization epidermotsitov, thickening of the horny
layer without structural changes in the cells.
Specify the kind of pathological process:
A. parakeratosis
B. gipergranulamatosys
C. hyperkeratosis *
D. spongiosa
E. No correct answer
777. The cells of the stratum corneum are in the form:
A. ball
B. Records *
C. cylinder
D. cube
E. No correct answer
778. The cells of the stratum corneum are filled:
A. eleidin
B. keratin *
C. keratohyalin
D. glycin
E. no correct answer
779. What disease is characterized by a true polymorphism:
A. eczema *
B. pemphigus
C. vulgarnogo sycosis
D. shingles
E. No correct answer
780. The biopsy of the skin marked intracellular edema in the Malpighian layer with signs of
pyknosis of nucle
Specify the nature of pathological changes
A. spongiosa*
B. vacuolar degeneration
C. akantolizis
D. acanthosis
E. No correct answer
781. The Malpighian layer includes:
A. basal layer *
B. mesh layer
C. papillary layer
D. shiny coat
E. No correct answer
782. What disease is characterized by itching night:
A. Eczema
B. Real vesiculosys
C. scabies *
D. dermatitis Duhring
E. No correct answer
783. Patient urticaria rashes on the body, accompanied by itching.
Specify the kind of morphological elements:
A. papule
B. blister *
C. Bubble
D. Bubble
E. No correct answer
784. Specify the nature of the blister:
A. primary effusion*
B. primary proliferativny
C. secondary exudative
D. secondary proliferative
E. No correct answer
785. The child of 1.5 years in the congested skin face multiple microvesicles.
Otmechaetsyamoknutie bothers severe itching.
Select the method of external treatment:
A. 2% solution of brilliant green*
B. sernayamaz
C. ichtiol ointment
D. salicylic ointment
E. No correct answer
786. Variety of eczema:
A. children's eczema *
B. strofulyus
C. vulgar impetigo
D. strep impetigo
E. No correct answer
787. The appearance of eczema in children is usually preceded by:
A. exudative diathesis *
B. Scabies
C. pneumonia
D. anemia
E. No correct answer
788. Patient neurodermatitis by mechanical stimulation of the skin with a blunt object appears
neurovascular reaction in the form of a white lin
What do you call such a reaction:
A. dermographism *
B. "goose bumps"
C. erythema "embarrassment"
D. Strengthening kozhnogorisunka
E. No correct answer
789. At what diseases of the skin, a change in dermographism:
A. versicolor Gibert
B. psoriasis
C. neurodermatitis *
D. scabies
E. No correct answer
790. The patient in the skin of the trunk, limbs, multiple milliarnye, lenticular, numullyarnys
papules, plaques, covered by silvery scales.
Determine the nature of the rash:
A. monomorphic, proliferative*
B. monomorphic, exudative
C. monomorphic, urticaria
D. polymorphic, exudative
E. No correct answer
791. For the diagnosis of monomorphic proliferative characteristic rash:
A. urticaria
B. Scabies
C. psoriasis *
D. pyoderma
E. No correct answer
792. To clarify the diagnosis of psoriasis is used:
A. sample Minor
B. triad auspices*
C. Sample Balzer
D. Sample Jadassohn
E. No correct answer
793. The patient lichen planus rash appeared after severe stress, fright
What kind of treatment it is advisable to appoint a general:
A. suggestive therapy*
B. cytostatics
C. fungicidal
D. broadspectrum antibiotics
E. No correct answer
794. Specify the nature of lesions in lichen planus:
A. polimorfnaya
B. monomorphic Bugorkova
C. true polymorphic
D. monomorphic papular*
E. No correct answer
795. Specify the characteristic color of lesions in lichen planus:
A. yellowish brown
B. red with violet hue *
C. pinkred
D. dark brown
E. No correct answer
796. At the 2yearold child on the face with a loose tire phlyctenas, seropurulent exudate, erosion,
crusts. Diagnosis:
A. strep impetigo *
B. vulgar sycosis
C. hydradenitis
D. psevdofurunkulez Finger.
E. No correct answer
797. When strep impetigo affected:
A. smooth leather *
B. apocrine sweat glands
C. eccrine sweat glands
D. hair follicles
E. No correct answer
798. strep impetigo is more common:
A. male
B. elderly
C. Children *
D. Women
E. No correct answer
799. A patient of 22 years on the forearm painful knot the size of a hazelnut coneshaped with a
necrotic cor A presumptive diagnosis:
A. Gunma
B. carbuncle
C. furunkul *
D. turniol
E. No correct answer
800. The causative agent is furunkula:
A. aureus *
B. mushroom
C. virus
D. pallidum
E. No correct answer
801. The child in the skin of the cheeks, legs whitish spots, rounded, with abundant
melkoplastinchatym peeling. The scraping the surface of foci of fungi were foun Sample Balzer
negativ Your presumptive diagnosis:
A. simple zoster*
B. surface trichophytosis
C. microsporia
D. multicolored lichen
E. No correct answer
802. Those who often suffer from lichen simplex:
A. Adults
B. Women
C. male
D. Children *
E. No correct answer
803. Specify the pathogen simply depriving:
A. mite
B. aureus
C. Streptococcus*
D. Virus
E. No correct answer
804. With what diseases must be differentiated lichen simplex:
A. Vitiligo
B. strofulyus
C. versicolor otrubevidpy*
D. simple herpes
E. No correct answer
805. Newborn week after birth appeared on the skin of the chest bubbles with serouspurulent
exudate, located on a slightly erythematous background, erosion, crusts.
A. presumptive diagnosis:
B. epidemic pemphigus *
C. acantholytic pemphigus
D. syphilitic pemphigus
E. strep impetigo
F. No correct answer
806. The causative agent of epidemic pemphigus is:
A. hemolytic streptococcus
B. Pseudomonas aeruginosa
C. Staphylococcus aureus*
D. Virus
E. No correct answer
807.Pri epidemic pemphigus source of infection are:
A. medical staff *
B. fathers of newborn
C. domestic cats
D. domestic dog
E. No correct answer
808. epidemic pemphigus often suffer
A. Adults
B. Women
C. male
D. newborns *
E. No correct answer
809. A newborn skin on the neck, back multiple nodes reddishbluish color, the size of a pea,
palpation determined fluctuation. From penetrated nodes semiliquid purulenthemorrhagic content.
A presumptive diagnosis:
A. psevdofurunkulez Finger*
B. hydradenitis
C. infant eczema
D. Bullous impetigo
810. What factors contribute to the development psevdofurunkuleza Finger:
A. overeating
B. breast feeding
C. frequent bathing
D. overheating *
E. No correct answer
811. The patient in the left axilla painful, tight, welded assemblies undulating skin, reddishbluish
color, with a fluctuation. Necrotic core is missing. A presumptive diagnosis:
A. hydradenitis *
B. kollikvativnyytuberkulez
C. erythrasma
D. abrasions
E. No correct answer
812. Which group of diseases is hidradenitis:
A. streptococcal
B. tinea
C. stafilodermia*
D. Collagen
E. No correct answer
813. When gidradenitis injured:
A. hair follicle
B. sebaceous glands
C. smooth skin
D. apocrine sweat glands*
E. No correct answer
814. The average time flow gidradenita:
A. 1015 days *
B. 23 months
C. b months
D. 57nedel
E. No correct answer
815. The patient on the skin of the penis size of the ulcer 2x2 cm, round shape, with dense edges,
purulent discharge, painful infiltrate the base beyond the borders of ulcers. Wasserman negativ
A. presumptive diagnosis:
B. genital herpes
C. gonorrhea
D. shankriform pyoderma*
E. chancre
F. No correct answer
816. The causative agent is shankriformnoy pyoderma:
A. pale treponema
B. lichenification
C. Staphylococcus aureus *
D. Mite
E. No correct answer
817. When shankriformnoy pyoderma in place allowed the element remains:
A. atrophic scar *
B. vegetation
C. lichenification
D. mosaic scar
E. No correct answer
818. A woman of 40 years on the skin of the chin, nasolabial folds against the backdrop of
reddened skin are infiltrated pustules, telangiectasi Hypoacid suffers from gastritis. A presumptive
diagnosis:
A. photodermatitis
B. shancriform pyoderma
C. rosacea *
D. erythematosus
E. No correct answer
819. What factors are important in the etiology of rozovyx ugrey:
A. angioneurosis lowered vascular tone*
B. streptococci
C. itch mites
D. fungus
E. No correct answer
820. Distinguish following clinical variants rozovyx ugrey:
A. lichenoid
B. papulespustular
C. erythematous *
D. warty
E. No correct answer
821. With what diseases is carried diff. Diagnosis rozovyx ugrey:
A. perioral dermatitis
B. psoriasis
C. simple herpes
D. discoid lupus erythematosus *
E. No correct answer
822. The patient 30 years on the skin of the chin, cheeks multiple folliculitis, sycosis, clusters of
offyellow purulent crusts. The process recurs. A presumptive diagnosis:
A. vulgar sycosis *
B. acne vulgaris
C. Chronic ulcerative pyoderma
D. Psoriasis
E.
823. Which group No correct answer of diseases is vulgar sycosis:
A. streptoderma
B. ringworm
C. stafilodermiya *
D. collagen
E. No correct answer
824. The etiopathogenesis vulgar sycosis matters:
A. neuroendocrine disorders
B. pyogenic staphylococci*
C. chesotochnye mites
D. overheating
E. No correct answer
825. The woman on the skin of the chest, abdomen, multiple excoriations, paired nodules,
pustules, linear scratching, intense itching, worse at night.
A. presumptive diagnosis:
B. scabies *
C. eczema
D. nodular pruritus
E. strofulyus
826. Specify the most common complication of scabies:
A. regional adenitis
B. neuralgia
C. pyoderma*
D. psychosis
E. No correct answer
827. A man 23 years in the pubic area, the penis, the inner thighs papules, pustules, purulent crust
erosion. Worried itching noted symptom GorchakovHard
A. presumptive diagnosis:
B. chancroid
C. scabies *
D. Lipschutz's disease
E. vulgar impetigo
F. No correct answer
828. Some other symptoms can be symptoms of scabies:
A. itchy skin only during the day
B. linear layout
C. bunching rash
D. pairing elements*
E. No correct answer
829. What are the laboratory methods should be carried out with scabies:
A. scraping on the itch mite*
B. blood tests eozonophili
C. urinalysis
D. Research on blood sugar
E. No correct answer
830. With what diseases is carried diff. diagnostics, scabies:
A. Shingles
B. lichen planus
C. syndrom SenirAschner
D. pruritus *
E. No correct answer
831. The patient was 25 years old, Single The diagnosis of scabies.
What laboratory studies should be included in the plan of the survey:
A. finding of scabies mite*
B. research on LE cells
C. Analysis on akantolys
D. Analysis of a bullock Borowski
E. No correct answer
832. The female scabies mite gets:
A. mesh layer in the dermis
B. horny layer of the epidermis*
C. in the subcutaneous fat
D. in the basal layer of the epidermis
E. No correct answer
833. The incubation period for scabies is:
A. 14 weeks *
B. 23 months
C. 2448 hours
D. 6 8 months
E. No correct answer
834. The fertilized female scabies mite lays:
A. B do100yaits
B. 1000 eggs
C. to 50 eggs *
D. to 200yaits
E. No correct answer
835. A patient with scabies after application of 33% sulfur ointment on the skin of the trunk
appeared :
Diagnose:
A. The contact allergic dermatitis. *
B. atopic dermatitis
C. toxicodermiya
D. erythema multiforme exudative
E. No correct answer
836. Specify drugs for the treatment of contact allergic dermatitis:
A. 1% ointment ichtiol
B. 0.1% calcium gluconate
C. Ampicillin
D. suprastin *
E. No correct answer
837. Specify the term applying sulfur ointment in the treatment of scabies:
A. 12 days
B. 4 5 days *
C. 8 days
D. 7 days
E. No correct answer
838. The varieties of scabies include:
A. dishydrotic
B. Dutch
C. Rural
D. Norwegian*
E. No correct answer
839. The patient skin spots round shape the size of 56 mm with a slight small laminar peeling.
When lubricating iodine stains darken.
Diagnosis:
A. multicolored shingles*
B. allergic dermatit
C. Dermatitis herpetiformis Duhring
D. Iododerma
E. No correct answer
840 Multicolored zoster is more common in persons:
A. pulmonary tuberculosis*
B. podagra
C. hypertensive disease
D. On fat seborroea
E. No correct answer
841. What other symptoms are characteristic of multicolored lichen:
A. true leucoderma
B. no subjective sensations
C. yellowishbrown spots *
D. papulosus rash
E. No correct answer
842. Select the drugs to treat multicolored lichen:
A. Nizoral *
B. 20% benzyl benzoate
C. tselestoderm
D. erythromycin
E. No correct answer
843. A patient of 56 years on the skin in the groin creases clearly limited noninflammatory spot
brickred color. There sweating.
Diagnosis:
A. candidiasis
B. limited neurodermatitis
C. erythrasma*
D. chromophytosis
E. No correct answer
844. What is important in the etiology of erythrasma:
A. neurotropic virus
B. Corynebacterium*
C. Streptococcus
D. Red trihofiton
E. No correct answer
845. The patient 23 years in the interdigital gaps feet maceration, fracture fragments of the
epidermis around the edges. The nails of the thumbs of both feet are yellow, dim, crumbl
Diagnosis:
A. mycosis fungoides
B. aktinomikoz
C. mycetoma of the foot
D. athlete's foot, onychomycosis. *
E. No correct answer
846. Specify the clinical variety of tinea pedis, onychomycosis:
A. intertriginosys *
B. dishidrotic
C. microbial
D. exudative
E. No correct answer
847. What research is needed to confirm the diagnosis of tinea pedis, onychomycosis:
A. microscopic examination of pathological material*
B. crop on Wednesday Saburo
C. CBC
D. urinalysis.
E. No correct answer
848. The pathology is characterized by a nail in the tinea pedis, onychomycosis:
A. 12 defeat of nail plastinoktolko toes*
B. defeat nail plates 34, 45 toes
C. lack of destruction of the nail plate brushes
D. losing all nail plates
E. No correct answer
849. A patient with diabetes at the head of the penis itchy erythematous border erosion macerated
epidermis Worried itching, burning
Diagnosis:
A. esthiomenous chancre
B. contact dermatitis
C. candida balanoposthitis *
D. genital herpes
E. No correct answer
850. What research is needed to confirm the diagnosis Candida balanoposthitis:
A. study on cell acantholytic
B. iodine sample Jadassohn
C. Research on pale treponema
D. microscopic study on yeasts*
E. No correct answer
851. What else could be affected by Candida balanoposthitis:
A. The corners of the mouth
B. interdigital folds
C. oral mucosa *
D. scalp
E. No correct answer
852. Select the drugs to treat Candida balanoposthitis:
A. Nizoral
B. kenolog
C. Diflucan *
D. Nystatin
E. No correct answer
853. The patient 10 years on the scalp rounded center 2x2 cm. Hair broken at 68 mm at the base
ofmufflike Case consists.
Diagnosis:
A. microsporia*
B. alopecia baldness
C. seborrhea
D. syphilitic alopecia
E. No correct answer
854. What other symptoms characteristic of microsporia:
A. BTE increase in lymph nodes
B. emerald glow in the rays of the lamp Wood *
C. Wood in the glow of the lamp is marked shining
D. burning and painful lesions
E. No correct answer
855. Patient For 15 years on the scalp is clearly limited erythematoussquamous foci to be broken
off hair at the level of 68 mm. Under Wood's lampgreen glow.
Diagnosis:
A. trihofitya
B. favus
C. psoriasis
D. microsporia *
E. No correct answer
856. Specify a possible source of contamination at mikrosporia:
A. sick person
B. cats and dogs *
C. Cattle
D. steppe rodents
E. No correct answer
857. What is striking when microsporia:
A. smooth leather *
B. organs
C. bones
D. nails
E. No correct answer
858 Select treatments for microsporia:
A. 610% sulfurtar ointment
B. griseofulvin forte*
C. kloforan
D. acyclovir
E. No correct answer
859. patient diagnosed as athlete's foot, dishydrotic form.
What are the clinical signs are characteristic of this disease:
A. localization in the arch of the foot*
B. formation of bubbles and bubble multi
C. the appearance of paired vesiclespapular elements
D. groin limfadenit
E. No correct answer
860. Where infection occurs most often in the athlete's foot, disgidroticheskoy form:
A. in the treatment room
B. bedside
C. At the beach
D. in the pool, saun *
E. No correct answer
861. Who suffers most athlete's foot, dishydrotic form:
A. athletes *
B. sellers
C. builders
D. vets
E. No correct answer
862. With what diseases is carried diff. diagnosis of athlete's foot, dishydrotic form:
A. palmoplantar pustular psoriasis
B. lichen planus
C. dishydrotic eczema*
D. dermatitis Duhring
E. No correct answer
863. The child of 9 years at the head in the neck, there are two large pockets rezkoocherchennyh
infiltrated the size of 5x6 cm, covered with a large number of purulent rich crusts. After removing
the crusts noted pus from each follicle individuall The patient noted an increase in temperature
headache, increase in cervical lymph nodes.
A. presumptive diagnosis:
B. abrasions
C. infiltrative suppurative trichophytosis *
D. microsporia
E. pustular psoriasis
864. Who is most often the source of infection at the infiltrativesuppurative trihofitia:
A. cattle*
B. cats, dogs
C. steppe rodents
D. Rat
E. No correct answer
865. Which symptom characteristic of infiltrativesuppurative trihofitia:
A. fish eggs
B. ladies heel
C. honeycomb*
D. apple jelly
E. No correct answer
866. What research is needed to infiltrativesuppurative trihofitiia
A. microscopy of hair*
B. crop on Wednesday Saburo
C. Skin biopsy histology
D. Research at Wood's lamp
E. No correct answer
867. A child 6 years of age on the skin in the shoulder, chest and back are eritematosquamous
lesions proper size 2x2,3x3 cm roundshaped lesions with clear boundaries, the edges of which are
raised roll shaped and they can see small papules vesicular rashes, scales and position crusts.
A. presumptive diagnosis:
B. focal neurodermatitis
C. microbial eczema
D. psoriasis
E. microsporia smooth leather*
868. What is striking when microsporia smooth skin:
A. nails
B. hair *
C. internal organs
D. mucous eye
E. No correct answer
869. In an infant with an inflammation of the oral mucosa diagnosed as candidiasis of the oral
mucosa (thrush).
What features are typical for this disease:
A. superficial erosion in the lesions
B. "cheesy" raid
C. whitish film can be easily removed*
D. ulcerated lesions
E. No correct answer
870. What can be combined lesion of the oral mucosa for candidiasis:
A. defeat of the cornea
B. defeat corners of the mouth
C. defeat red border
D. losing the language *
E. No correct answer
871. What are the allergic rashes with candidiasis of the oral mucosa (thrush):
A. levurid *
B. lentikulid
C. mikrobid
D. epidermofitid
E. No correct answer
872. Select drugs for the treatment of candidiasis of the oral mucosa:
A. 1% solution of methylene blue*
B. prednisolone
C. Ampicillin
D. Deksametozon
E. No correct answer
873. The patient on the body multiple yellowishbrown spot with defurfuration which subjectively
did not bother the patient. Sample Balzer and symptom of "chips" are positiv
Diagnosis:
A. pink zoster Gibert
B. secondary syphilis pigmentosa
C. versicolor tinea*
D. dry streptoderma
E. No correct answer
874. For samples Holds Balzer used:
A. 50% potassium iodide ointment
B. 5% tincture of iodine*
C. A 1% solution of nicotinic acid
D. 5% salicylic acid
E. No correct answer
875. What factors are important in the etiopathogenesis of candidiasis:
A. sweating
B. Corynebacterium
C. streptococci
D. Mushrooms *
E. No correct answer
876. Select drugs for the treatment of candidiasis:
A. mikozolon *
B. tetracycline
C. erythromycin
D. deksametozon
E. No correct answer
877. The patient on the scalp are ocheryellow rind with impression in the center, when removing
crusts visible scar atroph Hair dull, as if "eclipsing" comes "barn" smell.
Diagnosis:
A. trichophytosis
B. favus *
C. erythematosus
D. streptoderma
E. No correct answer
878. Which way happens infection favus:
A. A noncommunicable disease
B. sexual contact
C. airborne droplets
D. in direct contact with the patient*
E. No correct answer
879. What is striking in this pathology:
A. A hair
B. smooth skin
C. nails
D. All of the above *
E. No correct answer
880. Specify the clinical varieties favus:
A. A pustular
B. squamous
C. disgidrotic
D. skutular *
E. No correct answer
881. The child of 7 years diagnosed trichophytosis scalp.
A. What features are typical for this disease:
B. breaking of the hair at the level of 12 mm in the outbreaks*
C. whitish, defurfuration
D. imposition of silverwhite scales
E. Case consists mufflike at the base of the hair
F. No correct answer
882. Specify the clinical varieties trihofitia:
A. A sharp
B. infiltrativesuppurative *
C. disgidrotic
D. pustular
E. No correct answer
883. The patient on the face, forearms and feet are ulcer size 2x2,3x3 cm with uneven scalloped
edges on the bottom of the ulcer has granular granulation as "fish eggs". Ulcers were 6 weeks after
the arrival of Turkmenistan, where he was on a business trip in Ma
Diagnosis:
A. rural type of cutaneous leishmaniasis*
B. abrasions
C. ecthyma
D. tertiary syphilis
E. No correct answer
884. What research is needed to confirm the diagnosis of cutaneous leishmaniasis:
A. study on the pale treponema
B. research on calf Borowski *
C. Wasserman
D. All of the above
E. No correct answer
885. What other clinical signs are characteristic of cutaneous leishmaniasis:
A. symptom of "beads" *
B. symptom of falling through the probe
C. symptom of "fisheye"
D. cimptom ladies Heel
E. No correct answer
886. Select drugs for the treatment of cutaneous leishmaniasis:
A. monomitsin *
B. ampicillin
C. prednisolone
D. Nizoral
E. No correct answer
887. A patient diagnosed as leishmaniasis skin, rural styl
What are the symptoms characteristic of this disease:
A. The presence of ulcers with scalloped edges
B. symptom of "the head of Medusa"
C. ulcers with even steep edges
D. symptom of "fish eggs"*
E. No correct answer
888. Specify the source of infection of cutaneous leishmaniasis:
A. A cat, dog
B. gophers *
C. chickens
D. Bird
E. No correct answer
889. Who is the carrier of cutaneous leishmaniasis:
A. Mosquitoes Phlebotomus genus*
B. flies
C. sick person
D. cockroaches
E. No correct answer
890. The causative agent of cutaneous leishmaniasis is:
A. mycobacteria
B. Leschmania tropica major *
C. Fungus
D. Virus
E. No correct answer
891. A patient diagnosed as leishmaniasis skin, urban styl
What is the incubation period for this disease:
A. 26 months *
B. 23 days
C. 24 hours
D. 5b years
E. No correct answer
892. Enter the characteristics of the skin leyshmanioz, city type:
A. A rapid development pathologic process
B. slow development process*
C. appearing of furunkullike infiltrate at the bite site
D. scar formation after 6 months of infection from moment
E. No correct answer
893. Who is the source of infection in the skin leyshmanioze, city type:
A. A prairie rodents
B. Cattle
C. sick person *
D. cats, dogs
E. No correct answer
894. A patient of 16 years diagnosed metaleyshmaniosys
Specify the clinical manifestations of this disease:
A. The appearance of tubercles on the scars*
B. predominant localization on the extremities
C. on the scars do not appear
D. predominant localization on the face
E. No correct answer
895. What are the symptoms characteristic of metaleyshmanioz:
A. symptom of "apple jelly" *
B. symptom "necklace of Venus"
C. symptom of "thimble"
D. symptom "lady’s heel"
E. No correct answer
896. Specify the prevention metaleyshmanioz:
A. A fight with flies
B. Competition mosquito *
C. control of stray cats and dogs
D. strict isolation of patients in specialized hospitals
E. No correct answer
897. A patient of 22 years on the skin of the face, there are bumps the size of 0.70.8 mm in
diameter, brownred, soft pastry consistenc In place of the allowed elements are marked atrophic
scars resembling tissue paper. In the words of a patient sick since childhoodpresumptive
diagnosis:
A. tertiary syphilis
B. lupus *
C. leprosy
D. planus, warty form
E. No correct answer
898. What are the symptoms characteristic of tuberculosis of the skin:
A. symptom of "apple jelly"
B. symptom of "falling through the probe"*
C. symptom of "thimble"
D. symptom of "fish eggs"
E. No correct answer
899. At the slaughterhouse worker on the skin back of the hand and fingers were bluishred bumps
the size of a pea, on the surface of which there are warty growths. Onsite allowed members
celebrated scar atroph
A. presumptive diagnosis:
B. Syphilis
C. lichen planus
D. psoriasis
E. warty lupus *
900. What is important in the etiology of warty tuberculosis of the skin:
A. contact with sick animals
B. pale treponema
C. mycobacteria Koch*
D. Hansen bacillus
E. No correct answer
901. Select the drugs to treat TB warty skin:
A. rifampicin
B. tubazid *
C. acyclovir
D. orungal
E. No correct answer
902 in patients with active pulmonary tuberculosis in the mucosa of the lower lip has an ulcer 2
cm in diameter, with scalloped edges, a pale red color with grainy uneven bottom, having a
sluggish, gray, granulation, bleed easil At the bottom there are sores around the yellow lumps.
A. A presumptive diagnosis:
B. ulcerative tuberculosis*
C. simple herpes
D. shingles
E. StevensJohnson syndrome
903. What other symptoms characteristic of ulcerative tuberculosis:
A. localization have orifices
B. localization only in the mouth
C. painless
D. apple jem *
E. No correct answer
904. In the face of the patient in the brow, forehead, cheeks, nose, there are infiltrates nodes.
Broken facial expressions, has a fierce infection, hair loss is noted in the lateral part of the
eyebrow. There incomplete closure of the eyelids, hoarseness golos The patient marks a change of
pain and temperature sensitivit
A. A presumptive diagnosis:
B. lupus
C. leprosy *
D. tertiary syphilis
E. dermatomyositis
905. What other symptoms characteristic of leprosy:
A. triad Auspitts
B. "lion face*"
C. symptom butterfly
D. «crown of Venus"
E. No correct answer
906. What kind of research is necessary to clarify to leprosy:
A. scraping from the nasal mucosa on the shelves Hansen*
B. sputum on mycobacteria Koch
C. Sample production Jadassohn
D. Thompson production sample
E. No correct answer
907. Specify the clinical kind of leprosy:
A. warty type
B. Tuberculoid type
C. lepromatous type *
D. hemorrhagic type
E. No correct answer
908. The patient 38 years on the skin in the chest area, the lateral surface of the abdomen are
sharply defined plaques size 3x4 cm. On the periphery of the plaques are dense polygonal flat
papules with a purple tint in the central part of the atrophy and depigmentation observed in the
lesions is not the temperature, and pain sensitivity .
presumptive diagnosis:
A. lupus
B. lichen planus
C. leprosy *
D. lupus erythematosus
E. No correct answer
909. Specify the clinical kind of leprosy:
A. City type
B. Rural Type
C. tuberculoid type*
D. milliar type
E. No correct answer
910. A patient diagnosed lepromatous type of lepros
What are the clinical manifestations characteristic of this disease:
A. logoftalm
B. lion face *
C. aksifoidia
D. labyrinthitis
E. No correct answer
911. What is the basis sample Minor:
A. hypofunction of the sweat glands*
B. hyperactivity of the sweat glands
C. hyperactivity of the sebaceous glands
D. hypertrichosis
E. No correct answer
912. What are the diseases carried diff. diagnosis of leprosy:
A. with all of the *
B. tertiary syphilis
C. lupus and multicolored shingles
D. vitiligo
E. No correct answer
913. Enter the result lepramin sample at lepromatous type of leprosy:
A. negative *
B. Positive
C. The early positive
D. Late positive
E. No correct answer
914. On the skin of the perianal area warty growths like "color cabbage" on a narrow basis. PB
negative cells Ttsanka absent. Diagnosis:
A. genital warts*
B. extensive warts
C. warty lichen planus
D. pseudosifilis papular
915. Which group of diseases is genital warts:
A. buleznym dermatoses
B. fungal infections
C. viral dermatoses*
D. vasculitis
E. No correct answer
916. Manifestations sharp ended warts in men most commonly are located:
A. on the inner layer of the foreskin*
B. around the anus
C. In language
D. Groin
E. No correct answer
917. What are the means used for the treatment of genital warts sharp ended:
A. flutsinar
B. selestoderm
C. solkoseril
D. solkoderm *
E. No correct answer
918. A child of 5 years on the face, neck, back of hands on the size of the nodules are 57mm in
diameter with a pearlgray color, hemispherical shape, with "umbilicated" in the center. Subjective
feelings are absent.
Diagnosis:
A. molluscum contagiosum*
B. vulgaris Warts
C. verrucose psoriasis
D. A warts
E. No correct answer
919. Microscopic examination of the contents of nodules when molluscum contagiosum
determine:
A. calf Borowski
B. Hansen sticks
C. shellfish calf *
D. virus
E. No correct answer
920. A man 23 years in the border of the pubic area and abdomen are manifestations
molluscum contagiosum. What rash characteristic of this disease:
A. spherical nodules with umbilicated in the center*
B. polygonal papules with umbilicated
C. villous proliferation
D. conical tubercles
E. No correct answer
921. In what way is transmitted molluscum contagiosum:
A. animals
B. casual contact *
C. After insects
D. transmissible by
E. No correct answer
922. Which group of diseases is molluscum contagiosum:
A. viral dermatoses*
B. dermatozoonoz
C. pyoderma
D. fungal infections
E. No correct answer
923. The etiopathogenesis of molluscum contagiosum matters:
A. Hypersensitivity Koufo
B. hypersensitivity to animal proteins
C. filterable virus *
D. simple
E. No correct answer
924. The patient 56 years old diagnosed with herpes zoster:
What features are typical for this disease:
A. severe itching of the skin appearance
B. appearance of grouped polymorphic eruptions
C. grouped blisters *
D. paired vesiclespapular rash
E. No correct answer
925. Which group of diseases is herpes zoster:
A. viral dermatoses *
B. dermatozoonoz
C. pyoderma
D. fungal infections
E. No correct answer
926. The patient 62 years of age on the skin of the left half of the chest, along the intercostal nerve
has grouped rash with blisters, is located on the hyperemic basis. Rash preceded paroxysmal pain,
burning, fever.
Prepolagaet diagnosis:
A. pemphigus vulgaris
B. dermatitis Duhring
C. shingles *
D. bullous toksikodermia
E. No correct answer
927. Which group of diseases is herpes zoster:
A. viral dermatoses*
B. dermatozoonoz
C. pyoderma
D. fungal infections
E. No correct answer
928. What medications are used to treat herpes zoster:
A. Nizoral
B. ceftriaxone
C. diutsifon
D. acyclovir *
E. No correct answer
929. The causative agent of shingles is:
A. Streptococcus
B. neurotrophic filterable virus*
C. coli Dederleyn
D. papilomatoz human virus
E. No correct answer
930. A child 12 years of age on the skin of the dorsum of the hands are nodular lesions, the size
0,81sm in diameter, significantly protruding above the skin, fleshcolored, dense consistency,
without subjective sensations,
Diagnosis:
A. lichen planus
B. psoriasis
C. vulgaris warts *
D. warty lupus
E. No correct answer
931. Specify the clinical varieties of warts:
A. verrucous
B. infiltrative nagnoitslnye
C. flat, Youth *
D. Palmoplantar
E. No correct answer
932. B etiology of the disease is set to:
A. HSV1
B. Koch's bacillus
C. etiology of the disease is unknown
D. dermotropny filterable virus*
E. No correct answer
933. For the treatment of warts used:
A. solkoderm *
B. 5% ointment tebrofen
C. 5% Naftalan ointment
D. predinisolon ointment
E. No correct answer
934. The patient 35 years after supercooling on the red border of lips there was redness, swelling.
After 2 days, the rash appeared grouped fine bubble filled with serous flui Worried tingling and
burning.
offered diagnosis:
A. simple herpes*
B. multiforme exudative erythema
C. dishydrotic eczema
D. Stevens Johnson syndrome
E. No correct answer
935. Onsite allowed rash herpes remains:
A. atrophic scar
B. mosaic scar
F. dishydrotic eczema
G. Stevens Johnson syndrome
C. No correct answer
936. What is important in the etiology of herpes:
A. Human Papilloma Virus
B. HSV1 *
C. retrovirus
D. cytomegalovirus
E. No correct answer
937. For the treatment of herpes used:
A. 3% oxolinic ointment
B. Zovirax *
C. solkoderm
D. tselestoderm
E. No correct answer
938. A woman of 36 years of casual sex in the mucosa of the labia minora
there are small, the size of millet grain grouped vesicles, which are formed after the resolution of
the erosion of brightred color with polycyclic outlines. The rash of bubbles accompanied by a
feeling of pain, burning sensation, fever do37,8.
presumptive diagnosis:
A. genital herpes*
B. Acute ulcer LyupshittsaChalin
C. erythema multiforme exudative
D. dermatitis Duhring
E. No correct answer
939. In the etiology of genital gerpesa matters:
A. pale treponema
B. retrovirus
C. HSV2 *
D. Cytomegalovirus
E. No correct answer
940. The patient addressed to the doctor with complaints of general malaise, weakness, fatigu
Over the last month weight loss of 10 kg. A history of recurrent herpes. When viewed in the oral
cavity, on the hard palate are solitary, sharply circumscribed, painless, bluishred, testovatoy
consistency tumor formation size of a cherry ston
It noted an increase in BTE and cervical lymph nodes.
presumptive diagnosis:
A. HIV *
B. Pemphigus
C. Tuberculosis
D. StevensJohnson syndrome
E. No correct answer
941. For HIV associated diseases include
A. syndrome SenirAschner
B. Kaposi's sarcoma*
C. Syndrome MelkerssonRosenthal
D. Bazin induratum erythema
E. No correct answer
942. What laboratory studies are needed to clarify the HIV infection:
A. Enzyme Immunoassay
B. research on cells akantolic
C. Wasserman
D. Response immunoblotting*
F. No correct answer
943. Who is the main risk group for HIV infection:
A. staff kitchens
B. addicts *
C. homosexuals
D. Guides
E. No correct answer
944. The patient on the skin of the face there is a lesion, accompanied by a change in color, with
indistinct boundaries, the size of 45 cm, pink, not protruding above the skin. When pressed
elements disappear.
Specify the nature of the morphological elements:
A. vascular inflammatory spot *
B. hemorrhagic spot
C. artificial spot
D. lentigo
E. No correct answer
945. When any skin diseases encountered inflammatory vascular spot:
A. Psoriasis
B. Syphilis *
C. skrofuloderma
D. fixed erythema
E. No correct answer
946. A patient diagnosed with HIV infection:
What research is needed to confirm the diagnosis:
A. Enzyme Immunoassay
B. reaction immunoblotting*
C. Wasserman
D. RIBT
E. No correct answer
947. The causative agent of HIV infection is:
A.renotrophic filtrative virus
B.HSV1
C.HSV2
D.lymphotropic retrovirus *
E.No correct answer
948. The patient in the face and chest are acute inflammatory abdominal rash, protruding above
the skin filled with serous fluid, the size of 0.30.5 cm in diameter.
At a resolution of the elements left behind:
A.pigmentation *
B.scar atrophy
C.ulcers
D.hypertrophic scars
E.No correct answer
949. The bubble can be located:
A. intraepidermal
B. under the stratum corneum*
C. in the reticular dermis
D. in the subcutaneous fat
E. No correct answer
950 in patients with multiple skin lesions of the trunk round shape, the size of a penny coin 2,
proliferative, protruding above the skin surface, dens
Which diseases of the skin rash may appear like:
A. scabies
B. psoriasis *
C. tertiary syphilis
D. pemphigus
E. No correct answer
951. Specify the kind of node:
A. primary proliferative*
B. primary effusion
C. secondary proliferative
D. secondary exudative
E. No correct answer
952. In the study of skin biopsy revealed fusion of epithelial intercellular bridges in the spinous
layer of cells found epidermisa Ttsank
For a skin disease characterized histologically by this:
A. true pemphigus *
B. shingles
C. eczema
D. erythema multiforme exudative
E. No correct answer
953. Cells Ttsanka means:
A. modified cells of the granular layer
B. modified cells of the dermis
C. Changes spinous layer of cells*
D. Modification of the basal layer cells
E. No correct answer
954. Patient urticaria rashes on the body, accompanied by itching.
Urtica this:
A. pleural cavity element
B. pleural cavity element comprising blood
C. discoloration on the limited area as a result of vasodilatation surface
D. choroid plexus
E. Limitation acute inflammatory edema of the papillary dermis*
955. Select the method for topical treatment of hives:
A. lotions furacillin
B. Water talker menthol *
C. Sulfuric ointment
D. Varnish
E. No correct answer
956. Patient neurodermatitis by mechanical stimulation of the skin with a blunt object
neurovascular reaction occurs in the form of a white strip.
When any skin diseases often marked white dermographism:
A. Cutaneous leishmaniasis
B. atopic dermatitis *
C. psoriasis
D. lichen planus
E. No correct answer
957. Which drugs should not be prescribed in the white dermographism:
A. calcium supplements*
B. sulfa drugs
C. corticosteroid hormones
D. Vitamin
E. No correct answer
958. A patient of 22 years on the forearm painful knot the size of a hazelnut coneshaped with a
necrotic cor
After allowing the element will remain in place:
A. vegetation
B. lichenification
C. bridgelike scar
D. atrophic scar *
E. No correct answer
959. Select the drug for the treatment of boils:
A. corticosteroid hormones
B. antifungals
C. Antibiotics
D. Anabolic
E. No correct answer
960. A newborn skin on the neck, back multiple nodes reddishbluish color, the size of a pea,
palpation determined fluctuation. From penetrated nodes semiliquid purulenthemorrhagic content.
The disease occurs as a result of the defeat:
A. pilosebaceous unit
B. excretory ducts and glomeruli eccrine sweat glands*
C. smooth skin
D. follicle
E. No correct answer
961. With what diseases conduct differential diagnostics vesiculopustulose
A. abrasions *
B. strofulus
C. exudative diathesis
D. hydrodeniti
E. No correct answer
962. The woman on the skin of the chest, abdomen, multiple excoriations, nodules, abscesses,
linear hatchlike scratching, intense itching, worse at night.
Some other symptoms are symptoms of the disease:
A. The presence of paired vesiclespapular elements*
B. symptom Hissar
C. symptom BesnierMesherskogo
D. Net Wickham
E. No correct answer
963. Select the drugs to treat scabies:
A. 90% sulfuric ointment
B. 20% salicylic ointment
C. 20% benzyl benzoate *
D. 5% ointment naftolan
E. No correct answer
964. A patient of 56 years on the skin in the groin creases clearly limited noninflammatory spot
brickred color. There sweating.
After the resolution process is in place:
A. depigmentation
B. skin atrophy
C. mosaic scar
D. hyperpigmentation *
E. No correct answer
965. Tattooing refers to:
A. artificial stains *
B. vascular spots
C. hemorrhagic spots
D. pigment spot
E. No correct answer
966. The patient 10 years on the scalp rounded center 2x2 cm. Hair broken at 68 mm at the base of
mufflike Case consists.
A. What is important in the etiology of the disease:
B. yeasts
C. rusty mikrosporum*
D. gipsoid trihosporum
E. trihofiton Shonleyna
F. No correct answer
967. The most vulnerable age for this pathology:
A. 612 years *
B. 4060let
C. infancy
D. 1620 years
E. No correct answer
968. A child 6 years of age on the skin in the shoulder, chest and back are eritematosquamous
lesions proper size 2x2,3x3 cm roundshaped lesions with clear boundaries, the edges of which are
raised rollshaped and they can see small papules vesicular rashes, scales and position crusts.
What matters in the etiopathogenesis of the disease:
A. Age
B. pyogenic flora
C. mushroom mikrosporum *
D. Seasonality
E. No correct answer
969. Select the drugs to treat microsporia:
A. griseofulvin forte *
B. Nizoral
C. Ampicillin
D. Zovirax
E. No correct answer
970. The child of 7 years diagnosed trichophytosis scalp:
What the fungus causes this pathology:
A. trihofiton ektotrix *
B. mikrosporum rusty
C. S. candida
D. None of the above
E. No correct answer
971. What is striking the fungus Trichophyton ektotriks:
A. hair *
B. joints
C. oral mucosa
D. nails
E. No correct answer
972. A patient of 22 years on the skin of the face, there are bumps the size of 0.70.8 mm in
diameter, brownred, soft pastry consistenc In place of the allowed elements are marked atrophic
scars resembling tissue paper. In the words of a patient sick with childhood
What research is needed to confirm the diagnosis:
A. sample of histamine
B. Sample Pirke
C. probaYadassona
D. Mantoux*
E. No correct answer
973. Clinical Pathology is a kind induratum erythema Bazin
A. lupus
B. skrofuloderma
C. lupus *
D. psoriasis
E. No correct answer
974. A patient with active tuberculosis in the lung mucosa of the lower lip has an ulcer the size of
2 cm diameter, with scalloped edges, a pale red color with grainy uneven bottom, having a
sluggish, gray granulation, bleed easil At the bottom there are sores around the yellow lumps.
What research is needed to confirm the diagnosis:
A. direct microscopic research on mycobacteria Koch*
B. crop on flora and sensitivity
C. blood on RV
D. None of the above
E. No correct answer
975. Which way is most often infected with tuberculosis of the skin:
A. lymphogenous
B. hematogenous *
C. airborne
D. transmissible
E. No correct answer
976. The patient 38 years on the breast skin, the side surface of the stomach, there are sharply
defined plaques size 3x4 cm. On the periphery of the plaques are dense polygonal flat papules
with a purple tint; in the central part is marked atrophy and depigmentation. In the lesions and no
thermal pain sensitivit
What tests are used to confirm the diagnosis:
A. sample of histamine
B. Nicotine trial
C. Sample Jadassohn
D. lepromin sample*
F. No correct answer
977. Select the drugs to treat leprosy:
A. penicillin
B. sulfetron
C. DDS *
D. acyclovir.
E. No correct answer
978. A child of 5 years on the face, neck, back of hands on the size of the nodules are 57mm in
diameter with a pearlgray color, hemispherical shape, with "umbilicate pressure" in the center.
Subjective feelings are absent.
What group of diseases is the pathology:
A. neyrodermatosys
B. viral dermatoses*
C. collagen
D. pyoderma
E. No correct answer
979. The patient 56 years old diagnosed with herpes zoster:
How often seen exacerbation of the disease:
A. year in spring and autumn
B. leaves behind immunity, relapses were not observed*
C. exacerbated once in 34 years
D. Each summer
E. No correct answer
980. Shingles can be a manifestation of:
A. Tuberculosis
B. leukemia
C. HIV *
D. Rheumatism
E. No correct answer
981. A woman of 36 years of casual sex in the mucosa of the labia minora were small, the size of
millet grain grouped vesicles, which are formed after the resolution of the erosion of brightred
color with polycyclic outlines. The rash of bubbles accompanied by a feeling of pain, burning,
increased temperature do37,8.
A. .What group of diseases is the pathology:
B. STI: to viral dermatoses*
C. cystic dermatoses
D. etiology of the disease is unknown
E. EpsteinBarr virus
982. For the treatment of dermatoses, viral used:
A. acyclovir *
B. Nizoral
C. amphotericin
D. Lamisil
E. No correct answer
983. In what marked several dermatoses
morphological types of primary elements:
A. dermatitis Duhring*
B. In atopic dermatitis
C. Psoriasis
D. Abrasions
E. No correct answer
984. Under what dermatoses observed monomorphic eruption:
A. A lichen planus*
B. In erythema multiforme exudative
C. On the Microbial eczema
D. A dermatitis Duhring
E. No correct answer
985. Under what dermatoses observed akantolizis:
A. A true pemphigus*
B. The herpes simplex
C. With bullous form strofulus
D. A shingles
E. No correct answer
986. If there is any dermatoses
monomorphic papular rash:
A. A lichen planus*
B. In urticaria
C. With Eczema
D. A pink shingles
E. No correct answer
987. In what marked acanthosis dermatoses:
A. A lichen planus*
B. In turniol
C. With a simple zoster
D. A simple herpes
E. No correct answer
988. In what marked hyper dermatoses
perkeratozis:
A. A wart*
B. The bullous impetigo
C. With Sycosis
D. A folliculitis
E. No correct answer
989. What dosage forms are
the most superficial skin:
A. A lotion*
B. The ointment
C. With Pasta
D. A patch
E. No correct answer
990. What type of treatment should be external
choose when moist skin inflammation:
A. A lotion*
B. The ointment
C. With chatterbox
D. A cream
E. No correct answer
991. What kind of lotion should choose
for weeping infected surfaces
minute skin:
A. A rivanol*
B. The zinc
C. With lead
D. A tannic
E. No correct answer
992. What type of external therapy should be chosen with chronic infiltrative
inflammation of the skin:
A. A 5% Naftalan ointment*
B. The zinc paste
C. With oil mash
D. A water mash
E. No correct answer
993. What external agents have antipruritic property:
A. A menthol*
B. The tar
C. With sulfur
D.A salicylic No correct answer
E.acid
994. What means external treatment
possess the following property:
A. A solution podofillin*
B. The solution of sodium giposulfat
C. With the solution rivanol
D. A solution furatsillin
E. No correct answer
995. For the destruction of the skin are used:
A. A liquid nitrogen*
B. The zinc oxide
C. With potassium permanganate
D. A paint Castellane
E. No correct answer
996. Cryotherapy is used for:
A. A genital warts*
B. In general warts
C. With plaque psoriasis
D. A limited neurodermatitis
E. No correct answer
997. In the study of skin biopsy revealed to the melting of epithelial
A. intercellular bridges in spinous*
B. layer of the epidermis. Cells are found
C. Ttsank.
D. Smash
E. No correct answer
998. Specify the nature of the pathological process:
A. A acantholysis*
B. The granulosa
C. With acanthosis
D. A parakeratosis
E. No correct answer
999. The skin biopsy revealed violation
A. keratinization of epidermocytes*
B. thickening of the horny layer without structural
C. governmental cell changes and thickening of the granular layer
D. dermatosys
E. No correct answer
1000. Specify the kind of pathological process:
A. A hyperkeratosis, gipergranulosis*
B. The parakeratosis, gipergranulosis
C. With hyperkeratosis, acanthosis
D. dermatosys
E. No correct answer
1001. proliferative histomorphological changes include:
A. granulosa, acanthosis, hyperkeratosis *
B. hyperkeratosis, acanthosis, parakeratosis, spongiosis
C. parakeratosis, akantolizis, granulosa
D. acanthosis, parakeratosis, karyorrhexis, hyperkeratosis
E. all answer’s are wrong.
1002. There are no sweat glands in the skin;
A. soles
B. armpits
C. palms
D. red border *
E. all answer’s are wrong.
1003. There are no sebaceous glands in the skin;
A scalp
B. nasolabial triangle
C. Breast
D. palms, soles *
E. all answer’s are wrong.
1004. Which element is a kind of blisters:
A. telangiectasia
B. conflicts *
C. excoriation
D. vesicle
E. all answer’s are wrong.
1005. What kind of spots:
A Rupee
B. Acne
C.. roseola*
D. ecthyma
E. all answer’s are wrong.
1006. What morphological element is characterized by ephemeral:
A blister *
B. pustule
C. papule
D. tuberkulum
E. all answer’s are wrong.
1007. Where are the "tizonievye-sebaceous glands":
A red border
B. Internal sheet of the foreskin *
C. Leather breast
D. perianal
E. all answer’s are wrong.
1008. "Meybonievye" sebaceous glands are located:
A. on the red border of lips
B. genital skin
C. skin ears
D. By the edge of the eyelids *
E. all answer’s are wrong.
1009. In what areas of the skin sebaceous glands open directly into the epidermis:
A. The skin of the nose
B. scalp
C. glans penis *
D. breast skin
E. all answer’s are wrong.
1010. Akantolizis (akantoliz. is:
A thickening of the granular layer
B. Strengthening cell proliferation thorny layer
C. Violations of keratinization
D. Loss of communication between the cells of the thorny layer *
E. all answer’s are wrong.
1011. parakeratosis is incomplete keratinization epidermotsitov accompanied by dystrophy:
A basal layer
B. spinous layer
C. granular layer *
D. papillary layer
E. all answer’s are wrong.
1012. granulosa is:
A thinning of the granular layer
B. thickening of the stratum corneum
C. granular layer thickening *
D. Structural changes in squamous cell carcinomas
E. all answer’s are wrong.
1013. anaplasia is:
A loss of communication between the cells of the thorny layer
B. shrinkage nuclei '
C. granular cytoplasm of cells
D. atypia in tumors of epithelial nuclei *
E. all answer’s are wrong.
1014. Spongiosis is:
A vacuolar degeneration in the Malpighian layer
B. intercellular edema in the stratum corneum
C. cheesy disintegration of tissues
D. intercellular edema in the Malpighian layer *
E. all answer’s are wrong.
1015. Acanthosis not properties:
A. lichen planus
B. scleroderma *
C. neurodermatitis
D. psoriasis,
E. all answer’s are wrong.
1016. Under what dermatoses marked acantholysis:
A. Eczema
B. vulgar pemphigus *
C. contact dermatitis
D. vulgar ecthyma
E. all answer’s are wrong.
1017. Under what dermatoses granulosa notes:
A. lichen planus *
B. turniol
C. lichen simplex
D. simple herpes
E. all answer’s are wrong.
1018. What function is not peculiar to the skin:
A formation of toxins *
B. absorption
C. selection
D. thermoregulation
E. all answer’s are wrong.
1019. What morphological element is not proliferative:
A node
B. blister *
C. knot
D. bump
E. all answer’s are wrong.
1020. What morphological element is not proliferative:
A bundle
B. The Node
C. abscess *
D. bump
E. all answer’s are wrong.
1021. What morphological element is not proliferative:
A bump
B. The Node
C. knot
D. vial *
E. all answer’s are wrong.
1022. What morphological element is not proliferative:
A bundle
B. The Node
C. bubble *
D. bump
E. all answer’s are wrong.
1023. What is exudative morphological element:
A papule
B. nodus
C. tuberkulum
D. vesicle *
E. all answer’s are wrong.
1024. What is exudative morphological element:
A. tuberkulum
B. nodus
C. Bull *
D. papule
E. all answer’s are wrong.
1025. What is exudative morphological element:
A. tuberkulum
B. nodus
C. urtika *
D. papule
E. all answer’s are wrong.
1026. What is exudative morphological element:
A. nodus
B. pustule *
C. tuberkulum
D. papule
E. all answer’s are wrong.
1027. What is exudative morphological element:
A. tuberkulum
B. nodus
C. conflicts *
D. papule
E. all answer’s are wrong.
1028. What is the proliferative morphological element:
A bubble
B. bubble
C. Node *
D. blister
E. all answer’s are wrong.
1029. What is the proliferative morphological element:
A bundle *
B. bubble
C. Bubble
D. blister
E. all answer’s are wrong.
1030. What is the proliferative morphological element:
A bubble
B. bubble
C. tubercle *
D. blister
E. all answer’s are wrong.
1031. What is the proliferative morphological element:
A. urtika
B. vesicle
C. Bull
D. nodus *
E. all answer’s are wrong.
1032. What is the proliferative morphological element:
A vesicle
B. papule *
C. Bull
D. urtika
E. all answer’s are wrong.
1033. What is the proliferative morphological element:
A. Bull
B. vesicle
C. tuberkulum *
D. urtika
E. all answer’s are wrong.
1034 for any symptom of the disease is characterized by Gorachakogo-Hardy:
A. Pemphigus
B. scabies *
C. leprosy
D. skrofuloderma
E. all answer’s are wrong.
1035. The causative agent of scabies is:
A. Flebotomus Papatachi
B. Sarcoptis Hominis *
C. Demodex folliculorum
D. Hansen bacillus
E. all answer’s are wrong.
1036. Angular stomatitis is a variation:
A. stafilodermii
B. streptoderma *
C. candidiasis
D. pemphigus
E. all answer’s are wrong.
1037. Shankriformnaya pyoderma most often begins:
A. with pustules
B. with erosion *
C. with the spot
D. On the bubble
E. all answer’s are wrong.
1038. Which group of dermatoses is Sycosis:
A viral dermatoses
B. disease, sexually transmitted infections
C. pyoderma *
D. cystic dermatosis
E. all answer’s are wrong.
1039. Ritter Eksfolliativny dermatitis is a type of:
A streptococcal
B. stafilodermii *
C. contact dermatitis
D. toxicoderma
E. all answer’s are wrong.
1040. When exfoliative dermatitis Ritter can be detected positive symptom:
A. Uikhsma
B. Nikolsky *
C. Gorchakov-Hardy
D. Sisto
E. all answer’s are wrong.
1041. When psevdofurunkuleze Finger in the pathological process involved:
A. apocrine sweat gland ducts
B. hair follicles
C. sebaceous glands
D. ducts of sweat glands ekkrinnyh *
E. all answer’s are wrong.
1042. What factors contribute to the development of acne:
A frequent colds
B. hyperactivity of the sebaceous glands *
C. Violations of thermoregulation
D. exercise
E. all answer’s are wrong.
1043. With a variety of acne supporting factor are mites Zheleznitsa:
A. abscessed acne
B. acne vulgaris
C. rosacea *
D. spherical acne
E. all answer’s are wrong.
1044. What kind of spots characteristic of the erythematous stage demodekoza:
A. vibitsess
B. petechiae
C. ecchymosis
D. telangiectasia *
E. all answer’s are wrong.
1045. For Rosacea is characterized by the absence of:
A. telangiectasia
B. comedogenic *
C. follicular papules
D. pustules
E. all answer’s are wrong.
1046. What are the morphological features characteristic of hidradenitis sup-:
A thick, welded to the skin painful nodes *
B. superficial pustules
C. firm, mobile, painless nodes
D. blisters
E. all answer’s are wrong.
1047. With the opening of units hidradenitis stands at:
A gummy exudate opalescent
B. creamy pus mixed with blood *
C. cheesy contents
D. serous fluid
E. all answer’s are wrong.
1048. favus called:
A. antropofilnymi fungus *
B. Virus Filter
C. korinobakteriyami
D. Chlamydia
E. all answer’s are wrong.
1049. The typical localization at erythrasma:
A scalp
B. nails
C. inguinal folds *
D. okolonogtevogo roller
E. all answer’s are wrong.
1050. What is a symptom characteristic of infiltrative-suppurative trichophytosis:
A symptom Pincus
B. glow under a Wood's lamp
C. Sample Balzer
D. honeycomb *
E. all answer’s are wrong.
1051. With the laboratory diagnosis of fungal infections is used:
A. ELISA diagnostics
B. microscopy damaged hair and flakes *
C. histological examination
D. PCR diagnostics
E. all answer’s are wrong.
1052. Fungi affect all of the above except:
A. Skin
B. Internal organs
C. Hair *
D. mucous
E. all answer’s are wrong.
1053. The source of infection with the disease Borowski is urban:
A. The wild animals
B. rodents
C. Hymenoptera
D. people *
E. all answer’s are wrong.
1054. The source of infection with the disease Borowski rural type is:
A. rodents *
B. People
C. Hymenoptera
D. wild animals
E. all answer’s are wrong.
1055. Enter the average length for urban cutaneous leishmaniasis:
A. 2 weeks
B. a year *
C. 3-4 months
D. 2-8 weeks
E. all answer’s are wrong.
1056. Enter the average length for cutaneous leishmaniasis rural type:
A few years
B. a year
C. 2-6 months *
D. days
E. all answer’s are wrong.
1057. The source of infection of leprosy is:
A. rodents
B. wild animals
C. pets
D. people *
E. all answer’s are wrong.
1058. The material from which the patient is most often found Mycobacterium leprae:
A. in scrapings from leprosy and nasal mucosa *
B. secret sweat glands
C. urine
D. in sebum
1059. Which viruses cause genital warts:
A. herpesviruses
B. Virus Filter *
C. Herpes Simplex Virus
D. CMV
E. all answer’s are wrong.
1060 Molluscum contagiosum is ...
A pustule
B. erythema
C. papule *
D. vesicle
E. all answer’s are wrong.
1061. Enter variety of true eczema:
A varicose
B. nummulyarnaya
C. seborrheic
D. pruriginous *
E. all answer’s are wrong.
1062. What is a symptom seen with pink zoster Gibert:
A symptom of the "mother" plaque *
B. symptom chips
C. symptom of tissue paper
D. symptom ladies Heel
E. all answer’s are wrong.
1063. Monomorphic rash may be at:
A. erythema multiforme exudative
B. psoriasis *
C. microbial eczema
D. dermatitis Duhring
E. all answer’s are wrong.
1064. Akantolizis observed at:
A. bullous form strofulyusa
B. herpes simplex
C. true pemphigus *
D. shingles
E. all answer’s are wrong.
1065. Monomorphic papular rash occurs when:
A pink shingles
B. urticaria
C. eczema
D. planus *
E. all answer’s are wrong.
1066. Acanthosis typical for:
A. lichen planus *
B. turniol
C. lichen simplex
D. simple herpes
E. all answer’s are wrong.
1067. hyperkeratosis characteristic of:
Bullous impetigo A.
B. warts *
C. Sycosis
D. folliculitis
E. all answer’s are wrong.
1068. The most superficial skin are:
A paste
B. Ointment
C. gadgets *
D. patch
E. all answer’s are wrong.
1069. What type of topical treatment should be selected when moist skin inflammation:
A. Cream
B. Ointment
C. chatterbox
D. gadgets *
E. all answer’s are wrong.
1070. What type of lotions should be selected for the weeping of the infected skin surface:
A. rivanolevuyu *
B. zinc
C. Lead
D. tannic
E. all answer’s are wrong.
1071. What type of external therapy to choose in chronic infiltrative inflammation of the skin:
A zinc paste
B. 5% of Naftalan ointment *
C. the oil mash
D. water mash
E. all answer’s are wrong.
1072. What are the external agents have antipruritic property:
A. Sera
B. tar
C. Menthol *
D. salicylic acid
E. all answer’s are wrong.
1073. Which means external treatment has cauterizing property:
A solution furatsillina
B. A solution of sodium giposulfata
C. The solution rivanola
D. solution podofillina *
E. all answer’s are wrong.
1074. For the destruction of the skin are used:
A liquid nitrogen *
B. zinc oxide
C. Potassium permanganate
D. paint Castellane
E. all answer’s are wrong.
1075. The method is suitable for cryoablation treatment:
A. warts *
B. extensive warts
C. plaque psoriasis
D. Limitation neurodermatitis
E. all answer’s are wrong.
1076. If the skin biopsy revealed fusion of epithelial intercellular bridges in the spinous layer of
the epidermis. Ttsanka detected cells. Specify the nature of the disease process:
A. granulosa
B. acantholysis *
C. acanthosis
D. parakeratosis
E. all answer’s are wrong.
1077. What factors contribute to the development of pyoderma:
A. UVR exposure
B. gene mutations
C. Microfracture *
D. increase in blood pressure
E. all answer’s are wrong.
1078. What factors contribute to the development of pyoderma:
A violation of keratinization
Hemophilia B.
C. Violations of the fission mechanism epidermotsitov
D. diabetes *
E. all answer’s are wrong.
1079. Stafilodermiyam 1079. It includes:
A slit impetigo
B. impetigo nail ridges
C. lichen simplex
D. impetigo Bockhart *
E. all answer’s are wrong.
1080. What pyoderma found only in newborns and infants:
A. vezikulopustulez *
B. hydradenitis
C. pemphigus vulgaris
D. chronic ulcerative pyoderma
E. all answer’s are wrong.
1081. What pustular skin diseases do not occur in the newborn:
A. epidemic pemphigus
B. hydradenitis *
C. vezikulopustulez
D. psevdofurunkulez Finger
E. all answer’s are wrong.
1082. What pustular disease does not occur in adults:
A. turniol
B. boils
C. exfoliative dermatitis Ritter *
D. angular stomatitis
E. all answer’s are wrong.
1083. What are the morphological features characteristic of dermatitis Ritter:
A. bumps, crust
B. papules, blisters
C. crust bubbles *
D. lihenifikatsiya bubbles
E. all answer’s are wrong.
1084. What dermatoses in the first place should be differentiated eksfolliativny dermatitis Ritter:
A children's eczema
Dermatitis herpetiformis Duhring B.
C. bullous form strofulyusa
D. epidermolysis bullosa *
E. all answer’s are wrong.
1085. Multiple abscesses in children causes:
A calf Donovan
B. streptobatsilla Dyukreya-Petersen
C. Chlamydia
D. Staphylococcus aureus *
E. all answer’s are wrong.
1086. Psevdofurunkulez Finger refers to the group:
A. stafilodermy *
B. streptoderma
C. parasitic skin diseases
D. Viral Diseases
E. all answer’s are wrong.
1087. What factors contribute to the development psevdofurunkuleza Finger:
A. overheating of the newborn *
B. old age
C. Violations of the functions of the sebaceous glands
D. endocrinopathies
E. all answer’s are wrong.
1088. What factors contribute to the development of acne:
A. exercise
B. frequent colds
C. Violations of thermoregulation
D. Dysfunction sebaceous glands *
E. all answer’s are wrong.
1089. What kind of spots characteristic of the erythematous stage demodekoza:
A. erythema
B. petechiae, roseola
C. ecchymosis, spots
D. telangiectasia, erythema *
E. all answer’s are wrong.
1090. "pineal" the nose is typical for:
A vulgar lupus
B. acne vulgaris
C. rosacea *
D. discoid lupus
E. all answer’s are wrong.
1091. What factors are important in the development of seborrhea:
A thyroid dysfunction
B. ekkrinnyh sweat gland dysfunction
C. dysfunction apocrine sweat glands
D. Dysfunction sebaceous glands *
E. all answer’s are wrong.
1092. Enter the varieties of oily seborrhea:
A. mukovidnaya
B. viscous
C. liquid *
D. purulent
E. all answer’s are wrong.
1093. Development of dermatosis facilitated by the presence of seborrhea:
A. impetigo
B. trihofitii
C. microsporia
D. baldness *
E. all answer’s are wrong.
1094. For a dermatosis characterized by the presence of comedones:
A. vulgaris *
B. Rosacea
C. Limitations of atopic dermatitis
D. psoriasis
E. all answer’s are wrong.
1095. If any skin disease alopecia can be observed:
A. trichophytosis
B. psoriasis
C. hydradenitis
D. planus *
E. all answer’s are wrong.
1096. What factors contribute most to the development of alopecia areata:
A. The vitamin deficiency
B. calcium deficiency
C. neuroendocrine abnormalities *
D. C deficiency disease
E. all answer’s are wrong.
1097. What factors contribute to the development of hidradenitis sup-:
A hypersensitivity to iodine
B. hyperactivity of the sebaceous glands
C. Competencies skin contamination
D. sweating *
E. all answer’s are wrong.
1098. Enter the location of nodes in the hydradenitis:
A. shin
B. nasal bridge, sky
C. armpits *
D. under the breasts
E. all answer’s are wrong.
1099. Favourite localization rash scabies:
A. Eye
B. interdigital folds of brushes *
C. the skin of the scalp
D. skin of the face, neck,
E. all answer’s are wrong.
1100. For scabies is characterized by:
A peeling
B. polygonal papules
C. sgruppirvannyh small bubbles
D. paired papules vesicular elements *
E. all answer’s are wrong.
1101. Specify a rare localization of scabies rash in children:
A scalp *
B. genital skin
C. Leather breast
D. abdominal skin
E. all answer’s are wrong.
1102. syphilis is
A pale treponema *
Coli B. Hansen
C. Koch's bacillus
D. diplokokki
E. all answer’s are wrong.
1103 is a carrier of leishmaniasis
A. flebotomus papatachi *
B. rodents
C. People
D. Leishmania
E. all answer’s are wrong.
1104. The causative agent of leprosy is:
A. Hansen bacillus *
B. pale treponema
C. Koch's bacillus
D. leshmanii
E. all answer’s are wrong.
1105. lupus causes:
A. diplokokki
Coli B. Hansen
C. mycoplasma
D. Koch's bacillus *
E. all answer’s are wrong.
1106 in the etiology of multi-colored lichen plays a role
A. trihofiton
Coli B. Hansen
C. Koch's bacillus
D. malacesia furfur *
E. all answer’s are wrong.
1107. Enter the transmission path tinea versicolor
A contact-household *
B. sexual
C. airborne
D. alimentary
E. all answer’s are wrong.
1108. wary of any disease is to be a doctor, if the patient is often common recurrent pityriasis
versicolor
A. Psoriasis
B. AIDS *
C. hypertension
D. trichomoniasis
E. all answer’s are wrong.
1109. gidroadenit calls
A. ureaplasma
B. streptococcus
C. aureus *
D. Human Papilloma Virus
E. all answer’s are wrong.
1110. Unlike boil on gidroadenita
A. The presence of redness
B. absence of necrotic rod
C. the presence of pain
D. the presence of necrotic rod *
E. all answer’s are wrong.
1111. Which of the following processes among superficial pyoderma:
A. ostiofollikulit *
B. carbuncle
C. boil
D. gidroadenit
E. all answer’s are wrong.
1112 with the causative agent of the disease is considered to be identical to the herpes zoster virus
A. warts
B. molluscum contagiosum
C. genital warts
D. varicella *
E. all answer’s are wrong.
1113. predisposing factor for the appearance of youthful warts
A. Psoriasis
B. sweating hands *
C. diabetes
D. vitamins
E. all answer’s are wrong.
1114. symptom of Pospelov determined by
A leprosy
B. psoriasis
C. lupus *
D. pemphigus
E. all answer’s are wrong.
1115. The symptom of asbestos-positive at Hansen
A. leshmanioze
B. dermatitis Duhring
C. pemphigus *
D. molluscum contagiosum
E. all answer’s are wrong.
1116 at which the disease is determined Ttsanka cells and a positive sign of Nikolsky
A. Lyell's syndrome
Dermatitis herpetiformis Duhring B.
C. Stevens-Johnson syndrome
D. pemphigus *
E. all answer’s are wrong.
1117. At what disease symptom and not a positive Nikolsky defined cells Ttsanka
A. pemphigus
Dermatitis herpetiformis Duhring B.
C. Stevens-Johnson syndrome
D. Lyell's syndrome *
E. all answer’s are wrong.
1118. erythema multiforme exudative determined symptom
A. Pospelov
B. Nikolsky
C. «bird's eye" *
D. fish eggs
E. all answer’s are wrong.
1119. symptom of "fish eggs" is typical for
A. leishmaniasis *
B. Leprosy
C. lupus
D. syphilis
E. all answer’s are wrong.
1120. iodine sample positive for Jadassohn
A. dermatitis herpetiformis Duhring *
B. pemphigus
C. microsporia
D. multi-colored shingles
E. all answer’s are wrong.
1121. What changes in laboratory assays characteristic of dermatitis herpetiformis Duhring
A blood eosinophilia above 20-30% *
B. anemia
C. leukocytosis
D. Improved bilirubin
E. all answer’s are wrong.
1122. who is ill often dry streptoderma
Starikov
B. Children and women *
C. male
D. No, depending on age
E. all answer’s are wrong.
1123. What layers of the skin are affected in pityriasis versicolor
A basal
B. horny *
C. spinous
D. papillary
E. all answer’s are wrong.
1124. Which of the following layers of the skin to the dermis is
A. spinous
B. basal
C. Net *
D. brilliant
E. all answer’s are wrong.
1125. Which of the following layers of the skin to the dermis is
A. spinous
B. basal
C. papillary *
D. brilliant
E. all answer’s are wrong.
1126. symptom of "necklace of Venus" is observed at
A. psoriasis
B. gonorrhea
C. syphilis *
D. CPL
E. all answer’s are wrong.
1127. symptom of "necklace of Venus" is
A papular eruption in psoriasis on the chest and neck
B. papular eruptions in syphilis in the chest and neck
C. false leucoderma in syphilis in the chest and neck
D. true leucoderma in syphilis in the chest and neck *
E. all answer’s are wrong.
1128. The level at which the hair breaks off at mikrosporii
A. 1-2mm
B. 1-4cm
C. 5-8mm *
D. 3-5cm
E. all answer’s are wrong.
1129. The level at which the hair breaks off at trihofitii
A. 5-8mm
B. 1-4cm
C. 1-2mm *
D. 3-5cm
E. all answer’s are wrong.
1130 for characteristic microsporia
A breaking of hair at 1-2mm
B. honeycomb
C. Case consists muftoobrazny around the hair *
D. symptom chips
E. all answer’s are wrong.
1131. "asbestiform" plaque characteristic of
A. trihofitii
B. microsporia *
C. multi-colored lichen
D. athlete
E. all answer’s are wrong.
1132. effective cure for scabies is necessary
A. treat all family members regardless of the presence of clinical signs of scabies *
B. treat only those family members who have a night itching
C. Add to the treatment of immune stimulants
D. add to treatment vitamins
E. all answer’s are wrong.
1133. some of the below listed products used in the treatment of fungal diseases
A. acyclovir
B. spregal
C. griseofulvin *
D. ceftriaxone
E. all answer’s are wrong.
1134. What are the disadvantages has 33% sulfuric ointment in the treatment of scabies
A frequent development allergodermatita *
B. high cost
C. cause skin atrophy
D. addictive
E. all answer’s are wrong.
1135. The number of layers of the epidermis is
3 A.
B. 2
C. 5 *
D. 7
E. all answer’s are wrong.
1136. Mask "St. Anthony" is observed at
A. leprosy *
B. pemphigus
C. Psoriasis
D. syphilis
E. all answer’s are wrong.
1137. For systemic sclerosis is pathognomonic:
A symptom score
B. symptom of ladies heels
C. symptom of falling through the probe
D. CREST - Syndrome
E. all answer’s are wrong.
1138. For systemic lupus erythematosus characterized by all except:
A CREST - Syndrome
B. lupus nephritis
C. lupus carditis
D. arthralgia
E. all answer’s are wrong.
1139. What disease refers to collagen?
A. pemphigus
B. chromomycosis
C. dermatomyositis *
D. skrofuloderma
E. all answer’s are wrong.
1140. What disease refers to collagen
A. pemphigus
B. chromomycosis
C. scleroderma *
D. skrofuloderma
E. all answer’s are wrong.
1141. What disease refers to collagen
A. pemphigus
B. chromomycosis
C. lupus erythematosus *
D. skrofuloderma
E. all answer’s are wrong.
1142. Pathologic process which underlies the symptom Nikolsky
A. spongiosa
B. acanthosis
C. akantolizis *
D. papillomatosis
E. all answer’s are wrong.
1143. Pathologic process which underlies the phenomenon of stearin spot in psoriasis
A. papillomatosis
B. acanthosis
C. spongiosa
D. parakeratosis *
E. all answer’s are wrong.
1144. Pathologic process which underlies the phenomenon of the terminal films in psoriasis
A. papillomatosis
B. parakeratosis
C. spongiosa
D. acanthosis *
E. all answer’s are wrong.
1145. Pathologic process which underlies the phenomenon of blood dew in psoriasis
A. spongiosa
B. acanthosis
C. papillomatosis *
D. parakeratosis
E. all answer’s are wrong.
1146 a method of diagnosis of syphilis becomes positive after all
A. RIBT *
B. PB
C. RIF
D. ELISA
E. all answer’s are wrong.
1147. Jarisch -Lukashevicha observed at
A. syphilis *
B.puzyrchatke
C. scabies
D. psoriasis
E. all answer’s are wrong.
1148. atypical chancre is
A chancre - amigdolit *
B. bipolar chancre
C. extragenital chancre
D. dwarf chancre
E. all answer’s are wrong.
1149. atypical chancre is
A chancre - felon *
B. bipolar chancre
C. extragenital chancre
D. dwarf chancre
E. all answer’s are wrong.
1150 is atypical chancre
A dwarf chancre
B. bipolar chancre
C. extragenital chancre
D. indurativnyy edema *
E. all answer’s are wrong.
1151. What characterizes acute gonorrhea in men
A chancre
B. sharp pain during urination, urethral suppuration *
C. painless ulcer on the penis
D. frothy discharge from the urethra
E. all answer’s are wrong.
1152. The disease is characterized by a loss of sensitivity
A. Tuberculosis
B. leprosy *
C. lupus
D.opoyasyvayuschy zoster
E. all answer’s are wrong.
1153. True eczema differs from other types of eczema
A. asymmetry defeats
B. symmetrical lesions *
C. lack of vesicles
D. the presence of pustules
E. all answer’s are wrong.
1154. in the pathogenesis of the disease plays a role of gluten intolerance
A. Pemphigus
B. dermatitis herpetiformis Duhring *
C. erythema multiforme exudative
D. lupus erythematosus
E. all answer’s are wrong.
1155. What is the drug most often fixed erythema
A DDS
B. GCS
C. ceftriaxone
D. Biseptol *
E. all answer’s are wrong.
1156. "duty plaques" characteristic of
A. psoriasis *
B. CPL
C. GDD
D. pemphigus
E. all answer’s are wrong.
1157. characteristic for neurodermatitis
A. vesicles
B. vegetation
C. lihenifikatsiya *
D. tuberkulum
E. all answer’s are wrong.
1158 with any items not allocated pale treponema
A pustule
B. extensive warts
C. erosive papule
D. roseola *
E. all answer’s are wrong.
1159. evidenced by turbidity in 1 cup of the sample at Thomson
A. of pyelonephritis
B. a posterior urethral
C. of total urethritis
D. of anterior urethral *
E. all answer’s are wrong.
1160. evidenced in the turbidity in the sample cup 2 Thomson
A. a posterior urethral *
B. of anterior urethral
C. of total urethritis
D. of pyelonephritis
E. all answer’s are wrong.
1161. What morphological element is not proliferative:
A bump
B. knot
C. blister *
D. Node
E. all answer’s are wrong.
1162. Add a drug used in the pemphigus vulgaris:
A. monomitsin
B. Valtrex
C. prednisolone *
D. orungal
E. all answer’s are wrong.
1163. Enter the drug used in the treatment of uncomplicated scabies:
A. 20% benzyl benzoate *
B. Acyclovir
C. Vishnevsky ointment
D. 10% ointment sintomitsinovaja
E. all answer’s are wrong.
1164. Enter the drug used in skin candidiasis:
A. acyclovir
B. orungal *
C. furazolidon
D. Bactrim
E. all answer’s are wrong.
1165. Enter the drugs used in the progressive stage of psoriasis:
A. psoriasin ointment
B. 10% ointment sintomitsinovaja
C. Cream Unna *
D. ointment dermatol
E. all answer’s are wrong.
1166. Specify a phenomenon characteristic of lichen planus:
A fish eggs
B. Nikolsky simtom
C. Wickham Net *
D. tissue paper
E. all answer’s are wrong.
1167. Enter the symptoms characteristic of the multi-colored lichen:
A sample Wickham
B. triad Auspittsa
C. peeling *
D. Kebnera
E. all answer’s are wrong.
1168. Enter the drug used with shingles:
A. delagil
B. 10% ointment sintomitsinovaja
C. bonafton *
D. dapsone
E. all answer’s are wrong.
1169. Enter the clinical signs characteristic of sycosis:
A chronically relapsing course *
B. night itching
C. rheumatic pains
D. nodules
E. all answer’s are wrong.
1170. The stimuli that cause a simple contact dermatitis:
A. Mechanical *
B. Psychoemotional
C. Chemical - optional
D. dosage per os
E. all answer’s are wrong.
1171. Characteristic of eczema:
A. gumma
B. Moisture "serous wells" *
C. roseola
D. bumps
E. all answer’s are wrong.
1172. Clinical signs of syphilitic roseola:
A purple
B. pale pink *
C. rises above the skin
D. itchy
E. all answer’s are wrong.
1173. Clinical signs of tinea versicolor:
A papule
B. erosion
C. secondary leucoderma *
D. bumps
E. all answer’s are wrong.
1174. A defeat in the course of the nerve endings *
B. lentigo
C. blisters
D. nodules
E. all answer’s are wrong.
1175. The following morphological features:
A node *
B. bubble
C. Bubble
D. conflict
E. all answer’s are wrong.
1176. monomorphic rash when dermatoses:
A. Eczema
B. secondary syphilis
C. urticaria *
D. dermatitis Duhring
E. all answer’s are wrong.
1177. Clinical signs of psoriasis:
A. bumps
B. the presence of papules *
C. nodes
D. scars
E. all answer’s are wrong.
1178. For planus is characterized by:
A. blisters
B. bubbles
C. polygonal papules *
D. collar Biett
E. all answer’s are wrong.
1179. The main clinical forms of psoriasis:
A. atrophic
B. subacute
C. Acute
D. vulgaris *
E. all answer’s are wrong.
1180. Clinical phase currents and types of psoriasis:
A pustular
B. vezikuleznaya
C. stationary *
D. subacute
E. all answer’s are wrong.
1181. The phenomena characteristic of psoriasis:
A collar Biett
B. Stevens-Johnson
C. stearin spots *
D. Besnier-Meshcherskiy
E. all answer’s are wrong.
1182. For the characteristic symptoms of lupus:
A symptom Kebnera
B. mukovidnoe peeling
C. Erythema *
D. depigmentation
E. all answer’s are wrong.
1183. Periods of syphilis:
A subacute
B. Progressive
C. Acute
D. incubation *
E. all answer’s are wrong.
1184. Diseases related to dermatophytes:
A lichen simplex
B. chromophytosis
C. psoriasis
D. trichophytosis *
E. all answer’s are wrong.
1185. For hidradenitis sup- characterized by:
A. fever, malaise, headache *
B. papules
C. paired elements
D. blister
E. all answer’s are wrong.
1186. What pustular skin disease found only in newborns:
A. Ritter dermatitis *
B. carbuncle
C. Sycosis
D. boil
E. all answer’s are wrong.
1187. What pustular diseases occur in adults:
A. psevdofurunkulez Finger
B. vezikulopustulez
C. Sycosis *
D. epidemic pemphigus
E. all answer’s are wrong.
1188. If any of these viral diseases affected mucosa of the mouth:
A youthful warts
B. molluscum contagiosum
C. shingles *
D. plantar warts
E. all answer’s are wrong.
1189. Enter the drug used in psoriasis:
A. blemaren
B. ethambutol
C. Lorinden *
D. Cream Unna
E. all answer’s are wrong.
1190. How is a clinical sign is characterized by a rash in true eczema:
A. tubercles
B. asymmetry
C. induratum edema
D. true polymorphism *
E. all answer’s are wrong.
1191. Enter the variety of scleroderma:
A vulgar
B. intertriginoznoy
C. strip *
D. vulgar
E. all answer’s are wrong.
1192. Enter the drugs used in pemphigus vulgaris:
A. acyclovir
B. dapsone
C. monomitsin
D. prednizalon *
E. all answer’s are wrong.
1193. Lichen planus is characterized by:
A. tubercles
B. urticaria rashes
C. papular rash *
D. polymorphism
E. all answer’s are wrong.
1194. What are the clinical symptom pathognomonic of lichen planus:
A. * Net Wickham
B. cells Ttsanka
C. "honeycomb Celsus"
D. paired elements
E. all answer’s are wrong.
1195. What diseases do not belong to a group of viral dermatoses:
A wide warts *
B. genital warts
C.borodavki
D.kontagiozny clam
E. all answer’s are wrong.
1196. Primary morphological elements:
A scar
B. erosion
C. bubble *
D. lehenifikatsiya
E. all answer’s are wrong.
1197. Secondary morphological elements:
A. Lace
B. Ulcer *
C. Roseola
D. Bubble
E. all answer’s are wrong.
1198. Clinical signs of psoriasis:
A. units
B. the presence of papules *
C. blistering
D. hemorrhagic spots
E. all answer’s are wrong.
1199. Signs of true polymorphism rash when dermatitis Dühring:
A. Bubbles *
B. The Node
C. lihenifikatsiya
D. vegetation
E. all answer’s are wrong.
1200. Typical elements sprinkler planus have the following features, in addition to:
A. Wickham grid on the surface of papules
B. preferential localization on the face *
C. papules flat, polygonal, reddish-purple color
D. preferential localization in the flexor surface of the limbs
E. all answer’s are wrong.
1201. By the frequent complications of atopic dermatitis are all listed, except:
A polyadenylation
B. urethritis *
C. a painful and persistent itching
D. secondary infection in the outbreak of atopic dermatitis
E. all answer’s are wrong.
1202. What are the exogenous factors contribute to the development of pyoderma:
Hypervitaminosis A.
A. worm infestation
B.Microfracture *
C.gene mutations
D.UVR exposure
E.Stafilodermiyam
1203. It includes:
Bullous impetigo A.
B. slit impetigo
C. vulgar ecthyma
D. impetigo Bockhart *
E. all answer’s are wrong.
1204. Clinical symptoms of discoid lupus erythematosus everything except:
A symptom Besnier-Meshcherskiy
B. tightly fitting scales
C. atrophy
D. Bubbles *
E. all answer’s are wrong.
1205. For scabies is characterized by:
A symptom of "ladies' heel"
B. symptom of "apple jelly"
C. paired papules vesicular elements *
D. polygonal papules with central retraction
E. all answer’s are wrong.
1206. Diagnosis of acute prostatitis is based on the results of these studies:
A biopsy
B. blood chemistry
C. Survey of the skin
D. palpation of the prostate gland *
E. all answer’s are wrong.
1207. Which drug is effective for genital herpes:
A. Trichopolum
B. Diprospan
C.Lamizil
D. Ulkaril *
E. all answer’s are wrong.
1208. The drugs used in the treatment of fungal infections:
A. penicillin
B. ceftriaxone
C. griseofulvin *
D. linkomitsin
E. all answer’s are wrong.
1209. The main clinical signs of scabies:
A. blisters
B. acantholysis
C. itching in the evening and at night *
D.otsutstvie itching
E. all answer’s are wrong.
1210. The patient on the red border of lips accumulation of small cavity elements containing
serous fluid. Specify the nature of the morphological elements:
A. urticaria
B. bulls
C. papules
D. vesicles *
E. all answer’s are wrong.
1211. The patient on the red border of lips accumulation of small cavity elements containing
serous fluid. Which diseases of the skin may appear like a rash?
A simple herpes *
B. lichen planus
C. microsporia
D. leprosy
E. all answer’s are wrong.
1212. Histological examination of the preparation of the skin revealed a significant thickening of
the granular layer of the epidermis. Specify the nature of the disease process:
A. spongiosa
B. vacuolar degeneration
C. granulёz *
D. acantholysis
E. all answer’s are wrong.
1213. If the skin biopsy revealed fusion of epithelial intercellular bridges in the spinous layer
epidermisa, found Ttsanka cells. Specify the nature of the disease process:
A. papillomatosis
B.parakeratoz
C. acantholysis *
D. acanthosis
E. all answer’s are wrong.
1214. The skin biopsy revealed a violation of keratinization epidermotsitov, thickening of the
horny layer without structural changes in the cells. Specify the kind of pathological process:
A. parakeratosis
B. hyperkeratosis *
C. gipergranuyaez
D.akantoz
E. all answer’s are wrong.
1215. The biopsy of the skin marked intracellular edema in the Malpighian layer with signs of
pyknosis of nuclei. Specify the nature of pathological changes:
A vacuolar degeneration *
B. spongiosa
C. akantolizis
D. ballooning degeneration
E. all answer’s are wrong.
1216. The patient urticaria rashes on the body, accompanied by itching. Specify the kind of
morphological elements:
A papule
B. blister *
C. Spot
D.puzyr
E. all answer’s are wrong.
1217. The child of 1.5 years in the face of multiple microvesicles on congested skin notes
Moisture, worried about severe itching. Select the method of external treatment:
A. alcoholic solution of calendula
B. 2% tincture of iodine
C. 2% solution of brilliant green
D. 2% solution of methylene blue *
E. all answer’s are wrong.
1218. The patient neurodermatitis by mechanical stimulation of the skin with a blunt object
appears neuro-vascular reaction in the form of a white strip. What do you call such a reaction?
A. erythema "embarrassment"
B. marble skin
C. «Goosebumps"
D. dermographism *
E. all answer’s are wrong.
1219. The patient in the skin of the trunk, limbs, multiple milliarnye, lenticular, numullyarnys
papules, plaques, covered by silvery scales. Determine the nature of the rash:
A polymorph, exudative
B.monomorfnaya, exudative
C. monomorphic, urticaria
D. monomorphic, proliferative *
E. all answer’s are wrong.
1220. The patient has lichen planus rash appeared after severe stress, fear. What kind of treatment
it is advisable to appoint a general?
A. fuzhitsidnye funds
B. corticosteroids
C. cytostatics
D. suggestive therapy *
E. all answer’s are wrong.
1221. At the 2-year-old child on the face with a loose tire phlyctenas, sero-purulent exudate,
erosion, crusts. Diagnosis:
A. hydradenitis
B. epidemic pemphigus
C. vulgar sycosis
D. strep impetigo *
E. all answer’s are wrong.
1222. The patient 22 is not on the forearm painful knot the size of a hazelnut cone-shaped with a
necrotic core. A presumptive diagnosis:
A. Gunma
B. carbuncle
C. boil *
D.turniol
E. all answer’s are wrong.
1223. The child in the skin of the cheeks, legs whitish spots, rounded, with abundant
melkoplastinchatym peeling. The scraping the surface of foci of fungi were found. Sample Balzer
negative. Your presumptive diagnosis:
A. microsporia
B. surface trichophytosis
C. lichen simplex *
D.raznotsvetny zoster
E. all answer’s are wrong.
1224. A newborn in a week after birth appeared on the skin of the chest bubbles with serouspurulent exudate, located on a slightly erythematous background, erosion, crusts. A presumptive
diagnosis:
A. vezikulopustulez
B. syphilitic pemphigus
S. acantholytic pemphigus
D. epidemic pemphigus *
E. all answer’s are wrong.
1225. A newborn skin on the neck, back multiple nodes reddish-bluish color, the size of a pea,
palpation determined fluctuation. From penetrated nodes semi-liquid purulent-hemorrhagic
content. Diagnosis:
A children's eczema
B. hydradenitis
C. vezikulopustulez
D. psevdofurunkulez Finger *
E. all answer’s are wrong.
1226. The patient in the left axilla painful, tight, welded assemblies undulating skin, reddishbluish color, with a fluctuation. Necrotic core is missing. A presumptive diagnosis:
A. hydradenitis *
B. syphilitic lymphadenitis
C. kollekvativny tuberculosis
D. erythrasma
E. all answer’s are wrong.
1227. The patient on the skin of the penis size of the ulcer 2x2 cm, round shape, with dense
valikoobraznymi edges, purulent discharge, painful infiltrate from osnovaniya.Reaktsiya
Wasserman negative. A presumptive diagnosis:
A. shankrformnaya pyoderma *
B. genital herpes
C. chancroid
D. itch ecthyma
E. all answer’s are wrong.
1228. A woman of 40 years on the skin of the chin, nasolabial folds against the backdrop of
reddened skin are infiltrated pustules, TV angiectasis. Hypoacid suffers from gastritis. The
expected diagnosis:
A. photodermatitis
B. vulgaris
C. shankriformnaya pyoderma
D. rosacea *
E. all answer’s are wrong.
1229. The patient 30 years on the skin of the chin, cheeks multiple folliculitis, sycosis, clusters of
off-yellow purulent crusts. The process recurs. A presumptive diagnosis:
A. demodekoz
B. vulgar sycosis *
C. acne vulgaris
D. chronic ulcerative pyoderma
E. all answer’s are wrong.
1230. The woman on the skin of the chest, abdomen, multiple excoriations, nodules, abscesses,
linear shtrihoobraznye scratching, intense itching, worse at night. The expected diagnosis:
A. Eczema
B. scabies *
C. nodular pruritus
D. strofulyus
E. all answer’s are wrong.
1231. A man 23 years in the pubic area, the penis, the inner thighs papules, pustules, purulent
crust erosion. Worried itching noted symptom Gorchakov-Hardy. A presumptive diagnosis:
A vulgar sycosis
B. chancroid
C. scabies *
D. Lipschutz's disease
E. all answer’s are wrong.
1232. The patient was 25 years old Single The diagnosis of scabies. What laboratory studies
should be included in the plan of the survey?
A study on the LE cells
B. Research of blood PB
C. finding scabies *
D. Analysis on acantholytic cells
E. all answer’s are wrong.
1233. Patient itch after application of 33% sulfur ointment on the skin of the trunk appeared
diffuse erythema, rash melkopapuleznaya. Diagnosis:
A. atopic dermatitis
B. actinic dermatitis
C. contact dermatitis allergic *.
D. toksikodermiya
E. all answer’s are wrong.
1234. The patient skin spots round shape size of 5-6 mm with a slight melkoplastinchatym
peeling. When lubricating iodine stains darken. Diagnosis:
Dermatitis herpetiformis Duhring A.
B. erythema multiforme exudative
C. allergic dermatitis
D. multicolored shingles *
E. all answer’s are wrong.
1235. The patient 56 years old skin in the inguinal folds clearly limited noninflammatory spot
brick-red color. There sweating. Diagnosis:
A. candidiasis
B. erythrasma *
C. Limitations of atopic dermatitis
D.numullyarnaya eczema
E. all answer’s are wrong.
1236. The patient 23 years in the interdigital gaps feet maceration, fracture fragments of the
epidermis around the edges. The nails of the thumbs of both feet are yellow, dim, crumble.
Diagnosis:
A. aktinomikoz
B. mycosis fungoides
C. athlete's foot, onychomycosis. *
D. mitseloma foot
E. all answer’s are wrong.
1237. The patient suffering from diabetes at the head of the penis itchy erythematous border
erosion macerated epidermis. Worried itching, burning. Diagnosis:
A contact dermatitis
B. fagodenichesky chancre
C. candida balanoposthitis *
D. fixed erythema
E. all answer’s are wrong.
1238. The patient 10 years on the scalp rounded center 2x2 cm. The hair broken off at 6-8 mm at
the base - Case consists muftoobrazny. Diagnosis:
A. seborrhea
B. surface trichophytosis
C. alopecia baldness
D. microsporia *
E. all answer’s are wrong.
1239. Patient K. 15 years on the scalp is clearly limited erythematous-squamous foci to be broken
off hair at the level of 6-8 mm. Under Wood's lamp - green glow. Diagnosis:
A. favus
B. trihofitnya
C. mikrospornya *
D. psoriasis
E. all answer’s are wrong.
1240. The patient diagnosed as athlete's foot, disgidroticheskaya form. What are the clinical signs
are characteristic of this disease?
A bubbling and blistering multiB. Localization on the fingers and toes
C. localization in the arch of the foot *
D. appearance of paired vesicles-papular elements
E. all answer’s are wrong.
1241. Enter the drug used with shingles:
A. delagil
B. 10% ointment sintomitsinovaja
C. bonafton *
D. dapsone
E. all answer’s are wrong.
1242. Enter the clinical signs characteristic of sycosis:
A. nodules
B. night itching
C. rheumatic pains
D. chronically relapsing course *
E. all answer’s are wrong.
1243. The stimuli that cause a simple contact dermatitis:
A psycho-emotional
B. Mechanical *
C. Chemical - optional
D. dosage per os
E. all answer’s are wrong.
1244. Characteristic of eczema:
A. gumma
B. Moisture "serous wells" *
C. roseola
D. bumps
E. all answer’s are wrong.
1245. Clinical signs of syphilitic roseola:
A purple
B. pale pink *
C. rises above the skin
D. itchy
E. all answer’s are wrong.
1246. Clinical signs of tinea versicolor:
A papule
B. erosion
C. secondary leucoderma *
D. bumps
E. all answer’s are wrong.
1247. For zoster is characterized by:
A. blisters
B. lentigo
C. defeat the course of the nerve endings *
D. nodules
Do not have a cavity
E. all answer’s are wrong.
1248. The following morphological features:
A bubble
B. The Node *
C. Bubble
D. conflict
E. all answer’s are wrong.
1249. monomorphic rash when dermatoses:
A. Eczema
B. secondary syphilis
C. urticaria *
D. dermatitis Duhring
E. all answer’s are wrong.
1250. Clinical signs of psoriasis:
A. bumps
B. the presence of papules *
C. nodes
D. scars
E. all answer’s are wrong.
1251. For planus is characterized by:
A. blisters
B. bubbles
C. polygonal papules *
D. collar Biett
E. all answer’s are wrong.
1252. The main clinical forms of psoriasis:
A. atrophic
B. subacute
C. Acute
D. vulgaris *
E. all answer’s are wrong.
1253. Clinical phase currents and types of psoriasis:
A pustular
B. vezikuleznaya
C. stationary *
D. subacute
E. all answer’s are wrong.
1254. The phenomena characteristic of psoriasis:
A. Besnier-Meshcherskiy
B. Stevens-Johnson
C. collar Biett
D. stearin spots *
E. all answer’s are wrong.
1255. For the characteristic symptoms of lupus:
A. depigmentation
B. mukovidnoe peeling
C. symptom Kebnera
D. Erythema *
E. all answer’s are wrong.
1256. Periods of syphilis:
A subacute
B. Progressive
C. Acute
D. incubation *
E. all answer’s are wrong.
1257. Diseases related to dermatophytes:
A. psoriasis
B. chromophytosis
C. trichophytosis *
D. lichen simplex
E. all answer’s are wrong.
1258. For hidradenitis sup- characterized by:
A pair of elements
B. papules
C. fever, malaise, headache *
D. blister
E. all answer’s are wrong.
1259. What pustular skin disease found only in newborns:
A. Sycosis
B. carbuncle
C. dermatitis Ritter *
D. boil
E. all answer’s are wrong.
1260. What pustular diseases occur in adults:
A. Sycosis *
B. vezikulopustulez
C. psevdofurunkulez Finger
D. epidemic pemphigus
E. all answer’s are wrong.
1261. If any of these viral diseases affected mucosa of the mouth:
A plantar wart
B. molluscum contagiosum
C. youthful warts
D. shingles *
E. all answer’s are wrong.
1262. Enter the drug used in psoriasis:
A. ethambutol
B. Lorinden *
C. blemaren
D. Cream Unna
E. all answer’s are wrong.
1263. How is a clinical sign is characterized by a rash in true eczema:
A. asymmetry
B. true polymorphism *
C. induratum edema
D. tubercles
E. all answer’s are wrong.
1264. Enter the variety of scleroderma:
A. intertriginoznoy
B. strip *
C. vulgar
D. vulgar
E. all answer’s are wrong.
1265. Enter the drugs used in pemphigus vulgaris:
A. acyclovir
B. dapsone
C. monomitsin
D. prednizalon *
E. all answer’s are wrong.
1266. Lichen planus is characterized by:
A. tubercles
B. urticaria rashes
C. papular rash *
D. polymorphism
E. all answer’s are wrong.
1267. What clinical symptom pathognomonic of lichen planus:
A "honeycomb Celsus"
B. cells Ttsanka
C. Wickham Net *
D. paired elements
E. all answer’s are wrong.
1268. What diseases do not belong to a group of viral dermatoses:
A. warts
B. genital warts
C. wide warts *
D.kontagiozny clam
E. all answer’s are wrong.
1269. Primary morphological elements:
A scar
B. erosion
C. bubble *
D. lehenifikatsiya
E. all answer’s are wrong.
1270. Secondary morphological features:
A. Roseola
B. Lace
C. Ulcer *
D. Bubble
E. all answer’s are wrong.
1271. Clinical signs of psoriasis:
A. bubbles
B. nodes
C. the presence of papules *
D. hemorrhagic spots
E. all answer’s are wrong.
1272. Signs of true polymorphism rash when dermatitis Dühring:
A vegetation
B. The Node
C. lihenifikatsiya
D. Bubbles *
E. all answer’s are wrong.
1273. Typical elements sprinkler planus have the following features, in addition to:
A. papules flat, polygonal, reddish-purple color
B. Wickham grid on the surface of papules
C. preferential localization on the face *
D. preferential localization in the flexor surface of the limbs
E. all answer’s are wrong.
1274. By the frequent complications of atopic dermatitis are all listed, except:
A polyadenylation
B. urethritis *
C. a painful and persistent itching
D. secondary infection in the outbreak of atopic dermatitis
E. all answer’s are wrong.
1275. What are the exogenous factors contribute to the development of pyoderma:
Hypervitaminosis A.
B. worm infestation
C. Microfracture *
D. UVR exposure
E. all answer’s are wrong.
1276. It includes Stafilodermiyam :
A vulgar ecthyma
B. slit impetigo
C. impetigo Bockhart *
Bullous impetigo D.
E. all answer’s are wrong.
1277. Clinical symptoms of discoid lupus erythematosus everything except:
A. atrophy
B. tightly fitting scales
C. Bubbles *
D. symptom Besnier-Meshcherskiy
E. all answer’s are wrong.
1278. For scabies is characterized by:
A pair papules vesicular elements *
B. symptom of "apple jelly"
C. symptom of "ladies' heel"
D. polygonal papules with central retraction
E. all answer’s are wrong.
1279. Diagnosis of acute prostatitis is based on the results of these studies:
cA biopsy
B. blood chemistry
C. Survey of the skin
D. palpation of the prostate gland *
E. all answer’s are wrong.
1280. Which drug is effective for genital herpes:
A. Lamisil
B. Diprospan
C. Ulkaril *
D.Trihopol
E. all answer’s are wrong.
1281. The drugs used in the treatment of fungal infections:
A. penicillin
B. ceftriaxone
C. griseofulvin *
D. linkomitsin
E. all answer’s are wrong.
1282. The main clinical signs of scabies:
A. blisters
B. acantholysis
C. itching in the evening and at night *
D.otsutstvie itching
E. all answer’s are wrong.
1283. The patient on the red border of lips accumulation of small cavity elements containing
serous fluid. Specify the nature of the morphological elements:
A. urticaria
B. bulls
C. papules
D. vesicles *
E. all answer’s are wrong.
1284. The patient on the red border of lips accumulation of small cavity elements containing
serous fluid. Which diseases of the skin may appear like a rash?
A leprosy
B. lichen planus
C. microsporia
D. simple herpes *
E. all answer’s are wrong.
1285. Histological examination of the preparation of the skin revealed a significant thickening of
the granular layer of the epidermis. Specify the nature of the disease process:
A. acantholysis
B. vacuolar degeneration
C. spongiosa
D. granulёz *
E. all answer’s are wrong.
1286. If the skin biopsy revealed fusion of epithelial intercellular bridges in the spinous layer
epidermisa, found Ttsanka cells. Specify the nature of the disease process:
A. parakeratosis
B. acantholysis *
C. papillomatosis
D. acanthosis
E. all answer’s are wrong.
1287. The skin biopsy revealed a violation of keratinization epidermotsitov, thickening of the
horny layer without structural changes in the cells. Specify the kind of pathological process:
A. parakeratosis
B. hyperkeratosis *
C. gipergranuyaez
D.akantoz
E. all answer’s are wrong.
1288. The biopsy of the skin marked intracellular edema in the Malpighian layer with signs of
pyknosis of nuclei. Specify the nature of pathological changes:
A. akantolizis
B. spongiosa
C. vacuolar degeneration *
D. ballooning degeneration
E. all answer’s are wrong.
1289. The patient urticaria rashes on the body, accompanied by itching. Specify the kind of
morphological elements:
A spot
B. papule
C. blister *
D.puzyr
E. all answer’s are wrong.
1290. The child of 1.5 years in the face of multiple microvesicles on congested skin notes
Moisture, worried about severe itching. Select the method of external treatment:
A. 2% tincture of iodine
B. 2% solution of methylene blue *
C. 2% solution of brilliant green
D. alcoholic solution of calendula
E. all answer’s are wrong.
1291. The patient neurodermatitis by mechanical stimulation of the skin with a blunt object
appears neuro-vascular reaction in the form of a white strip. What do you call such a reaction?
A. dermographism *
B. marble skin
C. «Goosebumps"
D. erythema "embarrassment"
E. all answer’s are wrong.
1292. The patient in the skin of the trunk, limbs, multiple milliarnye, lenticular, numullyarnys
papules, plaques, covered by silvery scales. Determine the nature of the rash:
A monomorphic, urticaria
B.monomorfnaya, exudative
C. monomorphic, proliferative *
D. polymorphic, exudative
E. all answer’s are wrong.
1293. The patient has lichen planus rash appeared after severe stress, fear. What kind of treatment
it is advisable to appoint a general?
A suggestive therapy *
B. corticosteroids
C. cytostatics
D. fuzhitsidnye funds
E. all answer’s are wrong.
1294. At the 2-year-old child on the face with a loose tire phlyctenas, sero-purulent exudate,
erosion, crusts. Diagnosis:
A strep impetigo *
B. epidemic pemphigus
C. vulgar sycosis
D. hydradenitis
E. all answer’s are wrong.
1295. The patient 22 is not on the forearm painful knot the size of a hazelnut cone-shaped with a
necrotic core. A presumptive diagnosis:
A. Gunma
B. carbuncle
C. boil *
D.turniol
E. all answer’s are wrong.
1296. The child in the skin of the cheeks, legs whitish spots, rounded, with abundant
melkoplastinchatym desquamation. The scraping the surface of foci of fungi were found. Sample
Balzer negative. Your presumptive diagnosis:
A. microsporia
B. surface trichophytosis
C. lichen simplex *
D.raznotsvetny zoster
E. all answer’s are wrong.
1297. A newborn in a week after birth appeared on the skin of the chest bubbles with serouspurulent exudate, located on a slightly erythematous background, erosion, crusts. A presumptive
diagnosis:
A. vezikulopustulez
B. syphilitic pemphigus
S. acantholytic pemphigus
D. epidemic pemphigus *
E. all answer’s are wrong.
1298. A newborn skin on the neck, back multiple nodes reddish-bluish color, the size of a pea,
palpation determined fluctuation. From penetrated nodes semi-liquid purulent-hemorrhagic
content. Diagnosis:
A. vezikulopustulez
B. hydradenitis
C. psevdofurunkulez Finger *
D. Child Eczema
E. all answer’s are wrong.
1299. The patient in the left axilla painful, tight, welded assemblies undulating skin, reddishbluish color, with a fluctuation. Necrotic core is missing. A presumptive diagnosis:
A. erythrasma
B. syphilitic lymphadenitis
C. kollekvativny tuberculosis
D. hydradenitis *
E. all answer’s are wrong.
1300. The patient on the skin of the penis size of the ulcer 2x2 cm, round shape, with dense
valikoobraznymi edges, purulent discharge, painful infiltrate from osnovaniya.Reaktsiya
Wasserman negative. A presumptive diagnosis:
A. chancroid
B. genital herpes
C. shankrformnaya pyoderma *
D. itch ecthyma
E. all answer’s are wrong.
1301. A woman of 40 years on the skin of the chin, nasolabial folds against the backdrop of
reddened skin are infiltrated pustules, TV angiectasis. Hypoacid suffers from gastritis. The
expected diagnosis:
A. photodermatitis
B. vulgaris
C. shankriformnaya pyoderma
D. rosacea *
E. all answer’s are wrong.
1302. The patient 30 years on the skin of the chin, cheeks multiple folliculitis, sycosis, clusters of
off-yellow purulent crusts. The process recurs. A presumptive diagnosis:
A. demodekoz
B. vulgar sycosis *
C. acne vulgaris
D. chronic ulcerative pyoderma
E. all answer’s are wrong.
1303. The woman on the skin of the chest, abdomen, multiple excoriations, nodules, abscesses,
linear shtrihoobraznye scratching, intense itching, worse at night. The expected diagnosis:
A. Eczema
B. scabies *
C. nodular pruritus
D. strofulyus
E. all answer’s are wrong.
1304. A man 23 years in the pubic area, the penis, the inner thighs papules, pustules, purulent
crust erosion. Worried itching noted symptom Gorchakov-Hardy. A presumptive diagnosis:
A. chancroid
B. scabies *
C. vulgar sycosis
D. Lipschutz's disease
E. all answer’s are wrong.
1305. The patient was 25 years old Single The diagnosis of scabies. What laboratory studies
should be included in the plan of the survey?
A study on the LE cells
B. Research of blood PB
C. finding scabies *
D. Analysis on acantholytic cells
E. all answer’s are wrong.
1306. Patient itch after application of 33% sulfur ointment on the skin of the trunk appeared
diffuse erythema, rash melkopapuleznaya. Diagnosis:
A. atopic dermatitis
B. actinic dermatitis
C. contact allergic dermatitis *
D. toksikodermiya
E. all answer’s are wrong.
1307. The patient skin spots round shape size of 5-6 mm with a slight melkoplastinchatym
peeling. When lubricating iodine stains darken. Diagnosis:
A. allergic dermatitis
B. erythema multiforme exudative
C. versicolor colorful *
Dermatitis herpetiformis Duhring D.
E. all answer’s are wrong.
1308. The patient 56 years old skin in the inguinal folds clearly limited noninflammatory spot
brick-red color. There sweating. Diagnosis:
A. candidiasis
B. erythrasma *
C. Limitations of atopic dermatitis
D.numullyarnaya eczema
E. all answer’s are wrong.
1309. The patient 23 years in the interdigital gaps feet maceration, fracture fragments of the
epidermis around the edges. The nails of the thumbs of both feet are yellow, dim, crumble.
Diagnosis:
A. mycosis fungoides
B. athlete's foot, onychomycosis *
C. aktinomikoz
D. mitseloma foot
E. all answer’s are wrong.
1310. The patient suffering from diabetes at the head of the penis itchy erythematous border
erosion macerated epidermis. Worried itching, burning. Diagnosis:
A contact dermatitis
B. fagodenichesky chancre
C. candida balanoposthitis *
D. fixed erythema
E. all answer’s are wrong.
1311. The patient 10 years on the scalp rounded center 2x2 cm. The hair broken off at 6-8 mm at
the base - Case consists muftoobrazny. Diagnosis:
A. seborrhea
B. surface trichophytosis
C. alopecia baldness
D. microsporia *
E. all answer’s are wrong.
1312. Patient K. 15 years on the scalp is clearly limited erythematous-squamous foci to be broken
off hair at the level of 6-8 mm. Under Wood's lamp - green glow. Diagnosis:
A. favus
B. trihofitnya
C. mikrospornya *
D. psoriasis
E. all answer’s are wrong.
1313. 27y.o. The patient diagnosed as athlete's foot, disgidroticheskaya form. What are the clinical
signs are characteristic of this disease?
A localization in the arch of the foot *
B. Localization on the fingers and toes
C. Education and multi bubbles bubbles
D. appearance of paired vesicles-papular elements
E. groin lamfadenit
E. all answer’s are wrong.
1314. Enter the diseases which are appointed by the tests for fungi
A. microsporia *
B. mycosis fungoides
C. shingles
D. psoriasis - psoriasis
E. all answer’s are wrong.
1315. Indicate the correct answers which correspond to the route of infection
diagnosis
A. transfusion - shingles
B. transmissible - leishmaniasis *
C. airborne - warty tuberculosis of the skin
D. transmissible - trichophytosis
E. all answer’s are wrong.
1316. Enter the focal forms of skin tuberkuleza
A. lichenoid tuberculosis skin
B. induratum erythema
C. miliary tuberculosis
D. lupus *
E. all answer’s are wrong.
1317. Enter disseminirovannnye TB skin
A. vulgar lupus
B. miliary tuberculosis *
C. infiltrative tuberculosis
D. skrofuloderma
E. all answer’s are wrong.
1318. What are the signs characteristic of cutaneous leishmaniasis
A. finding the virus cells to the lesion
B. symptom of "apple jelly"
C. transmissible transmission *
D. reservoir of infection is cattle
E. all answer’s are wrong.
1319. Under what dermatoses appointed by the local corticosteroid ointment
A. Atopic dermatitis *
B. Leishmaniasis
C. Colorful lichen
D. Shingles
E. all answer’s are wrong.
1320. For what dermatoses uncharacteristically strong subjective feeling of itching
A. Duhring Dermatitis
B. Microbial eczema
C. Hives
D. Shingles *
E. all answer’s are wrong.
1321. Find mismatch harkter rash - diagnosis
A. Typhoid fever - roseolous
B. Colorful lichen - roseolous-papular *
C. Strofulyus- papules, urticaria, vesicular
D. Duhring Dermatitis - polymorphic herpeticum
E. all answer’s are wrong.
1322. Search for a diagnosis - the nature of the rash
A. Microbial eczema - monomorphic pustular
B. Systemic lupus erythematosus - roseolous squamous skin atrophy
C. Skrofuloderma - Bugorkova *
D. Leishmaniasis skin - papular - patchy
E. all answer’s are wrong.
1323. Which of dermatoses characterized by defeat and nail polish
Roller
A. Skrofuloderma
B. Athlete *
Pink Gibert C. versicolor
D. Duhring Dermatitis
E. all answer’s are wrong.
1324. A sign of some dermatoses may be desquamation, hyperkeratosis,
cracks in the skin of the palms and fingertips
A. Lyell Syndrome
B. Stevens-Zhonsona
C. Disseminated form of lupus erythematosus
Ichthyosis D. *
E. all answer’s are wrong.
1325. What are the most common opportunistic infections in AIDS
A. Leishmania
B. Korinobakterii
C. Pneumocystis infection *
D. Trichomonas
E. all answer’s are wrong.
1326. What forms of Kaposi's sarcoma you know
A. Pozdnoizyazvlyayuschaya
B. AIDS-related *
C. Undifferentiated
D. Paratravmaticheskaya
E. all answer’s are wrong.
1327. What form of AIDS on the prevalence of symptoms in the clinical
picture (according to WHO)
A. Pulmonary *
B. gepatolienalny
C. Gangrenous
D. lymphoproliferative
E. all answer’s are wrong.
1328. Enter the ratio of T-helper / T-suppressor in carriers of HIV
A. within 10 *
B. within 15
C. <1
D. more than 10
E. all answer’s are wrong.
1329 What laboratory methods allow to confirm the diagnosis
Vulgar lupus
A. smears
B. Mantoux *
C. Sample Jadassohn
D. Response Wassermann
E. all answer’s are wrong.
1330. What diseases are at bullous dermatosis
A. Pemphigus *
B. chickenpox
C. shingles
D. streptoderma
E. all answer’s are wrong.
1331. Under what bullous dermatosis observed eruptions
A. Eczema
B. lichen planus
C. streptoderma
D. Lyell's syndrome *
E. all answer’s are wrong.
1332. The classification includes true pemphigus
A. hand-foot
B. vulgar pemphigus *
C. disgidroticheskaya
D. pustular
E. all answer’s are wrong.
1333. What are the symptoms characteristic of true pemphigus
A. Nikolsky *
B. Pospelov
C. "apple jelly"
D. Gorchakov-Hardy
E. all answer’s are wrong.
1334. In what diseases there is a positive sign
Nikolsky
A. Pemphigus *
B. dermatitis Duhring
C. bullous streptoderma
D. nodules doilschits (false cow ospa)
E. all answer’s are wrong.
1335. What are the main rules of the COP in the treatment of hormonepemphigus
A. used pulse therapy
B. taking hormones administered on the evening
C. Treatment starts with a loading dose *
D. hormones are prescribed only for external treatment
E. all answer’s are wrong.
1336. What are the complications observed in the treatment of pemphigus
A. oppression of the hematopoietic system
B. cushingoid Syndrome
C. amyloidosis
D. paralysis
E. all answer’s are wrong.
1337. What are the characteristic clinical signs of dermatitis Duhring
A. infiltrative lesions are composed of elements
B. positive symptom Nikolsky
C. hypersensitivity to cereals *
D. losing the oral mucosa
E. all answer’s are wrong.
1338. What are the morphological features are characteristic rash
dermatitis Duhring
A. vesicular *
B. bumps
C. nodes
D. vegetation
E. all answer’s are wrong.
1339. What are the laboratory findings are characteristic of dermatitis Duhring
A. lymphocytosis
B. leykotsitozv cystic fluid
C. eosinophilia in the blood *
D. Histomorphological - acantholysis
E. all answer’s are wrong.
1340. Which dermatosis characterized by the appearance of rashes on
oral mucosa
A. Psoriasis
B. shingles
C. pemphigus *
D. hydradenitis
E. all answer’s are wrong.
1341. What a rash with a clear viral etiology
A. dermatitis Duhring
B. lichen planus
C. pemphigoid
D. herpes *
E. all answer’s are wrong.
1342. What are the characteristics of lesions in the herpes simplex
A. monomorphic bubble elements *
B. subjectively - itching prevails over the soreness and burning
C. grouped cystic lesions
D. polymorphism rash
E. all answer’s are wrong.
1343. What are the distinguishing features of genital herpes from a solid
chancre
A. painless erosions
B. tenderness *
C. Regional bubo
D. the presence of a solid infiltrate the base erosion
E. all answer’s are wrong.
1344. What are the clinical forms of herpes zoster
A. vegetating
B. abortive *
C. intercostal
D. infiltrative-suppurative
E. all answer’s are wrong.
1345. Enter the drug has prtivovirusnym action
A. Sulphur cleared
B. FIBS
C. Rifampicin
Acyclovir D. *
E. all answer’s are wrong.
1346. Assign the drug to patients with a diagnosis of herpes zoster
A. viroleks tablets *
B. ampicillin
C. Aevitum
D. Sodium bromo drops
E. all answer’s are wrong.
1347. Enter the characteristics of molluscum contagiosum
A. scales on the surface of papules
B. symptom of "net Wickham" positive
C. preimuschestveeno common in children *
D. transmission-transmissive
E. all answer’s are wrong.
1348. What forms of lupus are distinguished according to the classification
A. pustular
B. discoid *
C. annular
D. atrophic
E. all answer’s are wrong.
1349. What are the cardinal symptoms of lupus rash
A. pustular infiltration
B. follicular hyperkeratosis *
C. keloid scars
D. lihenifikatsiya
E. all answer’s are wrong.
1350. What are the symptoms observed in discoid lupus erythematosus
A. "mesh" Wickham
B. "honeycomb" Celsus
Film Terminal C.
D. "ladies' heel" *
E. all answer’s are wrong.
1351. What are the laboratory findings are characteristic of SLE (acute form)
A. "LE" - cells *
B. Positive RVs
C. bilirubinemia
D. ketoacidosis
E. all answer’s are wrong.
1352. With what diseases nebhodimo differentiate red
erythematosus
A. Eczema
B. tuberculosis Lupus *
C. pemphigus
D. molluscum contagiosum
E. all answer’s are wrong.
1353. What are the stages are distinguished in the clinical course of scleroderma
A. exudation
B. hyperkeratosis
C. izyazvlenie
D. atrophy *
E. all answer’s are wrong.
1354. What dermatitis are at kollagenozov
A. dermatitis Duhring
B. genital warts
C. sklerdermiya *
D. Leishmaniasis
E. all answer’s are wrong.
1355. What are considered benign neoplasms of the skin
A. melanoma
B. bazalioma
C. Paget's disease
D. lipoma *
E. all answer’s are wrong.
1356. Enter the malignant skin lesions
A. papilloma
B. lipoma
C. melanoma *
D. lymphangioma
E. all answer’s are wrong.
1357. What are the clinical varieties of acne vulgaris
A. abstsediruyuschie *
B. conical
C. eritemato-squamous
D. erosive and ulcerative
E. all answer’s are wrong.
1358. Enter the current stage of rosacea
A. follicular hyperkeratosis
B. Stage atrophy
C. erythematous *
D. sealing step
E. all answer’s are wrong.
1359. What are the exogenous factors that may cause abnormal
condition in the skin
A. immunodeficiency
B. pathogenic fungi *
C. hypersensitization
D. organic and functional disorders of the nervous system
E. all answer’s are wrong.
1360. Endogenous physiological factors that may cause
vospalitenye include changes in the skin
A. hyperthermia *
B. Chemical
C. physical
D. Bacteria
E. all answer’s are wrong.
1361. Primary morphological elements may be
A. induratum
B. exudative *
C. follicular
D. nefollikulyarnye
E. all answer’s are wrong.
1362. Which of the following applies to exudative lesions
A. unit
B. bubble *
C. tuberkulum
D. papule
E. all answer’s are wrong.
1363. What are the primary elements of infiltrative
A. blister
B. vesicle
C. Bull
D. knot *
E. all answer’s are wrong.
1364. The spot is the primary element, resulting from
A. hyperkeratinization
B. Hemorrhage *
C. Changes color as a result of acantholysis
D. Changes in the color due to melanocytes hyperfunction
E. all answer’s are wrong.
1365. What kind of spots are distinguished
A. disgidroticheskie
B. serous
C. melanocytic
D. pigment *
E. all answer’s are wrong.
1366. What kinds of vascular patches
A. petechiae
B. lentigo
C. roseola *
D. leucoderma
E. all answer’s are wrong.
1367. What kinds of hemorrhagic spots
A. lentigo
B. chloasma
C. petechiae *
D. telangiectasia
E. all answer’s are wrong.
1368. What are the characteristics of voldyrya
A. is the result of acantholysis
B. is ephemeral (quickly appears and disappears without a trace) *
And C. In the process of the return of the resulting secondary depigmentation
D. Nature for autoimmunnyhzabolevany
E. all answer’s are wrong.
1369. What are the characteristics of a bubble
A. ulceration with ulceration and atrophy
B. is a proliferative element
C. formed by the swelling of papillary dermis
D. exudative is the primary element and contains fluid *
E. all answer’s are wrong.
1370. What is different from the bubble of the bubble
A. subjective sensations
B. size - less than 05 cm bubble, the bubble more *
C. contents exudate
D. localization elements
E. all answer’s are wrong.
1371. What are the layers of the epidermis
A. horny *
B. papillary
C. reticular
D. fat
E. all answer’s are wrong.
1372. What applies to the skin appendages
A. subcutaneous fat
B. epidermis
C. nails *
D. dermal papilla
E. all answer’s are wrong.
1373. What are the primary elements of mofologicheskie
A. papule *
B. atrophy
C. vegetation
D. excoriation
E. all answer’s are wrong.
1374. What kinds of pustules you know
A. skrofuloderma
B. excoriation
C. petechiae
D. folliculitis *
E. all answer’s are wrong.
1375. As a result, any process can patogistomorfologicheskih
form a bundle
A. anaplasia
B. akantoz *
C. vacuolar degeneration
D. acantholysis
E. all answer’s are wrong.
1376. What kind of papules vary in size
A. lichenoid
B. miliary *
C. dwarf
D. tuberculoid
E. all answer’s are wrong.
1377. What are the characteristics of the nodule
A. is elusive
B. contains serous ekksudat
C. bespolostnoy primary element *
D. infiltrate located in the dermis and hypodermis
E. all answer’s are wrong.
1378. What are the characteristics of the node, and for tubercle
A. refers to the elements of the exudative
B. infiltrate located in the epidermis
C. are bespolostnoy elements *
D. are ephemeral formations
E. all answer’s are wrong.
1379. Please, primary cells and by appropriate them again
elements
A. bubble - atrophy
B. bubble - acantholysis *
C. tuberkulum - secondary depigmentation
D. node - Erosion
E. all answer’s are wrong.
1380. Find a secondary morphological elements
A. papule
D. vegetation *
C. conflicts
D. tuberkulum
E. all answer’s are wrong.
1381. What are the characteristics of lihenifikatsii
A. pronounced skin pattern *
B. hyperhidrosis on the hearth
C. anesthesia on the hearth
D. warty growths on the hearth
E. all answer’s are wrong.
1382. What are the primary elements and their corresponding inverse
the development of secondary cells
A. bubble - erosion *
B. bubble - atrophic changes
C. papule - scar atrophy
D. spot - flaking and crust
E. all answer’s are wrong.
1383. What pyoderma typical of childhood
A. hydradenitis
B. bazalioma
C. epidemic pemphigus *
D. discoid lupus erythematosus
E. all answer’s are wrong.
1384. What are clinical variants of streptococcal impetigo
A. surface felon *
D. centrifugal
C. papules necrotic
D. seborrheic
E. all answer’s are wrong.
1385. What are the main clinical signs of scabies
A. symptom of Pospelov
B. sgruppirovannnye bubbles
C. paired papular or vesicular papules elements *
D. bites itch mites
E. all answer’s are wrong.
1386. What tools are used to treat scabies
A. A solution of chlorhexidine
B. gray mercurial ointment
C. 33 percent sulfuric ointment *
D. kerosene mixed with vegetable oil
E. all answer’s are wrong.
1387. What are the main clinical signs of lice
A. finding nits *
B. finding ticks
C. paired papular elements
D. symptom Gorchakov-Hardy
E. all answer’s are wrong.
1388. With what diseases differentiate scabies
A. Shingles
B. itching *
C. exudative erythema
D. psoriasis
E. all answer’s are wrong.
1389. What a rash characteristic of the lepromatous type of leprosy
A. gummy
B. nodes *
C. vegetating papules
D. pockets lihenifikatsii
E. all answer’s are wrong.
1390. What are the symptoms occur due to various motor
disorders in leprosy
A. "paw" monkey *
B. symptom of "butterfly"
C. honeycomb Celsus
D. staggering gait
E. all answer’s are wrong.
1391. What laboratory and instrumental methods used to
diagnosis of leprosy
A. Mantoux
B. reaction Cana
C. leprominovaya sample *
D. Sample Jadassohn
E. all answer’s are wrong.
1392. Which of the provisions are considered correct
A. type of TB - negative test
B. lepromatous - negative test
C. undifferentiated type - ustochivaya trial
D. lepromatous type - negative *
E. all answer’s are wrong.
1393. Enter any skin diseases considered contagious
A. scabies *
B. psoriasis
Pink Gibert C. versicolor
D. dermatitis Duhring
E. all answer’s are wrong.
1394 Specify zveno in the epidemiological chain of the disease Borowski
A. Hansen bacillus
B. Leishmania *
C. Cattle
D. fleas
E. all answer’s are wrong.
1395. Enter the reservoir of infection in the leishmaniasis
A. gophers *
B. mosquitoes and fleas
C. Leishmania
D. cockroaches
E. all answer’s are wrong.
1396. List the basic preventive measures when
leishmaniasis
A. disinfection of the premises and linen
B. early detection and treatment of patients *
C. destruction of stray dogs
D. immunizations
E. all answer’s are wrong.
1397. What laboratory and instrumental methods of diagnosis
are used for the diagnosis of tuberculosis of the skin
A. Sample Jadassohn
B. smear imprint
C. Research on the LE-cells
D. Sample Pirke *
E. all answer’s are wrong.
1398. Enter the localized forms of tuberculosis of the skin
A. papulonekrotichesky
B. vulgar lupus *
C. indurativnyy
D. miliary
E. all answer’s are wrong.
1399. What are the current stage of psoriasis:
A. latent
B. Incubation
C. Progressive *
D. progredient
E. all answer’s are wrong.
1400. Indicate the most frequent localization of sites on the skin
psoriatic lesions:
A. facial skin
B. extensor surfaces of the lower extremities *
C. flexor surfaces of the upper extremities
D. leather back
E. all answer’s are wrong.
1401. What are clinical variants of psoriasis:
A. papular psoriasis
B. exfoliative psoriasis (leaf-shaped)
C. Dissemination of psoriasis *
D. infiltrative-suppurative form
E. all answer’s are wrong.
1402. What are the clinical types of psoriasis:
A. disgidroticheskaya form
B. papules necrotic form
C. pustular psoriasis *
D. psoriatic lymphadenopathy
E. all answer’s are wrong.
1403. What osobennno psoriatic plaques:
A. in the central part have umbilicated
B. papules covered with silvery-white scales *
C. covered with serous crusts
D. are placed are grouped (herpetiformis location)
E. all answer’s are wrong.
1404. What does the triad Auspittsa:
A. symptom thimble
B. symptom of apple jelly
C. symptom oil stains
D. symptom of terminal films *
E. all answer’s are wrong.
1405. Enter the pathognomonic symptoms of psoriasis:
A. symptom Jadassohn
B. symptom of "butterfly"
C. triad Auspittsa *
D. Wickham Grid
E. all answer’s are wrong.
1406. Which nail changes observed in the defeat of their
psoriatic process:
A. symptom of "thimble" *
B. purulent inflammation of the nail ridges
C. cyanotic nail plate
D. affects only the nail plate brushes
E. all answer’s are wrong.
1407. List the clinical manifestations common to psoriasis and lichen planus:
A. acantholysis
B. papules *
C. Education infectious granuloma Langhans cells
D. spongiosa
E. all answer’s are wrong.
1408. With what diseases it is necessary to differentiate psoriasis:
A. papular syphilis *
B. syphilitic roseola
C. Herpes Simplex
D. dermatitis Duhring
E. all answer’s are wrong.
1409. What diseases are characterized by monomorphic eruptions:
A. psoriasis *
B. eczema
C. dermatitis Duhring
Erythema multiforme D. ekssudativeya
E. all answer’s are wrong.
1410. What are the drugs used for the treatment of psoriasis:
A. Sulfuric simple ointment
B. griseofulvin
C. hormonal ointment *
D. anticoagulants
E. all answer’s are wrong.
1411. What a rash characteristic of a simple contact dermatitis:
A. Bugorkova
B. vesicular
C. urticaria
D. erythema with sharp edges *
E. all answer’s are wrong.
1412. proliferative histomorphological changes include:
A. karyorrhexis
B. spongiosa
C. akantolizis
D. hyperkeratosis *
E. all answer’s are wrong.
1413. There are no sweat glands in the skin;
A red border *
B. armpits
S. palms
D. soles
E. all answer’s are wrong.
1414. There are no sebaceous glands in the skin;
A. nasolabial triangle
B. palms, soles *
C. scalp
D. interscapulum.
E. all answer’s are wrong.
1415. Which element is a kind of blisters:
A conflict *
B. telangiectasia
C. excoriation
D. Bull
E. all answer’s are wrong.
1416. What kind of spots:
A. Acne
B. Rupee
C. roseola *
D. conflict
E. all answer’s are wrong.
1417. What morphological element is characterized by ephemeral:
A pustule
B. papule
C. tuberkulum
D. blister *
E. all answer’s are wrong.
1418. Where are the "tizonievye sebaceous glands":
A. The inner layer of the foreskin *
B. red border
C. Leather breast
Dmezhlopatochnoe space
E. all answer’s are wrong.
1419. "Meibomian" sebaceous glands are located:
A. on kozhegrudi
B. By the edge of the eyelids *
C. skin ears
D. on the red border of lips
E. all answer’s are wrong.
1420. In what areas of the skin sebaceous glands open directly into the epidermis:
A. The head of the penis *
B. scalp
C. skin of the nose
D. breast skin
E. all answer’s are wrong.
1421. Akantolizis (acantholysis) is:
A. increased cell proliferation thorny layer
B. narushenieprotsessa keratinization
C. Loss of communication between the cells of the thorny layer *
D. atypical nuclei of epithelial cells priopuholyah
E. all answer’s are wrong.
1422. parakeratosis is incomplete keratinization epidermotsitov accompanied by dystrophy:
A. granular layer *
B. basal layer
C. papillary layer
D. mesh layer
E. all answer’s are wrong.
1423. granulosa is:
A thickening of the granular layer *
B. thinning of the granular layer
C. Structural changes squamous cell
D. intracellular edema
E. all answer’s are wrong.
1424. anaplasia is:
A nuclear atypia of epithelial cells in tumors *
B. zernistosttsitoplazmy cells
C. Loss of communication between the cells of the thorny layer
D. vacuolization of the cytoplasm
E. all answer’s are wrong.
1425. Spongiosis is:
A. intercellular edema vMalpigievom layer *
B. cheesy disintegration of tissues
C. vacuolar degeneration in the Malpighian layer
D. degenerative changes in the dermis
E. all answer’s are wrong.
1426. Acanthosis not properties:
A. scleroderma *
B. lichen planus
C. neurodermatitis
D. psoriasis
E. all answer’s are wrong.
1427. Under what dermatoses marked acantholysis:
A. Eczema
V. contact dermatitis
C. vulgar ecthyma
D. vulgar pemphigus *
E. all answer’s are wrong.
1428. Under what dermatoses granulosa notes:
A. lichen planus *
B. turniol
C. lichen simplex
D. genital warts
E. all answer’s are wrong.
1429. What function is not peculiar to the skin:
A suction
V. formation of toxins *
C. thermoregulation
D. sensory organ
E. all answer’s are wrong.
1430. Angular stomatitis is a variation:
* A streptococcal
B. stafilodermii
C. candidiasis
D. pemphigus
E. all answer’s are wrong.
1431. Shankriformnaya pyoderma most often begins:
A. erosion *
V. with spots
C. with the bubble
D. On the nodule
E. all answer’s are wrong.
1432. Which group of dermatoses is shankriformnaya pyoderma:
A disease, sexually transmitted infections
B. pyoderma *
C. cystic dermatosis
D. dermatitis
E. all answer’s are wrong.
1433. Ritter exfoliative dermatitis is a type of:
A streptococcal
V. contact dermatitis
C. toxicoderma
D. stafilodermii *
E. all answer’s are wrong.
1434. When exfoliative dermatitis Ritter can be detected positive symptom:
A. Nikolsky *
V. Gorchakov-Hardy
C. Sisto
D. Kebnera
E. all answer’s are wrong.
1435. Who is more common psevdofurunkuleze Finger:
A. neonates and infants *
B. old people
C. teenagers
D. in women
E. all answer’s are wrong.
1436. What factors contribute to the development of acne:
A hyperactivity of the sebaceous glands *
B. frequent colds
C. Violations of thermoregulation
D. exercise
E. all answer’s are wrong.
1437. With a variety of acne supporting factor are mites Zheleznitsa:
A. acne vulgaris
B. rosacea *
C. spherical acne
D. nekroticheskne acne
E. all answer’s are wrong.
1438. What kind of spots characteristic of the erythematous stage demodekoza:
A. telangiectasia *
B. petechiae
C. ecchymosis
D. vibitsess
E. all answer’s are wrong.
1439. For Rosacea is characterized by the absence of:
A. komedonov *
B. follicular papules
C. pustules
D. rhinophyma
E. all answer’s are wrong.
1440. What are the morphological features characteristic of hidradenitis sup-:
A thick, welded to the skin painful nodes *
B. dense, mobile, painless nodes
C. blisters
D. lenticular papules
E. all answer’s are wrong.
1441. With the opening of units hidradenitis stands at:
A gummy exudate opalescent
B. creamy pus mixed with blood *
C. serous fluid
D. purulent necrotic core
E. all answer’s are wrong.
1442. favus called:
A virus filter
B. corynebacteria
C. Chlamydia
D. antropofilnymi fungus *
E. all answer’s are wrong.
1443. The typical localization at erythrasma:
A. inguinal folds *
B. scalp
C. okolonogtevogo roller
D. interdigital gaps feet
E. all answer’s are wrong.
1444. What is a symptom characteristic of infiltrative-suppurative trihofitii:
A glow under Wood's lamp
B. bee combs *
C. symptom Pincus
D. Nikolsky sign
E. all answer’s are wrong.
1445. With the laboratory diagnosis of fungal infections is used:
A. microsporia damaged hair and flakes *
B. histological examination
C. fluorescent diagnostics
D. smears from lesions
E. all answer’s are wrong.
1446. What is characteristic of psoriasis:
Net A. Wickham
B. Symptom falling through probe
C. triad Auspittsa *
Sample D. Balzer
E. all answer’s are wrong.
1447. The source of infection with the disease Borowski is urban:
A.gryzuny
B. Hymenoptera
C. People *
D. cloven-hoofed
E. all answer’s are wrong.
1448. The source of infection with the disease Borowski rural type is:
A. People
B. rodents *
C. wild animals
D. cattle
E. all answer’s are wrong.
1449. Enter the average length of the current urban cutaneous leishmaniasis:
A. 2 weeks
B. 3-4 months
C. for about a year *
D. days
E. all answer’s are wrong.
1450. Enter the average length of the current cutaneous leishmaniasis rural type:
A 2-6 month *
B. a year
C. for several years
D. days
E. all answer’s are wrong.
1451. The source of infection of leprosy is:
A. The wild animals
B. persons *
C. rodents
D. insects
E. all answer’s are wrong.
1452. The material from which the patient is most often found Mycobacterium leprae:
A secret sweat glands
B. urine
C. in sebum
The D. scrapings from leprosy and nasal mucosa *
E. all answer’s are wrong.
1453. Which viruses cause genital warts:
A filterable viruses *
B. herpesviruses
C. Herpes Simplex Virus
D. CMV
E. all answer’s are wrong.
1454. Molluscum contagiosum is ...
A. erythema
B. pustule
C. vesicle
D. papule *
E. all answer’s are wrong.
1455. Enter variety of true eczema:
A. pruriginous *
B. nummulyarnaya
C. seborrheic
D. varicose
E. all answer’s are wrong.
1456. What is a symptom seen with pink zoster Gibert:
A symptom of chips
Vsimptom "parent" plaque *
C. symptom ladies Heel
D. Koebner phenomenon
E. all answer’s are wrong.
1457. Monomorphic rash may be at:
AB erythema multiforme exudative
B. Microbial eczema
C. dermatitis Duhring
D. psoriasis *
E. all answer’s are wrong.
1458. Akantolizis observed at:
A herpes simplex
B. true pemphigus *
C. shingles
D. atopic dermatitis
E. all answer’s are wrong.
1459. Monomorphic papular rash occurs when:
A. lichen planus *
B. urticaria
C. eczema
D. lupus erythematosus
E. all answer’s are wrong.
1460. Acanthosis typical for:
A. lichen planus *
B. turniol
C. lichen simplex
D. simple herpes
E. all answer’s are wrong.
1461. hyperkeratosis characteristic of:
Bullous impetigo A.
B. warts *
C. folliculitis
D. demodekoz
E. all answer’s are wrong.
1462. The most superficial skin are:
A. ointment
B. paste
C. patch
D. gadgets *
E. all answer’s are wrong.
1463. What type of topical treatment should be selected when moist skin inflammation:
A. ointment
B. chatterbox
C. gadgets *
D. varnishes
E. all answer’s are wrong.
1464. What type of lotions should be selected for the weeping of the infected skin surface:
A zinc
B. rivanolevuyu *
C. tannic
D. boric acid
E. all answer’s are wrong.
1465. What type of external therapy to choose in chronic infiltrative inflammation of the skin:
A zinc paste
B. Oil-mash
C. 5% Naftalan ointment *
D. drying up wet bandage
E. all answer’s are wrong.
1466. What are the external agents have antipruritic property:
A. tar
B. menthol *
C. salicylic acid
D. Zinc
E. all answer’s are wrong.
1467. Which means external treatment has cauterizing property:
A solution of sodium giposulfata
V. solution podofillina *
C. The solution furatsillina
D. methylene blue solution
E. all answer’s are wrong.
1468. For the destruction of the skin are used:
A liquid nitrogen *
B. zinc oxide
C. Potassium permanganate
D. paint Castellane
E. all answer’s are wrong.
1469. The method is suitable for cryoablation treatment:
A broad warts
B. plaque psoriasis
C. Limitations of neurodermatitis
D. In warts *
E. all answer’s are wrong.
1470. If the skin biopsy revealed fusion of epithelial intercellular bridges in the spinous layer of
the epidermis. Ttsanka detected cells. Specify the nature of the disease process:
A. granulosa
B. acantholysis *
C. parakeratosis
D. papillomatosis
E. all answer’s are wrong.
1471. What factors contribute to the development of pyoderma:
A gene mutations
B. UVR exposure
C. Microfracture *
D. worm infestation
E. all answer’s are wrong.
1472. What factors contribute to the development of pyoderma:
A. diabetes *
Hemophilia B.
C. Violations of the fission mechanism epidermotsitov
D. Violations of keratinization
E. all answer’s are wrong.
1473. It includes Stafilodermiyam:
A. hydradenitis *
B. lichen simplex
S. slit impetigo
D. vulgar ecthyma
E. all answer’s are wrong.
1474. What pyoderma found only in newborns and infants:
A. hydradenitis
B. pemphigus vulgaris
C. Chronic ulcerative pyoderma
D. vezikulopustulez *
E. all answer’s are wrong.
1475. What pustular skin diseases do not occur in the newborn:
A. epidemic pemphigus
B. hydradenitis *
C. psevdofurunkulez Finger
D. Ritter dermatitis
E. all answer’s are wrong.
1476. What pustular disease does not occur in adults:
A. boils
B. turniol
C. angular stomatitis
Exfoliative dermatitis D. Ritter *
E. all answer’s are wrong.
1477. What are the morphological features characteristic of dermatitis Ritter:
A. papules, blisters
V. bumps, crust
C. crust bubbles *
D. vesicles, papules
E. all answer’s are wrong.
1478. What dermatoses in the first place should be differentiated eksfolliativny dermatitis Ritter:
Dermatitis herpetiformis Duhring A.
B. bullous form strofulyusa
C. epidermolysis bullosa *
D. herpes simplex
E. all answer’s are wrong.
1479. Multiple abscesses in children causes:
A. streptobatsilla Dyukreya-Petersen
V. Staphylococcus aureus *
C. Donovan corpuscles
D. Streptococcus
E. all answer’s are wrong.
1480. Psevdofurunkulez Finger refers to the group:
A. stafilodermy *
B. parasitic skin diseases
C. Viral Diseases
D. complications of scabies
E. all answer’s are wrong.
1481. What factors contribute to the development psevdofurunkuleza Finger:
A. overheating of the newborn *
B. disruption of the sebaceous glands
C. endocrinopathies
D. Pregnancy
E. all answer’s are wrong.
1482. What factors contribute to the development of acne:
A frequent colds
B. Violations of thermoregulation
C. Dysfunction sebaceous glands *
D. Violations of the functions of the sweat glands
E. all answer’s are wrong.
1483. What kind of spots characteristic of the erythematous stage demodekoza:
A. telangiectasia, erythema *
B. ecchymosis, spots
C. erythema
D. roseola, pustules
E. all answer’s are wrong.
1484. pineal "the nose is typical for:
A. acne vulgaris
B. rosacea *
C. discoid lupus
D. leprosy
E. all answer’s are wrong.
1485. What factors are important in the development of seborrhea:
A dysfunction of the sweat glands ekkrinnyh
B. dysfunction apocrine sweat glands
C. thyroid dysfunction
D. Dysfunction sebaceous glands *
E. all answer’s are wrong.
1486. Enter the varieties of oily seborrhea:
A. Dry
B. mukovidnaya
C. liquid *
D. the leaf
E. all answer’s are wrong.
1487. Development of dermatosis facilitated by the presence of seborrhea:
A. trihofitii
V. baldness *
C. impetigo
D. pink lichen Gibert
E. all answer’s are wrong.
1488. For a dermatosis characterized by the presence of comedones:
A. Rosacea
V. limited neurodermatitis
C. psoriasis
D. vulgaris *
E. all answer’s are wrong.
1489. If any skin disease alopecia can be observed:
A. Psoriasis
B. hydradenitis
C. Lichen planus *
D. erythema nodosum
E. all answer’s are wrong.
1490. What factors contribute most to the development of alopecia areata:
A. neuroendocrine abnormalities *
B. The deficiency disease
C. With vitamin deficiency
D. Improved insolation
E. all answer’s are wrong.
1491. What factors contribute to the development of hidradenitis sup-:
A. sweating *
V. professional skin contamination
C. hypersensitivity to iodine
D. micronutrient deficiencies
E. all answer’s are wrong.
1492. Enter the location of nodes in the hydradenitis:
A bridge of the nose, the sky
B. shin
C. armpits *
D. interscapulum
E. all answer’s are wrong.
1493. Favourite localization rash scabies:
A. interdigital gaps feet
B. interdigital folds of brushes *
C. skin of the face, neck,
D. leather trunk
E. all answer’s are wrong.
1494. For scabies is characterized by:
A pair papules vesicular elements *
B. polygonal papules
C. sgruppirvannyh small bubbles
D. peeling
E. all answer’s are wrong.
1495. Specify a rare localization of scabies rash in children:
A scalp *
B. genital skin
C. Leather breast
D. abdominal skin
E. all answer’s are wrong.
1496. Enter the drug used in the pemphigus vulgaris:
A. prednisolone *
B. dapsone
C. monomitsin
D. Nystatin
E. all answer’s are wrong.
1497. What is the primary morphological element is preceded by erosion:
A papule
B. vesicle *
C. urtika
D. macula
E. all answer’s are wrong.
1498. What disease is not included in the group of dermatophytosis:
A. favus
B. B. rubrofitii
C. trichophytosis
D. chromomycosis *
E. all answer’s are wrong.
1499. Enter the drug used in leprosy:
A. norsulfazol
B. monomitsin
C. delagil
D. dapsone *
E. all answer’s are wrong.
1500. In the treatment of candidiasis is not used:
A. acyclovir *
B. Nizoral
C. Nystatin
D. Diflucan
E. all answer’s are wrong.
1501. Symptom Nikolsky positive at:
A true pemphigus *
B. skrofuloderma
Dermatitis herpetiformis Duhring C.
D. focal neurodermatitis
E. all answer’s are wrong.
1502. pathognomonic for eczema sprinkler elements are:
A. erythema
B. peel
C. oozing erosion *
D. papules
E. all answer’s are wrong.
1503. What are the main clinical signs are not specific for chronic eczema:
A. infiltration
B. lichenification
C. bright redness, swelling *
D. venous congestion
E. all answer’s are wrong.
1504. If any of these viral diseases of the mucosa of the mouth is not affected:
A. molluscum contagiosum *
B. Chickenpox
C. simple herpes
D. shingles
E. all answer’s are wrong.
1505. What's not affected in rubrofitii:
A. soles
B. hair *
C. nail brushes
D. nails feet
E. all answer’s are wrong.
1506. The method of staining for Ziehl-Nielsen which pathogens are defined by:
A. Chlamydia
B. pale treponema
C. Koch's bacillus *
D. ureaplasma
E. all answer’s are wrong.
1507. Enter the characteristic appearance and location of the rash of pink lichen:
A congested spot on the body *
B. blister
C. vesicle
D. Facial
E. all answer’s are wrong.
1508. Enter the clinic at toxicoderma:
A. syndrome asthenoneurotic
B. Sezary syndrome
C. Cushing's syndrome-Itsengo
D. Lyell's syndrome *
E. all answer’s are wrong.
1509. Enter the drug used in psoriasis:
A. blemaren
B. ethambutol
C. Dexamethasone *
D. dithranol
E. all answer’s are wrong.
1510. Enter the symptoms of lupus erythematosus
A symptom Gorchakov -Ardi
B. symptom of asbestos-Hansen
C. symptom Nikolsky
D. symptom Besnier -Mescherskogo *
E. all answer’s are wrong.
1511. After some initial elements formed by erosion:
A. abscess *
B. Spot
C. tubercle
D. Node
E. all answer’s are wrong.
1512. What are the phenomena included in psoriatic triad:
A ladies' heel
B. phenomenon apple jelly
C. phenomenon stearin spots *
St. Anthony D. Mask
E. all answer’s are wrong.
1513. Streptococcal pyoderma differentiated with what disease:
A herpes simplex *
B. erythrasma
C. CPL
D. psoriasis
E. all answer’s are wrong.
1514. Deep trichophytosis differentiated c what diseases:
A. erythrosis
B. eczema
C. microsporia *
D. psoriasis
E. all answer’s are wrong.
1515. Which diseases are not observed site:
A leprosy
B. herpes *
C. cutaneous tuberculosis
D. tertiary syphilis
E. all answer’s are wrong.
1516. In the dermis layers are distinguished:
A grainy
Rogov
C. papillary *
D. basal
E. all answer’s are wrong.
1517. By the atypical chancre include:
A wide warts
B. balanopostit
C. chancre-amygdala *
D. balanitis
E. all answer’s are wrong.
1518. Enter the incorrect statement: primary syphiloma can oslozhnyatsyaA. orchitis *
B. paraphimosis
C. gangrenizatsiya
D. fagedinizmom
E. all answer’s are wrong.
1519. Enter the incorrect statement: possible causes of simple dermatitovA low temperatures
B. High Temperature
C. ingestion of drugs *
D. friction pressure
E. all answer’s are wrong.
1520. For localized scleroderma is uncommon stage:
A. induration
B. nefollikulyarnogo hyperkeratosis *
C. atrophy
D. edema
E. all answer’s are wrong.
1521. In the treatment of candidiasis is not used:
A. Nystatin
B. polkortolon *
C. intrakenazol
D. levorin
E. all answer’s are wrong.
1522. In the treatment of head lice is not used:
A. mikozolon *
B. 10-20% benzyl benzoate emulsion
C. spregal
D. nittifor
E. all answer’s are wrong.
1523. erythrodermic psoriasis can develop after:
A topical ointment 5% salicylic acid
B. Application of UFO patients with psoriasis type summer
C. emotional experiences
D. Designated ACTH *
E. all answer’s are wrong.
1524. Enter the secondary elements that are the result of transformation of the node:
A scar atrophy
B. secondary pigmentation *
C. ulcer
D. scar
E. all answer’s are wrong.
1525. Enter the clinical signs are not specific to tertiary syphilis:
A. gumma palate
B. mezaortit
C. nodes
D. A warts *
E. all answer’s are wrong.
1526. What are the main clinical signs are not specific for chronic eczema:
A bright redness *
B. infiltration
C. lichenification
D. crack
E. all answer’s are wrong.
1527. Under soak for topical treatment of eczema only apply:
A. creams
B. gadgets *
C. powders
D. powder
E. all answer’s are wrong.
1528. Enter the feature is not characteristic for discoid lupus erythematosus form:
A. Symptom "butterfly".
B. Symptom Besnier-Meshchersky.
C. The symptom of "apple jelly" *
D. follicular hyperkeratosis.
E. all answer’s are wrong.
1529. What disease is not included in the group neyrodermatozov:
A. scrapie
B. urticaria
C. itching
D. allergiichesky dermatitis *
E. all answer’s are wrong.
1530. What is not typical in a clinical picture of rosacea:
A. follicular hyperkeratosis *
B. erythema
C. infiltration of the skin of the nose
D. increase in the size of the nose
E. all answer’s are wrong.
1531. Indicate the most characteristic feature of scabies:
A serous well
B. evening and night itching *
C. infiltration lesions
D. polyadenylation
E. all answer’s are wrong.
1532. Enter the following among stafilodermii surface shape:
A boil
B. osteofollikulit *
C. carbuncle
D. hydradenitis
E. all answer’s are wrong.
1533. In the treatment of lichen planus can be used:
A. novopassit *
B. prednisolone
C. androgens
D. All of these
E. all answer’s are wrong.
1534. In the primary period of syphilis patients can meet all of the features kromeA negative Wasserman
B. lymphangitis
C. chancre
D. non-erosive papules *
E. all answer’s are wrong.
1535. Choose the most effective means for the treatment of chlamydial urethritis:
A. penicillin
B. tetracycline *
C. metronidazole
D. retarpen
E. all answer’s are wrong.
1536. Of these clinical signs are not typical of AIDS patients:
A. lymphoma
B. recurrent herpes
C. diarrhea
D. vitiligo *
E. all answer’s are wrong.
1537. Enter the drug used in the pemphigus vulgaris:
A. monomitsin
B. orungal
C. prednisolone *
D. dapsone
E. all answer’s are wrong.
1538. Specify the medication used to treat scabies:
A. Vishnevsky ointment
B. 10% ointment sintomitsinovaja
C. Spray Spregal *
D. 2% salicylic ointment
E. all answer’s are wrong.
1539. Enter the drug used in skin candidiasis:
A. orungal *
B. furazolidon
C. Bactrim
D. Biseptol
E. all answer’s are wrong.
1540. Enter the drugs used in the progressive stage of psoriasis:
A 2% ointment salitsilovayay *
B. 10% ointment salitsilovayay
C. ointment dermatol
D. 10% ointment ihtiolovaya
E. all answer’s are wrong.
1541. Enter the phenomena characteristic of lichen planus:
A fish eggs
B. tissue paper
C. Wickham Net *
D. «Ladies' heel"
E. all answer’s are wrong.
1542. Enter the symptoms characteristic of the multi-colored lichen:
A sample Wickham
B. Balzer sample *
C. tissue paper
D. Triad auspices
E. all answer’s are wrong.
1543. Enter the drug that you can not appoint resistant white dermographism:
A sodium thiosulfate
B. Calcium Gluconate *
C. Medicine Pavlova
D. potassium iodide
E. all answer’s are wrong.
1544. Enter the drug used with shingles:
A. delagil
B. dapsone
C. oxoline *
D. prednisolone
E. all answer’s are wrong.
1545. Enter the clinical signs characteristic of sycosis:
A chronic relapsing course *
B. rheumatic pains
C. nodules
D. excessive sweating
E. all answer’s are wrong.
1546. Clinical signs of herpes simplex virus does not apply:
A. keratoconjunctivitis
B. lichen sclerosus *
C. eczema herpeticum
D. meningoencephalitis
E. all answer’s are wrong.
1547. Which of the diseases are not a manifestation of HIV infection:
A. Kaposi's sarcoma
B. diarrhea
C. herpes simplex
D. vitiligo *
E. all answer’s are wrong.
1548. What is not typical for psoriasis:
A. may first appear after birth
B. You can call the animals by laboratory *
C. associated with stress
D. in the occurrence of the disease plays a role heredity
E. all answer’s are wrong.
1549. Enter the characteristic sign of a blister:
A. is due to intracellular edema *
V. leaves behind erosion
C. peeling on the surface of the blister
D. Lack of subjective sensations
E. all answer’s are wrong.
1550. What are the clinical manifestations characteristic of secondary recurrent syphilis:
A. parenchymal keratitis
B. indurativnyy swelling
C. fagedenizatsiya
D. leucoderma *
E. all answer’s are wrong.
1551. What are the clinical symptoms in patients with primary syphilis:
A. indurativnyy swelling
B. extensive warts *
C. regional lymphangitis
D. ulcerative chancre
E. all answer’s are wrong.
1552. Specify the characteristic signs of uncomplicated chancre:
A. saped edge
B. dense infiltrate the base of the ulcer *
C. treponem absence in the discharge sores
D. purulent discharge
E. all answer’s are wrong.
1553. For patients with chancroid is unusual:
A. The signs of inflammation
B. marked tendency to autoinnokulyatsii
C. solid infiltration *
D. lesions formed after 2-3 days on-site implementation streptobatsill
E. all answer’s are wrong.
1554. In the treatment of bacterial vaginosis is used:
A. penicillin
B. Erythromycin
C. Metronidazole *
D. ampicillin
E. all answer’s are wrong.
1555. For the diagnosis of the disease applied research using Wood's lamp:
A. microsporia *
B. Pink zoster.
C. vitiligo.
D. psoriasis.
E. all answer’s are wrong.
1556. What is the primary morphological element is preceded by erosion:
A. abscess *
B. none of these
C. tubercle
D. Node
E. all answer’s are wrong.
1557. What is the primary morphological element is preceded by an ulcer:
A bubble
B. blister
C. tubercle *
D. None of the above
E. all answer’s are wrong.
1558. Enter the signs typical for uncomplicated chancre:
A. saped edge
B. purulent discharge
C. dense infiltrate the base of the ulcer *
D. signs of inflammation
E. all answer’s are wrong.
1559. What pustular skin diseases do not occur in the newborn:
A. Sycosis *
B. vezikulopustulez
C. psevdofurunkulez Finger
Exfoliative dermatitis D. Ritter
E. all answer’s are wrong.
1560. Enter the varieties of seborrhea:
A leafB. Dry *
C. papulonekroticheskaya
D. scaly
E. all answer’s are wrong.
1561. What is the clinical symptom seen with lupus erythematosus:
A. "ladies' heel" *
B. "blood dew"
C. symptom Pincus
D. grain Trela
E. all answer’s are wrong.
1562. What are the atypical form of psoriasis:
A scalp
B. intertriginoznoy *
Coin-C.
D. numulyarny
E. all answer’s are wrong.
1563. Indicate the most severe form of psoriasis:
A. Geography
B. erythrodermic *
C. numulyarny
D. palmoplantar
E. all answer’s are wrong.
1564. What are the elements in the localization of pustular psoriasis Barbera:
A red border
B. soles *
C. Leather body
D. scalp
E. all answer’s are wrong.
1565. What kind of pustular psoriasis:
A. lichenoid
B. psoriasis Tsumbusha *
C. figured
D. erythrosis
E. all answer’s are wrong.
1566. What is the clinical sign characteristic of dermatitis Dühring:
A small focal baldness
B. sclerodactyly
C. esophagitis
D. eosinophilia content bladder *
E. all answer’s are wrong.
1567. What are the clinical varieties of cutaneous leishmaniasis you know:
Undifferentiated type A.
B. erythematous-squamous type
C. lepromatous type
D. late-ulcerated type *
E. all answer’s are wrong.
1568. What kinds of clinical disease Borowski:
A city model *
B. undifferentiated
C. infiltrative-suppurative
D. papuzezny
E. all answer’s are wrong.
1569. Which layer is not part of the epidermis:
A grainy
B. papillary *
C. basal
D. horn
E. all answer’s are wrong.
1570. Enter the varieties of seborrhea:
A leaf-
B. fat *
C. vulgar
D. disgidroticheskaya
E. all answer’s are wrong.
1571. What diseases are a group of stafilodermy:
A. epidemic pemphigus *
B. impetigo nail ridges
C. slit ecthyma vulgar impetigo
D. botryomycoma
E. all answer’s are wrong.
1572. What means have antipruritic property:
A. tar
B. menthol *
C. Sulfur
D. ihtiola
E. all answer’s are wrong.
1573. What pyoderma found only in newborns and infants:
A. hydradenitis
B. boil
C. Chronic ulcerative pyoderma
D. vezikulopustulez *
E. all answer’s are wrong.
1574. What are the morphological features characteristic of dermatitis Ritter:
A papule
B. bubbles *
C. Small bubbles
D. blisters
E. all answer’s are wrong.
1575. Localization rash scabies:
A neck skin
B. the skin around the navel, inner thighs *
C. interdigital gaps feet
D. places where sebaceous glands
E. all answer’s are wrong.
1576. What are the secondary morphological element:
A vegetation *
B. knot
C. abscess
D. bump
E. all answer’s are wrong.
1577. What proliferative morphological element:
A. abscess
B. bubble
C. knot *
D. bubble
E. all answer’s are wrong.
1578. acantholysis is the morphological basis of symptom:
A. Hansen of asbestos *
V. Pospelov
C. Wickham
D. Andogsky
E. all answer’s are wrong.
1579. What is the clinical evidence harakterendlya dermatitis Dühring:
A. cryoglobulinemia
B. leukopenia
C. eosinophilia in the blood *
D. leukocytosis
E. all answer’s are wrong.
1580. What tests are used for the diagnosis of dermatitis herpetiformis Duhring:
A3's a glass sample Thompson
B. iodine sample Balzer
C. Sample Minor
D. 50% potassium iodide ointment *
E. all answer’s are wrong.
1581. Which drugs are most effective in dermatitis Dühring:
A. Biseptol
B. VAT *
C. cytostatic
D. antibiotics
E. all answer’s are wrong.
1582. Enter papules especially when krasnomploskom zoster:
A copious desquamation
B. peeling edge
C. umbilicated *
D. peripheral bead
E. all answer’s are wrong.
1583. What kind of pustular psoriasis:
A. psoriasis sarcoma
B. lichenoid
C. annular
D. Barber psoriasis *
E. all answer’s are wrong.
1584. What are the areas of skin are usually free from lesions in the CPL:
A. palms, soles *
B. flexor surface of the limbs
C. wrist joint
D. flexor surface of the forearm
E. all answer’s are wrong.
1585. What should be differentiated dermatoses lichen planus:
A. the artificial dermatitis
B. chromophytosis
C. psoriasis *
D. dermatitis Duhring
E. all answer’s are wrong.
1586. What are the elements in the localization of pustular psoriasis Barbera:
A. palms, soles *
B. scalp
C. red border
D. Opening of the parts of the body
E. all answer’s are wrong.
1587. Enter variety of true eczema:
A. pruriginous, disgidroticheskaya *
B. nummulyarnaya
C. seborrheic, varicose
D. mycotic
E. all answer’s are wrong.
1588. rash of erythema multiforme exudative located:
A. grouped along nerves
B. pairs
C. in the form of rings *
D. asymmetrically platform
E. all answer’s are wrong.
1589. Enter the characteristic signs of scabies:
A merged papules
B. symptom Gorchakov-Hardy *
C. grouped vesicles plaque
D. Net Wickham
E. all answer’s are wrong.
1590. Which drugs are used to lecheniyachesotki:
A. 20% benzyl benzoate ointment *
B. 5% of Naftalan ointment
C. 5% sulfuric ointment
D. hydrofoil
E. all answer’s are wrong.
1591. With what diseases should be differentiated scabies:
A. atopic dermatitis *
B. Pink zoster
C. shingles
D. multicolored lichen
E. all answer’s are wrong.
1592. What obligate factors for dermatitis:
A gold jewelery
B. detergents
C. Concentrated lye *
D. jewelry made of platinum
E. all answer’s are wrong.
1593. Allergic reactions to the medication may occur as:
A symptom Pilnova
B. symptom Jadassohn
C. Erythema *
D. symptom Balzer
E. all answer’s are wrong.
1594. Enter toxicoderma varieties:
A fixed erythema *
B. psoriasis
C. perfrigeration
D. pellagrozny dermatitis
E. all answer’s are wrong.
1595. What medications often cause erythema fixed:
A. tavegil
B. tetracycline
C. analginum
D. sulfadimetoksin *
E. all answer’s are wrong.
1596. What should be differentiated disease spread toxicoderma:
A. exfoliative dermatitis *
B. pyoderma
C. rosacea
D. disease Borowski
E. all answer’s are wrong.
1597. Enter the typical localization exudative erythema multiforme at:
A scalp
B. interdigital spaces of hands
C. mucosal *
D. neck
E. all answer’s are wrong.
1598. What factors are important in the development of exudative erythema multiforme:
A hypersensitivity to fluoride
B. foci of chronic infection *
C. hypersensitivity to iodine
D. photosensitivity
E. all answer’s are wrong.
1599. What dermatoses differentiate erythema multiforme:
A vulgar pemphigus *
B. strofulyus
C. skrofuloderma
D. Bazin induratum erythema
E. all answer’s are wrong.
1600. Enter the pathogenetic factors of pink lichen Gibert:
A. yeasts
B. hypofunction of the sebaceous glands
C. viral infection *
D. idiosyncrasy to iodine
E. all answer’s are wrong.
1601. Enter the variety of microbial eczema:
A varicose *
V. adult
C. idiopathic
D. horn
E. all answer’s are wrong.
1602. How is a clinical sign is characterized by a rash in true eczema:
A symptom of "serous well" *
B. tubercles
C. monomorphic papules
D. ephemeral blisters
E. all answer’s are wrong.
1603. What are the symptoms seen with discoid lupus:
A collar Biett
B. symptom of "ladies' heel" *
C. Net Uithema
D. silvery white, easily removable scales
E. all answer’s are wrong.
1604. Enter the variety of scleroderma:
A vulgar
B. erythematous
C. infiltrative
D. disease blind spots *
E. all answer’s are wrong.
1605 Lichen planus is characterized by:
A polymorphism
B. lack of itching
C. Wickham Net *
D. urticaria rashes
E. all answer’s are wrong.
1606. For planus is characterized by:
A. parakeratosis
B. vnutrienidermalnye mikroabtsessy
C. uneven thickening of the granular layer of the epidermis *
D. akantolizis
E. all answer’s are wrong.
1607. What are the clinical symptoms pathognomonic planus:
A grain Trela
B. symptom auspices
C. umbilicated *
D. "honeycomb Celsus"
E. all answer’s are wrong.
1608. What diseases do not belong to a group of viral dermatoses:
A. Molluscum Contagiosum
B. shingles
C. simple herpes
D. psoriasis *
E. all answer’s are wrong.
1609. Vascular spot is:
A. Erythema *
B. Lentigo
C. Leucoderma
D. Nevus
E. all answer’s are wrong.
1610. Primary morphological elements:
A. lihenifikatsiya
B. atrophy
C. ulcer
D. vial *
E. all answer’s are wrong.
1611. Secondary morphological features:
A bubble
B. crust *
C. blister
D. knot
E. all answer’s are wrong.
1612. The bubble is formed by:
A. psoriasis
B. CPL
C. multicolored shingles
D. herpes simplex *
E. all answer’s are wrong.
1613. The outcome of the node is:
A scar
B. flake *
C. atrophy
D. crust
E. all answer’s are wrong.
1614. exudative morphological elements are:
A. abscess *
B. The Node
C. papule
D. bump
E. all answer’s are wrong.
1615. The layers of the epidermis:
A. Net
B. papillary
C. fat
D. granular *
E. all answer’s are wrong.
1616. Have cavity morphological elements:
A bubble *
B. papule
C. Node
D. blister
E. all answer’s are wrong.
1617. rash polymorphic at:
A. warts
B. eczema *
C. lichen planus
D. molluscum contagiosum
E. all answer’s are wrong.
1618. monomorphic rash when dermatoses:
A rash *
B. dermatitis Duhring
C. tertiary syphilis
D. eczema
E. all answer’s are wrong.
1619. Clinical signs of psoriasis:
A phenomenon of Koebner *
B. blistering
C. hemorrhagic spots
D. blisters
E. all answer’s are wrong.
1620. Clinical signs of lichen planus:
A. vorotnichek Biett
B. blisters
C. pupkoobraznym depressions in the center *
D. bumps
E. all answer’s are wrong.
1621. Means, used locally for the treatment of psoriasis:
A. Castellani paint
B. Naftalan ointment
C. benzyl benzoate ointment
D. salicylic ointment *
E. all answer’s are wrong.
1622. The phenomena characteristic of psoriasis:
A stearin spots *
B. paired elements
C. Wickham
D. ladies Heel
E. all answer’s are wrong.
1623. Signs of true polymorphism rash when dermatitis Dühring:
A. bubbles *
B. blister
C. tubercle
D. Node
E. all answer’s are wrong.
1624. Clinical varieties of pemphigus:
A pustular
B. vulgar *
C. exudative
D. papular
E. all answer’s are wrong.
1625. For the characteristic symptoms of lupus:
A. erythema *
B. Net Wickham
C. mukovidnoe peeling
D. symptom Kebnera
E. all answer’s are wrong.
1626. For the characteristic symptoms of lupus:
Net A. Wickham
B. follicular hyperkeratosis *
C. symptom Kebnera
D. triad auspices
E. all answer’s are wrong.
1627. Clinical forms of athlete's foot:
A. psoriaziformnaya
B. erased
C. disgidroticheskaya *
D. Dissemination
E. all answer’s are wrong.
1628. Diseases related to dermatophytes:
A. trichophytosis *
B. erythrasma
C. chromomycosis
D. impetigo
E. all answer’s are wrong.
1629. Antibiotics used in the treatment of candidiasis:
A. macrolides
B. flunol *
C. doxycycline
D. kanamycin
E. all answer’s are wrong.
1630. When artropaticheskom advisable to appoint all of the above, kromeA. essentiale
B. antioxidants
C. mineralocorticoid drugs
D. Nizoral *
E. all answer’s are wrong.
1631. Typical elements sprinkler planus have the following features, kromeA preferential localization on the surface of the flexion of the lower limbs
B. symptom Pilnova *
C. waxy shine
D. Wickham grid on the surface of papules
E. all answer’s are wrong.
1632. Monomorphic rash may be at:
A. psoriasis *
B. erythema multiforme exudative
C. microbial eczema
D. dermatitis Duhring
E. all answer’s are wrong.
1633. hyperkeratosis characteristic of:
A. ichthyosis *
Bullous impetigo B.
C. Sycosis
D. folliculitis
E. all answer’s are wrong.
1634. A woman of 25 years old, married for 3 years. The examination revealed chronic
endocervicitis, adnexitis. In smears - leukocytosis. Gonorrhea and trichomonas are not found. The
daughter of 1.5 years of acute vulvovaginal gonorrheal etiology. The correct tactics of the doctor:
A. The purpose of the combination of provocation followed by a bacteriological examination and
bacterioscopic *
B. appointment antihistamines
C. antifungals
D. vaccination
E. all answer’s are wrong.
1635. What type of topical treatment should be selected when moist skin inflammation:
A. ointment
B. aniline dyes *
C. Cream
D. varnishes
E. all answer’s are wrong.
1636. The patient was diagnosed with anterior urethritis chlamydial infection. Antibiotic group to
appoint a patient:
A. aminoglycosides *
B. antimalarial
C. fluorinated quinolones
D. In-generation cephalosporins
E. all answer’s are wrong.
1637. Girls vulvovaginitises trichomonas etiology characterized by all of the above symptoms,
kromeA cheesy discharge from the genital slit *
B. infiltrative foci
C. frequent and painful urination
D. itch in the vulva
E. all answer’s are wrong.
1638. What type of lotions should be selected for the weeping of the infected skin surface:
A potassium permanganate with *
B. zinc
C. Lead
D. tannic
E. all answer’s are wrong.
1639. Diagnostic tests are tests of allergic dermatitis.
A. Balzer
B. Jadassohn
C. skarifikatsionnye *
D. Minor
E. all answer’s are wrong.
1640. By the frequent complications of atopic dermatitis are all listed, except
A. hyperthyroidism *
B. secondary infection in the outbreak of atopic dermatitis
C. skin atrophy with chronic treatment
D. poor sleep, irritability
E. all answer’s are wrong.
1641. Diagnosis of acute prostatitis is based on the results of these studies kromeA. palpation of the prostate
B. ultrasound prostate
C. blood chemistry *
D. Analysis of 3 servings of urine
E. all answer’s are wrong.
1642. Examination of children with atopic dermatitis is to find out all of the above, except
A revealing state immunodeficiency
B. Identification of the source of microbial sensitization *
C. Identification of other allergenic factors
D. Assessment of endogenous intoxication
E. all answer’s are wrong.
1643. What are the exogenous factors contribute to the development of pyoderma:
A gene mutations
B. gipervitaminoz
C. hypothermia and hyperthermia *
D. worm infestation
E. all answer’s are wrong.
1644. What are the endogenous factors contribute to the development of pyoderma:
A vitamin deficiencies *
B. eosinophilia
C. hemophilia
D. Violations of the fission mechanism epidermotsitov
E. all answer’s are wrong.
1645. In acute and sub-acute uncomplicated gonorrhea in men and women in the complex
Treatment includes all of the above, kromeA. antispasmodics
B. gonovaktsiny *
C. sulfonamides
D. protivogonokokkovye antibiotics
E. all answer’s are wrong.
1646. Enter the correct tactics of treatment of chronic and torpid gonorrhea infektsii
A. antihistamines
B. calcium supplements
C. detoxification drugs
D. antibacterial *
E. all answer’s are wrong.
1647. Enter the correct tactics of treatment of chronic and torpid gonorrhea infektsii
A. antihistamines
B. calcium supplements
C. local treatment *
D. Antifungal
E. all answer’s are wrong.
1648. It includes Stafilodermiyam:
A carbuncle *
Bullous impetigo B.
C. impetigo nail ridges
D. lichen simplex
E. all answer’s are wrong.
1649. Clinical symptoms of discoid lupus volchankiA. bubbles
B. bubbles
C. atrophy *
D. pustules
E. all answer’s are wrong.
1650. Koebner phenomenon is observed in the following diseases:
A leprosy
B. folliculitis
C. Lichen planus *
D. allergic dermatitis
E. all answer’s are wrong.
1651. Under what dermatoses rash element is the bubble:
A. Psoriasis
B. Pink zoster Gibert
C. pemphigus
D. shingles *
E. all answer’s are wrong.
1652. The true characteristic of polymorphism:
A. eczema *
B. psoriasis
C. planus
D. tinea versicolor
E. all answer’s are wrong.
1653. Find the appropriate answers: proliferative morphological ChangesA. spongiosa,
B. hyperkeratosis *
C. vacuolar degeneration
D. balloniruyaschaya degeneration
E. all answer’s are wrong.
1654. Diffuse hyperkeratosis characteristic of:
A. pemphigus
B. ichthyosis *
C. allergic dermatitis
D. vitiligo
E. all answer’s are wrong.
1655. What factors contribute to the development of pyoderma:
A carbohydrate metabolism *
B. Increased blood clotting
C. gipervitaminoz
D. exercise
E. all answer’s are wrong.
1656. For scabies is characterized by:
A polygonal papules with central retraction
B. follicular hyperkeratosis
C. pair papules vesicular elements *
D. symptom of "apple jelly"
E. all answer’s are wrong.
1657. From the ectoderm develop:
A hair muscles
B. vessels of the skin
C. covering the epidermis *
Dermis D.
E. all answer’s are wrong.
1658. For hidradenitis sup- characterized by:
A. rupioidnye pustules
B. Localization favorite armpits *
C. papules
D. paired elements
E. all answer’s are wrong.
1659. Enter the clinical kind of scabies (form):
A. erythematous
B. squamous
C. Chronic
Norwegian D. *
E. all answer’s are wrong.
1660. It is a mixed pyoderma:
A vulgar impetigo *
B. osteofollikulit
C. carbuncle
D. boil
E. all answer’s are wrong.
1661. Enter the clinical varieties akantolicheskoy pemphigus:
A leaf-shaped *
B. annular
C. Stevens Johnson
D. epidemic pemphigus newborns
E. all answer’s are wrong.
1662. At what dermatosis morphological element is the primary node:
A. Psoriasis
B. leprosy *
C. angioedema
D. urticaria
E. all answer’s are wrong.
1663. Under what dermatoses main element of the rash is pimple:
A. Pemphigus
B. planus *
Pink Gibert C. versicolor
D. shingles
E. all answer’s are wrong.
1664. Clinical symptoms of discoid lupus erythematosus:
A. atrophy *
B. bubbles
C. bubbles
D. blister
E. all answer’s are wrong.
1665. Koebner phenomenon is observed in the following diseases:
A leprosy
B. folliculitis
C. psoriasis *
D. allergic dermatitis
E. all answer’s are wrong.
1666. Under what dermatoses main element of the rash is pimple:
A. psoriasis *
B. Pemphigus
C. shingles
D. erythrasma
E. all answer’s are wrong.
1667. Under what dermatoses rash element is the bubble:
A. Psoriasis
B. eczema *
C. pemphigus
D. erythematosus
E. all answer’s are wrong.
1668. At what dermatosis morphological element is the primary node:
A. Psoriasis
B. leishmaniasis of the skin *
C. angioedema
D. urticaria
E. all answer’s are wrong.
1669. True polymorphism characteristic forA. Duhring dermatitis *
B. psoriasis
C. planus
D. tinea versicolor
E. all answer’s are wrong.
1670. What are the atypical form of primary syphiloma:
A chancre-amygdala *
B. diffuse pigmentation
C. fagedenizm
D. phimosis
E. all answer’s are wrong.
1671. Enter the secondary elements that are the result of transformation of the node:
A scar atrophy
B. complete disappearance *
C. ulcer
D. scar
E. all answer’s are wrong.
1672. Complications chancre:
A. balanopostit *
B. impetigo
C. stafilodermiya Bockhart
D. osteofollikulit
E. all answer’s are wrong.
1673. Which drugs are used for photochemotherapy in psoriasis:
A. acyclovir
B. puvalen *
C. Nystatin
D. Lamisil
E. all answer’s are wrong.
1674. What type of external therapy is prescribed for the progressive stage of psoriasis:
A corticosteroid ointment *
B. 20% benzyl benzoate
C. ointment "psoriasin"
D. Methods Dem'yanovich
E. all answer’s are wrong.
1675. For secondary syphilis is characterized by:
A. bumps
B. roseolous rash *
C. chancre and polyadenylation
D. infiltration Gohzingera
E. all answer’s are wrong.
1676. Which drug is effective for genital herpes:
A. Lamisil
B. Trichopolum
C. Ulkaril *
D. Diprospan
E. all answer’s are wrong.
1677. The clinical picture of the acute form of athlete's foot:
A. redness *
B. bumps
C. peel
D. lihenifikatsiya
E. all answer’s are wrong.
1678. The clinical picture of infiltrative-suppurative trihofitii:
A. lihenifikatsiya
B. blisters
C. inflammatory infiltrate *
D. nodes
E. all answer’s are wrong.
1679. Clinical signs of jock itch:
A. blisters
B. nodes
C. bumps
D. red spots *
E. all answer’s are wrong.
1680. The most frequent localization of the surface of the skin candidiasis:
A. The corners of the mouth *
B. flexor surfaces of the wrists
C. vulva
D. cuticles
E. all answer’s are wrong.
1681. Fungal diseases of the skin causing:
A. trihofiton *
B. Corynebacterium
C. Hansen mycobacterium
D. Mycobacterium Koch
E. all answer’s are wrong.
1682. The main clinical signs of mucosal candidiasis:
A. mucous papules
B. bumps
C. White Film *
D. nodes
E. all answer’s are wrong.
1683. Antibiotics used in the treatment of fungal infections:
A. lincomycin
B. penicillin
C. Nizoral *
D. lindamitsin
E. all answer’s are wrong.
1684. Localization of lesions in scabies:
A scalp
B. nails
C. palm
D. interdigital folds of brushes *
E. all answer’s are wrong.
1685. The main clinical signs of scabies:
A. itching in the evening and at night *
B. nodes
C. acantholysis
D. blisters
E. all answer’s are wrong.
1686. Drugs used to treat scabies:
A tincture of iodine
B. prednisolone ointment
C. nistatinovaya ointment
D. 60% sodium thiosulfate and hydrochloric acid 6% *
E. all answer’s are wrong.
1687. Morphological element characteristic clinic uncomplicated scabies:
A spot
B. bump
C. abscess
D. knot *
E. all answer’s are wrong.
1688. With characteristic symptom of scabies:
A. Gorchakov-Hardy *
B. Besnier-Meshcherskiy
C. Nikolsky
D. Hensen of asbestos
E. all answer’s are wrong.
1689. Pathogens pustular skin diseases:
Av
B.tuberkuleznaya coli
C. aureus *
D. Corynebacterium
E. all answer’s are wrong.
1690. Streptococcus causes of the disease:
A. versicolor opoyasyvayushy
B. simple herpes
C. Sycosis
D. impetigo *
E. all answer’s are wrong.
1691. Species strep impetigo:
A. Zayed *
B. boil
C. simple herpes
D. epidemic pemphigus
E. all answer’s are wrong.
1692. Possible localization of boils:
A neck *
V. oral mucosa
C. lip
D. soles
E. all answer’s are wrong.
1693. Clinical signs typical of staphylococcal deep pustules:
A liquid pus
B. is located on the smooth skin
C. surrounded by a rim of hyperemia
D. riddled hair *
E. all answer’s are wrong.
1694. hydradenitis localized:
A palm
B. shin
C. soles
D. armpits *
E. all answer’s are wrong.
1695. The main clinical forms of staphylococcal skin lesions:
A. chromophytosis
B. shingles
C. vulgar ecthyma
D. folliculitis *
E. all answer’s are wrong.
1696. Clinical varieties of strep impetigo:
Bullous impetigo A. *
B. Sycosis
C. deep folliculitis
D. simple herpes
E. all answer’s are wrong.
1697. Complications occurring in patients with the localization of boils on his face:
A. phlebitis cerebral blood vessels *
B. neuritis of the facial nerve
C. endocarditis
D. neuralgia
E. all answer’s are wrong.
1698. Deep streptococcal form:
A strep impetigo
B. vulgar ecthyma *
Bullous impetigo C.
D. lichen simplex
E. all answer’s are wrong.
1699. Factors causing toxicoderma:
A sulfa drugs *
B. acid
C. alkalis
D. metals (cobalt, nickel, chromium)
E. all answer’s are wrong.
1700. Clinical signs of hives:
A blister *
Node B.
C. Pain
D. pustules
E. all answer’s are wrong.
1701. The stimuli that cause a simple contact dermatitis:
A dosage per os
B. nutritional
C. physical (high and low temperature) *
D. Psychoemotional
E. all answer’s are wrong.
1702. For atopic dermatitis is characterized by:
A. bumps
B. nodes
C. blister
D. Erythema *
E. all answer’s are wrong.
1703. Signs of the true characteristic of eczema:
A. blisters
B. scar atrophy
C. true polymorphism *
D. bumps
E. all answer’s are wrong.
1704.Dlya eczema is characterized by:
A. Moisture "serous wells" *
V. bumps
C. nodes
D. gumma
E. all answer’s are wrong.
1705. Clinical manifestations of fresh secondary period of syphilis:
A. roseola *
B. bubbles
C. blisters
D. A warts
E. all answer’s are wrong.
1706. Clinical signs of syphilitic roseola:
A. itchy
B. painful
C. Do not merge *
D. purple
E. all answer’s are wrong.
1707. Clinical signs of tinea versicolor:
A. defurfuration *
B. papules
C. bumps
D. nodes
E. all answer’s are wrong.
1708. The source of infection at mikrosporii:
A cat, a dog *
B. Cattle
C. rodents
D. Water
E. all answer’s are wrong.
1709. candidiasis affects:
A muscular system
B. follicular unit
C. Skeletal System
D. mucous *
E. all answer’s are wrong.
1710 Primary cells in the herpes simplex:
A blister
B. abscess
C. vial *
D. knot
E. all answer’s are wrong.
1711. For viral diseases include:
A. shingles *
B. eczema
C. lupus erythematosus
D. vulgar sycosis
E. all answer’s are wrong.
1712. For zoster is characterized by:
A. units
B. damage along the nerve endings *
C. blisters
D. nodules
E. all answer’s are wrong.
1713. Clinical manifestations characteristic of occupational eczema:
A. bumps
B. nodes
C. Moisture *
D. vegetation
E. all answer’s are wrong.
1714. The bubble is allowed to form:
A. lihenifikatsiya
B. vibitsess
C. erosion *
D. ulcer
E. all answer’s are wrong.
1715. Have cavity morphological elements:
A papule
B. blister
C. Spot
D. bubble *
E. all answer’s are wrong.
1716. rash polymorphic at:
A. psoriasis
B. eczema *
C. lichen planus
D. molluscum contagiosum
E. all answer’s are wrong.
1717. monomorphic rash when dermatoses:
A. erythema multiforme exudative
B. eczema
C. dermatitis Duhring
D. psoriasis *
E. all answer’s are wrong.
1718. Clinical signs of psoriasis:
A. The presence of papules *
B. hemorrhagic spots
C. blistering
D. bumps
E. all answer’s are wrong.
1719. Clinical signs of lichen planus:
A. vorotnichek Biett
B. pupkoobraznym depressions in the center *
C. blisters
D. Node
E. all answer’s are wrong.
1720. The main clinical forms of psoriasis:
A subacute
B. atrophic
C. hemorrhagic
D. vulgaris *
E. all answer’s are wrong.
1721. Histologic features characteristic of psoriasis:
A. spongiosa
B. acantholysis
C. parakeratosis *
D. ballooning degeneration
E. all answer’s are wrong.
1722. Clinical stage course of psoriasis:
A subacute
* Progressive B.
C. Acute
D. Chronic
E. all answer’s are wrong.
1723. Clinical varieties of lichen planus:
A. arthropathic
B. atrophic *
C. pemfigoidnaya
D. Bugorkova
E. all answer’s are wrong.
1724. The most common form of psoriasis with the flow:
A. autumn
B.-season
C. Winter *
D. total
E. all answer’s are wrong.
1725. Psoriasis must be differentiated from:
A. pemphigus
B. eczema
C. warts
D. lichen planus *
E. all answer’s are wrong.
1726. The phenomena characteristic of psoriasis:
A blood dew *
B. Besnier-Meshcherskiy
C. Nikolsky
D. Hansen of asbestos
E. all answer’s are wrong.
1727. For the characteristic symptoms of lupus:
Net A. Wickham
B. follicular hyperkeratosis *
C. depigmentation
D. symptom Kebnera
E. all answer’s are wrong.
1728. Stage scleroderma:
A peeling
B. Seal *
C. hyperkeratosis
D. parakeratosis
E. all answer’s are wrong.
1729. Drugs used to treat scleroderma:
A nicotinic acid
B. lidasa *
C. erythromycin
D. biyohinol
E. all answer’s are wrong.
1730. Connective tissue diseases:
A. lupus erythematosus *
B. psoriasis
C. allergic vasculitis
D. planus
E. all answer’s are wrong.
1731. Clinical varieties of lupus:
A. seborrheic
B. discoid *
C. osteo-articular
D. muscle
E. all answer’s are wrong.
1732. Periods of syphilis:
A sharp
B. subacute
C. Primary *
D. All of the right
E. all answer’s are wrong.
1733. The main clinical forms of leprosy:
A. lepromatous *
B. kollikvativnaya
C. Acute
D. Chronic
E. all answer’s are wrong.
1734. For tinea versicolor is characterized by:
A. Balzer sample *
B. symptom Besnier - Meshcherskiy
C. green glow when the luminescence
D. roseola
E. all answer’s are wrong.
1735. Clinical forms of athlete's foot:
A. erased
B. intertriginoznoy *
C. Chronic
D. subacute
E. all answer’s are wrong.
1736. Diseases related to dermatophytes:
A. athlete's groin *
B. psoriasis
C. lichen simplex
D. erythrasma
E. all answer’s are wrong.
1737. What kind of pustules:
A. folliculitis *
B. telangiectasia
C. excoriation
D. vesicle
E. all answer’s are wrong.
1738. What kind of spots:
A. roseola *
B. Acne
C. papule
D. conflict
E. all answer’s are wrong.
1739. The lesion with indistinct borders, the size of 4-5 cm, pink, not protruding above the skin.
When the pressure-sensitive element disappears.
Specify the morphological element:
A spot *
B. knot
C. blister
D. bump
E. all answer’s are wrong.
1740. The spot size of 4-5 cm in diameter. Specify the kind of morphological elements:
A. roseola
B. erythema *
S. Purple
D. vibitsecc
E. all answer’s are wrong.
1741. What dermatological manifestations may be markers of AIDS:
Dermatitis herpetiformis Duhring A.
B. neurofibromatosis
C. Lichen planus
D. Kaposi's sarcoma *
E. all answer’s are wrong.
1742. Under which developed AIDS observed:
A. sarkomaKaposhi *
B. gonorrhea
S. dermatitis Duhring
D. Hutchinson's triad
E. all answer’s are wrong.
1743. The patient on the skin surface of the forearm extensor bespolostnoe proliferative
precipitation measuring 0.5 0,7sm diameter, thick consistency, raised above the skin. The
elements are clear lines, flat, dull shape.
Specify the morphological element:
A bundle *
B. bubble
S. blister
D. bubble
E. all answer’s are wrong.
1744. What morphological element has the ephemeral:
A spot
B. The Node
C. blister *
D. bump
E. all answer’s are wrong.
1745. In place of the node is allowed:
A plague
B. pigmentation *
C. crack
D. scar
E. all answer’s are wrong.
1746. For any skin disease characterized by monomorphic papuleznayasyp:
A. psoriasis *
B. Pemphigus
C. vulgar sycosis
D. Herpes
E. all answer’s are wrong.
1747. Bugorok- proliferative cavity element towering above the skin, often ulcerate and
zakanchivayuschiysyarubtsevaniem or scar atrophy
Find erroneous statements in this definition:
A. ulcerate
B. proliferative
C. cavity *
D. ends scarring
E. all answer’s are wrong.
1748. Find the right signs of tubercle:
A proliferative *
B. nevozvyshaetsya
C. neizyazvlyayushiysya
D. ends without scar pigmentation
E. all answer’s are wrong.
1749. The tubercle is:
A. infectious granuloma occurring in the reticular dermis *
B. cavity element occurs in the epidermis
C. ostrovospalitelny swelling papillary dermis
D. ostrovospalitelny cavity element of the papillary dermis
E. all answer’s are wrong.
1750. If any skin diseases occur tubercles:
A. Psoriasis
B. secondary syphilis
S. dermatitis Duhring
D. lupus *
E. all answer’s are wrong.
1751. The patient in the face and chest are abdominal elements protruding above the skin filled
with serous fluid, the size of 0.3-0.5 cm in diameter.
Identify morphological element:
A. abscess
B. vial *
C. papule
D. bubble
E. all answer’s are wrong.
1752. In the formation of the bubble histologically observed:
A vacuolar degeneration *
B. acanthosis
C. granulosa
D. akantolizis
E. all answer’s are wrong.
1753. The patient in the forearm skin lesion size of 5x6 cm, bright red, do not rise above the level
of the skin. Specify the nature of the morphological elements:
A bump
B. knot
C. Spot *
D. lihenifikatsiya
E. all answer’s are wrong.
1754. Enter the kind of element size of 5x6 cm, towering above the skin, bright red color:
A. roseola
B. erythema *
C. purpura
D. Lentigo
E. all answer’s are wrong.
1755. Enter the morphological element characteristic erythema sulfa:
A spot *
B. knot
C. Node
D. bump
E. all answer’s are wrong.
1756. The patient on the back of the neck lesion size 5x6 cm. The neck skin is dry, rough, thick.
Skin pattern in the outbreak is significantly strengthened.
Specify the nature of the morphological elements:
A. excoriation
V. vegetation
C. lihenifikatsiya *
D. scar atrophy
E. all answer’s are wrong.
1757. Lihenifikatsiya character for:
A. shingles
B. simple bubble stripping
C. pruriginous eczema
D. neurodermatitis *
E. all answer’s are wrong.
1758. Lihenifikatsiya appears:
A primary
B. a second time at the confluence papular elements *
C. primarily due to prolonged irritation of the skin when scratching
D. primarily as a result of burns
E. all answer’s are wrong.
1759. The patient in the tibia lesion, accompanied by a deep defect in the epidermis and dermis,
the size of 2-3 cm oval, smooth edges, with sero-purulent discharge.
Specify the nature of the morphological elements:
A. Erosion
B. crack
C. ulcer *
D. excoriation
E. all answer’s are wrong.
1760. In place of the ulcer will been resolved:
A scar *
B. pigmentation
C. excoriation
D. lihenifikatsiya
E. all answer’s are wrong.
1761. In what appears ulcer diseases of the skin:
A tertiary syphilis *
B. lichen planus
C. scabies
D. psoriasis
E. all answer’s are wrong.
1762. The patient skin rashes torso multiple pale pink in color, the size of 0.3-0.5 cm, towering
above the skin, disappearing when pressed.
Specify the kind of morphological elements:
A. roseola *
B. erythema
C. petechiae
D. ecchymosis
E. all answer’s are wrong.
1763. Indicate the nature of roseola:
A vascular hemorrhagic spot
B. primary lentigo
C. inflammatory vascular spot *
D. lenticular papule
E. all answer’s are wrong.
1764. What are the diseases of the skin accompanied by roseola:
A. erythema multiforme exudative
B. secondary syphilis *
C. psoriasis
D. planus
E. all answer’s are wrong.
1765. The patient in the skin of the trunk multiple lesions rounded shape, the size of a penny coin
2, proliferative, protruding above the skin surface, dense.
Specify the nature of the morphological elements:
A. nummulyarnye papules *
B. lenticular papules
C. Spot
D. blisters
E. all answer’s are wrong.
1766. The papules are located:
A. in the epidermis with the capture of the papillary dermis
B. the reticular dermis
C. Surface only within the epidermis *
D. in the subcutaneous fat
E. all answer’s are wrong.
1767. The patient on the red border of lips accumulation of small cavity elements containing
serous fluid.
Specify the nature of the morphological elements:
A bull
B. vesicles *
S. urticaria
D. pustules
E. all answer’s are wrong.
1768. To what morphological elements include vesicle:
A primary effusion *
B. primary proliferative
C. secondary exudative
D. specific granuloma
E. all answer’s are wrong.
1769. Which diseases of the skin may appear vesicle:
A. lichen planus
B. microsporia
C. leprosy
D. simple herpes *
E. all answer’s are wrong.
1770. At the site revealed the remains of vesicles:
A. erosion *
B. excoriation
C. scar
D. ulcer
E. all answer’s are wrong.
1771. Histological examination of the preparation of the skin revealed a significant thickening of
the granular layer of the epidermis.
Specify the nature of the disease process:
A vacuolar degeneration
B. granulёz *
C. acantholysis
D. hyperkeratosis
E. all answer’s are wrong.
1772. The granular layer consists of:
A 2-4 series *
B. 5-10ryadov
C. 5-6 rows of cells
D. 10-12 rows of cells
E. all answer’s are wrong.
1773. The cytoplasm of the cells of the granular layer comprises:
A. eleidin
B. keratin
C. keratohyalin *
D.net correct answer
E. all answer’s are wrong.
1774. If the skin biopsy revealed fusion of epithelial intercellular bridges in the spinous layer of
cells found epidermisa Ttsanka.
Specify the nature of the disease process:
A. parakeratosis
B. papillomatosis
C. acanthosis
D. acantholysis *
E. all answer’s are wrong.
1775. spinous layer consists of:
A. 3-6 rows of cells *
V. 10-18 series
C. 12-15 series
D. 20-25 series
E. all answer’s are wrong.
1776. The skin biopsy revealed a violation of keratinization epidermotsitov, thickening of the
horny layer without structural changes in the cells.
Specify the kind of pathological process:
A. parakeratosis
B. gipergranuyaez
C. hyperkeratosis *
D. spongiosa
E. all answer’s are wrong.
1777. The cells of the stratum corneum are in the form:
A ball
B. Records *
C. cylinder
D. cube
E. all answer’s are wrong.
1778. The cells of the stratum corneum are filled:
A. eleidin
B. keratin *
C. keratohyalin
D.net correct answer
E. all answer’s are wrong.
1779. For what disease is characterized by a true polymorphism:
A. eczema *
B. pemphigus
C.vulgarnogo sycosis
D. shingles
E. all answer’s are wrong.
1780. The biopsy of the skin marked intracellular edema in the Malpighian layer with signs of
pyknosis of nuclei.
Specify the nature of pathological changes
A. spongiosa *
B. vacuolar degeneration
C. akantolizis
D. acanthosis
E. all answer’s are wrong.
1781. The Malpighian layer includes:
A basal layer *
B. mesh layer
C. papillary layer
D. shiny coat
E. all answer’s are wrong.
1782. For what disease is characterized by itching night:
A. Eczema
B. istinnayapuzyrchatka
C. scabies *
D. dermatitis Duhring
E. all answer’s are wrong.
1783. The patient urticaria rashes on the body, accompanied by itching.
Specify the kind of morphological elements:
A papule
B. blister *
C. Bubble
D. bubble
E. all answer’s are wrong.
1784. Indicate the nature of the blister:
A primary effusion *
B. pervichnyyproliferativny
C. secondary exudative
D. secondary proliferative
E. all answer’s are wrong.
1785. The child of 1.5 years in the congested skin face multiple microvesicles. There Moisture,
worried about severe itching.
Select the method of external treatment:
A 2% solution of brilliant green *
B. sernayamaz
C. ihtiolovaya ointment
D. salicylic ointment
E. all answer’s are wrong.
1786. Types of eczema:
A.bolalar ekzemasi *
B. stroflyus
C. vulgar impetigo
D. streptakokli impetigo
E. all answer’s are wrong.
1787. bolalarda ekzemani paydo bo'lishi nima bilan kuzatiladi:
A.ekssudativ diatez *
B. qo'tir
C. pnevmaniya
D. anemiya
E. all answer’s are wrong.
1788. neyrodermit bilan og'rigan bemorda terini mexanik to'mtoq predmet bilan ta'sirlanishida
nerv-tomili reaksiya oq chiziq ko'rinishida kuzatiladi. Bu reaksiya nima deyiladi:
A. dermografizm *
V. «g'oz terisi»
C. eritema «uyalgandigi»
D. teri suratini kuchayishi
E. all answer’s are wrong.
1789. Which diseases of the skin, a change in dermographism:
A. versicolor Gibert
B. psoriasis
C. neurodermatitis *
D. scabies
1790. The patient in the skin of the trunk, limbs, multiple milliarnye, lenticular, numullyarnys
papules, plaques, covered by silvery scales.
Determine the nature of the rash:
A monomorphic, proliferative *
B. monomorphic, exudative
C. monomorphic, urticaria
D. polymorphic, exudative
E. all answer’s are wrong.
1791. For the diagnosis of monomorphic proliferative characteristic rash:
A. urticaria
B. Scabies
C. psoriasis *
D. pyoderma
E. all answer’s are wrong.
1792. To clarify the diagnosis of psoriasis is used:
A sample Minor
B. triad auspices *
C. Sample Balzer
D. Sample Jadassohn
E. all answer’s are wrong.
1793. The patient has lichen planus rash appeared after severe stress, fright
What kind of treatment it is advisable to appoint a general:
A suggestive therapy *
B. cytostatics
C. fungicidal
D. broad-spectrum antibiotics
E. all answer’s are wrong.
1794. Enter the character of lesions in lichen planus:
A.polimorfnaya
B. monomorphic Bugorkova
C. true polymorphic
D. monomorphic papular *
E. all answer’s are wrong.
1795. Enter the characteristic color of lesions in lichen planus:
A yellowish brown
B. red with violet hue *
S. pink-red
D. dark brown
E. all answer’s are wrong.
1796. At the 2-year-old child on the face with a loose tire phlyctenas, sero-purulent exudate,
erosion, crusts. Diagnosis:
A strep impetigo *
B. vulgar sycosis
S. hydradenitis
D. psevdofurunkulez Finger.
E. all answer’s are wrong.
1797. When strep impetigo affected:
A smooth leather *
B. apocrine sweat glands
C. eccrine sweat glands
D. hair follicles
E. all answer’s are wrong.
1798. strep impetigo is more common:
A. male
B. elderly
C. Children *
D. Women
E. all answer’s are wrong.
1799. The patient 22 years on his forearm a painful knot the size of a hazelnut cone-shaped with a
necrotic core. A presumptive diagnosis:
A. Gunma
B. carbuncle
C.furunkul *
D. turniol
E. all answer’s are wrong.
1800Furunkula. The causative agent is:
A. aureus *
B. mushroom
C. virus
D. pallidum
E. all answer’s are wrong.
1801. The child in the skin of the cheeks, legs whitish spots, rounded, with abundant
melkoplastinchatym peeling. The scraping the surface of foci of fungi were found. Sample Balzer
negative. Your presumptive diagnosis:
A simple zoster *
B. surface trichophytosis
C. microsporia
D. multicolored lichen
E. all answer’s are wrong.
1802. Who suffers most simple zoster:
A. Adults
B. Women
C. male
D. Children *
E. all answer’s are wrong.
1803. Enter the pathogen simply depriving:
A mite
V. aureus
C. Streptococcus *
D. virus
E. all answer’s are wrong.
1804. With what diseases must be differentiated lichen simplex:
A. Vitiligo
B. strofulyus
C. versicolor otrubevidpy *
D. simple herpes
E. all answer’s are wrong.
1805. A newborn in a week after birth appeared on the skin of the chest bubbles with serouspurulent exudate, located on a slightly erythematous background, erosion, crusts.
A presumptive diagnosis:
A. epidemic pemphigus *
B. acantholytic pemphigus
C. syphilitic pemphigus
D. strep impetigo
E. all answer’s are wrong.
1806. The causative agent of epidemic pemphigus is:
A hemolytic streptococcus
V. Pseudomonas aeruginosa
C. Staphylococcus aureus *
D. virus
E. all answer’s are wrong.
1807.Pri epidemic pemphigus source of infection are:
A medical staff *
B. fathers of newborn
C. domestic cats
D. domestic dog
E. all answer’s are wrong.
1808 epidemic pemphigus often suffer
A. Adults
B. Women
C. male
D. newborns *
E. all answer’s are wrong.
1809. A newborn skin on the neck, back multiple nodes reddish-bluish color, the size of a pea,
palpation determined fluctuation. From penetrated nodes semi-liquid purulent-hemorrhagic
content.
A presumptive diagnosis:
A. psevdofurunkulez Finger *
B. hydradenitis
C. infant eczema
Bullous impetigo D.
E. all answer’s are wrong.
1810. What factors contribute to the development psevdofurunkuleza Finger:
A. overeating
B. breast feeding
C. frequent bathing
D. overheating *
E. all answer’s are wrong.
1811. The patient in the left axilla painful, tight, welded assemblies undulating skin, reddishbluish color, with a fluctuation. Necrotic core is missing. A presumptive diagnosis:
A. hydradenitis *
B. kollikvativnyytuberkulez
C. erythrasma
D. abrasions
E. all answer’s are wrong.
1812. Which group of diseases is hidradenitis:
A streptococcal
B. tinea
C. stafilodermiyam *
D. Collagen
E. all answer’s are wrong.
1813. When gidradeniteporazhayutsya:
A hair follicle
B. sebaceous glands
S. smooth skin
D. apocrine sweat glands *
E. all answer’s are wrong.
1814. The average time flow gidradenita:
A. 10-15 days *
B. 2-3 months
C. b months
D. 5-7nedel
E. all answer’s are wrong.
1815. The patient on the skin of the penis size of the ulcer 2x2 cm, round shape, with dense
valikoobraznymi edges, purulent discharge, painful infiltrate the base beyond the borders of
ulcers. Wasserman negative.
A presumptive diagnosis:
A genital herpes
B. gonorrhea
C. shankriformnaya pyoderma *
D. chancre
E. all answer’s are wrong.
1816. The causative agent is shankriformnoy pyoderma:
A pale treponema
B. sinegnoynayapalochka
C. Staphylococcus aureus *
D. mite
E. all answer’s are wrong.
1817. When shankriformnoy pyoderma in place allowed the element remains:
A. atrophic scar *
V. vegetation
C. lihenifikatsiya
D. mosaic scar
E. all answer’s are wrong.
1818. A woman of 40 years on the skin of the chin, nasolabial folds against the backdrop of
reddened skin are infiltrated pustules, telangiectasia. Hypoacid suffers from gastritis. A
presumptive diagnosis:
A. photodermatitis
B. shankriformnaya pyoderma
C. rosacea *
D. erythematosus
E. all answer’s are wrong.
1819. What factors are important in the etiology of rozovyx ugrey:
A. angioneurosis lowered vascular tone *
B. streptococci
C. itch mites
D. fungus
E. all answer’s are wrong.
1820. Distinguish following clinical variants rozovyx ugrey:
A. lichenoid
B. papules-pustular
C. erythematous *
D. warty
E. all answer’s are wrong.
1821. With what diseases is carried diff. Diagnosis rozovyx ugrey:
A. perioral dermatitis
B. psoriasis
C. simple herpes
D. discoid lupus erythematosus *
E. all answer’s are wrong.
1822. The patient 30 years on the skin of the chin, cheeks multiple folliculitis, sycosis, clusters of
off-yellow purulent crusts. The process recurs. A presumptive diagnosis:
A vulgar sycosis *
V. acne vulgaris
C. Chronic ulcerative pyoderma
D. psoriasis
E. all answer’s are wrong.
1823. Which group of diseases is vulgar sycosis:
A. streptoderma
B. ringworm
S. stafilodermiya *
D. collagen
E. all answer’s are wrong.
1824. The etiopathogenesis vulgar sycosis matters:
A. neuroendocrine disorders
B. pyogenic staphylococci *
C.chesotochnye mites
D. overheating
E. all answer’s are wrong.
1825. The woman on the skin of the chest, abdomen, multiple excoriations, paired nodules,
pustules, linear shtrihoobraznye scratching, intense itching, worse at night.
A presumptive diagnosis:
A scabies *
B. eczema
C. nodular pruritus
D. strofulyus
E. all answer’s are wrong.
1826. Indicate the most common complication of scabies:
A regional adenitis
B. neuralgia
C. pyoderma *
D. psychosis
E. all answer’s are wrong.
1827. A man 23 years in the pubic area, the penis, the inner thighs papules, pustules, purulent
crust erosion. Worried itching noted symptom Gorchakov-Hardy.
A presumptive diagnosis:
A. chancroid
B. scabies *
C. Lipschutz's disease
D. vulgar impetigo
E. all answer’s are wrong.
1828. Some other symptoms can be symptoms of scabies:
A. itchy skin only during the day
B. linear layout
C. bunching rash
D. pairing elements *
E. all answer’s are wrong.
1829. What laboratory methods of research should be conducted in scabies:
A scraping on the itch mite *
B. blood tests naeozinofiliyu
S. urinalysis
D. Research on blood sugar
E. all answer’s are wrong.
1830. With what diseases is carried diff. diagnostics, scabies:
A. Shingles
B. lichen planus
C. snndrom Senir-Aschner
D. pruritus *
E. all answer’s are wrong.
1831. The patient was 25 years old, Single The diagnosis of scabies.
What laboratory studies should be included in the plan of the survey:
A finding of scabies mite *
V. research on LE cells
C. Analysis on akantoliticheskiekletki
D. Analysis of a bullock Borowski
E. all answer’s are wrong.
1832. The female scabies mite gets:
A mesh layer in the dermis
B. horny layer of the epidermis *
C. in the subcutaneous fat
D. in the basal layer of the epidermis
E. all answer’s are wrong.
1833. The incubation period for scabies is:
A. 1-4 weeks *
B. 2-3 months
C. 24-48 hours
D. 6 -8 months
E. all answer’s are wrong.
1834. The fertilized female scabies mite lays:
AB do100yaits
B. 1000 eggs
C. to 50 eggs *
D. to 200yaits
E. all answer’s are wrong.
1835. Patient itch after application of 33% sulfur ointment on the skin of the trunk appeared
diffuse erythema, rash melkopapuleznaya
Diagnose:
The contact allergic dermatitis. *
B. atopic dermatitis
C. toksikodermiya
D. erythema multiforme exudative
E. all answer’s are wrong.
1836. Specify drugs for the treatment of contact allergic dermatitis:
A 1% ointment ihtiolovaya
B. 0.1% calcium gluconate
C. Ampicillin
D. suprastin *
E. all answer’s are wrong.
1837. Enter the term applying sulfur ointment in the treatment of scabies:
A. 1-2 days
B. 4- 5 days *
C. 8 days
D. 7 days
E. all answer’s are wrong.
1838. The varieties of scabies include:
A. disgidroticheskaya
B. Dutch
C. Rural
Norwegian D. *
E. all answer’s are wrong.
1839. The patient skin spots round shape size of 5-6 mm with a slight melkoplastinchatym
peeling. When lubricating iodine stains darken.
Diagnosis:
A multi-colored shingles *
B. allergicheskiydermatit
Dermatitis herpetiformis Duhring C.
D. yododerma
E. all answer’s are wrong.
1840. Multicolored zoster is more common in persons:
A pulmonary tuberculosis *
B. stradayuschihpodagroy
C. hypertensive disease
D. On zhirnoyseboreey
E. all answer’s are wrong.
1841. What other symptoms are characteristic of multi-colored lichen:
A true leucoderma
V. no subjective sensations
S. yellowish-brown spots *
D.papuleznaya rash
E. all answer’s are wrong.
1842. Click drugs for the treatment of multi-colored lichen:
A. Nizoral *
B. 20% benzyl benzoate
C. tselestoderm
D. erythromycin
E. all answer’s are wrong.
1843. The patient 56 years old skin in the inguinal folds clearly limited noninflammatory spot
brick-red color. There sweating.
Diagnosis:
A. candidiasis
V. limited neurodermatitis
C. erythrasma *
D. chromophytosis
E. all answer’s are wrong.
1844. What is important in the etiology of erythrasma:
A neurotropic virus
B. Corynebacterium *
C. Streptococcus
D. Red trihofiton
E. all answer’s are wrong.
1845. The patient 23 years in the interdigital gaps feet maceration, fracture fragments of the
epidermis around the edges. The nails of the thumbs of both feet are yellow, dim, crumble.
Diagnosis:
A. mycosis fungoides
B. aktinomikoz
C. mycetoma of the foot
D. athlete's foot, onychomycosis. *
E. all answer’s are wrong.
1846. Enter the clinical variety of tinea pedis, onychomycosis:
A. intertriginoznoy *
B. disgidroticheskaya
S. microbial
D. exudative
E. all answer’s are wrong.
1847. What research is needed to confirm the diagnosis of tinea pedis, onychomycosis:
A microscopic examination of pathological material *
B. crop on Wednesday Saburo
C. CBC
D. urinalysis.
E. all answer’s are wrong.
1848. The pathology is characterized by nail in the athlete's foot, onychomycosis:
A 1-2 defeat of nail plastinoktolko toes
B. defeat nail plates 3-4, 4-5 toes *
C. lack of destruction of the nail plate brushes
D. losing all nail plates
E. all answer’s are wrong.
1849. The patient suffering from diabetes at the head of the penis itchy erythematous border
erosion macerated epidermisa. Worried itching, burning
Diagnosis:
A. esthiomenous chancre
V. contact dermatitis
C. candida balanoposthitis *
D. genital herpes
E. all answer’s are wrong.
1850. What research is needed to confirm the diagnosis - Candida balanoposthitis:
A study on cell acantholytic
B. iodine sample Jadassohn
C. Research on pale treponemu
D. microscopic study on yeasts *
E. all answer’s are wrong.
1851. What else could be affected by Candida balanoposthitis:
A. The corners of the mouth
B. interdigital folds
C. oral mucosa *
D. scalp
E. all answer’s are wrong.
1852. Click drugs for the treatment of Candida balanoposthitis:
A. Nizoral
B. kenolog
C. Diflucan *
D. Nystatin
E. all answer’s are wrong.
1853. The patient 10 years on the scalp rounded center 2x2 cm. The hair broken off at 6-8 mm at
the base of-muftoobrazny Case consists.
Diagnosis:
A. microsporia *
B. alopecia baldness
C. seborrhea
D. syphilitic alopecia
E. all answer’s are wrong.
1854. What other symptoms characteristic of microsporia:
A BTE increase in lymph nodes
B. emerald glow in the rays of the lamp Wood *
C. Wood in the glow of the lamp is marked svecheniyane
D. burning and painful lesions
E. all answer’s are wrong.
1855. Patient For 15 years on the scalp is clearly limited erythematous-squamous foci to be broken
off hair at the level of 6-8 mm. Under Wood's lamp-green glow.
Diagnosis:
A. trihofitnya
B. favus
C. psoriasis
D. microsporia *
E. all answer’s are wrong.
1856. Enter the possible source of contamination at mikrosporii:
A sick person
B. cats and dogs *
C. Cattle
D. steppe rodents
E. all answer’s are wrong.
1857. What is striking when microsporia:
A smooth leather *
B. organs
C. bones
D. nails
E. all answer’s are wrong.
1858. Click treatments for microsporia:
A. 6-10% sulfur-tar ointment
B. griseofulvin forte *
C. kloforan
D. acyclovir
E. all answer’s are wrong.
1859 27y.o. The patient diagnosed as athlete's foot, disgidroticheskaya form.
What are the clinical signs are characteristic of this disease:
A localization in the arch of the foot *
V. formation of bubbles and bubble multiC. the appearance of paired vesicles-papular elements
D. groin lamfadenit
E. all answer’s are wrong.
1860. Where infection occurs most often in the athlete's foot, disgidroticheskoy form:
A. in the treatment room
B. bedside
C. At the beach
D. in the pool, sauna. *
E. all answer’s are wrong.
1861. Who suffers most athlete's foot, disgidroticheskoy form:
A. athletes *
V. sellers
C. builders
D. vets
E. all answer’s are wrong.
1862. With what diseases is carried diff. diagnosis of athlete's foot, disgidroticheskoy form:
A palmoplantar pustular psoriasis
B. lichen planus
C. disgidroticheskaya eczema *
D. dermatitis Duhring
E. all answer’s are wrong.
1863. The child of 9 years at the head in the neck, there are two large pockets rezkoocherchennyh
infiltrated the size of 5x6 cm, covered with a large number of purulent rich crusts. After removing
the crusts noted pus from each follicle individually. The patient noted an increase in temperature
headache, increase in cervical lymph nodes.
A presumptive diagnosis:
A. abrasions
B. infiltrative-suppurative trichophytosis *
C. microsporia
D. pustular psoriasis
E. all answer’s are wrong.
1864. Who is most often the source of infection at the infiltrative-suppurative trihofitii:
A cattle *
B. cats, dogs
C. steppe rodents
D. Rat
E. all answer’s are wrong.
1865. What is a symptom characteristic of infiltrative-suppurative trihofitii:
A fish eggs
B. ladies heel
S. honeycomb *
D. apple jelly
E. all answer’s are wrong.
1866. What research is needed to infiltrative-suppurative trihofitii:
A. microscopy of hair *
B. crop on Wednesday Saburo
C. Skin biopsy histology
D. Research at Wood's lamp
E. all answer’s are wrong.
1867. A child 6 years of age on the skin in the shoulder, chest and back are eritemato-squamous
lesions proper size 2x2,3x3 cm round-shaped lesions with clear boundaries, the edges of which are
raised valikoobrazno and they can see small papules vesicular rashes, scales and position crusts.
A presumptive diagnosis:
A focal neurodermatitis
B. microbial eczema
C. psoriasis
D. microsporia smooth leather *
E. all answer’s are wrong.
1868. What is striking when microsporia smooth skin:
A. nails
V. hair *
C. internal organs
D. mucous eye
E. all answer’s are wrong.
1869. In the infant with an inflammation of the oral mucosa diagnosed as candidiasis of the oral
mucosa (thrush).
What features are typical for this disease:
A superficial erosion in the lesions
B. "cheesy" raid
C. whitish film can be easily removed *
D. ulcerated lesions
E. all answer’s are wrong.
1870. What can be combined lesion of the oral mucosa for candidiasis:
A defeat of the cornea
B. defeat corners of the mouth
C. defeat red border
D. losing the language *
E. all answer’s are wrong.
1871. What are the allergic rashes with candidiasis of the oral mucosa (thrush):
A. levuridy *
B. lentikulidy
C. mikrobidy
D. epidermofitidy
E. all answer’s are wrong.
1872. Click drugs for the treatment of candidiasis of the oral mucosa:
A 1% solution of methylene blue *
B. prednisolone
C. Ampicillin
D. deksametozon
E. all answer’s are wrong.
1873. The patient on the body multiple yellowish-brown spot with defurfuration which
subjectively did not bother the patient. Sample Balzer and symptom of "chips" are positive.
Diagnosis:
A pink zoster Gibert
B. secondary syphilis pigmentosa
C. versicolor raznotsvetpy *
D. dry streptoderma
E. all answer’s are wrong.
1874. RESOLVED to sample Balzer used:
A 50% potassium iodide ointment
B. 5% tincture of iodine *
C. A 1% solution of nicotinic acid
D. 5% salicylic kisdota
E. all answer’s are wrong.
1875. What factors are important in the etiopathogenesis of candidiasis:
A. sweating
B. Corynebacterium
S. streptococci
D. Mushrooms *
E. all answer’s are wrong.
1876. Click drugs for the treatment of candidiasis:
A. mikozolon *
B. tetracycline
C. erythromycin
D. deksametozon
E. all answer’s are wrong.
1877. The patient on the scalp are ocher-yellow rind with impression in the center, when removing
crusts visible scar atrophy. Hair dull, as if "eclipsing" comes "barn" smell.
Diagnosis:
A. trichophytosis
B. favus *
C. erythematosus
D. streptoderma
E. all answer’s are wrong.
1878. Which way prioskhodit infection favus:
A non-communicable disease
B. sexual contact
C. airborne droplets
D. in direct contact with the patient *
E. all answer’s are wrong.
1879. What is striking in this pathology:
A hair
B. smooth skin
C. nails
D. All of the above *
E. all answer’s are wrong.
1880. Enter favus clinical varieties:
A pustular
B. squamous
C. disgidroticheskaya
D. skutulyarnaya *
E. all answer’s are wrong.
1881. A child of 7 years diagnosed trichophytosis scalp.
What features are typical for this disease:
A breaking of the hair at the level of 1-2 mm in the outbreaks *
B. whitish, defurfuration
C. imposition of silver-white scales
D. Case consists muftoobrazny at the base of the hair
E. all answer’s are wrong.
1882. Enter trihofitii clinical varieties:
A sharp
B. infiltrative-suppurative *
C. disgidroticheskaya
D. pustular
E. all answer’s are wrong.
1883. The patient on the face, forearms and feet are ulcer size 2x2,3x3 cm with uneven scalloped
edges on the bottom of the ulcer has granular granulation as "fish eggs". Ulcers were 6 weeks after
the arrival of Turkmenistan, where he was on a business trip in May.
Diagnosis:
A rural type of cutaneous leishmaniasis *
B. abrasions
C. ecthyma
D. tertiary syphilis
E. all answer’s are wrong.
1884. What research is needed to confirm the diagnosis of cutaneous leishmaniasis:
A study on the pale treponemu
V. research on calf Borowski *
C. Wasserman
D. All of the above
E. all answer’s are wrong.
1885. What other clinical signs are characteristic of cutaneous leishmaniasis:
A symptom of "beads" *
B. symptom of falling through the probe
C. symptom of "fish-eye"
D. cimptom ladies Heel
E. all answer’s are wrong.
1886. Click drugs for the treatment of cutaneous leishmaniasis:
A. monomitsin *
B. ampicillin
C. prednisolone
D. Nizoral
E. all answer’s are wrong.
1887. The patient diagnosed as leishmaniasis skin, rural style.
What are the symptoms characteristic of this disease:
A. The presence of ulcers with scalloped edges
B. symptom of "the head of Medusa"
C. ulcers with even steep edges
D. symptom of "fish eggs" *
E. all answer’s are wrong.
1888. Indicate the source of infection of cutaneous leishmaniasis:
A cat, dog
B. gophers *
C. chickens
D. Bird
E. all answer’s are wrong.
1889. Who is the carrier of cutaneous leishmaniasis:
A. Mosquitoes Phlebotomus genus *
B. flies
C. sick person
D. cockroaches
E. all answer’s are wrong.
1890. The causative agent of cutaneous leishmaniasis is:
A. mycobacteria
B. Leschmania tropica major *
C. Fungus
D. virus
E. all answer’s are wrong.
1891. The patient diagnosed as leishmaniasis skin, urban style.
What is the incubation period for this disease:
A. 2-6 months *
B. 2-3 days
C. 24 hours
D. 5-b years
E. all answer’s are wrong.
1892. Enter the characteristics of the skin leyshmanioza, city type:
A rapid development patologicheskogoprotsessa
B. slow development process *
C. poyavleniefurunkulopodobnogo infiltrate at the bite site
D. scar formation after 6 months of infection smomenta
E. all answer’s are wrong.
1893. Who is the source of infection in the skin leyshmanioze, city type:
A prairie rodents
B. Cattle
C. sick person *
D. cats, dogs
E. all answer’s are wrong.
1894. The patient 16 years diagnosed metaleyshmanioz
Specify the clinical manifestations of this disease:
A. The appearance of tubercles on the scars *
B. predominant localization on the extremities
C. bugorkinikogda on the scars do not appear
D. predominant localization on the face
E. all answer’s are wrong.
1895. What are the symptoms characteristic of metaleyshmanioza:
A symptom of "apple jelly" *
B. symptom "necklace of Venus"
C. symptom of "thimble"
D. symptom of "ladies' heel"
E. all answer’s are wrong.
1896. Enter prevention metaleyshmanioza:
A fight with flies
B. Competition mosquito *
C. control of stray cats and dogs
D. strict isolation of patients in specialized hospitals
E. all answer’s are wrong.
1897. The patient of 22 years on the skin of the face, there are bumps the size of 0.7-0.8 mm in
diameter, brown-red, soft testovatoy consistency. In place of the allowed elements are marked
atrophic scars resembling tissue paper. In the words of a patient sick since childhood.
A presumptive diagnosis:
A tertiary syphilis
B. lupus *
C. leprosy
D. planus, warty form
E. all answer’s are wrong.
1898. What are the symptoms characteristic of tuberculosis of the skin:
A symptom of "apple jelly"
B. symptom of "falling through the probe" *
C. symptom of "thimble"
D. symptom of "fish eggs"
E. all answer’s are wrong.
1899. At the slaughterhouse worker on the skin back of the hand and fingers were bluish-red
bumps the size of a pea, on the surface of which there are warty growths. On-site allowed
members celebrated scar atrophy.
A presumptive diagnosis:
A. Syphilis
B. lichen planus
C. psoriasis
D. warty lupus *
E. all answer’s are wrong.
1900. What is important in the etiology of warty tuberculosis of the skin:
A. contact with sick animals
B. pale treponema
S. mycobacteria Koch *
D. Hansen bacillus
E. all answer’s are wrong.
1901. Click drugs to treat warty tuberculosis of the skin:
A. rifampicin
B. tubazid *
C. acyclovir
D. dorungal
E. all answer’s are wrong.
1902. The patient with active TB lung mucosa of the lower lip has an ulcer 2 cm in diameter, with
scalloped edges, a pale red color with grainy uneven bottom, having a sluggish, gray, granulation,
bleed easily. At the bottom there are sores around the yellow lumps.
A presumptive diagnosis:
A. ulcerative tuberculosis *
B. simple herpes
S. shingles
D. Stevens-Johnson syndrome
E. all answer’s are wrong.
1903. What other symptoms characteristic of ulcerative tuberculosis:
A. localization have orifices
B. localization only in the mouth
S. painless
D. Trela grain *
E. all answer’s are wrong.
1904. The patient on his face in the brow, forehead, cheeks, nose, there are infiltrates nodes.
Broken facial expressions, has a fierce infection, hair loss is noted in the lateral part of the
eyebrow. There incomplete closure of the eyelids, hoarseness golosa. The patient marks a change
of pain and temperature sensitivity.
A presumptive diagnosis:
A. lupus
B. leprosy *
C. tertiary syphilis
D. dermatomyositis
E. all answer’s are wrong.
1905. What other symptoms characteristic of leprosy:
A triad Auspittsa
B. "lion face" *
C. symptom butterfly
D. «crown of Venus"
E. all answer’s are wrong.
1906. What research is necessary to clarify to leprosy:
A scraping from the nasal mucosa on the shelves Hansen *
B. sputum on mycobacteria Koch
C. Sample production Jadassohn
D. Thompson production sample
E. all answer’s are wrong.
1907. Enter the clinical kind of leprosy:
A warty type
Tuberculoid type B.
C. lepromatous type *
D. hemorrhagic type
E. all answer’s are wrong.
1908. The patient 38 years on the skin in the chest area, the lateral surface of the abdomen are
sharply defined plaques size 3x4 cm. On the periphery of the plaques are dense polygonal flat
papules with a purple tint in the central part of the atrophy and depigmentation observed in the
lesions is not the temperature, and pain sensitivity .
A presumptive diagnosis:
A. lupus
B. lichen planus
C. leprosy *
D. lupus erythematosus
E. all answer’s are wrong.
1909. Enter the clinical kind of leprosy:
A City type
V. Rural Type
C. tuberculoid type *
D. milliarny type
1910. The patient diagnosed lepromatous type of leprosy.
What are the clinical manifestations characteristic of this disease:
A. logoftalm
B. lion face *
S. aksifoidiya
D. labyrinthitis
E. all answer’s are wrong.
1911. What is the basis sample Minor:
A hypofunction of the sweat glands *
B. hyperactivity of the sweat glands
C. hyperactivity of the sebaceous glands
D. hypertrichosis
E. all answer’s are wrong.
1912. What are the diseases carried diff. diagnosis of leprosy:
A. with all of the *
B. tertiary syphilis
C. lupus and multicolored shingles
D. vitiligo
1913. Enter the result lepraminovoy sample at lepromatous type of leprosy:
A negative *
B. Positive
C. The early positive
D. Late positive
E. all answer’s are wrong.
1914. On the skin of the perianal area warty growths like "cauliflower", on a narrow basis. PB
negative cells Ttsanka absent. Diagnosis:
A. genital warts *
B. extensive warts
C. warty lichen planus
D. psevdosifilis papular
E. all answer’s are wrong.
1915. Which group of diseases is genital warts:
A. buleznym dermatoses
B. fungal infections
C. viral dermatoses *
D. vasculitis
E. all answer’s are wrong.
1916. Manifestations ostrokonechnyx warts in men most commonly are located:
A. on the inner layer of the foreskin *
B. around the anus
C. In language
D. groin
E. all answer’s are wrong.
1917. What tools are used to treat warts ostrokonechnyx:
A. flutsinar
B. tselestoderm
C. solkoseril
D. solkoderm *
E. all answer’s are wrong.
1918. A child of 5 years on the face, neck, back of hands on the size of the nodules are 5-7mm in
diameter with a pearl-gray color, hemispherical shape, with "umbilicated" in the center. Subjective
feelings are absent.
Diagnosis:
A. molluscum contagiosum *
B. vulgaris Warts
C. verrucose psoriasis
D. A warts
E. all answer’s are wrong.
1919. Microscopic examination of the contents of nodules when molluscum contagiosum
determine:
A calf Borowski
V. Hansen sticks
C. shellfish calf *
D. virus
E. all answer’s are wrong.
1920. A man 23 years in the border of the pubic area and abdomen are manifestations
molluscum contagiosum. What rash characteristic of this disease:
A spherical nodules with umbilicated in the center *
B. polygonal papules with umbilicated
C. villous proliferation
D. conical tubercles
E. all answer’s are wrong.
1921. Which way is transmitted molluscum contagiosum:
A. animals
B. casual contact *
C. After insects
D. transmissible by
E. all answer’s are wrong.
1922. Which group of diseases is molluscum contagiosum:
A viral dermatoses *
B dermatozoonozam
C. pyoderma
D. fungal infections
E. all answer’s are wrong.
1923. In the etiopathogenesis of molluscum contagiosum matters:
A. Hypersensitivity Koufo
B. hypersensitivity to animal proteins
C. filterable virus *
D. simple
E. all answer’s are wrong.
1924. At 56 years old patient diagnosed with herpes zoster:
What features are typical for this disease:
A severe itching of the skin appearance
B. appearance of grouped polymorphic eruptions
C. grouped blisters *
D. paired vesicles-papular rash
E. all answer’s are wrong.
1925. Which group of diseases is herpes zoster:
A viral dermatoses *
B dermatozoonozam
C. pyoderma
D. fungal infections
E. all answer’s are wrong.
1926. The patient 62 years of age on the skin of the left half of the chest, along the intercostal
nerve has grouped rash with blisters, is located on the hyperemic basis. Rash preceded paroxysmal
pain, burning, fever.
Prepolagaet diagnosis:
A. pemphigus vulgaris
B. dermatitis Duhring
S. shingles *
D. buleznaya toksikodermiya
E. all answer’s are wrong.
1927. Which group of diseases is herpes zoster:
A viral dermatoses *
B. dermatozoonozam
C. pyoderma
D. fungal infections
E. all answer’s are wrong.
1928. Which drugs are used to treat herpes zoster:
A. Nizoral
B. ceftriaxone
S. diutsifon
D. acyclovir *
E. all answer’s are wrong.
1929. The causative agent of shingles is:
A Streptococcus
B. neurotrophic filterable virus *
C. coli Dederleyna
D. papilomatozny human virus
E. all answer’s are wrong.
1930. A child 12 years of age on the skin of the dorsum of the hands are nodular lesions, the size
0,8-1sm in diameter, significantly protruding above the skin, flesh-colored, dense consistency,
without subjective sensations,
Diagnosis:
A. lichen planus
B. psoriasis
C. vulgaris warts *
D. warty lupus
E. all answer’s are wrong.
1931. Enter the clinical varieties of warts:
A. verrucous
B. infiltrative nagnoitslnye
C. flat, Youth *
D. Palmoplantar
E. all answer’s are wrong.
1932. B etiology of the disease is set to:
A. HSV-1
V. Koch's bacillus
C. etiology of the disease is unknown
D. dermotropny filterable virus *
E. all answer’s are wrong.
1933. For the treatment of warts used:
A. solkoderm *
B. 5% ointment tebrofenovaya
C. 5% Naftalan ointment
D. predinizolonovaya ointment
E. all answer’s are wrong.
1934. The patient 35 years after supercooling on the red border of lips there was redness, swelling.
After 2 days, the rash appeared grouped fine bubble filled with serous fluid. Worried tingling and
burning.
Predlolagaemy diagnosis:
A simple herpes *
W. multiforme exudative erythema
C. disgidroticheskaya eczema
D. Stevens-Johnson syndrome
E. all answer’s are wrong.
1935. At the site allowed rash herpes remains:
A. atrophic scar
B. mosaic scar
C. extruded scar
D. pigmented spot *
E. all answer’s are wrong.
1936. What is important in the etiology of herpes:
A human papilloma virus
B. HSV-1 *
S. retrovirus
D. cytomegalovirus
E. all answer’s are wrong.
1937. For the treatment of herpes used:
A 3% oxolinic ointment
B. Zovirax *
C. solkoderm
D. tselestoderm
E. all answer’s are wrong.
1938. A woman of 36 years of casual sex in the mucosa of the labia minora
there are small, the size of millet grain grouped vesicles, which are formed after the resolution of
the erosion of bright-red color with polycyclic outlines. The rash of bubbles accompanied by a
feeling of pain, burning sensation, fever do37,8.
A presumptive diagnosis:
A genital herpes *
B. Acute ulcer Lyupshittsa-Chalin
C. erythema multiforme exudative
D. dermatitis Duhring
E. all answer’s are wrong.
1939. The etiology of genital gerpesa matters:
A pale treponema
B. retrovirus
C. HSV-2 *
D. cytomegalovirus
E. all answer’s are wrong.
1940. The patient addressed to the doctor with complaints of general malaise, weakness, fatigue.
Over the last month weight loss of 10 kg. A history of recurrent herpes. When viewed in the oral
cavity, on the hard palate are solitary, sharply circumscribed, painless, bluish-red, testovatoy
consistency tumor formation size of a cherry stone.
It noted an increase in BTE and cervical lymph nodes.
A presumptive diagnosis:
A. HIV *
B. Pemphigus
C. Tuberculosis
D. Stevens-Johnson syndrome
E. all answer’s are wrong.
1941 HIV - associated diseases include
A. syndrome Senir-Aschner
B. Kaposi's sarcoma *
S. Syndrome Melkersson-Rosenthal
D. Bazin induratum erythema
E. all answer’s are wrong.
1942. What laboratory studies are needed to clarify the HIV infection:
A. Enzyme Immunoassay
V. research on cells akantolicheskie
C. Wasserman
D. Response immunoblotting *
E. all answer’s are wrong.
1943. Who is the main risk group for HIV infection:
A staff kitchens
B. addicts *
C. homosexuals
D. Guides
E. all answer’s are wrong.
1944. The patient on the skin of the face there is a lesion, accompanied by a change in color, with
indistinct boundaries, the size of 4-5 cm, pink, not protruding above the skin. When pressed
elements disappear.
Specify the nature of the morphological elements:
A vascular inflammatory spot *
B. hemorrhagic spot
C. artificial spot
D. lentigo
E. all answer’s are wrong.
1945. If any skin diseases encountered inflammatory vascular spot:
A. Psoriasis
B. Syphilis *
S. skrofuloderma
D. fixed erythema
E. all answer’s are wrong.
1946. The patient has a diagnosis of HIV infection:
What research is needed to confirm the diagnosis:
A. Enzyme Immunoassay
B. reaction immunoblotting *
S. Wasserman
D. RIBT
E. all answer’s are wrong.
1947. The causative agent of HIV infection is:
A. renotrophic filtriruyuschchysya virus
B. HSV-1
C. HSV-2
D. lymphotropic retrovirus *
E. all answer’s are wrong.
1948. The patient in the face and chest are ostrovospalitelnye abdominal rash, protruding above
the skin filled with serous fluid, the size of 0.3-0.5 cm in diameter.
At a resolution of the elements left behind:
A. pigmentation *
B. scar atrophy
C. ulcers
D. hypertrophic scars
E. all answer’s are wrong.
1949. The bubble can be located:
A. intraepidermal
V. under the stratum corneum *
C. in the reticular dermis
D. in the subcutaneous fat
E. all answer’s are wrong.
1950. The patient in the skin of the trunk multiple lesions rounded shapes, the size of a penny coin
2, proliferative, protruding above the skin surface, dense.
Which diseases of the skin rash may appear like:
A scabies
B. psoriasis *
C. tertiary syphilis
D. pemphigus
E. all answer’s are wrong.
1951. Enter the kind of node:
A primary proliferative *
B. primary effusion
C. secondary proliferative
D. secondary exudative
E. all answer’s are wrong.
1952. If the skin biopsy revealed fusion of epithelial intercellular bridges in the spinous layer of
cells found epidermisa Ttsanka.
For a skin disease characterized histologically by this:
A true pemphigus *
B. shingles
C. eczema
D. erythema multiforme exudative
E. all answer’s are wrong.
1953. Cells Ttsanka means:
A modified cells of the granular layer
B. modified cells of the dermis
C. Changes spinous layer of cells *
D. Modification of the basal layer cells
E. all answer’s are wrong.
1954. The patient urticaria rashes on the body, accompanied by itching.
Urtika- this:
A pleural cavity element
B. pleural cavity element comprising blood
C. discoloration on the limited area as a result of vasodilatation surface
choroid plexus
D. Limitation ostrovospalitelny edema of the papillary dermis *
E. all answer’s are wrong.
1955. Select the method of external treatment of hives:
A. lotions furatsillinovye
B. Water talker menthol *
C. Sulfuric ointment
D. varnish
E. all answer’s are wrong.
1956. The patient neurodermatitis by mechanical stimulation of the skin with a blunt object
neuro-vascular reaction occurs in the form of a white strip.
When any skin diseases often marked white dermographism:
A. Cutaneous leishmaniasis
B. atopic dermatitis *
C. psoriasis
D. lichen planus
E. all answer’s are wrong.
1957. Which drugs should not be prescribed in the white dermographism:
A calcium supplements *
B. sulfa drugs
C. corticosteroid hormones
Vitamin D.
E. all answer’s are wrong.
1958. The patient 22 years on his forearm a painful knot the size of a hazelnut cone-shaped with a
necrotic core.
After allowing the element will remain in place:
A vegetation
B. lihenifnkatsiya
S. mostikoobrazny scar
D. atrophic scar *
E. all answer’s are wrong.
1959 Select a drug for the treatment of boils:
A corticosteroid hormones
B. antifungals
C. Antibiotics
D. anabolics
E. all answer’s are wrong.
1960. A newborn skin on the neck, back multiple nodes reddish-bluish color, the size of a pea,
palpation determined fluctuation. From penetrated nodes semi-liquid purulent-hemorrhagic
content.
The disease occurs as a result of the defeat:
A pilosebaceous unit
B. excretory ducts and glomeruli eccrine sweat glands *
C. smooth skin
D. follicle
E. all answer’s are wrong.
1961. With what diseases conduct differential diagnostics vezikulopustuleza
A. abrasions *
B. strofulyusom
C. exudative diathesis
D. gidradenity
E. all answer’s are wrong.
1962. The woman on the skin of the chest, abdomen, multiple excoriations, nodules, abscesses,
linear shtrihoobraznye scratching, intense itching, worse at night.
Some other symptoms are symptoms of the disease:
A. The presence of paired vesicles-papular elements *
B. symptom Hissar
C. symptom Besnier-Mesherskogo
D. Net Wickham
E. all answer’s are wrong.
1963. Click medicines to treat scabies:
A 90% sulfuric ointment
B. 20% salicylic ointment
C. 20% benzyl benzoate *
D. 5% ointment naftolanovaya
E. all answer’s are wrong.
1964. The patient 56 years old skin in the inguinal folds clearly limited noninflammatory spot
brick-red color. There sweating.
After the resolution process is in place:
A. depigmentation
V. skin atrophy
C. mosaic scar
D. hyperpigmentation *
E. all answer’s are wrong.
1965 Tattoo refers to:
A. artificial stains *
B. vascular spots
C. hemorrhagic spots
D. pigment spots
E. all answer’s are wrong.
1966. The patient 10 years on the scalp rounded center 2x2 cm. The hair broken off at 6-8 mm at
the base of-muftoobrazny Case consists.
What is important in the etiology of the disease:
A. yeasts
B. rusty mikrosporum *
C. gipsovidny trihosporum
D. trihofiton Shonleyna
E. all answer’s are wrong.
1967. The most vulnerable age for this pathology:
A. 6-12 years *
B. 40-60let
C. infancy
D. 16-20 years
E. all answer’s are wrong.
1968. A child 6 years of age on the skin in the shoulder, chest and back are eritemato-squamous
lesions proper size 2x2,3x3 cm round-shaped lesions with clear boundaries, the edges of which are
raised valikoobrazno and they can see small papules vesicular rashes, scales and position crusts.
What matters in the etiopathogenesis of the disease:
A. Age
B. pyogenic flora
C. mushroom mikrosporum *
D. Seasonality
E. all answer’s are wrong.
1969. Click treatments for microsporia:
A. griseofulvin forte *
B. Nizoral
C. Ampicillin
D. Zovirax
E. all answer’s are wrong.
1970. A child of 7 years diagnosed trichophytosis scalp:
What the fungus causes this pathology:
A. trihofiton ektotriks *
B. mikrosporum rusty
S. candida
D. None of the above
E. all answer’s are wrong.
1971. What is striking about the fungus Trichophyton ektotriks:
A hair *
V. joints
C. oral mucosa
D. nails
E. all answer’s are wrong.
1972. The patient of 22 years on the skin of the face, there are bumps the size of 0.7-0.8 mm in
diameter, brown-red, soft testovatoy consistency. In place of the allowed elements are marked
atrophic scars resembling tissue paper. In the words of a patient sick with childhood
What research is needed to confirm the diagnosis:
A sample of histamine
B. Sample Pirke
C. probaYadassona
D. Mantoux *
E. all answer’s are wrong.
1973. Clinical Pathology is a kind induratum erythema Bazin
A. lupus
B. skrofulodermy
C. lupus *
D. psoriasis
E. all answer’s are wrong.
1974. The patient with active TB lung mucosa of the lower lip has an ulcer the size of 2 cm
diameter, with scalloped edges, a pale red color with grainy uneven bottom, having a sluggish,
gray granulation, bleed easily. At the bottom there are sores around the yellow lumps.
What research is needed to confirm the diagnosis:
A direct microscopic research on mycobacteria Koch *
B. crop on flora and sensitivity
C. blood on RV
D. None of the above
E. all answer’s are wrong.
1975. How often do you get infected with tuberculosis of the skin:
A. lymphogenous
B. hematogenous *
C. airborne
D. transmissible
E. all answer’s are wrong.
1976. The patient 38 years in the skin of the breast, the side surface of the stomach, there are
sharply defined plaques size 3x4 cm. On the periphery of the plaques are dense polygonal flat
papules with a purple tint; in the central part is marked atrophy and depigmentation. In the lesions
and no thermal pain sensitivity.
What tests are used to confirm the diagnosis:
A sample of histamine
B. Nicotine trial
C. Sample Jadassohn
D. leprominovaya sample *
E. all answer’s are wrong.
1977. Click drugs to treat leprosy:
A. penicillin
B. sulfetron
S. DDS *
D. acyclovir.
E. all answer’s are wrong.
1978. A child of 5 years on the face, neck, back of hands on the size of the nodules are 5-7mm in
diameter with a pearl-gray color, hemispherical shape, with "umbilicate pressure" in the center.
Subjective feelings are absent.
What group of diseases is the pathology:
A. neyrodermatozy
B. viral dermatoses *
C. collagen
D. pyoderma
E. all answer’s are wrong.
1979. At 56 years old patient diagnosed with herpes zoster:
How often seen exacerbation of the disease:
A year in spring and autumn
V. leaves behind immunity, relapses were not observed *
C. exacerbated once in 3-4 years
D. Each summer
E. all answer’s are wrong.
1980. Shingles can be a manifestation of:
A. Tuberculosis
B. leukemia
C. HIV *
D. rheumatism
E. all answer’s are wrong.
1981. A woman of 36 years of casual sex in the mucosa of the labia minora were small, the size of
millet grain grouped vesicles, which are formed after the resolution of the erosion of bright-red
color with polycyclic outlines. The rash of bubbles accompanied by a feeling of pain, burning,
increased temperature do37,8.
What group of diseases is the pathology:
A. STI: to viral dermatoses *
B. cystic dermatoses
C. etiology of the disease is unknown
D. Epstein-Barr virus
E. all answer’s are wrong.
1982. For the treatment of dermatoses, viral used:
A. acyclovir *
B. Nizoral
C. amphotericin
D. Lamisil
E. all answer’s are wrong.
1983. What should be differentiated disease spread
toxicoderma
A. Herpes
B. pyoderma
S. vulgar pemphigus *
D. disease Borowski
E. all answer’s are wrong.
1984. Enter the typical localization at multiforme exudative
erythema
A scalp
V. rear brake *
S. neck
D. interdigital spaces of hands
E. all answer’s are wrong.
1985. What factors are important in the development multiforme
A.exudative erythema
B.hereditary
C.photosensitization
D.foci of chronic infection *
E.hyperactivity of the sebaceous glands
1986. What dermatoses differentiate erythema multiforme
A. Lyell's syndrome *
B. strofulyus
S. scrapie
D. psoriasis
E. all answer’s are wrong.
1987. Enter the pathogenetic factors of pink lichen Gibert
A. introduction in skin mites
B. yeasts
S. idiosyncrasy to iodine
D. viral infection *
E. all answer’s are wrong.
1988. Enter the variety of microbial eczema
A. Adult
B. paratravmaticheskaya *
S. Idiopathic
D. horn
E. all answer’s are wrong.
1989. How is a clinical sign is characterized by a rash in true
A.eczema
B.symptom <serous wells> *
C.tubercles
D.asymmetry
E.induratum edema
1990. What are the symptoms seen with discoid lupus
Net A. Wickham
B. collar Biett
C. symptom of "ladies' heel" *
D. paired elements
E. all answer’s are wrong.
1991. Enter the variety of scleroderma
A. vulgar
B. disease blind spots *
C. infiltrative
D. intertriginoznoy
E. all answer’s are wrong.
1992 Lichen planus is characterized by
A. lack of itching
B. tubercles
C. Wickham Net *
D. urticaria rashes
E. all answer’s are wrong.
1993. For planus characterized by
A. intercellular edema spinous layer
B. akantolizis
C. vacuolar degeneration
D. uneven thickening of the granular layer of the epidermis *
E. all answer’s are wrong.
1994. What clinical symptom pathognomonic for red
lichen planus
A."Honey cell Celsus"
B.Net Wickham *
C.cells Ttsanka
D.symptom auspices
E. all answer’s are wrong.
1995. What diseases do not belong to a group of viral dermatoses
A. warts
B. genital warts
C. psoriasis *
D. shingles
E. all answer’s are wrong.
1996. Vascular spot is
A. Erythema *
B. Chloasma
C. Nevus
D. Leucoderma
E. all answer’s are wrong.
1997. Primary morphological elements
A. scar
B. vial *
C. Erosion
D. ulcer
1998. Secondary morphological elements
A. Roseola
B. Lace
C. Node
D. ulcer *
E. all answer’s are wrong.
1999. The bubble is formed at
A. Urticaria
B. CPL
C. Herpes Simplex *
D. Psoriasis
E. all answer’s are wrong.
2000. The end result is a node
A. Spot *
B. Atrophy
C. Ulcer
D. Tripe
E. all answer’s are wrong.
With two correct answers
2001. Specify drugs used in pemphigus vulgaris:
A. prednisolone *
B. triamcinolon *
In sulfadimetoksin
D. Dapsone
2002. What primary morphological element precedes erosion:
A. Bulla*
B. vesicle*
C. Urtica
D. macula
2003. What disease is not part of dermatophytosis:
A. sporotrichosis *
B. hromomikoza*
C. rubrofitii
D. trichophytosis
2004. Specify the drug used in leprosy:
A. Dapsone*
B. monomitsin
C. sulfation*
D. delagil
2005. In the treatment of candidiasis is not used:
A. polkortolon*
B. nystatin
C. nizoral
D. acyclovir*
2006 Nikolsky sign is positive in:
A. true pemphigus*
B. medicamentous allergic stomatitis*
C. schoolderman
D. dermatitis herpetiformis during the
2007. Pathognomonic for eruptive eczema are:
A. the microvesicles*
B. peel
C. weeping erosions*
D. scales
2008. What are the main clinical signs are not characteristic of chronic eczema:
A. bright redness, swelling*
B. microerosion, brown*
C. infiltration
D. licensure
2009 for any of these viral diseases of the oral mucosa is not affected:
A. molluscum contagiosum*
B. simple bubble herpes
C. varicella
D. warts *
2010 Exclude lesions not typical for rubrofitia:
A. hair*
B. the mouth *
C. sole
D. palm
2011. what pathogens are determined by colouring on Cells. Nelson:
A. Leishmania
B. the tubercle Bacillus *
C. Treponema pallidum
D. leprosy *
2012. Specify a characteristic appearance and location of lesions pink lichen:
A. hyperemic spot*
B. vesicle
C. blister
D. on the trunk *
2013. Specify clinic if toxicodermia:
A. Lyell's syndrome*
B. anaphylactic shock *
C. syndrome of Cesar
D. the syndrome of Irangos. Cushing's
2014. Specify drug used in psoriasis:
A. lorinden *
B. dexamethasone *
C. ethambutol
D. blemaren
2015. Specify the symptoms of lupus erythematosus
A. a symptom of a ladies ' heel *
B. a symptom of Asbos. Of Hansen
C. symptom Gorchakov C. Ardi
D. symptom Besnier –Meshchersky *
2016. After any primary cells formed by erosion:
A. bubble*
B. spot
C. abscess *
D. hump
2017. What phenomena are included in the psoriatic triad:
A. the phenomenon of a stearin stain*
B. the phenomenon terminalno film *
C. the phenomenon of Apple jelly
D. phenomenon and Lyell's syndrome
2018. Streptococcal pyoderma is differentiated with what disease:
A. herpes simplex *
B. eritrazma
C. bullous herpes*
D. CPL
2019. Deep dermatophytosis what differentiates diseases:
A. microsporia *
B. carbuncle *
C. eczema
D. eritrazma
2020. When the disease is not observed host:
A. herpes*
B. lichen planus *
C. leprosy
D. leishmaniasis
2021. In the dermis layers are distinguished:
A. papillary*
B. Horny
C. net*
D. granular
2022. By atypical solid ancram include:
A. indurative edema*
B. shanks. amygdala*
C. balanoposthitis
D. phlyctena
2023. Specify the incorrect statement:
primary syphiloma can oslojnyaetsya.
A. epididymitis*
B. paraphimosis
C. orchitis *
D. gangrenization
2024. Specify the incorrect statement:
possible causes simple dermatitis.
A. oral administration of medications*
B. parenteral introduction in case the administration of medicines*
C. low temperature
D. high temperature
2025. For localized scleroderma unusual stage:
A. follicular hyperkeratosis*
B. nepolitano hyperkeratosis*
C. pustular
D. induration
2026. In the treatment of candidiasis is not used:
A. polkortolon*
B. nystatin
C. hydrocortisone *
D. flunil
2027. In the treatment of head lice is not used:
A. antibi shampoo
B. cycloferon *
C. 10C. 20% emulsion of benzyl benzoate
D. mikozolon*
2028. Psoriatic erythroderma can develop after:
A. appointment ACTH *
B. external use only 1% ointment salicylic acid *
C. topical application of 5% ointment of salicylic acid
D. the application of ultraviolet irradiation to patients with the summer type of psoriasis
2029. Add the secondary elements resulting from the transformation of the burl:
A. scale*
B. secondary pigmentation *
C. cork
D. ulcer
2030. Specify the clinical signs, characteristic of tertiary syphilis:
A. extensive warts *
B. the nodes
C. masorti
D. roseola *
2031. What are the main clinical signs are not characteristic of chronic eczema:
A. bright redness *
B. edema, microerosion *
C. infiltration
D. licensure
2032. In the stage of vesicles for topical therapy eczema apply only:
A. lotions *
B. paste
C. creams
D. aerosols*
2033. Specify the symptom is not characteristic for discoid forms of lupus erythematosus:
A. Lichenification*
B. Symptom of "Apple jelly"*
C. the Symptom of "butterfly".
D. Symptom Benies. Meshchersky.
2034. What disease is among the group of neurodermatoses:
A. eczema *
B. allergicheskie dermatitis *
C. pruritus
D. urticaria
2035. What is unusual in the clinical picture of rosacea:
A. follicular hyperkeratosis *
B. erythema
C. follicular papules *
D. infiltration of the skin of the nose
2036. Specify the most characteristic symptom of scabies:
A. evening and night itching*
B. serous well
C. the paired arrangement of the elements*
D. impetiginized in the lesions
2037. Enter among the following surface form stafilodermii:
A. carbuncle
B. the osteo folliculitis *
C. furuncle
D. sycosis vulgaris*
2038. In the therapy of lichen planus may apply:
A. diphenhydramine*
B. novopassit *
C. prednisolone
D. androgens
2039. In the primary stage of syphilis patients may meet all the characteristics of chromes.
A. erosive papules *
B. lymphangitis
C. negative Wassermann reaction
D. non-erosive papules *
2040. Select the most effective agent for the treatment of chlamydial urethritis:
A. tetracycline*
B. doxycyclin *
C. penicillin
D. levamisole
2041. One of the clinical signs are not typical for AIDS patients:
A. vitiligo *
B. lymphoma
C. scabies *
D. recurrent herpes
E. diarrhea
2042. Specify drug, used in pemphigus vulgaris:
A. prednisolone *
B. dexamethasone *
C. monomitsin
D. Orungal
2043. Specify drug used in the treatment of scabies:
A. 20% benzyl benzoate *
B. Vishnevsky ointment
C. the aerosols of Spregal *
D. 10% syntomycin ointment
2044. Specify the medication used for candidiasis of the skin:
A. Orungal *
B. flonal *
Furazolidone C.
D. bactrim
2045. Specify drugs used in the progressive stage of psoriasis:
A. cream unna *
B. psoriasin ointment
C. 2% salicylic ointment *
D. ointment dermatol
2046. Specify the phenomenon are characteristic of lichen planus:
A. Cabrera *
B. grid Wickham *
C. fish caviar
D. tissue paper
2047. Specify the symptoms characteristic of multi-colored lichen:
A. peeling *
B. sample Signs *
C. sample Wickham
D. Kepner
2048. Specify a drug that cannot be set under a persistent white dermographism:
A. calcium chloride *
B. thiosulfate sodium
C. calcium gluconate*
D. pirogenal
2049. Specify the drug used in herpes zoster:
A. bonafton *
B. delagil
C. oxoline *
D. Dapsone
2050. Specify the clinical symptoms characteristic of sycosis:
A. chronically relapsing course*
B. rheumatic pain
C. localization on the skin of the Mons pubis *
D. nodules
2051 . Clinical signs of herpes simplex is not:
A. lichen sclerosis*
B. cicatricial atrophy *
C. eczema herpeticum
D. meningoencephalitis
2052. Which of diseases is not a manifestation of VICS. infection:
A. vitiligo *
B. Kaposi's sarcoma
C. scabies *
D. diarrhea
2053. That is not typical for psoriasis:
A. you can call animals in a laboratory *
B. not due to seasonality*
C., it can be first appearance after giving birth
D. due to seasonality
2054. Specify the characteristic symptom blister:
A. occurs due to the intra-cellular edema *
B. quickly appears and disappears*
C. leave behind after erosion
D. peeling on the surface of the blister
2055. What clinical manifestations are characteristic of secondary recurrent syphilis:
A. leukoderma*
B. parenchymatous keratitis
C. alopecia *
D. indurative edema
2056. What clinical symptoms are not in patients with primary syphilis:
A. ladonnas. plantar papules *
B. indurative edema
C. extensive warts *
D. shanks. felon
2057 . Specify the symptoms characteristic of uncomplicated chancre:
A. dense infiltration at the base of the ulcer *
B. saped edge
C. painless ulcer*
D. signs of inflammation
2058. For patients with chancroid is not typical:
A. the absence of pain*
B. the signs of inflammation
C. solid infiltration *
D. inclined to autoinoculation
2059. In the treatment of gardnerellosis used:
A. tinidazole *
B. metronidazole *
C. penicillin
D. erythromycin
2060. For the diagnosis of some diseases is applied research using
wood's lamp:
A. microsporia*
B. eritrazma *
C. pityriasis rosea.
D. vitiligo.
2061. What primary morphological element precedes erosion:
A. gnojnice *
B. bubble *
C. none of the
D. hump
2062. What primary morphological element precedes the ulcer:
A. the node *
B. bubble
C. bump *
D. blister
2063. Specify the symptoms characteristic of uncomplicated chancre:
A. dense infiltration at the base of the ulcer*
B. painless ulcer *
C. saped edge
D. purulent discharge
2064. What pustular disease of the skin
not found in newborns:
A. hydradenitis *
B. sycosis *
C. vesiculopustules
D. pseudoharengus finger
2065. Specify the types of seborrhea:
A. dry*
B. vulgar
C. leaf
D. bold*
2066. What clinical symptom is noted
when lupus erythematosus:
A. "ladies' heel*"
B. The Symptom Benies. Meshchersky *
C. "blood dew"
D. ymptom of Pincus
2067. Name the atypical form of psoriasis:
A. justiceability /rupiny/ *
B. scalp
C. intertriginous *
D. coin
2068. What is the most severe form of psoriasis:
A. artropaticheskom *
B. geographical
C. nummularnyi
D. erythrodermic *
2069. Name the localization of elements at
pustular psoriasis of the Barber:
A. palm *
B. the sole *
C. the red border of the lips
D. the skin of the genital area
2070. Name the type of pustular
psoriasis :
A. psoriasis Combucha *
B. nummularnyi
C. lichenoid
D. psoriasis of the Barber *
2071. What clinical symptom is typical
for dermatitis Duhring:
A. eosinophilia of content bubble *
B. blood eosinophilia *
C. microfocal pattern baldness
D. sclerodactyly
2072. What are the clinical varieties cogs.
tion of leishmaniasis You know:
A. Astros. nitrotyrosine type*
B. nedifferentsirovannost type
C. ulcerated type*
D. erythematous
2073. Name the clinical varieties
Borowski disease:
A. city type *
B. Ostro. nitrotyrosine type *
C. undifferentiated
D. infiltrative. suppurative
2074. Which layer is not part of the epidermis:
A. net*
B. granular
C. papillary *
D. prickly
2075. Specify the types of seborrhea:
A. dry*
B. bold *
C. leaf
D. vegetative
2076. What diseases belong to the group of stafilodermy:
A. impetigo of Bockhart *
B. epidemic pemphigus *
C. impetigo nail ridges
D. slit-like vulgar ecthyma impetigo
2077. What tools have antipruritic property:
A. salicylic acid
B. benzocaine *
C. tar
D. menthol *
2078. What pyoderma occur only in newborns and infants:
A. vesiculopustules *
B. exfoliative dermatitis *
C. hydradenitis
D. furuncle
2079. What are morphological elements typical of Ritter's dermatitis:
A. tubercles
B. erosion *
C. papules
D. bubbles *
2080. Localization of the rash in scabies:
A. interdigital spaces of the hands*
B. the skin around the navel, inner thighs*
C. interdigital spaces stop
D. places of a congestion of the sebaceous glands
2081. What are secondary morphological element:
A. the knot
B. ulcer *
C. vegetation *
D. abscess
2082. Name the proliferative morphological element:
A. a knot *
B. the node *
C. abscess
D. bubble
2083. Acantholysis is the morphological
the basis of the symptom:
Nicholas A. *
B. Wickham;
C. Pospelova
D. Asbestos-Hansen*
2084. What clinical symptom is typical
for dermatitis Duhring:
A. eosinophilia of content bubble*
B. eosinophilia in the blood*
C. cryoglobulinemia
D. leukopenia
2085. What are the tests uses for diagnostic.
Ki dermatitis herpetiformis Duhring:
A. sample 2C. 3% solution of potassium iodide *
B. 50% ointment with potassium iodide *
C. 3C. x test Thompson
D. iodine test Signs
2086. What medicines naibo.
effective for dermatitis Duhring:
A. DDS *
B. corticosteroids *
C. Biseptol
D. vitamins In
2087. Specify the features of papules with red
flat shingles:
A. umbilicate indentation *
B. edge peeling
C. rich peeling
D. purple tint *
2088. Name the type of pustular
psoriasis:
A. psoriasis Combucha *
B. psoriasis of the Barber *
C. psoriasis Kaposi
D. lichenoid
2089. What skin areas are usually free from
rashes when the LCP:
A. palm *
B. the sole *
C. flexion of the legs
D. wrist joints
2090. What dermatosis should different.
to differentiate lichen planus:
A. secondary syphilis *
B. psoriasis*
C. artifactual dermatitis
D. pityriasis versicolor
2091. Name the localization of elements at
pustular psoriasis of the Barber:
A. palm *
B. the red border of the lips
C. scalp
D. soles *
2092. Specify the true types of eczema:
A. prodigiosa *
B. disgidroticheskaya *
C. numulyarnaya
D. seborrheic
2093. The rash of exudative mnogoformnuû
the erythema is:
A. in the form of arcs*
B. grouped the course of nerves
C. in the form of rings *
D. in pairs
2094. Specify the symptoms characteristic of scabies:
A. symptom Gorchakovs. Ardi *
B. nocturnal itching *
C. paired papules
D. bubbles
2095. What drugs is used to treat
scabies:
A. 20 % benzyl –benzoate ointment *
B. spray Spregal *
C. 5 % naphthalan ointment
D. 5% sulfur ointment
2096. With what diseases should be differentiated scabies:
A. atopic dermatitis*
B. pityriasis rosea
C. pruritus*
D. tinea
2097. Name the obligate factor of dermatitis:
A. concentrated acid *
B. boiling water *
C. jewelry of gold
D. synthetic detergents
2098. Allergic reaction to the medication may occur:
A. bullous rash *
B. vesicular *
C. symptom Pylnova
D. papular rash
2099. Specify the kind of toxicodermia:
A. fixed erythema *
B. perniosis
C. psoriasis
D. iododerma *
2100. What medications are more likely to cause fixed erythema:
A. sulfadimetoksin *
B. Biseptol *
C. analgin
D. aspirin
2101. What disease should be differentiated common toksikodermiya:
A. pityriasis rosea Gibert *
B. the syndrome Stevens. Johnson *
C. pyoderma
D. acne rosacea
2102. Indicate the typical localization at the mnogoformnuû exudative erythema:
A. the back of the hand *
B. scalp
C. the rear brake *
D. interdigital spaces of the hands
2103. What factors are important in the development of mnogoformnuû of erythema multiforme:
A. seasonality *
B. hypothermia *
C. hypersensitivity to fluoride
D. hereditary
2104. What differentiate exudative dermatoses mnogoformnuû erythemal:
A. pemphigus vulgaris*
B. Lyell's syndrome *
C. strophulus
D. scrofuloderma
2105. Specify pathogenetic factors of pink lichen Gibert:
A. viral infection *
B. hypofunction of the sebaceous glands
C. yeast
D. streptococcal infection *
2106. Specify the variety of microbial eczema:
A. seksitarina *
B. varicose *
C. adult
D. idiopathic
2107. What clinical sign is characterized by a rash with true eczema:
A. true polymorphism *
B. symmetry *
C. ephemeral blisters
D. assimetria
2108. What are the symptoms seen with discoid lupus:
A. symptom Khachaturian *
B. a collar of Byetta
C. a symptom of "lady heel" *
D. Voronova collar
2109. Specify types of scleroderma:
A. stripy *
B. white spot disease *
C. infiltrative
D. intertriginous
2110. Lichen planus is characterized by:
A. papular rash *
B. umbilicated *
C. predominant localization on the flexor surfaces of the extremities
D. the absence of subjective feelings
2111. For lichen planus is characterized by:
A. uneven thickening of the granular layer of the epidermis *
B. parakeratosis
C. acanthosis *
D. vnutriarterialno microabcesses
2112. What clinical symptom is pathognomonic for lichen planus:
A. grid Wickham*
B. umbilicate indentation *
C. symptom of Auspice
D. cells Tzenka
2113. What diseases does not belong to the group of viral dermatoses:
A. extensive warts *
B. genital warts
C. a simple bubble herpes
D. psoriasis *
2114. Vascular spot is:
A. Erythema *
B. Roseola*
C. Lentigo
D. Leukoderma
2115. Primary morphological elements:
A. bubble *
B. bubble *
C. ulcer
D. erosion
2116. Secondary morphological elements:
A. Ulcer*
B. Voldyri
C. Erosion*
D. Knot
2117. The bubble is formed by:
A. Eczema*
B. herpes simplex*
C. Psoriasis
D. CPL
2118. The outcome of the nodule is:
A. hypopigmented stain*
B. Ulcer
C. vanish*
D. Scale *
2119. Exudative morphological element is:
A. Vesicle*
B. Blister*
C. Papule
D. Hump
2120. Layers of epidermis:
A. basal *
B. thorn*
C. adipose
D. hypodermally
2121. Have a cavity morphological elements:
A. abscess*
B. blister
C. node
D. bubble *
2122. The rash is polymorphic if:
A. secondary syphilis *
B. dermatitis Duhring *
C. warts
D. psoriasis
2123. Monomorphic rash in medicine:
A. psoriasis *
B. the LCP *
C. dermatitis Duhring
D. tinea
2124. Clinical signs of psoriasis:
A. the presence of papules *
B. peeling *
C. rash predominantly on the extensor surfaces of the extremities
D. blisters
2125. Clinical signs of lichen planus:
A. koebner's phenomenon *
B. polygonal papules *
C. blisters
D. nodes
2126. Agents used topically in the treatment of psoriasis:
A. the gel skins. cap *
B. corticosteroid ointment *
C. Castellani paint
D. ointment of benzyl benzoate
2127. Phenomena typical for psoriasis:
A. stearin spot *
B. Wickham;
C. matching items
D. terminal film*
2128. The signs of true polymorphism of rash for dermatitis Duhring:
A. node
B. nodules *
C. tubercle
D. bullas*
2129. Clinical varieties of pemphigus:
A. vulgar *
B. vegetative *
C. psoriasiform
D. exudative
2130. For lupus erythematosus symptoms:
A. follicular hyperkeratosis*
B. cicatricial atrophy*
C. grid Wickham
D. mukovidnoe peeling
2131. Clinical forms of athlete's foot:
A. worn
B. disgidroticheskaya *
C. psoriasiform
D. intertriginous *
2132. Diseases related to dermatophytosis:
A. athlete's groin *
B. microspores*
C. eritrazma
D. hromomikoza
2133. The antibiotics used in the treatment of candidiasis:
A. nystatin*
B. flonal *
C. cephalosporins
D. doxycyclin
2134. When artropaticheskom appropriate will appoint all of the above, chromes.
A. nizoral *
B. antioxidants
C. penicillin *
D. mineralocorticoid drugs
2135. Typical eruptive lichen planus elements have the following features, chromes.
A. preferential localization on the face *
B. symptom Pylnova *
C. waxy luster
D. Wickham mesh on the surface of papules
2136. Monomorphic rash can be:
A. psoriasis *
B. lichen planus *
C. erythema multiforme
D. microbial eczema
2137. Hyperkeratosis is characteristic of:
A. psoriasis *
B. sycosis
C. bullous impetigo
D. ichthyosis *
2138. Woman, 25 years old, married 3 gods. When the survey was diagnosed with chronic
endocervicitis, adnexitis. Smears C. leukocytosis, gonococcus and Trichomonas not found. My
daughter 1.5 years of acute gonorrheal vulvovaginitis etiologies. Correct doctor's tactics:
A. use of a combination of provocation with subsequent bacteriological and bacterioscopic study*
B. the appointment of antibacterial drugs according to the sensitivity of bacterial flora to
antibiotics *
C. appointment of corticosteroids
D. appointment of antihistamines
2139. What type of topical treatment should
to choose with weeping skin inflammations:
A. lotions *
B. Prater Park
C. aerosols *
D. cream
2140. The patient was diagnosed as anterior urethritis chlamydial etiologies. What antibiotic group
to prescribe to the patient:
A. aminoglycosides*
B. tetracycline *
C. antimalarial
D. fluorinated quinolones
2141. Girls vulvovaginitis Trichomonas etiology characterized by all of these
symptoms, chromes.
A. frothy discharge from the genital slit *
B. the absence of itching *
C. infiltrative lesions
D. frequent and painful urination
2142. What kind of lotions should choose to oozing of the infected skin surface:
A. ivanyevo *
B. lead
C. furatsilinovoy *
D. tannin
2143. Diagnostic tests of allergic dermatitis are samples.
A. drip *
B. compression *
C. Baltser
D. Adamson
2144. To frequent complications of atopic dermatitis include all of these except
A. membranes. destructive processes in the epithelium of the urinary organs during exacerbation
atopic dermatitis*
B. urethritis *
C. hyperthyroidism *
D. poor sleep, irritability
2145. Diagnosis of acute prostatitis, based on the results of the above studies, chromes.
A. General analysis of blood *
B. biochemical analysis of blood *
C. palpation of the prostate
D. ultrasound of the prostate
2146. Examination of children with atopic dermatitis with the aim to understand all of the above,
except
A. identifying the source of microbial sensitization *
B. identify the States of immunodeficiency*
C. identify indicatorii
D. identify other allergenic factors
2147. What exogenous factors contribute to the development of pyoderma:
A. microfracture*
B. helminthic invasion
C. disorders of keratinization
D. contamination of the skin*
2148. What endogenous factors contribute to the development of pyoderma:
A. diabetes mellitus*
B. hypovitaminosis*
C. eosinophilia
D. hemophilia
2149. In acute and subacute complicated gonorrhea in men and women in the complex treatment
included all of the above, chromes.
A. antihistamines *
B. gonovaccine *
C. protivolodochnyi antibiotics
D. antispasmodics
2150. Specify the correct tactics of treatment of torpid and chronic forms of gonorrheal infection.
A. immunotherapeutic drugs *
B. local treatment *
C. antibacterial
D. calcium supplements
2151. To stafilodermy include:
A. impetigo of Bockhart *
B. simple versicolor
C. deep folliculitis *
D. slit-like impetigo
2152. Clinical symptoms of discoid lupus erythematosus all but
A. bullas *
B. bubbles *
C. erythema
D. tightly sitting scales
2153. Koebner's phenomenon is observed in the following diseases:
A. lichen planus *
B. leprosy
C. psoriasis *
D. folliculitis
2154. If any element of dermatitis rash is the bubble:
A. eczema *
B. herpes simplex*
C. pityriasis rosea Gibert
D. pemphigus
2155. True polymorphism is typical for the following diseases except:
A. eczema
B. dermatitis Duhring
C. lichen planus*
D. psoriasis *
2156. Find relevant answers:
Proliferative morphological changes
A. granules,*
B. hyperkeratosis,*
C. spongiose,
D. acantholysis,
2157. Diffuse hyperkeratosis is not typical for:
A. ichthyosis
B. keratodermia
C. pemphigus*
D. pyoderma*
2158. What factors contribute to the development of pyoderma:
A. disorders of carbohydrate metabolism *
B. the presence of a large number of pathogenic strains *
C. increased clotting
D. hypervitaminosis
2159. For scabies is characterized by:
A. paired papules. vesicular elements *
B. comedones
C. itching in the night *
D. symptom of "Apple jelly"
2160. From the ectoderm develop:
A. hair *
B. sebaceous glands *
C. muscles of the hair
D. the vessels of the skin
2161. To has is characterized by:
A. fever, malaise, headache *
B. blister
C. opioidnye pustules
D. the formation of fistulous *
2162. Specify the clinical form of scabies (a form):
A. worn *
B. squamous
C. erythematous
D. clean *
2163. To a mixed pyoderma include:
A. brimicombe *
B. chancriform pyoderma *
C. hydradenitis
D. osteo folliculitis
2164. Indicate the clinical varieties of pemphigus acantholytic:
A. leaf *
B. circle
C. vulgar *
D. syndrome Stevenses. Jones
2165. Dermatosis at what primary morphological element is a node:
A. tuberculosis of the skin *
B. leishmaniasis of the skin *
C. psoriasis
D. genital warts
2166. Under what medicine the main element of the rash is pimple:
A. molluscum contagiosum *
B. pemphigus
C. pityriasis rosea Gibert
D. warts *
2167. Clinical symptoms of discoid lupus erythematosus:
A. erythema *
B. tightly sitting scales *
C. atrophy *
D. bubbles
2168. Koebner's phenomenon is not observed in the following diseases:
A. lichen planus
B. folliculitis*
C. vitiligo
D. leprosy*
2169. Under what medicine the main element of the rash is not pimple:
A. molluscum contagiosum
B. warts
C. pityriasis rosea Gibert*
D. pemphigus *
.
2170. If any element of dermatosis rash is not a bubble:
A. eczema
B. herpes simplex
In . pink ringworm Gibert*
D. pemphigus *
2171. Dermatosis at what primary morphological element is not a node:
A. tuberculosis of the skin
B. leishmaniasis of the skin
C. psoriasis *
D. genital warts *
2172. True polymorphism characteristic off.
A. eczema *
B. dermatitis Duhring *
C. psoriasis
D. lichen planus
2173. Name the atypical form of primary syphiloma:
A. indurative edema *
B. shanks. amygdala *
C. diffuse pigmentation
D. fagedenizm
2174. Add the secondary items that are not the result of the transformation of the knot:
A. secondary spot
B. vanish
C. erosion *
D. cicatricial atrophy *
2175. Complication of hard chancre is:
A. pagedevice *
B. phimosis *
C. impetigo
D. stricture of the urethra
2176. What drugs are used for photochemotherapy in psoriasis:
A. psoralen *
B. nystatin
C. acyclovir
D. pevalin *
2177. What type of external therapy is prescribed in case of progressive stage of psoriasis:
A. corticosteroid ointment *
B. 1C. 2% salicylic ointment *
C. ointment "Psoriasin"
D. method of dem'yanovich
2178. For secondary syphilis is characterized by:
A. pustular*
B. roseolous rash *
C. Gumma
D. chancre and polyadenylation
2179. Diagnosis of acute prostatitis, based on the results of the above studies:
A. General analysis of blood
B. palpation of the prostate *
C. examination of the skin
D. ultrasound of the prostate *
2180. What drug is effective for genital herpes:
A. Alkaryl *
B. Neovir *
C. Sandimmune
D. Diprospan
2181. The clinical picture disgidroticheskaya form of athlete's foot:
A. bullas *
B. bubbles *
C. bumps
D. peel
E. lichenification
2182. The clinical picture of infiltrative. nagnoitel'noj ringworm:
A. inflammatory infiltrate *
B. pustule *
C. lichenification
D. blisters
2183. Clinical symptoms Jock itch:
A. spot red *
B. the border is sharply limited *
C. blisters
D. nodes
2184. The most frequent localization of superficial candidiasis of the skin:
A. folds of skin *
B. the corners of the mouth *
C. flexor surface of wrists
D. vulva
2185. Fungal diseases of the skin are called:
A. epidermofiton *
B. Trichophyton *
C. Hansen of the Mycobacterium
D. Mycobacterium Koch
2186. The main clinical signs of lesions of the mucous membranes with candidiasis:
A. erosive surface*
B. white film *
C. mucous papules
D. bumps
2187. the drugs used in the treatment of fungal infections:
A. griseofulvin *
B. nizoral*
C. lindamycin
D. Ceftriaxone
2188. Localization of the rash in scabies in adults:
A. interdigital folds of the hands *
B. the flexor surface of forearms *
C. nails
D. palm
2189. The main clinical signs of scabies:
A. itching in the evening and at night *
B. the nodes
C. paired arrangement of morphological elements *
D. acantholysis
2190. Funds are used for the treatment of scabies:
A. sodium thiosulfate 60% and hydrochloric acid 6% *
B. benzyl benzoate 20% emulsion *
C. iodine tincture
D. prednisolone ointment
2191. Morphological elements characteristic of the clinic of uncomplicated scabies:
A. a knot *
B. spot
C. the vial *
D. hump
2192. Scabies is not characteristic symptom:
A. Gorchakov - Ardi
B. a symptom of the triangle
C. . Nicholas*
G Asbestos-Hansen*
2193. The causative agents of pustular diseases of the skin:
A. Streptococcus *
B. Staphylococcus *
C. Proteus
D. the tubercle Bacillus
2194. The Streptococcus causes the disease:
A. impetigo*
B. superficial panaritium *
C. sycosis
D. folliculitis
2195. Varieties of streptococcal impetigo:
A. impetigo nail ridges *
B. Zayed road *
C. furuncle
D. a simple bubble herpes
2196. The localization of furuncle:
A. the neck *
B. the region of the back *
C. mucosa of the oral cavity
D. lips
2197. Clinical signs specific for staphylococcal deep pustules:
A. riddled with hair *
B. hemispherical form *
C. pus liquid
D. is located on the smooth skin
2198. Hydradenitis is localized:
A. axillae*
B. peripapillary area*
C. palm
D. Shin
2199. Main clinical forms of staphylococcal skin lesions:
A. folliculitis *
B. carbuncle *
C. pityriasis versicolor
D. tinea
2200. Clinical varieties of streptococcal impetigo:
A. bullous impetigo *
B. deep folliculitis
C. sycosis
D. impetigo slit *
2201. Complications arising in patients with the localization of furuncle on the face:
A. phlebitis of the vessels of the brain*
B. sepsis *
C. neuritis of the facial nerve
D. endocarditis
2202. Deeper forms of streptodermii:
A. vulgar ecthyma *
B. drill ecthyma *
C. streptococcal impetigo
D. periungual conflicts
2203. The factors that cause toksikodermiya:
A. sulfanilamidnye drugs *
B. antibiotics *
C. acid
D. alkali
2204. Clinical signs of urticaria:
A. pruritus *
B. pain
C. node
D. blister *
2205. Irritants cause irritant contact dermatitis:
A. mechanical *
B. high temperature *
C. emotional
D. chemical C. optional
2206. For allergic dermatitis is characterized by:
A. erythema *
B. the nodes
C. bumps
D. bubbles *
2207. Symptoms typical of true eczema:
A. true polymorphism *
B. microvesicles *
C. blisters
D. cicatricial atrophy
2208. Characteristic of eczema:
A. erythema, swelling *
B. oozing "serous wells" *
C. Gumma
D. roseola
2209. Clinical manifestations fresh secondary period of syphilis:
A. chancre or its remains *
B. roseola *
C. bubbles
D. blisters
2210. Clinical signs of syphilitic roseola:
A. blades. pink *
B. does not merge *
C. flakes
D. purple
2211. Clinical signs of tinea versicolor:
A. secondary leukoderma *
B. erosion
C. nodes
D. defurfuration *
2222. The source of the infection by Microsporum:
A. cats, dogs *
B. people *
C. cattle
D. rodents
2223. When candidiasis affects:
A. mucous *
B. skin *
C. hair
D. the muscular system
2224. Primary cells for herpes simplex:
A. erythema *
B. bubble *
C. blister
D. abscess
2225. To viral diseases include:
A. eczema
B. herpes zoster *
C. molluscum contagiosum*
D. lupus erythematosus
2226. For herpes zoster is characterized by:
A. pain *
B. bubbles *
C. nodes
D. pigmented
2227. Clinical manifestations typical for professional eczema:
A. hyperemia *
B. vegetation
C. vesicularia *
D. bumps
2228. The bubble allowed the formation of:
A. spot *
B. peel *
C. ulcer
D. scar
2229. do not have cavity following morphological elements:
A. abscess
B. bubble
C. spot *
D. the node *
2230. The rash is polymorphic if:
A. secondary syphilis *
B. eczema *
C. psoriasis
D. lichen planus
2231. Monomorphic rash in medicine:
A. urticaria *
B. psoriasis *
C. secondary syphilis
D. eczema
2232. Clinical signs of psoriasis:
A. the presence of papules *
B. peeling *
C. bumps
D. nodes
2233. for lichen planus is characterized by:
A. polygonal papules *
B. pupkoobraznym depressions in the center *
C. bubbles
D. blisters
2234. Main clinical forms of psoriasis:
A. vulgar *
B. while
C. artropaticheskom *
D. subacute
2235. Histological features characteristic of psoriasis:
A. parakeratosis*
B. acanthosis *
C. spongiose
D. acantholysis
2236. Clinical stage and types of psoriasis:
A. stationary *
B. progressive *
C. chronic
D. vesicular
2237. Clinical types of lichen planus:
A. hypertrophic *
B. atrophic *
C. pemphigidae
D. artropaticheskom
2238. The most common form of psoriasis with the flow:
A. summer *
B. winter *
C. off-season
D. spring
2239. Psoriasis must be differentiated from:
A. lichen planus *
B. eczema
C. pemphigus
D. secondary syphilis *
2240. Phenomena typical for psoriasis:
A. stearin spot*
B. terminal protectors *
C. Asbestos-Hansen
D. Stevens - Jones
2241. For lupus erythematosus symptoms:
A. erythema *
B. follicular hyperkeratosis *
C. grid Wickham
D. mukovidnoe peeling
2242. Stage of scleroderma:
A. peeling
B. edema*
C. atrophy *
D. hyperkeratosis
2243. Drugs used for the treatment of scleroderma:
A. penicillin *
B. lidaza *
C. erythromycin
D. nicotinic acid
2244. Connective tissue disease:
A. lupus erythematosus *
B. dermatomyositis *
C. psoriasis
D. allergic vasculitis
2245. Clinical types of lupus erythematosus:
A. discoid*
B. disseminated *
C. seborrheic
D. annular
2246. Periods of the syphilis:
A. incubation *
B. sharp
C. all right
D. primary *
2247. Main clinical forms of leprosy:
A. Tuberculoid *
B. Undifferentiated *
C. Kollikvativnom
D. Tuberous
2248. For tinea versicolor is characterized by:
A. peeling *
B. sample Signs *
C. symptom C. Besnier-Meshchersky
D. svechenie with green luminescence
2249. Clinical forms of athlete's foot:
A. intertriginous*
B. disgidroticheskaya *
C. worn
D. sharp
2250. Diseases related to dermatophytosis:
A. dermatophytosis *
B. pityriasis versicolor
C. eritrazma
D. microspores *
2251. What are the main clinical signs are not characteristic of chronic eczema:
A. bright redness *
B. edema, microerosion *
C. infiltration
D. licensure
2252. In the stage of vesicles for topical therapy eczema apply only:
A. lotions *
B. aerosols *
C. creams
D. paste
2253. Specify the symptom is not characteristic for discoid forms of lupus erythematosus:
A. Lichenification. *
B. Symptom of "butterfly".
C. a Symptom of "Apple jelly" *
D. Symptom Benies. Meshchersky.
2254. What disease is among the group of neurodermatoses:
A. eczema *
B. allergicheskie dermatitis *
C. pruritus
D. urticaria
2255. What is unusual in the clinical picture of rosacea:
A. erythema
B. follicular papules *
C. follicular hyperkeratosis *
D. infiltration of the skin of the nose
2256. Specify the most characteristic symptom of scabies:
A. evening and night itching *
B. the paired arrangement of the elements *
C. serous well
D. impetiginized in the lesions
2257. Enter among the following surface form stafilodermii:
A. vulgaris sycosis *
B. the osteo folliculitis *
C. furuncle
D. carbuncle
2258. In the therapy of lichen planus may apply:
A. diphenhydramine *
B. androgens
C. prednisolone
D. novopassit *
259. In the primary stage of syphilis patients may meet all the characteristics of chromes.
A. erosive papules *
B. non-erosive papules *
C. negative Wassermann reaction
D. lymphangitis
2260. Select the most effective agent for the treatment of chlamydial urethritis:
A. tetracycline *
B. doxycyclin *
C. penicillin
D. levamisole
2261. One of the clinical signs are not typical for AIDS patients:
A. vitiligo *
B. scabies *
C. lymphoma
D. recurrent herpes
2262. Specify drug, used in pemphigus vulgaris:
A. prednisolone *
B. dexamethasone *
C. monomitsin
D. Orungal
2263. Specify drug used in the treatment of scabies:
A. 20% benzyl benzoate *
B. 10% syntomycin ointment
C. Vishnevsky ointment
D. the aerosols of Spregal*
2264. Specify the medication used for candidiasis of the skin:
A. Orungal *
B. flonal *
Furazolidone C.
D. bactrim
2265. Specify drugs used in the progressive stage of psoriasis:
A. cream unna *
B. 2% salicylic ointment *
C. psoriasin ointment
D. ointment dermatol
2266. Specify the phenomenon are characteristic of lichen planus:
A. Cabrera *
B. tissue paper
C. fish ROE
D. grid Wickham *
2267. Specify the symptoms characteristic of multi-colored lichen:
A. peeling *
B. sample Signs *
C. sample Wickham
D. Kepner
2268. Specify a drug that cannot be set under a persistent white dermographism:
A. calcium chloride *
B. calcium gluconate *
C. sodium thiosulfate
D. pirogenal
2269. Specify the drug used in herpes zoster:
A. bonafton *
B. Dapsone
C. delagil
D. oxoline *
2270. Specify the clinical symptoms characteristic of sycosis:
A. chronically relapsing course *
B. localization on the skin of the Mons pubis *
C. rheumatic pain
D. nodules
2271 . Clinical signs of herpes simplex is not:
A. lichen sclerosis *
B. cicatricial atrophy *
C. eczema herpeticum
D. meningoencephalitis
2272. Which of diseases is not a manifestation of VICS. infection:
A. vitiligo *
B. scabies *
C. Kaposi's sarcoma
D. diarrhea
2273. That is not typical for psoriasis:
A. you can call animals in a laboratory *
B. not due to seasonality*
C., it can be first appearance after giving birth
D. due to seasonality
2274. Specify the characteristic symptom blister:
A. occurs due to the intra-cellular edema *
B. leave behind after erosion
C. quickly appears and disappears *
D. peeling on the surface of the blister
2275. What clinical manifestations are characteristic of secondary recurrent syphilis:
A. leukoderma *
B. alopecia *
C. parenchymatous keratitis
D. indurative edema
2276. What clinical symptoms are not in patients with primary syphilis:
A. ladonnas. plantar papules *
B. a chancre
C. indurative edema
D. extensive warts *
2277 . Specify the symptoms characteristic of uncomplicated chancre:
A. dense infiltration at the base of the ulcer *
B. painless ulcer *
C. saped edge
D. signs of inflammation
2278. For patients with chancroid is not typical:
A. the absence of pain *
B. solid infiltration *
C. signs of inflammation
D. inclined to autoinoculation
2279. In the treatment of gardnerellosis used:
A. tinidazole *
B. erythromycin
C. penicillin
D. metronidazole *
2280. For the diagnosis of some diseases is applied research using
wood's lamp:
A. microsporia *
B. eritrazma *
C. pityriasis rosea.
D. vitiligo.
2281. What primary morphological element precedes erosion:
A. gnojnice *
B. bubble *
C. none of the listed
D. hump
2282. What primary morphological element precedes the ulcer:
A. the node *
B. bump *
C. bubble
D. blister
2283. Specify the symptoms characteristic of uncomplicated chancre:
A. dense infiltration at the base of the ulcer *
B. painless ulcer *
C. saped edge
D. purulent discharge
2284. What pustular disease of the skin
not found in newborns:
A. hydradenitis *
B. sycosis *
C. vesiculopustules
D. pseudoharengus finger
2285. Specify the types of seborrhea:
A. dry *
B. vulgar
C. leaf
D. bold *
2286. What clinical symptom is noted
when lupus erythematosus:
A. "lady's thumb" *
B. The Symptom Benies. Meshchersky *
C. "blood dew"
D. symptom of Pincus
2287. Name the atypical form of psoriasis:
A. justiceability /rupiny/ *
B. intertriginous *
C. scalp
D. coin
2288. What is the most severe form of psoriasis:
A. artropaticheskom *
B. nummularnyi
C. erythrodermic *
D. geographical
2289. Name the localization of elements at
pustular psoriasis of the Barber:
A. palm *
B. the sole *
C. the red border of the lips
D. the skin of the genital area
2290. Name the type of pustular
psoriasis :
A. lichenoid
B. psoriasis of the Barber *
C. psoriasis Combucha *
D. nummularnyi
2291. What clinical symptom is typical
for dermatitis Duhring:
A. eosinophilia of content bubble *
B. blood eosinophilia *
C. microfocal pattern baldness
D. sclerodactyly
2292. What are clinical types of leishmaniasis cozenage You know:
A. acute nitrotyrosine type *
B. nedifferentsirovannost type
C. late. ulcerated type *
D. erythematous
2293. Name the clinical varieties
Borowski disease:
A. city type *
B. Ostro. nitrotyrosine type *
C. undifferentiated
D. infiltrative
2294. Which layer is not part of the epidermis:
A. net *
B. papillary *
C. granular
D. prickly
2295. Specify the types of seborrhea:
A. dry *
B. bold *
C. leaf
D. vegetative
2296. What diseases belong to the group of stafilodermy:
A. impetigo of Bockhart *
B. impetigo nail ridges
C. epidemic pemphigus *
D. slit vulgar ecthyma impetigo
2297. What tools have antipruritic property:
A. menthol*
B. benzocaine *
C. tar
D. salicylic acid
2298. What pyoderma occur only in newborns and infants:
A. vesiculopustules*
B. exfoliative dermatitis *
C. hydradenitis
D. furuncle
2299. What are morphological elements typical of Ritter's dermatitis:
A. bubbles *
B. bumps
C. papules
D. erosion *
2300. What dermatosis should different.
to differentiate lichen planus:
A. secondary syphilis*
B. psoriasis *
C. artifactual dermatitis
D. pityriasis versicolor
2301. Name the localization of elements at
pustular psoriasis of the Barber:
A. palm *
B. the sole *
C. scalp
D. the red border of the lips
2302. Specify the true types of eczema:
A. prodigiosa *
B. disgidroticheskaya *
C. numulyarnaya
D. seborrheic
2303. The rash of exudative mnogoformnuû
the erythema is:
A. in the form of arcs *
B. in the form of rings *
C. grouped the course of nerves
D. in pairs
2304. Specify the symptoms characteristic of scabies:
A. symptom Gorchakov Ardi *
B. nocturnal itching *
C. paired papules
D. bubbles
2305. What drugs is used to treat
scabies:
A. 20 % benzyl –benzoate ointment *
B. hydrocream
C. 5 % naphthalan ointment
D. spray Spregal *
2306. With what diseases should be differentiated scabies:
A. atopic dermatitis *
B. pruritus *
C. pityriasis rosea
D. tinea
2307. Name the obligate factor of dermatitis:
A. concentrated acid *
B. boiling water*
C. jewelry of gold
D. synthetic detergents
2308. Allergic reaction to the medication may occur:
A. bullous rash *
B. papular rash
C. symptom Pylnova
D. vesicular *
2309. Specify the kind of toxicodermia:
A. fixed erythema *
B. iododerma *
C. psoriasis
D. perniosis
2310. What medications are more likely to cause fixed erythema:
A. sulfadimetoksin *
B. Biseptol *
C. analgin
D. aspirin
2311. What disease should be differentiated common toksikodermiya:
A. acne rosacea
B. the syndrome Stevens. Johnson *
C. pyoderma
D. pityriasis rosea Gibert *
2312. Indicate the typical localization at the mnogoformnuû exudative erythema:
A. the back of the hand *
B. the rear brake *
C. scalp
D. interdigital spaces of the hands
2313. What factors are important in the development of mnogoformnuû of erythema multiforme:
A. seasonality *
B. hypersensitivity to fluoride
C. hypothermia *
D. hereditary
2314. What differentiate exudative dermatoses mnogoformnuû erythemal:
A. pemphigus vulgaris*
B. Lyell's syndrome *
C. strophulus
D. scrofuloderma
2315. Specify pathogenetic factors of pink lichen Gibert:
A. viral infection *
B. streptococcal infection *
C. yeast
D. hypofunction of the sebaceous glands
2316. Specify the variety of microbial eczema:
A. seksitarina *
B. idiopathic
C. adult
D. varicose *
2317. What clinical sign is characterized by a rash with true eczema:
A. true polymorphism *
B. symmetry *
C. ephemeral blisters
D. assimetria
2318. What are the symptoms seen with discoid lupus:
A. Voronova collar
B. symptom of "lady heel" *
C. collar, Byetta
D. symptom Khachaturian *
2319. Specify types of scleroderma:
A. stripy *
B. white spot disease *
C. infiltrative
D. intertriginous
2320. Lichen planus is characterized by:
A. papular rash *
B. the absence of subjective feelings
C. predominant localization on the flexor surfaces of the extremities
D. umbilicated*
2321. For lichen planus is characterized by:
A. irregular thickening of the granular layer of the epidermis *
B. acanthosis*
C. parakeratosis
D. vnutriarterialno microabcesses
2322. What clinical symptom is pathognomonic for lichen planus:
A. grid Wickham *
B. umbilicate indentation*
C. symptom of Auspice
D. cells Tzenka
2323. What diseases does not belong to the group of viral dermatoses:
A. extensive warts *
B. simple bubble herpes
C. psoriasis *
D. genital warts
2324. Vascular spot is:
A. Erythema *
B. Roseola *
C. Lentigo
D. Naevus
2325. Primary morphological elements:
A. ulcer
B. bubble *
C. bubble *
D. erosion
2326. Secondary morphological elements:
A. Ulcer *
B. Erosion *
C. Voldyri
D. Knot
2327. The bubble is formed by:
A. Eczema *
B. herpes simplex*
C. Psoriasis
D. CPL
2328. The outcome of the nodule is:
A. hypopigmented spot *
B. Ulcer
C. Atrophy
D. Scale *
2329. Exudative morphological element is:
A. Vesicle *
B. Blister*
C. Papule
D. Hump
2330. What drug is effective for genital herpes:
A. Alkaryl*
B. Neovir *
C. Sandimmune
D. Diprospan
2331. The clinical picture disgidroticheskaya form of athlete's foot:
A. bullas *
B. lichenification
C. bumps
D. bubbles *
2332. The clinical picture of infiltrative. nagnoitel'noj ringworm:
A. inflammatory infiltrate *
B. pustule *
C. lichenification
D. blisters
2333. Clinical symptoms Jock itch:
A. spot red *
B. the border is sharply limited *
C. blisters
D. nodes
2334. The most frequent localization of superficial candidiasis of the skin:
A. folds of skin *
B. vulva
C. flexor surface of wrists
D. the corners of the mouth *
2335. Fungal diseases of the skin are called:
A. epidermofiton *
B. Trichophyton *
C. Hansen of the Mycobacterium
D. Mycobacterium Koch
2336. The main clinical signs of lesions of the mucous membranes with candidiasis:
A. erosive surface *
B. white film *
C. mucous papules
D. bumps
2337. the drugs used in the treatment of fungal infections:
A. Ceftriaxone
B. nizoral *
C. lindamycin
D. griseofulvin *
2338. Localization of the rash in scabies in adults:
A. interdigital folds of the hands *
B. the flexor surface of forearms *
C. nails
D. palm
2339. The main clinical signs of scabies:
A. itching in the evening and at night *
B. acantholysis
C. nodes
D. the paired arrangement of morphological elements *
2340. Funds are used for the treatment of scabies:
A. sodium thiosulfate 60% and hydrochloric acid 6% *
B. benzyl benzoate 20% emulsion *
C. iodine tincture
D. prednisolone ointment
2341. Morphological elements characteristic of the clinic of uncomplicated scabies:
A. a knot *
B. spot
C. the vial *
D. hump
2342. Scabies is not characteristic symptom:
A. Asbestos - Gentina*
B. a symptom of the triangle
C. nocturnal itching
D. Nicholas *
2343. The causative agents of pustular diseases of the skin:
A. Streptococcus *
B. Proteus
C. Staphylococcus aureus *
D. the tubercle Bacillus
2344. The Streptococcus causes the disease:
A. impetigo *
B. superficial panaritium *
C. sycosis
D. folliculitis
2345. Varieties of streptococcal impetigo:
A. impetigo nail ridges *
B. the boil
C. Zayed road *
D. simple bubble herpes
2346. The localization of furuncle:
A. the neck *
B. the region of the back *
C. mucosa of the oral cavity
D. lips
2347. Clinical signs specific for staphylococcal deep pustules:
A. is riddled with hair *
B. hemispherical form *
C. pus liquid
D. is located on the smooth skin
2348. Hydradenitis is localized:
A. axillae *
B. peripapillary area *
C. palm
D. Shin
2349. Main clinical forms of staphylococcal skin lesions:
A. folliculitis *
B. herpes zoster
C. pityriasis versicolor
D. carbuncle *
2350. Clinical varieties of streptococcal impetigo:
A. bullous impetigo*
B. impetigo slit *
C. sycosis
D. deep folliculitis
2351. Complications arising in patients with the localization of furuncle on the face:
A. phlebitis of the vessels of the brain*
B. sepsis *
C. neuritis of the facial nerve
D. endocarditis
2352. Deeper forms of streptodermii:
A. vulgar ecthyma*
B. periungual conflicts
C. streptococcal impetigo
D. drill ecthyma *
2353. The factors that cause toksikodermiya:
A. sulfanilamidnye drugs *
B. antibiotics *
C. acid
D. alkali
2354. Clinical signs of urticaria:
A. pruritus*
B. blister *
C. node
D. pain
2355. Irritants cause irritant contact dermatitis:
A. mechanical *
B. high temperature *
C. psycho-emotional
D. chemical C. optional
2356. For allergic dermatitis is characterized by:
A. erythema *
B. bubbles *
C. bumps
D. nodes
2357. Symptoms typical of true eczema:
A. true polymorphism *
B. microvesicles *
C. blisters
D. cicatricial atrophy
2358. Characteristic of eczema:
A. roseola
B. oozing "serous wells" *
C. Gumma
D. erythema, swelling*
2359. Clinical manifestations fresh secondary period of syphilis:
A. chancre or its remains *
B. roseola *
C. bubbles
D. blisters
2360. Clinical signs of syphilitic roseola:
A. blades. pink *
B. does not merge *
C. flakes
D. purple
2361. Clinical signs of tinea versicolor:
A. secondary leukoderma *
B. erosion
C. nodes
D. defurfuration *
2362. The source of the infection by Microsporum:
A. cats, dogs *
B. people*
C. cattle
D. rodents
2363. When candidiasis affects:
A. mucous *
B. skin *
C. hair
D. the muscular system
2364. Primary cells for herpes simplex:
A. erythema *
B. bubble *
C. blister
D. abscess
2365. To viral diseases include:
A. simple bubble herpes
B. herpes zoster *
C. molluscum contagiosum *
D. eczema
2366. What medications are more likely to cause fixed erythema
A. sulfadimetoksin*
B. analgin
C. aspirin
D. Biseptol*
2367. What disease should be differentiated common
toksikodermiya
A. herpes
B. eczema*
C. vulgaris pemphigus *
D. a disease of Borovsk
2368. Indicate the typical localization at mnogoformnuû exudative
the erythema
A. the scalp
B. the rear brake*
C. the neck
D. the rear palm*
2369. What factors are important in the development of mnogoformnuû
exudative erythema
A. Neurotic factor*
B. photosensitization
C. chronic infection*
D. hyperfunction of the sebaceous glands
2370. What dermatoses differentiate mnogoformnuû erythemal
A. Lyell's syndrome*
B. scrofulous
C. pruritus
D. Syndrome Steve-Zhons*
2371. Specify pathogenetic factors of pink lichen Gibert
A. introduction to skin mites
B. yeast
C. Infektsionno-allergic process*
D. viral infection*
2372. Specify the variety of microbial eczema
A. adult
B. paratravmaticheskoy*
C. idiopathic
D. Trophic ulcer*
2373. What clinical sign is characterized by a rash with true
eczema
A. symptom <serous wells>*
B. tubercles
C. cilnii itching*
D. indurative edema
2374. What are the symptoms seen with discoid lupus
A. grid Wickham
B. a collar of Byetta
C. a symptom of "lady heel"*
D. atrophy*
2375. Specify types of scleroderma
A. vulgar
B. white spot disease*
C. infiltrative
D. linear*
2376. Lichen planus is characterized by
A. polygonal papule*
B. tubercles
C. grid Wickham*
D. urticarial rash
2377. For lichen planus is characterized by
A. intercellular edema spinous layer
B. lichenoides response*
C. vacuolar degeneration
D. uneven thickening of the granular layer of the epidermis*
2378. What clinical symptom is pathognomonic for red
lichen planus
A."honeycomb celse"
B. grid Wickham*
C. koebner's phenomenon*
D. symptom of Auspice
2379. What diseases does not belong to the group of viral dermatoses
A. warts
B. genital warts
C. psoriasis*
D. mycosis*
2380. Vascular spot is
A. Erythema
B. roseola*
C. Georgia*
D. Leukoderma
2381. Primary morphological elements
A. scar
B. bubble*
C. erosion
D. papule*
2382. Secondary morphological elements
A. The Scar*
B. Burl
C. Node
D. Ulcer*
2383. The bubble is formed at
A. puzyrchatka*
B. the LCP
C. herpes simplex*
D. Psoriasis
2384. The outcome of the burl is
A. Spot*
B. Atrophy
C. nothing*
D. Scar
2385. Exudative morphological element is
A. Node
B. Tubercle
C. Vesicle*
D. Papule
2386. Specify dosage form for local treatment according to stage
eczema:
A. weeping – lotions*
B. stage C. secondary pyoderma gadgets
C. bubble step C. Masi
D. corticosteroids ointments*
.
2387. What diseases belong to the group of neurodermatoses:
A. urticaria*
B. dermatitis Duhring
C. lichen planus
D. Pruritus*
2388.Specify the reasons that may be causing localized
itching:
A. Contact dermatitis*
B. violation of function of endocrine glands
C. The Hemorrhoids*
D. Ketoacidosis
2389. What preparations have antipruritic effect:
A. anesthetics*
B. sulfonamides
C. antibiotics
D. an antihistamine*
2390. What are the comorbidities often found in
atopic dermatitis:
A. Asthma*
B. toxoplasmosis
C. tuberculosis
D. vasomotor rhinitis*
2391. When any disease occurs white dermographism:
A. scabies
B. allergic dermatitis
C. pruritus*
D. atopic dermatitis*
2392. Specify the basic pathogenetic mechanisms of development of child
pocesuha:
A. hereditary fermentopathy*
B. the hearth of focal infection
C. endocrine disorders
D. after self-poisoning from the bowel*
2393. What kind of rash is typical for psoriasis:
A. vegetative
B. peeling*
C. monomorphic pustular
D. Papular*
2394. What zone are distinguished in focal neurodermatitis:
A. peripheral C. area pustular rash
B. Central area C. lichenification*
C. peripheral zone papulezne infiltrate*
D. mid-zone of depigmentation
2395, What signs are characteristic for hives:
A. secondary atrophic changes in the skin
B. Krasne dermographism*
C. monomorphic wheals*
D. white dermographism
2396. In a patient with acute angioedema, beginning asphyxia. What kind of prepyou want to assign to rata for assistance:
A. korglikon 0.5/m
B. 200,0 gemodez drip into a vein
C. aminophylline 2.4%*
D. prednisolone 60-80 mg in a vein*
2397. What diseases belong to the group of bullous dermatoses:
A. stafilodermia
B. Vulgar pemphigus*
C. . tinea
D. Syndrome And Lyell's Syndrome*
2398. If any are observed dermatoses bullous rashes:
A. herpes
B. stafilodermia
C. Lyell's syndrome*
D. pemphigus Vulgaris*
2399. In the classification of pemphigus includes:
A. pemphigus vulgaris*
B. exudative
C. Palmar-plantar
D. Seborrheic pemphigus*
2400. What are the symptoms of true pemphigus:
A. Symptom Gorchakov-Ardi
B. Symptom of "Apple jelly"
C. Nikolsky Sign*
D. Asbestos-Hansen*
2401. Under what conditions there is a positive symptom
Nicholas:
A. contact dermatitis bullous stage
B. Vulgar pemphigus*
C. Lyell's Syndrome*
D. bullous streptoderma
2402. What are the basic rules for the use of corticosteroids-hormones in the treatment of
pemphigus:
A. hormones is prescribed only for topical treatment
B. begin treatment with a loading dose*
C. begin treatment with a gradual increase in dose up to shock
D. podderjivali dose*
2403. What complications occur in the treatment of pemphigus:
A. development of hypersensibility
B. oppression the function of hematopoietic system
C. Syndrome cancel*
D. The Syndrome Itsenko-Kushinga*
2404. What are the characteristic clinical signs of dermatitis Duhring:
A. lesion of the oral mucosa
B. positive Nikolsky sign
C. sgruppirovany rash*
D. A Polymorphism*
2405. What are the morphological elements characteristic of the rash
dermatitis Duhring:
A. pustule*
B. the nodes
C. bumps
D. papular*
2406. What laboratory indexes are characteristic of dermatitis Duhring:
A. eosinophils in the cystic fluid*
B. lymphocytosis
C. chuvstvitelnost to iodine*
D. histomorphological – acantholysis
2407. For any of dermatoses characterized by the appearance of rashes on
the oral mucosa:
A. psoriasis
B. candidiasis*
C. pityriasis rosea Gibert
D. CPL*
2408. Name rash with a clear viral etiology:
A. dermatitis Duhring
B. lichen planus
C. herpes*
D. containskey clam*
2409. Name the characteristics of the lesions for herpes simplex:
A. C. subjectively prevails over itching soreness and a burning sensation
B. polymorphism of the rash
C. monomorphic bubble elements*
D. erosion*
2410. What are the distinctive features of genital herpes from solid
chancre:
A. the lack of hard infiltration at the base*
B. a positive result PB
C. painless erosions
D. pain erosions*
2411. Name the clinical forms of tinea:
A. Dyshidrotic
B. generalized*
C. monomorphic bubble elements*
D. Neuralgic
2412. Specify medicines are antiviral
action:
A. methotrexate is used
B. purified sulfur
In gerpevir*.
D. acyclovir*
2413. Assign medicines to patient with a diagnosis of tinea versicolor:
A. the sodium-bromine drops
B. prednisolone ointment
C. analgesics*
D. acyclovir*
2414. Specify characteristic features of molluscum contagiosum:
A. the way of transmission of vector-borne C.
B. nodules contain inside a white cheesy mass*
C. hemispherical form*
D. symptom of "grid Wickham" positive
2415. What forms of lupus erythematosus are distinguished according to the classification:
A. discoid*
B. vegetative
C. disseminirovanne*
D. Pustular
.
2416. Name the cardinal symptoms of lupus erythematosus rash:
A. lichenification
B. keloid scars
C. follicularis hyperkeratosis*
D. infiltrative erythema*
2417. What symptoms are observed in discoid lupus erythematosus:
A. grid Wickham
B. "butterflies"*
C. "honeycomb" of Celsus
D. atrophy of the skin*
2418. With what diseases should be differentiated red
lupus:
scleroderma
A. dermatitis Duhring*
B. photodermatosis
C. Eczema
D. Scleroderma*
2419. What stage are distinguished in clinical course of scleroderma:
A. edema*
B. exudation
C. ulceration
D. Seal*
2420. Some dermatoses are a group of connective tissue:
A. discoid lupus erythematosus*
B. dermatitis Duhring
C. genital warts
D. scleroderma*
2421. What skin tumors are considered benign:
A. carcinoma
B. lipoma*
C. melanoma
D. naevus*
2422. Favourite localization molluscum contagiosum:
A. axillary region
B. hands and feet
C. face*
D. polowie bodies*
2423. Name the clinical varieties of acne:
A. conglobate*
B. erythematous-squamous
C. erosive and ulcerative
D. Pustular*
2424. Specify the stage of rosacea:
A. papulo-pustular*
B. follicular hyperkeratosis
C. stage of atrophy
D. erythematous*
2425. What are the exogenous factors which can cause pathological
the condition in the skin:
A. disorders of the nervous system
B. hypersensibility
C. pathogenic fungi*
D. pathogenic bacteria*
2426. To endogenous physiological factors that can cause
inflammatory changes in the skin include:
A. viruses
B. genetics factor*
C. physical
D. metabolic disorders*
2427. Primary morphological elements can be:
A. proliferative*
B. indurative
C. exudative*
D. Follicular
2428. Which of the following lesions refers to exudative:
A. roseola
B. vesicle*
C. tubercle
D. Blister*
2429. What are the primary proliferative elements:
A. a knot*
B. blister
C. vesicle
D. the node*
2430. Spot C. this is the primary element resulting from:
A. answer B*
B. as a result of the accumulation of melanocytes in a particular area of the skin*
C. change of skin color as a result of acantholysis
D. color change as a result of an overactive sebaceous glands
2431. What types of spots are distinguished:
A. papillomatous*
B. iron deficiency*
C. Pigment
D. depigment
2432. Name the types of vascular stains:
A. hyperpigmentation
B. petechia*
C. telangiectasias*
D. Tattoo
2433. What types of hemorrhagic spots:
A. nevus
B. petechiae*
C. lentigo
D. hematoma*
2434. What are not characteristic symptoms of the blister:
A. bespalatnoe element, resulting from edema of the papillary layer
dermis
B. occurs as a result of acantholysis*
C. characteristic of autoimmune diseases*
D. is accompanied by subjective sensations
2435. Name the characteristic features of the bubble:
A. is formed as a result of edema of the papillary dermis*
B. is a proliferative element*
C. exudative primary element contains a liquid
D. Exsudative element
2436. What distinguishes a bubble from the bubble:
A. localization of elements
B. content of the exudate
C. grouping*
D. size C. the bubble is less than 0.5 cm, the bubble more*
2437. Name the layer of the epidermis:
A. horn*
B. papillary
C. basal*
D. Fat
2438. That refers to the appendages of the skin:
A. papillae of the dermis
B. the epidermis
C. Hair*
D. Nails *
2439. What primary morphological elements:
A. atrophy
B. papule*
C. follicle
D. the node*
2440. What kind of pustules you know:
A. ecchymosis
B. petechia
C. impetigo*
D. conflicts*
2441. What types of papules are distinguished by the size:
A. lichenoid
B. dwarf
C. lenticular*
D. Miliary*
2442. Name the characteristics of the knot:
A. bespalatnoe primary element*
B. is ephemeral
C. contains serous exudate
D. proliferative element*
2443. Name the characteristics for tubercle and host:
A. refers to the elements of exudative
B. the infiltrate is located in the epidermis*
C. are besplatnymi elements*
D. are ephemeral formations
2444. Write to the primary element and the corresponding pathological process:
A. C. vesicle atrophy
B. tuberculum –
C. proliferative*
D. the node *
2445. Locate the secondary morphological elements:
A. papule
B. erosion*
C. conflicts
D. ulcer*
2446. What are not characteristic features of lichenification:
A. atrophic changes*
B. hyperhidrosis on the hearth*
C. exudation
D. enhanced skin pattern
2447. What are the primary elements and their corresponding reverse
the development of secondary elements:
A. node
B. secondary depigmentation
C. bladder – erosion*
D. tuberculum*
2448. Name the pyoderma is not specific to childhood:
A. hydradenitis*
B. basal cell carcinoma*
C. epidemic pemphigus newborns
D. Acne
2449. What factors contribute to the development of pyoderma:
A. diabetes mellitus*
B. violation of the mechanism of division of epidermocytes
C. acantholysis
D. immunodeficit*
2450. What are the main clinical signs of scabies:
A. symptom Pospelova
B. sgruppirovany bubbles
C. paired papular or papulo-vesicular elements*
D. itching nochyu*
2451. What funds are used for the treatment of scabies:
A. a solution of Dimexidum
B. method of dem'yanovich*
C. gray mercury ointment
D. sulfur ointment*
2452. Name the basic clinical signs of head lice:
A. finding ticks*
B. paired papular elements*
C. itching
D. finding lice
2453. With what diseases differentiate scabies:
A. pruritus*
B. herpes zoster
C. leishmaniasis
D. atopic dermatitis*
2454. What are the lesions characteristic of the lepromatous type of leprosy:
A. vesicular elements
B. lepromy*
C. gummy
D. mutilate*
2455. What symptoms occur due to a variety of motor
disorders in leprosy:
A. symptom of "fish eggs"
B. symptom of "butterfly"
C. "the masque of St. Anthony"*
D. the face of La*
2456. What laboratory and instrumental methods are used to
diagnosis of leprosy:
A. larobina sample*
B. Mantoux test
C. the reaction Kahn
D. bacterioscopy*
2457. Which of these provisions are not considered when setting the correct sample:
A. tuberculous type*
B. negative sample
C. lepromatous
D. undifferentiated type*
2458. Specify what skin diseases are considered contagious:
A. psoriasis
B. pink ringworm Gibert
C. mycosis*
D. Pediculosis*
2459. Specify proliferative elementi:
A. papule*
B. nodes*
C. vesicle
D. Bulla
2460. Specify the reservoir of infection in leishmaniasis:
A. poultry
B. mosquitoes and fleas
C. ground squirrels*
D. a sick man*
2461. List the main preventive measures when
leishmaniasis:
A. rodent control work in the centers*
B. disinfection of the room and linen
C. .destruction of stray dogs
D. prophylactic medical examination*
2462. What laboratory and instrumental methods of diagnosis
used for the diagnosis of tuberculosis of the skin:
A. sample of Minor
B. test Jadasson
C. Mantoux test*
D. Pirok*
2463. Specify the localized forms of tuberculosis of the skin:
A. papulonecrotic
B. ulcerative tuberculosis*
C. indurative
D. tuberculosis of the skin*
2464. What is called dermatosis scaly ringworm:
A. Allergic dermatitis
B. Toxicodermia
C. Psoriasis*
D. CPL*
2465. Specify where the correct compliance with the pathogen diagnosis:
A. Leprosy*
B. chopsticks
C. nevi
D. the Herpes virus*
2466. Specify where the correct compliance with the pathogen diagnosis:
A. Microsporia C. Mycoplasma
B. acne rosacea C. mites of the genus Demodex*
C. Dermatophytosis C. Trichomonas
D. The herpes virus*
2467. Specify the exogenous factors of nature cause
histopathological changes in the skin:
A. Immune deficiency
B. The Alkaline*
C. the State of hypersensibility
D. Acid*
2468. Specify the endogenous nature of factors causing
histopathological changes in the skin:
A. Immune deficiency*
B. Autoimmunization*
C. Substances oncogenic action
D. Pathological fungi
2469. What layers are distinguished in the epidermis:
A. Net
B. basal*
C. Granular*
D. Lipoid
2470. That refers to the appendages of the skin:
A. Hair*
B. keratinocytes
C. connective tissue of the dermis
D. Nails*
2471. Specify where on the skin of the human body most are
apocrine sweat glands:
A. On the skin of the palms and soles skin
B. In the armpit
C. On the face*
D. groin*
2472. What physiological functions are performed by skin:
A. involved in the synthesis of sex hormones
B. the Function of the depot steroids
C. Protective*
D. dyhatelnaya*
2473. Specify primary mifologicheskie elements:
A. A Vesicle
B. Erosion
C. Atrophy
D. papule*
2474. Specify which morphological elements are not primary:
A. Abscess
B. Hemorrhagic spot
C. ulcer*
D. Scratch*
2475. Specify exudative primary element:
A. Papule
B. Tubercle
C. Vesicle*
D. Urtica*
2476. Specify primary proliferative element:
A. A Vesicle
B. Knot*
C. Scratch
D. Tuberculum*
2477. What types of spots are distinguished:
A. Faux*
B. Exudative
C. Hypertrophic
D. defintatly*
2478. Which of the following refers to vascular spots:
A. Secondary pigmentation
B. Roseola*
C. Webiress
D. petechia*
2479. With what diseases should be differentiated leishmaniasis:
A. A Disease Of Borovsk
B. Dermatitis Duhring
C. Leprosy*
D. cutaneous tuberculosis*
2480. What tests should be appointed to confirm the diagnosis
scrofuloderma:
A. Biopsy*
B. Laromana sample
C. Mantoux Test*
D. Sample Of Minor
2481. What types of fungal infections differ according to the classification of A.Saklakova:
A. Condylomatosis
B. Sporadicity
C. Dermatomycosis*
D. keratomas*
2482. What diseases belong to the group of ringworm:
A. pahova epidermofitia*
B. psoriasis
C. lichen planus
G dermatophytosis*
2483. What clinical forms of rubromycosis are distinguished:
A. Rubromikoz of the oral mucosa
B. Rubromikoz scalp
C. Rubromikoz of the palms and soles*
D. deep*
2484. In what diseases may occur as a symptom of alopecia:
A. Rubromikoz
B. Leishmaniasis
C. Microspores*
D. dermatophytosis*
2485. What factors lead to enhancement of Candida infection:
A. Long-term antibiotic therapy*
B. the State of hypersensibility
C. Autonomic dysfunction
D. immunodeficit*
2486. Look what disease is not consistent with the assigned
drug:
A. Dermatophytosis – griseofulvin
B. Candidiasis – metronizadol*
C. C. Pemphigus polkortolon
G mycosis metronizadol*
2487. What spots are hemorrhagic:
A. Leukoderma
B. Erythema
C. Petechiae*
D. roseola*
2488. Specify the characteristic blister:
A. the Ephemeral nature of the rash*
B. Deep exudative primary element
C. Accompanied by a feeling of soreness
D. Characteristic of viral diseases*
2489. Specify the varieties of pustules:
A. Folliculitis*
B. Granuloma
C. Comedones
D. ecthyma*
2490. Specify pinaki characteristic papules:
A. Ephemeral element C. unstable
B. After it leaves scar atrophy
C. extends above the level of the skin*
D. up To 1 cm*
2491. What histopathological changes in the skin can lead to
the formation of papules:
A. Acantholysis
B. Acanthosis*
C. hyperkeratosis*
D. Pompholyx
2492. What are the papules vary in size:
A. Skutulârnaâ
B. Miliary*
C. Hypertrophic
D. numularia*
2493. Clinical stage and types of psoriasis:
A. stationary*
B. sharp
D. subacute
E. progressive*
2494. Phenomena typical for psoriasis:
A. stearin spot*
B. terminal film *
C. Asbestos-Hansen
D. Stevens-Jones
2495. For lupus erythematosus symptoms:
A. erythema*
B. follicular hyperkeratosis*
C. grid Wickham
D. mukovidnoe peeling
2496. Stage of scleroderma:
A. atrophy*
B. edema*
C. peeling
D. hyperkeratosis
2497. Periods of the syphilis:
A. incubation*
B. sharp
C. all right
D. primary*
2498. Main clinical forms of leprosy:
A. Tuberculoid*
B. Undifferentiated*
C. Kollikvativnom
D. Tuberous
2499. Clinical forms of athlete's foot:
A. intertriginous*
B. sharp
C. worn
D. disgidroticheskaya*
2500. Diseases related to dermatophytosis:
A. dermatophytosis*
B. microspores*
C. eritrazma
D. pityriasis versicolor
2501.What distinguishes a bubble from the bubble
A. the size of the bubble is less than 05 cm bubble more*
B. mechanism of formation of ( vacuolar degeneration and acantholysis)*
C. secondary changes
D. subjective sensations
2502. Exudative morphological element is:
A. papule
B. blister*
C. abscess*
D. hump
2503. Exudative morphological element is:
A. vesicle*
B. node
C. the knot
D. bubble*
2504. Have a cavity morphological elements:
A. blister
B. bubble*
C. tubercle
D. conflicts*
2505. Have a cavity morphological elements:
A. abscess*
B. node
C. bubble*
D. papule
2506. Clinical signs of lichen planus:
A. koebner's phenomenon*
B. the nodes
C. blisters
D. severe itching*
2507. Clinical signs of lichen planus:
A. abscesses
B. polygonal papules*
C. pupkoobraznym depressions in the center*
D. bumps
2508. Agents used topically in the treatment of psoriasis:
A. Castellani paint
B. corticosteroid ointment*
C. ointment benzyl benzoate
D. mitigating and indifferent ointment*
2509. Agents used topically in the treatment of psoriasis:
A. gel Skin cap*
B. sulfuric ointment 33%.
C. salicylic ointment*
D. nizoralbuy ointment
2510. Clinical varieties of pemphigus:
A. exudative
B. psoriasiform
C. leaf*
D. seborrheic*
2511. Clinical varieties of pemphigus:
A. vulgar*
B. vegetative*
C. papular
D. raspostraneny
2512. Clinical varieties of pemphigus:
A. exudative
B. vegetative*
C. raspostraneny
D. seborrheic*
2513. Clinical varieties of pemphigus:
A. vulgar*
B. raspostraneny
C. leaf*
D. psoriasiform
2514. The antibiotics used in the treatment of candidiasis:
A. cephalosporins
B. flonal*
C. kanamycin
D. diflucan*
2515. The antibiotics used in the treatment of candidiasis:
A. nystatin*
B. doxycyclin
C.levorin*
D. chloramphenicol
2516. The antibiotics used in the treatment of candidiasis:
A. kanamycin
B. flonal*
C. levorin*
D. doxycyclin
2517. When artropaticheskom appropriate will appoint all of the above, except:
A. antioxidants
B. penicillin*
C. nonsteroidal anti-inflammatory drugs
D. terbisil*
2518. When artropaticheskom appropriate will appoint all of the above, except:
A. nizoral*
B. mineralocorticoid drugs
C. synthetic anti-malarial drugs*
D. detoxifying tools
2519. When artropaticheskom appropriate will appoint all of the above, except:
A. mineralocorticoid drugs
B. non-steroidal anti-inflammatory drugs
C. synthetic anti-malarial drugs*
D. terbisil*
2520. What type of topical treatment should be chosen in a moist inflammation of the skin:
A. lotions*
B. Prater Park
C. cream
D. wet-drying bandages*
2521. What type of topical treatment should be chosen in a moist inflammation of the skin:
A. ointment
B. aerosols*
C. aniline dyes*
D. lucky
2522. What type of topical treatment should be chosen in a moist inflammation of the skin:
A. gadgets*
B. ointment
C. aniline dyes*
D. cream
2523.What kind of lotions should choose to oozing of the infected skin surface:
A. ivanyevo*
B. lead
C. with potassium permanganate*
D. tannin
2524.What kind of lotions should choose to oozing of the infected skin surface:
A. boric acid
B. furatsilinovoy*
C. zinc
D. with a dilute solution of citala*
2525. In acute and subacute complicated gonorrhea in men and women in the complex treatment
included all of the above, exceptA. antihistamines*
B. antispasmodics
C. sulfonamides
D. pirogenal*
2526. In acute and subacute complicated gonorrhea in men and women in the complex treatment
included all of the above, exceptA. local therapy
B. gonovaccine*
C. the desensitization*
D. protivolodochnyi antibiotics
2526.What kind of lotions should choose to oozing of the infected skin surface:
A. zinc
B. lead
C. with potassium permanganate*
D. with a dilute solution of citala*
2527. Specify the correct tactics of treatment of torpid and chronic forms of gonorrheal infection.
A. immunotherapeutic drugs*
B. antifungal
C. calcium supplements
D. Provocation*
2528. Specify the correct tactics of treatment of torpid and chronic forms of gonorrheal infection
A. detoxification drugs
B. local treatment*
C. antibacterial*
D. antihistamines
2529.If any element of dermatitis rash is the bubble:
A. pityriasis rosea Gibert
B. pemphigus
C. tinea*
D. genital herpes*
2530.What factors contribute to the development of pyoderma:
Hypervitaminosis A.
B. the presence of a large number of pathogenic strains*
C. hyperthyroidism
D. microtrauma of the skin*
2531. From the ectoderm develop:
A. hypodermis
B. sebaceous glands*
C. muscles of the hair
D. integumentary epidermis*
2532. From the ectoderm develop:
A. hair*
B. dermis
C. sweat glands*
D. the vessels of the skin
2533.If any element of dermatitis rash is the bubble:
A. eczema*
B. herpes simplex*
C. erythematous
D. psoriasis
2534.What factors contribute to the development of pyoderma:
A. disorders of carbohydrate metabolism*
B. physical activity
C. reduction of the barrier functions of the skin *
D. increased blood clotting
2535. To has is characterized by:
A. fever, malaise, headache *
B. estimatese pustules
C. favourite localization in the armpits, genitals *
D. the paired elements
2536. To a mixed pyoderma include:
A. carbuncle
B. chancriform pyoderma*
C. pyoderma of Bochart
D. vulgar impetigo*
2537. Specify the clinical form of scabies (form.:
A. squamous
B. clean*
C. erythematous
D. pastoriza lymphopenia*
2538. To has is characterized by:
A. papules
B. the formation of fistulous*
C. blister
D. the emergence of painful infiltration*
2539. To a mixed pyoderma include:
A. brimicombe*
B. the boil
C. yasenovitsa pyoderma *
D. osteo folliculitis
2540. Specify the clinical form of scabies (form.:
A. worn*
B. sharp
Norwegian C.*
D. chronic
2541. Indicate the clinical varieties of pemphigus acantholycosa:
A. circle
B. zosterifolia
C. vegeliusa*
D. seborrheic*
2542. Dermatosis at what primary morphological element is a node:
A. tuberculosis of the skin*
B. urticaria
C. tertiary syphilis*
D. psoriasis
2543. Indicate the clinical varieties of pemphigus acantholycosa:
A. leaf*
B. vulgar*
C. syndrome Stevens Johnson
D. epidemic pemphigus newborns
2544. Dermatosis at what primary morphological element is a node:
A. angioedema
B. leishmaniasis of the skin*
C. genital warts
D. leprosy*
2545. Under what medicine the main element of the rash is pimple:
A. molluscum contagiosum*
B. eritrazma
C. pemphigus
D. lichen planus*
2546. Under what medicine the main element of the rash is pimple:
A. herpes zoster
B. warts *
C. psoriasis*
D. pityriasis rosea Gibert
2547. Clinical symptoms of discoid lupus erythematosus:
A. erythema*
B. bubbles
C. atrophy*
D. blister
2548. Under what medicine the main element of the rash is not pimple:
A. pityriasis rosea Gibert*
B. psoriasis
C. tinea*
D. warts
2549. Clinical symptoms of discoid lupus erythematosus:
A. bullas
B. tightly sitting scales*
C. abscesses
D. deterioration after sun exposure*
2550. If any element of dermatosis rash is not a bubble:
A. herpes simplex
B. pemphigus *
C. eczema
D. psoriasis*
2551. Dermatosis at what primary morphological element is not a node:
A. the leishmaniasis of the skin
B. genital warts*
C. angioedema *
D. tuberculosis skin
2552. If any element of dermatosis rash is not a bubble:
A. pityriasis rosea Gibert *
B. genital herpes
C. erythematous*
D. tinea
2553. Dermatosis at what primary morphological element is not a node:
A. psoriasis*
B. tertiary syphilis
C. cutaneous leishmaniasis
D. urticaria*
2554. Add the secondary items that are not the result of the transformation of the knot:
A. secondary hypopigmented spot
B. vanish
C. scar*
D. cicatricial atrophy*
2555. Add the secondary items that are not the result of the transformation of the knot:
A. erosion*
B. ulcer *
C. scales
D. secondary hyperpigmented spot
2556. What drugs are used for photochemotherapy in psoriasis:
A. psoralen*
B. povalen*
C. acyclovir
D. nystatin
2557. What drugs are used for photochemotherapy in psoriasis:
A. psoralen*
B. povalen*
C. lamisil
D. vincristine
2558. For secondary syphilis is characterized by:
A. pustular*
B. chancre and polyadenylation
C. papular rash*
D. Gumma
2559. For secondary syphilis is characterized by:
A. syphilitic rhinitis
B. roseolous rash*
C. infiltration of Gachinger
D. extensive warts*
2560. The clinical picture disgidroticheskaya form of athlete's foot:
A. bullas*
B. bumps
C. edema*
D. peel
2561. The clinical picture disgidroticheskaya form of athlete's foot:
A. bullas*
B. blisters
C. edema*
D. lichenification
2562. The clinical picture of infiltrative-nagnoitel'noj ringworm:
A. bullas
B. pustule*
C. hair loss*
D. blisters
2563. The clinical picture of infiltrative-nagnoitel'noj ringworm:
A. inflammatory infiltrate*
B. bullas
C. nodes
D. scar*
2564.Fungal diseases of the skin are called:
A. Mycobacterium Koch
B. Trichophyton *
C. Hansen of the Mycobacterium
D. yeast*
2565.Fungal diseases of the skin are called:
A. epidermofiton*
B. Staphylococcus aureus
C. Microsporum*
D. mites
2566.Localization of the rash in scabies in adults:
A. interdigital folds of the hands*
B. nails
C. palm
D. skin inner thighs*
2567.Localization of the rash in scabies in adults:
A. mucous membranes
B. the flexor surface of forearms*
C. the skin of the abdomen*
D. soles
2568. Funds are used for the treatment of scabies:
A. sodium thiosulfate 60% and hydrochloric acid 6%*
B. benzyl benzoate 20% emulsion*
C. prednisolone ointment
D. iodine tincture
2569. Funds are used for the treatment of scabies:
A. sodium thiosulfate 6% hydrochloric acid 60%
B. nystatin cream
C. sulfuric ointment 33%*
D. spray Spregal*
2570.Scabies is not characteristic symptom:
A. Gorchakov-Ardi
B. nocturnal itching
C. Besnier-Meshchersky*
D. Nicholas *
2571.Scabies is not characteristic symptom:
A. Asbestos-Gentina*
B. Pilnov*
C. the Symptom triangle Merkelson
D. the pairing of elements
2572. The Streptococcus causes the disease:
A. impetigo*
B. folliculitis
C. Zayed road*
D. sycosis
2573. The Streptococcus causes the disease:
A. simple bubble herpes
B. superficial panaritium*
C. tinea
D. simple versicolor*
2574. Varieties of streptococcal impetigo:
A. impetigo nail ridges *
B. Zayed road *
C. furuncle
D. a simple bubble herpes
2575. Varieties of streptococcal impetigo:
A. syphilitic pemphigus
B. epidemic pemphigus
C. bullous impetigo*
D. intertriginous diaper rash*
2576. The outcome of the nodule is:
A. Ulcer
B. scale*
C. Atrophy
D. hyperpigmented spot*
2577. The outcome of the nodule is:
A. hypopigmented spot*
B. Peel
C. vanish*
D. Scar
2578.Main clinical forms of staphylococcal skin lesions:
A. pityriasis versicolor
B. herpes zoster
C. parasitic sycosis*
D. furuncle*
2579.Main clinical forms of staphylococcal skin lesions:
A. folliculitis *
B. carbuncle*
C. ecthyma vulgar
D. bullous impetigo
2580. To viral diseases include:
A. versicolor is a simple bubble*
B. lupus erythematosus
C. eczema
D. warts*
2581. To viral diseases include:
A. pemphigus vulgaris
B. herpes zoster *
C. molluscum contagiosum *
D. vulgaris sycosis
2582. Histological features characteristic of psoriasis:
A. acantholysis
B. acanthosis*
C. spongiose
D. Munro microabscesses*
2583. Histological features characteristic of psoriasis:
A. parakeratosis*
B. vacuolar degeneration
C. papillomatosis*
D. bulloneria degeneration
2584. The bubble allowed the formation of:
A. scar
B. peel*
C. ulcer
D. scale*
2585. The bubble allowed the formation of:
A. spot*
B. lichenification
C. erosion *
D. atrophy
2586. What primary morphological element precedes erosion:
A. Bulla*
B. vesicle *
C. Urtica
D. macula
2587. What disease is not part of dermatophytosis:
A. sporotrichosis*
B. hromomikoza*
C. rubrofitii
D. trichophytosis
2588. Specify the drug used in leprosy:
A. Dapsone *
B. sulfation *
C. monomitsin
D. delagil
2589. In the treatment of candidiasis is not used:
A. polkortolon *
B. acyclovir *
C. nizoral
D. nystatin
2590. Nikolsky sign is positive in:
A. true pemphigus*
B. medicamentous allergic stomatitis *
C. schoolderman
D. dermatitis herpetiformis during the
2591. Pathognomonic for eruptive eczema are:
A. the microvesicles *
B. peel
C. weeping erosions *
D. scales
2592. What are the main clinical signs are not characteristic of chronic eczema:
A. bright redness, swelling *
B. infiltration
C. microerosion, brown*
D. licensure
2593. For any of these viral diseases of the oral mucosa is not affected:
A. molluscum contagiosum *
B. simple bubble herpes
C. varicella
D. warts *
2594. Exclude lesions not typical for rubrofitia:
A. the palm
B. the mouth *
C. sole
D. hair *
2595. what pathogens are determined by the Method of staining by Ziehl-Nelson:
A. leprosy*
B. Leishmania
C. Treponema pallidum
D. the tubercle Bacillus *
2596. Specify a characteristic appearance and location of lesions pink lichen:
A. hyperemic spot*
B. vesicle
C. blister
D. on the trunk*
2597. Specify clinic if toxicodermia:
A. syndrome of Cesar
B. anaphylactic shock *
C. Lyell's syndrome*
D. Icinga syndrome-Cushing's
2598. Specify drug used in psoriasis:
A. blumarin
B. dexamethasone *
C. ethambutol
D. lorinden*
2599. Specify the symptoms of lupus erythematosus
A. a symptom of a ladies ' heel*
B. Nikolsky sign
C. symptom Gorchakov-Ardi
D. symptom Besnier –Meshchersky *
2600. After any primary cells formed by erosion:
A. bubble*
B. abscess *
C. spot
D. hump
2601. What phenomena are included in the psoriatic triad:
A. the phenomenon of a stearin stain*
B. the phenomenon of Apple jelly
C. the phenomenon terminalno film*
D. phenomenon and Lyell's syndrome
2602. Streptococcal pyoderma is differentiated with what disease:
A. eritrazma
B. bullous herpes *
C. herpes simplex*
D. CPL
2603. Deep dermatophytosis what differentiates diseases:
A. microsporia *
B. eritrazma
C. eczema
D. carbuncle*
2604. When the disease is not observed host:
A. herpes*
B. leprosy
C. lichen planus*
D. leishmaniasis
2605. In the dermis layers are distinguished:
A. Horny
B. net*
C. papillary*
D. granular
2606. By atypical solid ancram include:
A. indurative edema*
B. conflicts
C. balanoposthitis
D. chancre-amygdala*
2607. Specify the incorrect statement: the primary syphiloma can be complicated.
A. epididymitis*
B. orchitis*
C. paraphimosis
D. gangrenization
2608. Specify the incorrect statement: possible causes simple dermatitis.
A. oral administration of medications*
B. high temperature
C. low temperature
D. parenteral introduction in case the administration of medicines*
2609. For localized scleroderma unusual stage:
A. pustular
B. nepolitano hyperkeratosis *
C. follicular hyperkeratosis*
D. induration
2610. In the treatment of candidiasis is not used:
A. polkortolon*
B. nystatin
C. hydrocortisone*
D. flunil
2611. In the treatment of head lice is not used:
A. spregal
B. cycloferon*
C. 10C.20% emulsion of benzyl benzoate
D. mikozolon*
2612. Psoriatic erythroderma can develop after:
A. appointment ACTH*
B. application of ultraviolet irradiation to patients with the summer type of psoriasis
C. topical application of 5% ointment of salicylic acid
D. external use only 1% ointment salicylic acid *
2613. Add the secondary elements resulting from the transformation of the burl:
A. scale*
B. secondary pigmentation*
C. cork
D. ulcer
2614. Specify the clinical signs, characteristic of tertiary syphilis:
A. extensive warts *
B. the nodes
C. masorti
D. roseola *
2615. What are the main clinical signs are not characteristic of chronic eczema:
A. licensure
B. edema, microerosion *
C. infiltration
D. bright redness*
2616. In the stage of vesicles for topical therapy eczema apply only:
A. lotions*
B. creams
C. aerosols*
D. paste
2617. Specify the symptom is not characteristic for discoid forms of lupus erythematosus:
A. Lichenification*
B. Symptom of "Apple jelly" *
C. the Symptom of "butterfly".
D. Symptom Besnier-Meshchersky.
2618. What disease is among the group of neurodermatoses:
A. pruritus
B. allergicheskie dermatitis*
C. eczema*
D. urticaria
2619. What is unusual in the clinical picture of rosacea:
A. follicular hyperkeratosis*
B. erythema
C. follicular papules *
D. infiltration of the skin of the nose
2620. Specify the most characteristic symptom of scabies:
A. serous well
B. the paired arrangement of the elements *
C. evening and night itching*
D. impetiginized in the lesions
2621. Enter among the following surface form stafilodermii:
A. vulgaris sycosis*
B. carbuncle
C. furuncle
D. osteo folliculitis *
2623. In the therapy of lichen planus may apply:
A. diphenhydramine*
B. novopassit*
C. prednisolone
D. androgens
2624. In the primary stage of syphilis patients may meet all the characteristics of chromes.
A. lymphangitis
B. non-erosive papules *
C. negative Wassermann reaction
D. erosive papules*
2625. Select the most effective agent for the treatment of chlamydial urethritis:
A. tetracycline*
B. penicillin
C. doxycyclin*
D. levamisole
2626. One of the clinical signs are not typical for AIDS patients:
A. vitiligo*
B. recurrent herpes
C. lymphoma
D. scabies*
2627. Specify drug, used in pemphigus vulgaris:
A. prednisolone*
B. dexamethasone*
C. monomitsin
D. Orungal
2628. Specify drug used in the treatment of scabies:
A. 20% benzyl benzoate *
B. Vishnevsky ointment
C. the aerosols of Spregal*
D. 10% syntomycin ointment
2629. Specify the medication used for candidiasis of the skin:
A. Orungal*
B. flonal *
Furazolidone C.
D. bactrim
2630. Specify drugs used in the progressive stage of psoriasis:
A. psoriasin ointment
B. 2% salicylic ointment*
C. cream unna*
D. ointment dermatol
2631. Specify the phenomenon are characteristic of lichen planus:
A. Cabrera*
B. grid Wickham *
C. fish caviar
D. tissue paper
2632. Specify the symptoms characteristic of multi-colored lichen:
A. peeling*
B. Kerner
C. sample Wickham
D. sample Signs *
2633. Specify a drug that cannot be set under a persistent white dermographism:
A. calcium chloride*
B. pirogenal
C. sodium thiosulfate
D. calcium gluconate *
2634. Specify the drug used in herpes zoster:
A. delagil
B. oxoline *
C. bonafton*
D. Dapsone
2635. Specify the clinical symptoms characteristic of sycosis:
A. rheumatic pain
B. localization on the skin of the Mons pubis *
C. chronically relapsing course *
D. nodules
2636 . Clinical signs of herpes simplex is not:
A. lichen sclerosis*
B. eczema herpeticum
C. cicatricial atrophy*
D. meningoencephalitis
2637. Which of diseases is not a manifestation of HIV infection:
A. diarrhea
B. scabies *
C. Kaposi's sarcoma
D. vitiligo *
2638. That is not typical for psoriasis:
A. you can call animals in a laboratory*
B. may first appear after birth
C. not due to seasonality *
D. due to seasonality
2639. Specify the characteristic symptom blister:
A. occurs due to the intra-cellular edema *
B. peeling on the surface of the blister
C. leave behind after erosion
D. quickly appears and disappears *
2640. What clinical manifestations are characteristic of secondary recurrent syphilis:
A. indurative edema
B. alopecia *
C. parenchymatous keratitis
D. leukoderma *
2641. What clinical symptoms are not in patients with primary syphilis:
A. indurative edema
B. extensive warts *
C. Palmar-plantar papules *
D. chancre-panaritium
2642 . Specify the symptoms characteristic of uncomplicated chancre:
A. signs of inflammation
B. saped edge
C. painless ulcer *
D. dense infiltration at the base of the ulcer*
2643. For patients with chancroid is not typical:
A. signs of inflammation
B. inclined to autoinoculation
C. the absence of pain*
D. solid infiltration *
2644. In the treatment of gardnerellosis used:
A. tinidazole *
B. penicillin
C. metronidazole *
D. erythromycin
2645. For the diagnosis of some diseases is applied research using
wood's lamp:
A. microsporia*
B. eritrazma *
C. pityriasis rosea.
D. vitiligo.
2646. What primary morphological element precedes erosion:
A. none of the listed
B. tubercle
C. gnojnice *
D. bubble *
2647. What primary morphological element precedes the ulcer:
A. bubble
B. blister
C. the node*
D. bump *
2648. Specify the symptoms characteristic of uncomplicated chancre:
A. dense infiltration at the base of the ulcer *
B. purulent discharge
C. saped edge
D. painless ulcer *
2649. What pustular disease of the skin does not occur in infants:
A. vesiculopustules
B. sycosis *
C. hydradenitis *
D. pseudoharengus finger
2650. Specify the types of seborrhea:
A. dry *
B. vulgar
C. leaf
D. bold *
2651. What clinical symptom seen with lupus erythematosus:
A. symptom of Pincus
B. Symptom Besnier-Meshchersky *
C. "blood dew"
D. "ladies ' heel" *
2652. Name the atypical form of psoriasis:
A. justiceability /rupiny/ *
B. coin
C. scalp
D. intertriginous *
2653. What is the most severe form of psoriasis:
A. geographical
B. nummularnyi
C. erythrodermic *
D. artropaticheskom *
2654. Name the localization of elements at pustular psoriasis of the Barber:
A. red border of the lips
B. the sole *
C. palm*
D. the skin of the genital area
2655. Name the type of pustular psoriasis :
A. psoriasis Combucha*
B. psoriasis of the Barber*
C. lichenoid
D. nummularnyi
2656. What clinical sign is characteristic of dermatitis Duhring:
A. sclerodactyly
B. blood eosinophilia *
C. microfocal pattern baldness
D. eosinophilia content bubble *
2657. What are clinical types of cutaneous leishmaniasis You know:
A. nedifferentsirovannost type
B. late - ulcerated type *
C. acute - nitrotyrosine type*
D. erythematous-samotny type
2658. Name the clinical varieties of the disease Borovsky:
A. papistry
B. acute nitrotyrosine type*
C.undifferentiated
D. city type *
2659. Which layer is not part of the epidermis:
A. granular
B. papillary *
C. net*
D. prickly
2660. Specify the types of seborrhea:
A. dry *
B. vegetative
C. leaf
D. bold *
2661. What diseases belong to the group of stafilodermy:
A. slit-like vulgar ecthyma impetigo
B. epidemic pemphigus*
C. impetigo nail ridges
D. impetigo of Bockhart*
2662. What tools have antipruritic property:
A. menthol*
B. salicylic acid
C. tar
D. anestezin *
2663. What pyoderma occur only in newborns and infants:
A. hydradenitis
B. the boil
C. vesiculopustules*
D. exfoliative dermatitis *
2664. What are morphological elements typical of Ritter's dermatitis:
A. tubercles
B. erosion *
C. papules
D. bubbles*
2665. Localization of the rash in scabies:
A. interdigital spaces of the hands*
B. interdigital spaces stop
C. the skin around the navel, inner thighs*
D. places of a congestion of the sebaceous glands
2666. What are secondary morphological element:
A. vegetation*
B. abscess
C. the knot
D. ulcer *
2667. Name the proliferative morphological element:
A. abscess
B. the node *
C. burl*
D. bubble
2668. Acantholysis is the morphological basis of the symptom:
Nicholas A.*
B. Wickham;
C. Pospelova
D. Asbestos-Hansen *
2669. What clinical sign is characteristic of dermatitis Duhring:
A. leukopenia
B. cryoglobulinemia
C. eosinophilia in the blood *
D. eosinophilia content bubble *
2670. What are the tests uses for the diagnosis of dermatitis herpetiformis Duhring:
A. sample 2C.3% solution of potassium iodide *
B. 50% ointment with potassium iodide *
C. 3C.x test Thompson
D. iodine test Signs
2671. What medicines are most effective when dermatitis Duhring:
A. DDS *
B. Biseptol
C. corticosteroids *
D. vitamins In
2672. Specify the features of papules in lichen planus:
A. umbilicate indentation*
B. edge peeling
C. rich peeling
D. purple tint *
2673. Name the type of pustular psoriasis:
A. psoriasis Kaposi
B. psoriasis of the Barber*
C. psoriasis Combucha*
D. lichenoid
2674. What skin areas are usually free of lesions at the LCP:
A. radiocarpal joints
B. the sole *
C. flexion of the legs
D. hands*
2675. What dermatosis should be differentiated lichen planus:
A. artifactual dermatitis
B. psoriasis*
C. secondary syphilis*
D. pityriasis versicolor
2676. Name the localization of elements at pustular psoriasis of the Barber:
A. palm*
B. the red border of the lips
C. scalp
D. soles *
2677. Specify the true types of eczema:
A. numulyarnaya
B. disgidroticheskaya *
C. prodigiosa *
D. seborrheic
2678. When mnogoformnuû rash the exudative erythema is:
A. in the form of arcs*
B. in pairs
C. grouped the course of nerves
D. in the form of rings *
2679.Morphological elements typical of the clinic of uncomplicated scabies:
A. bubble
B. spot
C. the vial*
D. nodule*.
2680.The causative agents of pustular diseases of the skin:
A. Streptococcus*
B. E. coli
C. Staphylococcus aureus*
D. the tubercle Bacillus
2681.The localization of furuncle:
A. the neck *
B. lips
C. mucosa of the oral cavity
D. the region of the back*
2682. Hydradenitis is localized:
A. the palm
B. peripapillary area*
C. the underarm*
D. Shin
2683. Clinical varieties of streptococcal impetigo:
A. deep folliculitis
B. sycosis
C. impetigo slit*
D. bullous impetigo*
2684 . Clinical signs of herpes simplex is not:
A. eczema herpeticum
B. cicatricial atrophy*
C. lichen sclerosus*
D. meningoencephalitis
2685. Which of diseases is not a manifestation of HIV infection:
A. Kaposi's sarcoma
B. scabies*
C. vitiligo*
C. diarrhea
2686. That is not typical for psoriasis:
A. may first appear after birth
B. not due to seasonality*
C. you can call animals in a laboratory*
D. due to seasonality
2687. Specify the characteristic symptom blister:
A. scaling on the surface of the blister
B. quickly appears and disappears*
C. leave behind after erosion
D. occurs at the expense of intra-cellular edema*
2688. What clinical manifestations are characteristic of secondary recurrent syphilis:
A. parenchymatous keratitis
B. alopecia*
C. leukoderma*
D. indurative edema
2689. What clinical symptoms are not in patients with primary syphilis:
A. Palmar-plantar papules*
B. indurative edema
C. extensive warts*
D. chancre-panaritium
2690 . Specify the symptoms characteristic of uncomplicated chancre:
A. dense infiltration at the base of the ulcer*
B. the signs of inflammation
C. the absence of treponemes in the discharge of ulcers
D. painless ulcer*
2691. For patients with chancroid is not typical:
A. the absence of pain*
B. solid infiltration*
C. signs of inflammation
D. the lesions are formed after 2-3 days on-site implementation of streptobacilli
2692. In the treatment of gardnerellosis used:
A. erythromycin
B. metronidazole*
C. penicillin
D. tinidazole*
2693. For the diagnosis of some diseases is applied research using
wood's lamp:
A. pityriasis rosea
B. eritrazma*
C. microspores*
D. vitiligo
2694. What primary morphological element precedes erosion:
A. tubercle
B. bubble*
C. none of the listed
D. gnojnice*
2695. What primary morphological element precedes the ulcer:
A. bubble
B. blister
C. the node*
D. bump*
2696. Specify the symptoms characteristic of uncomplicated chancre:
A. dense infiltration at the base of the ulcer*
B. painless ulcer*
C. saped edge
D. purulent discharge
2697. What pustular disease of the skin does not occur in infants:
A. vesiculopustules
B. sycosis*
C. hydradenitis*
D. pseudoharengus finger
2698. Specify the types of seborrhea:
A. vulgar
B. bold*
C. leaf
D. dry*
2699. What clinical symptom seen with lupus erythematosus:
A. "lady's thumb"*
B. a symptom of the Pincus
C. "blood dew"
D. symptom Besnier-Meshchersky*
2700. Name the atypical form of psoriasis:
A. coin
B. intertriginous*
C. scalp
D. justiceability /rupiny/*
2701. What is the most severe form of psoriasis:
A. artropaticheskom*
B. nummularnyi
C. erythrodermic*
D. geographical
2702. Name the localization of elements at pustular psoriasis of the Barber:
A. red border of the lips
B. the sole*
C. palm*
D. the skin of the genital area
2703. Name the type of pustular psoriasis :
A. psoriasis Combucha*
B. lichenoid
C. psoriasis of the Barber*
D. nummularnyi
2704. What clinical sign is characteristic of dermatitis Duhring:
A. sclerodactyly
B. microfocal pattern baldness
C. blood eosinophilia*
D. eosinophilia content bubble*
2705. What are clinical types of cutaneous leishmaniasis You know:
A. acute nitrotyrosine type*
B. lepromatous type
C. late-ulcerated type*
D. erythematous-samotny type
2706. Name the clinical varieties of the disease Borovsky:
A. city type*
B. papular
C. acute-nitrotyrosine type*
D. infiltrative-suppurative
2707. Which layer is not part of the epidermis:
A. net*
B. prickly
C. granular
D. papillary*
2708. Specify the types of seborrhea:
A. leaf
B. vegetative
C. dry*
D. bold*
2709. What diseases belong to the group of stafilodermy:
A. impetigo of Bockhart*
B. epidemic pemphigus*
C. impetigo nail ridges
D. slit-like vulgar ecthyma impetigo
2710. What tools have antipruritic property:
A. tar
B. benzocaine*
C. menthol *
D. salicylic acid
2711. What pyoderma occur only in newborns and infants:
A. hydradenitis
B. exfoliative dermatitis*
C. vesiculopustules*
D. furuncle
2712. What are morphological elements typical of Ritter's dermatitis:
A. bubbles*
B. papules
C. erosion*
D. bumps
2713. Localization of the rash in scabies:
A. interdigital spaces of the hands *
B. places of a congestion of the sebaceous glands
C. interdigital spaces stop
D. the skin around the navel, inner thighs *
2714. What are secondary morphological element:
A. the knot
B. ulcer*
C. vegetation*
D. abscess
2715. Name the proliferative morphological element:
A. bubble
B. abscess
C. the node *
D. nodule*
2716. Acantholysis is the morphological basis of the symptom:
Nicholas A.*
B. Asbestos-Hansen*
C. Pospelova
D. Wickham
2717. What clinical sign is characteristic of dermatitis Duhring:
A. eosinophilia of content bubble*
B. eosinophilia in the blood*
C. cryoglobulinemia
D. leukopenia
2718. What are the tests uses for the diagnosis of dermatitis herpetiformis Duhring:
A. the sample with 2-3% solution potassium iodide*
B. 50% ointment with potassium iodide*
C. 3 test Thompson
D. iodine test Signs
2719. What medicines are most effective when dermatitis Duhring:
A. DDS*
B. Biseptol
C. corticosteroids*
D. vitamins In
2720. Specify the features of papules in lichen planus:
A. umbilicate indentation*
B. edge peeling
C. rich peeling
D. purple tint*
2721. Name the type of pustular psoriasis:
A. psoriasis Combucha*
B. psoriasis Kaposi
C. psoriasis of the Barber*
D. lichenoid
2722. What skin areas are usually free of lesions at the LCP:
A. flexor surfaces of the extremities
B. the sole*
C. palm*
D. wrist joints
2723. What dermatosis should be differentiated lichen planus:
A. secondary syphilis*
B. pityriasis versicolor
C. artifactual dermatitis
D. psoriasis*
2724. Name the localization of elements at pustular psoriasis of the Barber:
A. the scalp
B. the sole*
C. palm*
D. the red border of the lips
2725. Specify the true types of eczema:
A. prodigiosa*
B. seborrheic
C. numulyarnaya
D. disgidroticheskaya*
2726. When mnogoformnuû rash the exudative erythema is:
A. in pairs
B. in the form of rings*
C. grouped the course of nerves
D. in the form of arcs*
2727. Complications arising in patients with the localization of furuncle on the face:
A. phlebitis of the vessels of the brain*
B. sepsis*
C. neuritis of the facial nerve
D. endocarditis
2728. Deeper forms of streptodermii:
A. periungual conflicts
B. streptococcal impetigo
C. drill ecthyma*
D. vulgar ecthyma*
2729. The factors that cause toksikodermiya:
A. acid
B. antibiotics*
C. sulfanilamidnye drugs *
D. alkali
2730. Clinical signs of urticaria:
A. pruritus *
B. node
C. blister*
D. pain
2731.For allergic dermatitis is characterized by:
A. erythema *
B. the nodes
C. bumps
D. bubbles*
2732. Symptoms typical of true eczema:
A. blisters
B. microvesicles*
C. true polymorphism*
D. cicatricial atrophy
2733. Clinical manifestations fresh secondary period of syphilis:
A. blisters
B. roseola*
C. chancre or its remains *
D. bubbles
2734. The source of the infection by Microsporum:
A. rodents
B. people*
C. cattle
D. cats, dogs*
2735. Primary cells for herpes simplex:
A. blister
B. bubble*
C. erythema*
D. abscess
2736. The rash is polymorphic if:
A. psoriasis
B. eczema *
C. secondary syphilis*
D. lichen planus
2737. Clinical types of lichen planus:
A. hypertrophic*
B. atrophic*
C. pemphigidae
D. artropaticheskom
2738. The most common form of psoriasis with the flow:
A. spring
B. autumn
C. summer*
D. winter*
2739. Psoriasis must be differentiated from:
A. lichen planus *
B. secondary syphilis*
C. pemphigus
D. eczema
2740. Drugs used for the treatment of scleroderma:
A. erythromycin
B. lidaza*
C. penicillin*
D. nicotinic acid
2741. Connective tissue disease:
A. lupus erythematosus*
B. dermatomyositis*
C. psoriasis
D. allergic vasculitis
2742. Clinical types of lupus erythematosus:
A. seborrheic
B. disseminated*
C. discoid*
D. annular
2743. For tinea versicolor is characterized by:
A. peeling*
B. symptom Besnier - Meshchersky
C. sample Signs*
D. svechenie with green luminescence
2744. Name the type of pustules:
A. telangiectasia
B. scratch
C. conflicts*
D. folliculitis*
2745. Name the type of stains:
A. roseola*
B. conflicts
C. erythema*
D. rupee
2746. Clinical types of lichen planus:
A. pemphigidae
B. atrophic*
C. hypertrophic*
D. artropaticheskom
2747. The most common form of psoriasis with the flow:
A. off-season
B. winter*
C. summer*
D. spring
C. autumn
C. General
2748. Psoriasis must be differentiated from:
A. lichen planus *
B. secondary syphilis*
C. paasiaisen deprive
D. neurodermatitis
2749. Drugs used for the treatment of scleroderma:
A. penicillin*
B. lidaza*
C. biyohinol
D. delagil
2750. Connective tissue disease:
A. lupus erythematosus*
B. dermatomyositis*
C. lichen planus
D. strophulus
With 3 correct answers
2751. Specify drugs used in pemphigus vulgaris:
A. prednisolone *
B. triamcinolon*
C. monomitsin
D. sulfadimetoksin
E. Dapsone
F. dexamethasone*
2752. What drugs is used to treat
scabies:
A. 20 % benzyl –benzoate ointment*
B. spray Spregal *
C. hydrocream
D. 5 % naphthalan ointment
E. 5% sulfur ointment
F. 33% sulfuric ointment*
2753. With what diseases should be differentiated scabies:
A. atopic dermatitis *
B. pityriasis versicolor
C. pruritus*
D. pityriasis rosea
E. herpes zoster
F. pruritus*
2754. Name the obligate factor of dermatitis:
A. concentrated acid *
B. jewelry of gold
C. concentrated alkali*
D. boiling water*
E. synthetic detergents
F. jewelry made of platinum
2755. Allergic reaction to the medication may occur:
A. bullous rash*
B. vesicular*
C. erythema*
D. symptom Pylnova
E. papular rash
F. there is no right answer
2756 . Specify the kind of toxicodermia:
A. fixed erythema*
B. iododerma*
C. psoriasis
D. bromoderma*
E. perniosis
F. pellagrozny dermatitis
2757 . What medications are more likely to cause fixed erythema:
A. sulfadimetoksin*
B. Biseptol*
C. norsulfazol*
D. analgin
E. aspirin
F. tavegil
2758. What disease should be differentiated common toksikodermiya:
A. pityriasis rosea Gibert *
B. the syndrome of Stevens-Johnson*
C. acne rosacea
D. pyoderma
E. exfoliative dermatitis*
F. disease of Borovsk
2759. Indicate the typical localization at the mnogoformnuû exudative erythema:
A. the back of the hand*
B. the rear brake *
C. mucous membranes*
D. scalp
E. the interdigital spaces of the hands
F. the side surfaces of the extremities
2760. What factors are important in the development of mnogoformnuû of erythema multiforme:
A. seasonality*
B. hereditary
C. chronic infection*
D. hypersensitivity to fluoride
E. hypothermia*
F. hypersensitivity to iodine
2761. What differentiate exudative dermatoses mnogoformnuû erythemal:
A. pemphigus vulgaris *
B. scrofuloderma
C. exfoliative dermatitis*
D. strophulus
E. Lyell's syndrome*
F. erythema induratum of Bazin
2762. Specify pathogenetic factors of pink lichen Gibert:
A. viral infection*
B. hyperfunction of the sebaceous
C. hypofunction of the sebaceous glands
D. yeast
E. vaccination*
F. streptococcal infection* glands
2763. Specify the variety of microbial eczema:
A. seksitarina*
B. Horny
C. sysformat*
D. adult
E. idiopathic
F. varicose*
2764. What clinical sign is characterized by a rash with true eczema:
A. true polymorphism*
B. symmetry*
In assimetria
D. ephemeral blisters
E. . the symptom of "serous wells"*
F. indurative edema
2765. What are the symptoms seen with discoid lupus:
A. symptom Khachaturian*
B. the symptom of "lady heel"*
C. collar Voronova
D. the collar of Byetta
E. symptom Besnier-Meshchersky *
F. mesh Witham
2766. Specify types of scleroderma:
A. stripy*
B. intertriginous
C. surface*
D. infiltrative
E. white spot disease*
F. vulgar
2767. Lichen planus is characterized by:
A. papular rash*
B. umbilicated*
C. predominant localization on the flexor surfaces of the extremities*
D. the absence of subjective feelings
E. urticaria rash
F. tubercles
2768. For lichen planus is characterized by:
A. irregular thickening of the granular layer of the epidermis*
B. acanthosis*
C. parakeratosis
D. moderate hyperkeratosis*
E. vnutriarterialno microabcesses
F. disorganization of collagen fibers
2769. What clinical symptom is pathognomonic for lichen planus:
A. grid Wickham*
B. umbilicate indentation*
C. purple colour*
D. symptom of Auspice
E. cells Tzenka
F. all answers are not correct
2770. What diseases does not belong to the group of viral dermatoses:
A. extensive warts*
B. simple bubble herpes.
C. lichen planus*
D. psoriasis*
E. genital warts
F. all answers are not correct
2771. Vascular spot is:
A. Erythema*
B. Leukoderma
C. Teleangioektasia*
D. Lentigo
E. Roseola*
F. Naevus
2772. Primary morphological elements:
A. bubble*
B. ulcer
C. bump*
D. bubble*
E. erosion
F. all answers are not correct
2773. Secondary morphological elements:
A. Ulcer*
B. Voldyri
C. Cork*
D. Erosion*
E. Burl
F. all answers are not correct
2774. The bubble is formed by:
A. Eczema*
B. the LCP
C. herpes zoster*
D. Psoriasis
E. herpes simplex*
F. all the answers are not correct
2775. The outcome of the nodule is:
A. hypopigmented spot*
B. Scale*
C. vanish*
D. Ulcer
E. Atrophy
F. all answers are not correct
2776. Exudative morphological element is:
A. Vesicle*
B. Tubercle
C. Abscess*
D. Papule
E. The Blister*
F. all answers are not correct
2777. Layers of epidermis:
A. basal *
B. thorn*
C. granular*
D. fat
E. . hypodermally
F. all answers are not correct
2778. Have a cavity morphological elements:
A. abscess*
B. blister
C. bubble*
D. node
E. vial*
F. all answers are not correct
2779. The rash is polymorphic if:
A. secondary syphilis*
B. psoriasis
C. dermatitis Duhring*
D. warts
E. eczema*
F. all answers are not correct
2780. Monomorphic rash in medicine:
A. herpes zoster
B. the LCP*
C. urticaria*
D. dermatitis Duhring
E. psoriasis*
F. all answers are not correct
2781. Clinical signs of psoriasis:
A. the presence of papules*
B. peeling*
C. koebner's phenomenon*
D. blisters
E. nodes
F. all answers are not correct
2782. Clinical signs of lichen planus:
A. koebner's phenomenon*
B. polygonal papules*
C. pupkoobraznym depressions in the center*
D. blisters
D bumps
The pustules
2783. Agents used topically in the treatment of psoriasis:
A. gel Skin cap*
B. corticosteroid ointment*
C. salicylic ointment
D. Castellani paint
E. Diprosalic ointment*
F. all answers are not correct
2784 Phenomena typical for psoriasis:
A. stearin spot*
B. terminal film*
C. blood dew*
D. matching items
E. Wickham
F. ladies ' heel
2785. The signs of true polymorphism of rash for dermatitis Duhring:
A. bullas*
B. uslc*
C. spot*
D. hump
E. node
F. lichenification
2786. Clinical varieties of pemphigus:
A. vulgar*
B. exudative
C. leaf*
D. psoriasiform
E. vegetative*
F. papular
2787. For lupus erythematosus symptoms:
A. follicular hyperkeratosis*
B. cicatricial atrophy*
C. erythema*
D. grid Wickham
E. mukovidnoe peeling
F. the symptom of Cabrera
2788. Clinical forms of athlete's foot:
A. intertriginous*
B. psoriasiform
C. squamous*
D. disgidroticheskaya*
E. worn
F. pustular
2789. Diseases related to dermatophytosis:
A. athlete's groin*
B. hromomikoza
C. dermatophytosis*
D. eritrazma
E. microspores*
F. impetigo
2790 Antibiotics used in the treatment of candidiasis:
A. nystain*
B. flonal*
C. levorin*
D. cephalosporins
E. doxycyclin
F. kanamycin
2791. When artropaticheskom appropriate will appoint all of the above, exceptA. nizoral*
B. mineralocorticoid drugs
C. synthetic anti-malarial drugs*
D. antioxidants
E. penicillin*
F. nonsteroidal anti-inflammatory drugs
2792. Typical eruptive lichen planus elements have the following features, exceptA. preferential localization on the face*
B. symptom Pylnova*
C. pustules flat, polygonal reddish-crimson color, is a symptom of Asbestos-Hansen*
D. waxy Shine
E. Wickham mesh on the surface of papules
F. flat papules, polygonal, reddish-purple color
2793. Monomorphic rash can be:
A. psoriasis*
B. microbial eczema
C. warts *
D. erythema multiforme
E. lichen planus*
F. dermatitis Duhring
2794. Hyperkeratosis is characteristic of:
A. psoriasis*
B. ichthyosis*
C. warts *
D. bullous impetigo
E. sycosis
F. all answers are not correct
2795. Woman, 25 years old, married 3 годD. When the survey was diagnosed with chronic
endocervicitis, adnexitis. Smears D. leukocytosis, gonococcus and Trichomonas not found. My
daughter 1.5 years of acute gonorrheal vulvovaginitis etiologies. Correct doctor's tactics:
A. use of a combination of provocation with subsequent bacteriological and bacterioscopic study*
B. the appointment of antibacterial drugs according to the sensitivity of the bacterial
flora to antibiotics*
C. the treatment scheme of chronic gonorrhea*
D. prescription of corticosteroids
E. appointment of antihistamines
F. antifungal drugs
2796. What type of topical treatment should
to choose with weeping skin inflammations:
A.gadgets*
B. cream
C. aniline dyes*
D. Prater Park
E. aerosols*
F. all answers are not correct
2797. The patient was diagnosed as anterior urethritis chlamydial etiology. What antibiotic group
to prescribe to the patient:
A. aminoglycosides*
B. fluorinated quinolones
C. macrolides*
D. antimalarial
E. tetracycline*
F. cephalosporins
2798. Girls vulvovaginitis Trichomonas etiology characterized by all of these
symptoms except-
A. frothy discharge from the genital slit*
B. frequent and painful urination
C. atrophy of the external genitalia*
D. infiltrative lesions
E. no itching*
F. itch in the vulva
2799. What kind of lotions should choose to oozing of the infected skin surface:
A. ivanyevo*
B. tannin
C. with potassium permanganate*
D. lead
E. furatsilinovoy*
F. boric acid
2800. Diagnostic tests of allergic dermatitis are samples.
A. drip*
B. Adamson
C. prick*
D. Baltser
E. compression *
F. all answers are not correct
2801. To frequent complications of atopic dermatitis include all of these except
A. membrane-destructive processes in the epithelium of the urinary organs during exacerbation
atopic dermatitis*
B. urethritis*
C. hyperthyroidism *
D. poor sleep, irritability
E. polyadenylation of
F. painful and persistent itching
2802. Diagnosis of acute prostatitis, based on the results of the above studies, exceptA. General analysis of blood*
B. ultrasound of the prostate
C. examination of the skin*
D. palpation of the prostate
E. biochemical analysis of blood*
F. all answers are not correct
2803. Examination of children with atopic dermatitis with the aim to understand all of the above,
except
A. identifying the source of microbial sensitization*
B. identify indicatorii
C. identification of pustules in the affected skin lesions*
D. identify the States of immunodeficiency
E. identification of sepsis in affected skin lesions*
F. identify other allergenic factors
2804. What exogenous factors contribute to the development of pyoderma:
A. microfracture *
B. contamination of the skin*
C. hypothermia and hyperthermia*
D. disorders of keratinization
E. helminthic invasion
F. all answers are not correct
2805. What endogenous factors contribute to the development of pyoderma:
A. diabetes mellitus*
Hemophilia B.
C. intestinal toxemia*
D. eosinophilia
E. hypovitaminosis*
F. violations of the mechanism of division of epidermocytes
2806. In acute and subacute complicated gonorrhea in men and women in the complex treatment
included all of the above, exceptA. antihistamines*
B. gonovaccine*
C. the desensitization*
D. protivolodochnyi antibiotics
E. antispasmodics
F. sulfonamides
2807. Specify the correct tactics of treatment of torpid and chronic forms of gonorrheal infection
A. immunotherapeutic drugs*
B. detoxification drugs
C. antibacterial*
D. calcium supplements
E. local treatment*
F. all answers are not correct
2808. To stafilodermy include:
A. impetigo of Bockhart*
B. slit-like impetigo
C. carbuncle*
D. simple versicolor
E. deep folliculitis*
F. ecthyma vulgar
2809. Clinical symptoms of discoid lupus erythematosus all but
A. bullas *
B. tightly sitting scales
C. blister *
D. erythema
E. bubbles *
F. atrophy
2810. Koebner's phenomenon is observed in the following diseases:
A. lichen planus*
B. folliculitis
C. vitiligo*
D. leprosy
E. psoriasis*
F. tinea versicolor
2811. If any element of dermatitis rash is the bubble:
A. eczema*
B. herpes simplex*
C. tinea*
D. pityriasis rosea Gibert
E. pemphigus
F. all answers are not correct
2812. True polymorphism is typical for the following diseases except:
A. eczema
B. dermatitis Duhring
C. mnogoformnuû of erythema multiforme
D. psoriasis*
E. lichen planus*
F. tinea versicolor *
2813. Find relevant answers:
Proliferative morphological changes
A. granules, *
B. acantholysis,
C. acanthosis, *
D. sponges,
E.hyperkeratosis,*
F. all answers are not correct
2814. Diffuse hyperkeratosis is not typical for:
A. ichthyosis
B. keratodermia
C. pemphigus*
D. pyoderma*
E. eczema*
F. all answers are not correct
2815. What factors contribute to the development of pyoderma:
A. disorders of carbohydrate metabolism*
B. the presence of a large number of pathogenic strains*
C. reduction of the barrier functions of the skin *
D. increased blood clotting
E. hypervitaminosis
F. all answers are correct
2816. For scabies is characterized by:
A. paired papulo-vesicular elements*
B. itching at night*
C. symptom Gorchakov-Ardi *
D. comedones
E. the symptom of "Apple jelly"
F. the symptom of "lady heel"
2817. From the ectoderm develop:
A. hair*
B. the vessels of the skin
C. sweat glands*
D. hair muscles
E. sebaceous glands*
F. all answers are not correct
2818. To has is characterized by:
A. fever, malaise, headache *
B. estimatese pustules
C. favourite localization in the armpits, genitals *
D. blister
E. the formation of fistulous*
F. all answers are not correct
2819. Specify the clinical form of scabies (a form):
A. worn*
B. squamous
Norwegian C.*
D. erythematous
E. clean*
F. all the answers are not correct
2820. To a mixed pyoderma include:
A. brimicombe*
B. chancriform pyoderma *
C. ulcerative-vegetative pyoderma *
D. hydradenitis
E. osteo folliculitis
F. all answers are not correct
2821. Indicate the clinical varieties of pemphigus acantholytic:
A. leaf*
B. vulgar*
C. vegetative*
D. annular
E. syndrome Stevens-Jones
F. epidemic pemphigus newborns
2822. Dermatosis at what primary morphological element is a node:
A. tuberculosis of the skin*
B. genital warts
C. tertiary syphilis *
D. psoriasis
D leishmaniasis of the skin*.
F. angioedema
2823. Under what medicine the main element of the rash is pimple:
A. molluscum contagiosum*
B. warts *
C. psoriasis*
D. lichen planus
E. pityriasis rosea Gibert
F. pemphigus
2824. Clinical symptoms of discoid lupus erythematosus:
A. erythema*
B. blister
C. atrophy*
D. bubbles
E. tightly sitting scales*
F. abscesses
2825. Koebner's phenomenon is not observed in the following diseases:
A. lichen planus
B. psoriasis
C. vitiligo
D. leprosy *
E. folliculitis*
F. pityriasis versicolor*
2826. Under what medicine the main element of the rash is not pimple:
A. molluscum contagiosum
B. herpes zoster*
C. psoriasis
D. lichen planus
E. pityriasis rosea Gibert*
F. pemphigus*
2827. If any element of dermatosis rash is not a bubble:
A. herpes simplex
B. herpes zoster
C. genital herpes
D. pityriasis rosea Gibert *
E. pemphigus *
F. erythematous*
2828. Dermatosis at what primary morphological element is not a node:
A. tertiary syphilis
B. leprosy
C. psoriasis*
D. genital warts*
E. angioedema *
F. all answers are not correct
2829. True polymorphism is characteristic forA. eczema*
B. dermatitis Duhring*
C. mnogoformnuû exudative erythema*
D. psoriasis
E. lichen planus
F. tinea versicolor
2830. Name the atypical form of primary syphiloma:
A. indurative edema *
B. the chancre-amygdala*
C. chancre-panaritium*
D. diffuse pigmentation
E. fagedenizm
F. all answers are not correct
2831. Add the secondary items that are not the result of the transformation of the knot:
A. secondary spot
B. vanish
C. scales
D. erosion*
E. ulcer *
F. scar*
2832. Complication of hard chancre is:
A. pagedevice*
B. phimosis*
C. balanoposthitis*
D. impetigo
E. urethral stricture
F. all answers are not correct
2833. What drugs are used for photochemotherapy in psoriasis:
A. psoralen*
B. povalen*
C. ammifurin*
D. acyclovir
E. nystatin
F. lamisil
2834. What type of external therapy is prescribed in case of progressive stage of psoriasis:
A. corticosteroid ointment*
B. method of dem'yanovich
C. softening ointments*
D. ointment "Psoriasin"
E. 1-2% salicylic ointment*
F. all answers are not correct
2835. For secondary syphilis is characterized by:
A. pustular *
B. roseolous rash*
C. papular rash*
D. Gumma
E. chancre and polyadenylation
F. all answers are not correct
2836. Diagnosis of acute prostatitis, based on the results of the above studies:
A. General analysis of blood
B. biochemical analysis of blood
C. examination of the skin
D. palpation of the prostate*
E. ultrasound of the prostate*
F. ureteroscopy*
2837. What drug is effective for genital herpes:
A. Alkaryl *
B. Neovir*
C. Acyclovir*
D. Sandimmune
E. Diprospan
F. all answers are not correct
2838. The clinical picture disgidroticheskaya form of athlete's foot:
A. bullas*
B. peel
C. edema*
D. bumps
E. bubbles*
F. lichenification
2839. The clinical picture of infiltrative-nagnoitel'noj ringworm:
A. inflammatory infiltrate*
B. pustule*
C. hair loss*
D. lichenification
E. blisters
F. bullas
2840. Clinical symptoms Jock itch:
A. spot red*
B. the border is sharply limited*
C. Blisters*
D. nodes
E. hill
F. blisters
2841. The most frequent localization of superficial candidiasis of the skin:
A. folds of skin*
B. the corners of the mouth*
C. flexor surface of wrists*
D. vulva
E. hair
F. internal organs
2842. Fungal diseases of the skin are called:
A. epidermofiton*
B. Trichophyton *
C. Microsporum*
D. bacilli of Hansen
E. Mycobacterium Koch
F. all answers are not correct
2843. The main clinical signs of lesions of the mucous membranes with candidiasis:
A. erosive surface*
B. ulcers
C. papules of the mucous*
D. bumps
E. white films*
F. nodes
2844. the drugs used in the treatment of fungal infections:
A. griseofulvin*
B. Ceftriaxone
C. Fluconazol*
D. lindamycin
E. nizoral*
F. penicillin
2845 Localization of the rash in scabies in adults:
A. interdigital folds of the hands*
B. palm
C. the skin of the abdomen*
D. nails
E. the flexor surface of forearms*
F. soles
2846. The main clinical signs of scabies:
A. itching in the evening and at night*
B. the paired arrangement of morphological elements*
C. папулаD. vesicle*
D. nodes
E. acantholysis
F. blisters
2847. Funds are used for the treatment of scabies:
A. sodium thiosulfate 60% and hydrochloric acid 6%*
B. benzyl benzoate 20% emulsion*
C. prednisolone ointment
D. iodine tincture
E. sulfur ointment 33%*
F. nystatin cream
2848. Morphological elements characteristic of the clinic of uncomplicated scabies:
A. a knot*
B. bubble*
C. Spot*
D. hump
E. abscess
F. all answers are not correct
2849. Scabies is not characteristic symptom:
A. Gorchakov-Ardi
B. a symptom of the triangle
C. nocturnal itching
D. Nicholas *
E. Asbestos-Gentina*
F. Pylnova*
2850. The causative agents of pustular diseases of the skin:
A. Streptococcus *
B. Staphylococcus*
C. Proteus*
D. the tubercle Bacillus
E. Escherichia coli
F. Corynebacterium
2851. The Streptococcus causes the disease:
A. impetigo*
B. folliculitis
C. Zayed road*
D. sycosis
E. superficial panaritium*
F. shingles
2852. Varieties of streptococcal impetigo:
A. impetigo nail ridges *
B. Zayed road *
C. bullous impetigo*
D. furuncle
E. a simple bubble herpes
F. epidemic pemphigus
2853. Clinical signs specific for staphylococcal deep pustules:
A. is riddled with hair*
B. located on the smooth skin
C. pus thick*
D. pus liquid
E. hemispherical form*
F. surrounded by a halo of hyperemia
2854. Hydradenitis is localized:
A. armpits*
B. soles
C. palm*
D. Shin
E. peripapillary area*
F. scalp
2855. Main clinical forms of staphylococcal skin lesions:
A. folliculitis *
B. carbuncle*
C. parasitic sycosis*
D. pityriasis versicolor
E. herpes zoster
F. ecthyma vulgar
2856. Irritants cause irritant contact dermatitis:
A. mechanical*
B. drug parenteral introduction in case
C. low temperature*
D. alimentary
E. high temperature*
F. all answers are not correct
2857. Characteristic of eczema:
A. erythema, swelling*
B. oozing "serous wells"*
C. vesicles*
D. Gumma
E. roseola
F. tubercles
2858. To viral diseases include:
A. versicolor is a simple bubble*
B. vulgaris sycosis
C. molluscum contagiosum *
D. eczema
E. lupus erythematosus
F. herpes zoster *
2859. For herpes zoster is characterized by:
A. pain *
B. bubbles*
C. a lesion in the nerve endings*
D. nodes
E. pigmented
F. blisters
2860. What is the number of drugs not applicable in case of dermatitis Duhring
A. the number of Sulfonic
B. steroid hormones*
C. cytostatics*
D. eubiotics*
E. sulfon
F. all answers are not correct
2861. Clinical manifestations typical for professional eczema:
A. hyperemia*
B. vezikulyatsia *
C. weeping*
D. vegetation
E. tubercles
F. all answers are not correct
2862. The bubble allowed the formation of:
A. spot*
B. peel*
C. erosion *
D. ulcer
E. the scar
F. lichenification
2863. Monomorphic rash in medicine:
A. urticaria*
B. psoriasis *
C. lichen planus*
D. secondary syphilis
E. eczema
F. dermatitis Duhring
2864. What type of topical treatment is not prescribed for progressivnoj stage of psoriasis?
A. 2% salicylic ointment
B. 10% salicylic ointment*
C. 20% benzyl-benzoate*
D. cream unna
E. "ointment "psoriasin"*
F. all answers are not correct
2865. Clinical signs of psoriasis:
A. the presence of papules*
B. peeling*
C. koebner's phenomenon*
D. bumps
E. nodes
F. scars
2866. Main clinical forms of psoriasis:
A. vulgar *
B. subacute
C. pustular*
D. while
E. artropaticheskom*
F. sharp
2867. Histological features characteristic of psoriasis:
A. parakeratosis*
B. acanthosis*
C. papillomatosis*
D. sponges
E. acantholysis
F. vacuolar degeneration
2868. Clinical stage and types of psoriasis:
A. stationary*
B. progressive*
C. regressing*
D. subacute
E. sharp
F. all answers are not correct
2869. Phenomena typical for psoriasis:
A. stearin spot*
B. terminal film *
C. blood dew*
D. Asbestos-Hansen
E. Stevens-Jones
F. all answers are not correct
2870. For lupus erythematosus symptoms:
A. erythema*
B. follicular hyperkeratosis*
C. cicatricial atrophy*
D. grid Wickham
E. mukovidnoe peeling
F. the symptom of Cabrera
2871. Stage of scleroderma:
A. atrophy*
B. hyperkeratosis
C. seal*
D. peeling
E. edema*
F. parakeratosis
2872. Periods of the syphilis:
A. incubation*
B. primary*
C. secondary*
D. all right
E. sharp
F. subacute
2873. Main clinical forms of leprosy:
A. Tuberculoid*
B. Tuberous
C. Lepromatous*
D. Kollikvativnom
E. Undifferentiated*
F. Sharp
2874. Clinical forms of athlete's foot:
A. intertriginous*
B. disgidroticheskaya*
C. squamous*
D. worn
E. sharp
F. chronic
2875. Diseases related to dermatophytosis:
A. dermatophytosis*
B. pityriasis versicolor
C. athlete's groin*
D. eritrazma
E. microspores*
F. psoriasis
2876. Specify drugs used in pemphigus vulgaris:
A. prednisolone *
B. triamcinolon *
C. dexamethasone*
D. sulfadimetoksin
E. Dapsone
F. monomitsin
2877. What drugs is used to treat
scabies:
A. 20 % benzyl –benzoate ointment*
B. spray Spregal *
C. 33% sulfuric ointment*
D. 5 % naphthalan ointment
E. 5% sulfur ointment
F. hydrocream
2878. With what diseases should be differentiated scabies:
A. atopic dermatitis *
B. pruritus*
C. pruritus*
D. pityriasis rosea
E. herpes zoster
F. tinea versicolor
2879. Name the obligate factor of dermatitis:
A. concentrated acid *
B synthetic detergents
C. concentrated alkali*
D. jewelry gold
E. . boiling water*
F. jewelry made of platinum
2880. Allergic reaction to the medication may occur:
A. bullous rash*
B. vesicular*
C. erythema*
D. symptom Pylnova
E. papular rash
F. all answers are not correct
2881 . Specify the kind of toxicodermia:
A. fixed erythema*
B. iododerma*
C. bromoderma*
D. psoriasis
E. perniosis
F. pellagrozny dermatitis
2882 . What medications are more likely to cause fixed erythema:
A. sulfadimetoksin*
B. Biseptol*
C. norsulfazol*
D. analgin
E. aspirin
F. tavegil
2883. What disease should be differentiated common toksikodermiya:
A. pityriasis rosea Gibert *
B. acne rosacea
C. exfoliative dermatitis*
D. pyoderma
E. the syndrome of Stevens-Johnson*
F. disease of Borovsk
2884. Indicate the typical localization at the mnogoformnuû exudative erythema:
A. the back of the hand*
B. the rear brake *
C. mucous membranes*
D. scalp
E. the interdigital spaces of the hands
F. the side surfaces of the extremities
2885. What factors are important in the development of mnogoformnuû of erythema multiforme:
A. seasonality*
B. hypothermia*
C. chronic infection*
D. hypersensitivity to fluoride
E. hereditary
F. hypersensitivity to iodine
2886. What differentiate exudative dermatoses mnogoformnuû erythemal:
A. pemphigus vulgaris *
B. Lyell's syndrome*
C. scrofuloderma
D. strophulus
E. exfoliative dermatitis*
F. erythema induratum of Bazin
2887. Specify pathogenetic factors of pink lichen Gibert:
A. viral infection*
B. streptococcal infection*
C. vaccination*
D. yeast
E. hypofunction of the sebaceous glands
F. hyperfunction of the sebaceous glands
2888. Specify the variety of microbial eczema:
A. seksitarina*
B. varicose*
C. sysformat*
D. adult
E. idiopathic
F. horn
2889. What clinical sign is characterized by a rash with true eczema:
A. true polymorphism*
B. symmetry*
C. assimetria
D. ephemeral blisters
E. symptom of "serous wells"*
F. indurative edema
2890. What are the symptoms seen with discoid lupus:
A. symptom Khachaturian*
B. the symptom of "lady heel"*
C. symptom Besnier-Meshchersky *
D. the collar of Byetta
The collar E. Voronov
F. mesh Witham
2891. Specify types of scleroderma:
A. stripy*
B. white spot disease*
C. surface*
D. infiltrative
E. intertriginous
F. vulgar
2892. Lichen planus is characterized by:
A. papular rash*
B.. umbilicated*
C. predominant localization on the flexor surfaces of the extremities*
D. the absence of subjective feelings
E. urticaria rash
F. tubercles
2893. For lichen planus is characterized by:
A. irregular thickening of the granular layer of the epidermis*
B. vnutriarterialno microabcesses
C. moderate hyperkeratosis*
D. parakeratosis
E. acanthosis*
F. disorganization of collagen fibers
2894. What clinical symptom is pathognomonic for lichen planus:
A. grid Wickham*
B. umbilicate indentation*
C. purple colour*
D. symptom of Auspice
E. cells Tzenka
F. all answers are not correct
2895. What diseases does not belong to the group of viral dermatoses:
A. extensive warts*
B. psoriasis*
C. lichen planus*
D. simple bubble лишайD.
E. genital warts
F. all answers are not correct
2896. Vascular spot is:
A. Erythema*
B. Roseola*
C. Teleangioektasia*
D. Lentigo
E. Leucoderma
F. Naevus
2897. Primary morphological elements:
A. bubble*
B. bubble*
C. bump*
D. ulcer
E. Erosion
F. all answers are not correct
2898. Secondary morphological elements:
A. Ulcer*
B. Burl
C. Cork*
D. Volder
E. Erosion*
F. all answers are not correct
2899 . The bubble is formed by:
A. Eczema*
B. herpes simplex*
C. herpes zoster*
D. Psoriasis
E. the LCP
F. all answers are not correct
2900. The outcome of the nodule is:
A. hypopigmented spot*
B. Scale*
C. vanish*
D. Ulcer
E. Atrophy
F. all answers are not correct
2901. Exudative morphological element is:
A. Vesicle*
B. Tubercle
C. Abscess*
D. Papule
D Blister*.
F. all answers are not correct
2902. Layers of epidermis:
A. basal *
B. thorn*
C. granular*
D. fat
E. hypodermally
F. all answers are not correct
2903. Have a cavity morphological elements:
A. abscess*
B. bubble*
C. bubble*
D. node
E. blister
F. all answers are not correct
2904. The rash is polymorphic if:
A. secondary syphilis*
B. eczema*
C. dermatitis Duhring*
D. warts
E. psoriasis
F. all answers are not correct
2905. Monomorphic rash in medicine:
A. psoriasis*
B. CPL*
C. urticaria*
D. dermatitis Duhring
E. herpes zoster
F. all answers are not correct
2906. Clinical signs of psoriasis:
A. the presence of papules*
B. peeling*
C. koebner's phenomenon*
D. blisters
E. tubercles
F. all answers are not correct
2907. Clinical signs of lichen planus:
A. koebner's phenomenon*
B. polygonal papules*
C. pupkoobraznym depressions in the center*
D. blisters
E. nodes
F. tubercles
2908. Agents used topically in the treatment of psoriasis:
A. gel Skin cap*
B. corticosteroid ointment*
C. salicylic ointment
D. Castellani paint
D . Diprosalic ointment*
F. all the answers are not correct
2909. Phenomena typical for psoriasis:
A. stearin spot*
B. terminal film*
C. blood dew*
D. matching items
E. Wickham
F. ladies ' heel
2910. The signs of true polymorphism of rash for dermatitis Duhring:
A. bullas*
B. knot*
C. spot*
D. hump
E. node
F. lichenification
2911. Clinical varieties of pemphigus:
A. vulgar*
B. exudative
C. leaf*
D. psoriasiform
E. vegetative*
F. papular
2912. For lupus erythematosus symptoms:
A. follicular hyperkeratosis*
B. cicatricial atrophy*
C. erythema*
D. grid Wickham
E. mukovidnoe peeling
F. the symptom of Cabrera
2913. Clinical forms of athlete's foot:
A. intertriginous*
B. erased
C. squamous*
D. psoriasiform
E. disgidroticheskaya*
F. pustular
2914. Diseases related to dermatophytosis:
A. athlete's groin*
B. microspores*
C. dermatophytosis*
D. eritrazma
E. hromomikoza
F. impetigo
2915 Antibiotics used in the treatment of candidiasis:
A. nystain*
B. flonal*
C. levorin*
D. cephalosporins
E. doxycyclin
F. kanamycin
2916. When artropaticheskom appropriate will appoint all of the above, exceptA.nizoral*
B. penicillin*
C. synthetic anti-malarial drugs*
D. antioxidants
E. mineralocorticoid drugs
F. nonsteroidal anti-inflammatory drugs
2917. Typical eruptive lichen planus elements have the following features, exceptA. preferential localization on the face*
B. symptom Pylnova*
C. flat papules, polygonal, reddish-purple color
D. waxy luster*
E. Wickham mesh on the surface of papules
F. all answers are not correct
2918. Monomorphic rash can be:
A. psoriasis*
B. lichen planus*
C. warts *
D. erythema multiforme
E. microbial eczema
F. dermatitis Duhring
2919. Hyperkeratosis is characteristic of:
A. psoriasis*
B. ichthyosis*
C. sycosis
D. bullous impetigo
E. warts *
F. all answers are not correct
2920. Woman, 25 years old, married 3 годD. When the survey was diagnosed with chronic
endocervicitis, adnexitis. Smears D. leukocytosis, gonococcus and Trichomonas not found. My
daughter 1.5 years of acute gonorrheal vulvovaginitis etiologies. Correct doctor's tactics:
A. use of a combination of provocation with subsequent bacteriological and bacterioscopic study*
B. prescription of antibacterial drugs according to the sensitivity of the bacterial
flora to antibiotics*
C. the treatment scheme of chronic gonorrhea*
D. prescription of corticosteroids
E. appointment of antihistamines
F. antifungal drugs
2921. What type of topical treatment should
to choose with weeping skin inflammations:
A. lotions*
B. aerosols*
C. aniline dyes*
D. Prater Park
E. cream
F. all answers are not correct
2922. The patient was diagnosed as anterior urethritis chlamydial etiology. What antibiotic group
to prescribe to the patient:
A. aminoglycosides*
B. fluorinated quinolones
C. macrolides*
D. antimalarial
E. tetracycline*
F. cephalosporins In generation
2923. Girls vulvovaginitis Trichomonas etiology characterized by all of these
symptoms exceptA. frothy discharge from the genital slit*
B. the absence of itching*
C. atrophy of the external genitalia*
D. infiltrative lesions
E. frequent and painful urination
F. itch in the vulva
2924 What kind of lotions should choose to oozing of the infected skin surface:
A. ivanyevo*
B. furatsilinovoy*
C. with potassium permanganate*
D. lead
E. tannin
F. boric acid
2925. Diagnostic tests of allergic dermatitis are samples.
A. drip*
B. compression *
C. prick*
D. Baltser
E. Adamson
F. all answers are not correct
2926. To frequent complications of atopic dermatitis include all of these except
A. membrane-destructive processes in the epithelium of the urinary organs during exacerbation
atopic dermatitis*
B. polyadenylation of
C. hyperthyroidism *
D. poor sleep, irritability
E. urethritis*
F. painful and persistent itching
2927. Diagnosis of acute prostatitis, based on the results of the above studies, exceptA. General analysis of blood*
B. biochemical analysis of blood*
C. examination of the skin*
D. palpation of the prostate
E. ultrasound of the prostate
F. all answers are not correct
2928. Examination of children with atopic dermatitis with the aim to understand all of the above,
except
A. identifying the source of microbial sensitization*
B. identification of sepsis in affected skin lesions*
C. identification of pustules in the affected skin lesions*
D. identify the States of immunodeficiency
E. identify indicatorii
F. identify other allergenic factors
2929. What exogenous factors contribute to the development of pyoderma:
A. microfracture *
B. contamination of the skin*
C. helminthic infestations
D. disorders of keratinization
E. hypothermia and hyperthermia*
F. all answers are not correct
2930. What endogenous factors contribute to the development of pyoderma:
A. diabetes mellitus*
B. hypovitaminosis*
C. intestinal toxemia*
D. eosinophilia
E. hemophilia
F. violations of the mechanism of division of epidermocytes
2931. In acute and subacute complicated gonorrhea in men and women in the complex treatment
included all of the above, exceptA. antihistamines*
B. gonovaccine*
C. the desensitization*
D. protivolodochnyi antibiotics
E. antispasmodics
F. sulfonamides
2932. Specify the correct tactics of treatment of torpid and chronic forms of gonorrheal infection
A. immunotherapeutic drugs*
B. local treatment*
C. antibacterial*
D. calcium supplements
E. detoxification drugs
F. all answers are correct
2933. To stafilodermy include:
A. impetigo of Bockhart*
B. slit-like impetigo
C. carbuncle*
D. simple versicolor
E. deep folliculitis*
F. ecthyma vulgar
2934. Clinical symptoms of discoid lupus erythematosus all but
A. bullas *
B. bubbles *
C. blister *
D. erythema
E. densely sitting flakes
F. atrophy
2935. Koebner's phenomenon is observed in the following diseases:
A. lichen planus*
B. psoriasis*
C. folliculitis
D. leprosy
E. vitiligo*
F. tinea versicolor
2936. If any element of dermatitis rash is the bubble:
A. eczema*
B. herpes simplex*
C. tinea*
D. pityriasis rosea Gibert
E. pemphigus
F. all the answers are not correct
2937. True polymorphism is typical for the following diseases except:
A. eczema
B. dermatitis Duhring
C. mnogoformnuû of erythema multiforme
D. psoriasis*
E. lichen planus*
F. tinea versicolor *
2938. Find relevant answers:
Proliferative morphological changes
A. granules, *
B. hyperkeratosis,*
C. spongiose,
D. acanthosis, *
E. acantholysis,
F. all answers are not correct
2939. Diffuse hyperkeratosis is not typical for:
A. ichthyosis
B. keratodermia
C. pemphigus*
D. pyoderma*
E. eczema*
F. all answers are not correct
2940. What factors contribute to the development of pyoderma:
A. disorders of carbohydrate metabolism*
B. the presence of a large number of pathogenic strains*
C. reduction of the barrier functions of the skin *
D. increased blood clotting
E. hypervitaminosis
F. all answers are not correct
2941. For scabies is characterized by:
A. paired papulo-vesicular elements*
B. itching at night*
C. symptom Gorchakov-Ardi *
D. comedones
E. symptom of "Apple jelly"
F. the symptom of "lady heel"
2942. From the ectoderm develop:
A. hair*
B. sebaceous glands*
C. muscles of the hair
D. sweat glands*
E. the vessels of the skin
F. all answers are not correct
2943. To has is characterized by:
A. fever, malaise, headache *
B. blister
C. favourite localization in the armpits, genitals *
D. the formation of fistulous*
E. estimatese pustules
F. all answers are not correct
2944. Specify the clinical form of scabies (a form):
A. worn*
B. clean*
Norwegian C.*
D. erythematous
E. squamous
F. all answers are not correct
2945. To a mixed pyoderma include:
A. brimicombe*
B. chancriform pyoderma *
C. osteo folliculitis
D. hydradenitis
E. ulcerative-vegetative pyoderma *
F. all the answers are not correct
2946. Indicate the clinical varieties of pemphigus acantholytic:
A. leaf*
B. vulgar*
C. vegetative*
D. annular
E. syndrome Stevens-Jones
F. epidemic pemphigus newborns
2947. Dermatosis at what primary morphological element is a node:
A. tuberculosis of the skin*
B. psoriasis
C. tertiary syphilis*
D. cutaneous leishmaniasis*
E. genital warts
F. angioedema
2948. Under what medicine the main element of the rash is pimple:
A. molluscum contagiosum*
B. warts *
C. lichen planus
D. psoriasis*
E. pink ringworm Gibert
F. pemphigus
2949. Clinical symptoms of discoid lupus erythematosus:
A. erythema*
B. tightly sitting scales*
C. atrophy*
D. bubbles
E. blister
F. abscesses
2950. Koebner's phenomenon is not observed in the following diseases:
A. lichen planus
B. psoriasis
C. vitiligo
D. leprosy*
E. folliculitis*
F. pityriasis versicolor*
2951. Under what medicine the main element of the rash is not pimple:
A. molluscum contagiosum
B. warts
C. tinea*
D. lichen planus
E. pityriasis rosea Gibert*
F. pemphigus*
2952. If any element of dermatosis rash is not a bubble:
A. herpes simplex
B. herpes zoster
C. genital herpes
D. pityriasis rosea Gibert *
E. pemphigus *
F. erythematous*
2953. Dermatosis at what primary morphological element is not a node:
A. tertiary syphilis
B. leprosy
C. psoriasis*
D. genital warts*
E. angioedema *
F. all answers are not correct
2954. True polymorphism is characteristic forA. eczema*
B. dermatitis Duhring*
C. mnogoformnuû exudative erythema*
D. psoriasis
E. lichen planus
F. tinea versicolor
2955. Name the atypical form of primary syphiloma:
A. indurative edema *
B. the chancre-amygdala*
C. chancre-panaritium*
D. diffuse pigmentation
E. fagedenizm
F. all answers are not correct
2956. Add the secondary items that are not the result of the transformation of the knot:
A. secondary spot
B. vanish
C. scales
D. erosion*
E. ulcer *
F. scar*
2957. Complication of hard chancre is:
A. pagedevice*
B. phimosis*
C. balanoposthitis*
D. impetigo
E. urethral stricture
F. all answers are not correct
2958. What drugs are used for photochemotherapy in psoriasis:
A. psoralen*
B. povalen*
C. ammifurin*
D. acyclovir
E. nystatin
F. lamisil
2959. What type of external therapy is prescribed in case of progressive stage of psoriasis:
A. corticosteroid ointment*
B. 1-2% salicylic ointment*
C. softening ointments*
D. ointment "Psoriasin"
E. method of dem'yanovich
F. all answers are not correct
2960. For secondary syphilis is characterized by:
A. pustular*
B. roseolous rash*
C. papular rash*
D. Gumma
E. chancre and polyadenylation
F. all answers are not correct
2961. Diagnosis of acute prostatitis, based on the results of the above studies:
A. General analysis of blood
B. biochemical analysis of blood*
C. examination of the skin
D. palpation of the prostate*
E. ultrasound of the prostate
F. ureteroscopy*
2962. What drug is effective for genital herpes:
A. Alkaryl*
B. Neovir*
C. Acyclovir*
D. Sandimmune
E. Diprospan
F. all answers are not correct
2963. The clinical picture disgidroticheskaya form of athlete's foot:
A. bullas*
B. bubbles*
C. edema*
D. bumps
E. peel
F. lichenification
2964. The clinical picture of infiltrative-nagnoitel'noj ringworm:
A. inflammatory infiltrate*
B. pustule*
C. hair loss*
D. lichenification
E. blisters
F. bullas
2965. Clinical symptoms Jock itch:
A. spot red*
B. the border is sharply limited*
C. Blisters*
D. nodes
E. hill
F. blisters
2966. The most frequent localization of superficial candidiasis of the skin:
A. folds of skin*
B. the corners of the mouth*
C. vulva
D. flexor surface of wrists*
E. hair
F. internal organs
2967. Fungal diseases of the skin are called:
A. epidermofiton*
B. Trichophyton *
C. Hansen of the Mycobacterium
D. Microsporum*
E. Mycobacterium Koch
2968. The main clinical signs of lesions of the mucous membranes with candidiasis:
A. erosive surface*
B. white film*
C. bumps
D. papules of the mucous*
E. ulcers
F. nodes
2969. the drugs used in the treatment of fungal infections:
A. griseofulvin*
B. nizoral*
C. lindamycin
D. Fluconazol*
E. Ceftriaxone
F. penicillin
2970. Localization of the rash in scabies in adults:
A. interdigital folds of the hands*
B. the flexor surface of forearms*
C. nails
D. the skin of the abdomen*
E. palm
F. sole
2971. What group of dermatoses is not chancriform pyoderma :
A. cystic dermatosis *
B. sexually transmitted infections *
C. pyoderma
D. viral dermatoses*
E. stafilodermii
F. all answers are not correct
2972. Exfoliative dermatitis Ritter is not a form:
A. streptococcal *
B. contact dermatitis *
C. toxicodermia *
D. Stafilodermii
E. syphilis
F. all answers are not correct
2973. When exfoliative dermatitis Ritter not be detected positive symptom:
A. Nikolsky
B. Wickham *
C. Gorchakov-Ardi *
D. Sisto*
E. all answers are correct
F. all answers are not correct
2974. When pseudoformicaleo finger to pathological process involved:
A. hair follicles *
B. the ducts of sweat glands, eccrine glands, which
C. ducts of the sebaceous glands *
D. ducts of apocrine sweat glands*
E. all answers are correct
F. all answers are not correct
2975. What factors do not contribute to the development of acne:
A. frequent colds *
B. disturbances of thermoregulation *
C. physical activity *
D. hyperfunction of the sebaceous glands
E. all answers are correct
F. all answers are not correct
2976. What varieties of acne support factor are not ticks of zeleznica:
A. nodular acne *
B. acne *
C. acne rosacea
D. abstemious acne *
E. all answers are correct
F. all answers are not correct
2977. What kind of spots are not typical for erythematous stage of demodicosis:
A. telangiectasia
B. petechiae *
C. ecchymosis *
D. webiress *
E. all answers are correct
F. all answers are not correct
2978. For rosacea is not characterized by the absence of:
A. comedones *
B. telangiectasia
C. follicular papules *
D. pustules *
E. all answers are correct
F. all answers are not correct
2979. What morphological elements not characteristic of has:
A. superficial pustules *
B. dense, mobile, painless nodes *
C. blisters *
D. dense, cohesive with the skin painful knots
E. all answers are correct
F. all answers are not correct
2980. At the opening of the nodes has not allocated:
A. adhesive opalescent exudate *
B. cheesy contents*
C. creamy pus mixed with blood
D. serous fluid *
E. all answers are correct
F. all answers are not correct
2981. Favus is not called:
A. filterable virus*
B. anthropophily mushroom
C. Corynebacterium*
D. chlamydia*
E. all answers are correct
F. all answers are not correct
2982. Not the typical localization at eritrazme:
A. axillary folds*
B. scalp*
C. inguinal folds
D. periungual rollers*
E. all answers are correct
F. all answers are not correct
2983. What symptom is not characteristic of infiltrative-suppurative trichophytosis:
A. bee honeycombs
B. luminescence under wood's light*
C. sample Signs*
D. symptom of Pincus*
E. all answers are correct
F. all answers are not correct
2984. Laboratory diagnosis of mycoses is not used:
A. culture diagnostics *
B. microspores affected hair and scales
C. histological examination*
D. fluorescent diagnostics *
E. all answers are correct
F. all answers are not correct
2985. Fungi amaze:
A. internal organs*
B. skin*
C. mucous*
D. hair
E. all answers are correct
F. all answers are not correct
2986. The source of infection for the disease of Borovsk town type is:
A. rodents*
B. Hymenoptera*
C. people
D. wild animals*
E. all answers are correct
F. all answers are not correct
2987. The source of infection for the disease of Borovsk village is not:
A. people*
B. rodents
C. Hymenoptera*
D. wild animals*
E. all answers are correct
F. all answers are not correct
2988. Specify the average duration of cutaneous leishmaniasis urban type:
A. about a year*
B. 2 weeks
C. 8 month*
D. more than a year*
E. all answers are correct
F. all answers are not correct
2989. Specify the average duration of cutaneous leishmaniasis urban type:
A. about a year*
B. a few years *
C. 9 months*
D. 2-6 months
E. all answers are correct
F. all answers are not correct
2990. The source of infection in leprosy is not:
A. wild animals*
B. people
C. Pets*
D. rodents*
E. all answers are correct
F. all the answers are not correct
2991. What material from the patient often does not detect Mycobacterium leprae:
A. in the secretions of the sweat glands*
B. in the urine*
C. in the scrapings from leprom and nasal mucosa
D. a secret of sebaceous glands*
2992. What the virus do not cause genital warts:
A. herpes viruses*
B. filterable viruses
C. herpes simplex virus*
D. CMV*
E. all answers are correct
F. all answers are not correct
2993. Molluscum contagiosum is not...
A. papule
B. ar
item*
C. pustule*
D. vesicle*
E. all answers are correct
F. all answers are not correct
2994. Specify the variety of microbial eczema:
A. numular*
B. seborrheic*
C. varicose*
D. prodigiosa
E. all answers are correct
F. all answers are not correct
2995. Which symptom is not marked pink lichen Gibert:
A. a symptom of chips*
B. symptom of "mother" plaque
C. symptom tissue paper*
D. a symptom of a ladies ' heel*
E. all answers are correct
F. all answers are not correct
2996. Monomorphic rash may not be if:
A. mnogoformnuû the exudative erythema*
B. Microbial eczema*
C. psoriasis
D. dermatitis Duhring *
E. all answers are correct
F. all answers are not correct
2997. Acantholysis is not observed in:
A. herpes simplex*
B. bullous form of strophulus*
C. herpes zoster*
D. true pemphigus
E. all answers are correct
F. all answers are not correct
2998. Monomorphic papular rash may not occur if:
A. lichen planus
B. urticaria*
C. eczema*
D. pityriasis rosea*
E. all answers are correct
F. all answers are not correct
2999. Acanthosis is not typical for:
A. tournyol*
B. simple versicolor*
C. lichen planus
D. a simple bubble versicolor*
E. all answers are correct
F. all answers are not correct
3000. Hyperkeratosis is not typical for:
A. bullous impetigo*
B. warts
C. sycosis*
D. folliculitis*
E. all answers are correct
F. all answers are not correct